All In-service questions Flashcards

1
Q

A 30-year-old man with a history of radius and ulna midshaft fractures underwent fasciotomies for acute compartment syndrome of the nondominant left volar forearm with immediate return of normal perfusion 4 months ago. He is now pain-free with normal sensation but has persistent stiffness and weakness of the fingers, despite appropriate splinting and physiotherapy. His compartments are soft, and there are no joint contractures. He has full motion and normal strength, except the fingers and thumb can fully extend only with the wrist flexed, and finger and thumb flexion have MRC grade 4/5 strength. Which of the following is the most appropriate next step in management?

A) Dynamic splinting
B) Flexor tendon transfers
C) Intrinsic releases
D) Selective muscle origin slide
E) Strengthening physiotherapy

A

The correct response is Option D.

The patient is presenting with evidence of Volkmann ischemic contracture of his deep volar forearm compartment musculature, specifically flexor digitorum profundus and flexor pollicis longus. Flexor digitorum superficialis could be minimally involved, but the wrist flexors are spared. Mild median nerve involvement with full recovery and sparing of the ulnar nerve would support this diagnosis. The patient has already undergone appropriate physiotherapy. With persistent findings at 4 months, the most appropriate treatment is surgical exploration, debridement of necrotic muscle, with either selective muscle origin slide or tendon lengthening of preserved but contracted muscle.

Although continued dynamic physiotherapy could potentially provide further improvement in this patient’s muscle tightness, strengthening physiotherapy will not address the problem adequately. Dynamic splinting could complement physiotherapy and be helpful but has likely provided most of its benefit in the 4 months after initial surgery. Intrinsic releases would be indicated in intrinsic muscle contractures; however, this patient has involvement of the extrinsic flexors, not the intrinsic muscles. Finally, flexor tendon transfers would be indicated for more severe cases of Volkmann contractures, where there is no muscle function remaining. This patient’s examination suggests adequate muscle function remains.

References

Gulgonen A, Ozer K. Compartment Syndrome. In, Green’s Operative Hand Surgery, 6th edition. Ed. Wolfe et al. Philadelphia: Churchill Livingstone, 2011. 1929-1948.

Stevanovik MV, Sharpe F. Compartment Syndrome and Volkmann Ischemic Contracture. In, Green’s Operative Hand Surgery, 7th edition. Ed. Wolfe, et al. Philadelphia: Elsevier, 2017. 1763-1787.

Thevenin-Lemoine C, Denormandie P, Schnitzler A, et al. Flexor origin slide for contracture of spastic finger flexor muscles: a retrospective study. J Bone Joint Surg Am. 2013 Mar 6;95(5):446-53.

How well did you know this?
1
Not at all
2
3
4
5
Perfectly
2
Q

A 26-year-old man sustained a crush injury to the tip of the left middle finger with an associated fracture at the dorsal base of the distal phalanx with nail bed injury 6 months ago. No treatment was provided. Examination shows non-union of the distal phalanx. Which of the following is the most likely secondary deformity in this patient?

A) Boutonniere deformity
B) Jersey finger
C) Quadriga
D) Swan neck deformity
E) Trigger finger

A

The correct response is Option D.

The scenario described involves a bony mallet deformity in which a distal phalanx fracture is associated with disruption of terminal extension at the distal interphalangeal joint. If untreated, the DIP extension loss due to a non-union of a bony mallet injury may progress to a swan neck deformity through compensatory proximal phalangeal hyperextension in the setting of continued and persistent flexion at the distal interphalangeal joint (from unopposed pull of the flexor digitorum profundus tendon). A secondary swan neck deformity may occur because of dorsal subluxation of the lateral bands and attenuation of the volar plate and transverse retinacular ligament at the PIP joint level.

A jersey finger is caused by rupture of the terminal flexor digitorum profundus. A boutonniere deformity can be caused by an injury to the central slip (but not the terminal extensor tendon). Quadriga is due to loss of length of a repaired FDP tendon, causing the finger with the repaired tendon to reach terminal flexion sooner than the other fingers whose FDP tendons are of normal length. A trigger finger does not involve a fracture of the DIP joint.

How well did you know this?
1
Not at all
2
3
4
5
Perfectly
3
Q

A 30-year-old woman comes to the office because of a mass of the dorsum of the wrist for the past 5 months. She reports that the mass occasionally gets larger and then gets smaller. Physical examination shows the mass is mildly tender and transilluminates. From which of the following articulations is this lesion most likely to arise?

A) Pisotriquetral
B) Radioscaphoid
C) Scapholunate
D) Scaphotrapezial
E) Thumb carpometacarpal (CMC)

A

The correct response is Option C.

The mass in question is most likely a dorsal ganglion cyst of the wrist. Sixty to 70% of ganglion cysts are found in the dorsal aspect of the wrist. Dorsal wrist ganglion cysts usually communicate with the joint by a stalk. This stalk usually originates at the scapholunate interval, but it can also rarely arise from other aspects of the dorsal wrist joint.

Thirteen to 20% of ganglia are found on the volar aspect of the wrist, and they usually arise from the radioscaphoid, scapholunate, scaphotrapezial, or metacarpotrapezial joint, in decreasing order of frequency.

Ganglia arising from the flexor tendon sheath of the hand account for approximately 10%.

How well did you know this?
1
Not at all
2
3
4
5
Perfectly
4
Q

A 35-year-old woman is evaluated because of swelling of the right breast 3 years after undergoing augmentation mammaplasty. The implant type is unknown. Ultrasonography shows a seroma, and a fine-needle aspiration is performed. Which of the following immunohistochemical stains of the aspirate is most appropriate?

A) CCD79a
B) CD30
C) CK20
D) E-cadherin
E) p63

A

The correct response is Option B.

Patients who present with a late seroma should be evaluated for possible breast implant-associated anaplastic large cell lymphoma (BI-ALCL). A late seroma is usually accepted as occurring 1 year following surgery; however, there are cases of BI-ALCL seromas that have presented as early as 4 months.

The first step in evaluating BI-ALCL is ultrasonography, followed by fine-needle aspiration if indicated. The fluid requires evaluation beyond routine cell cytology. Immunohistochemistry test for CD30 was the most commonly positive marker for BI-ALCL. Immunohistochemistry stains specific antigens in cells by binding to this antigen in an antibody/antigen reaction. The specific stain can then be seen under light microscopy. The CD30 antibody labels anaplastic large cell lymphoma cells. CD30 is a transmembrane cytokine receptor belonging to the tumor necrosis factor receptor family.

CK20 and CCD79a were negative for tested BI-ALCL specimens.

P63 stains myoepithelial cells and is used to rule out invasive breast tumors.

E-cadherin helps distinguish ductal from lobular carcinoma.

How well did you know this?
1
Not at all
2
3
4
5
Perfectly
5
Q

A 45-year-old woman who has breast cancer comes to the office for consultation regarding bilateral breast reconstruction. Reconstruction using autologous abdominal tissue is considered. The risk of abdominal morbidity is discussed. Which of the following flap techniques is most likely to result in the lowest level of overall abdominal morbidity?

A ) Deep inferior epigastric artery perforator

B ) Free muscle-sparing transverse rectus abdominis musculocutaneous (TRAM)

C ) Free TRAM

D ) Pedicled TRAM

E ) Superficial inferior epigastric artery flap

A

The correct response is Option E.

The superficial inferior epigastric artery (SIEA) flap results in the lowest level of overall abdominal morbidity, as the technique used in harvesting this flap leaves the abdominal fascia intact. These vessels are only present in less than one third of patients, and only one half of those patients will have vessels of sufficient diameter to support a free tissue transfer. The SIEA flap is also associated with a higher frequency of total flap loss, in addition to a higher incidence of fatty necrosis.

The deep inferior epigastric artery perforator (DIEAP or DIEP) flap involves dissection of one or two (occasionally more) perforators through the rectus muscle to the inferior epigastric vessels. Although this technique does not include any rectus muscle or sheet/fasica within the flap itself, it does involve moderate-level trauma to those organs and can cause abdominal wall morbidity.

The free TRAM or free muscle-sparing TRAM techniques are free tissue transfer variants of the TRAM, whereby a small amount of the rectus is taken with the flap. In the more advanced MS free TRAM technique, the amount of muscle taken is only enough to allow safe transfer of the perforators. Although this technique is less invasive than the pedicled TRAM, it does still involve removal of a variable portion of the rectus muscle and fascia.

A pedicled TRAM flap technique involves transferring of the flap based on the superficial epigastric vessel that runs within the rectus muscle. Therefore, the entirety of the rectus (unilateral or bilateral) is elevated out of its native abdominal wall location.

Many publications have compared the other modes of breast reconstruction with reference to abdominal wall morbidity. This area remains controversial. The general consensus remains that

in a bilateral reconstruction, pedicled TRAM flaps are associated with higher levels of overall abdominal morbidity (hernias, bulges, weakness, intolerance to exercise, etc) when compared with the use of MS free TRAM, DIEP, or SIEA flaps. The use of SIEA flaps results in minimal to no abdominal wall morbidity.

How well did you know this?
1
Not at all
2
3
4
5
Perfectly
6
Q

A 35-year-old man comes to the office for follow-up 3 years after he sustained a scaphoid fracture of the dominant right wrist that was treated in a cast until radiographically healed. Examination shows reduced wrist extension of 35 degrees, weakened grip strength, and dorsoradial wrist pain. Scaphoid malunion is suspected, and an oblique sagittal CT scan is obtained. Which of the following is the minimum intrascaphoid angle at which surgical intervention is required?

A) 10 Degrees
B) 25 Degrees
C) 45 Degrees
D) 65 Degrees
E) 80 Degrees

A

The correct response is Option C.

Treatment of a scaphoid malunion or “humpback” nonunion deformity by means of an opening interposition wedge bone graft is indicated when the lateral intrascaphoid angle is greater than 45 degrees. The intrascaphoid angle is determined by drawing a line tangent to the dorsal cortex of the distal fragment and the palmar cortex of the proximal fragment. Normally, this angle is 30 to 40 degrees. Amadio and coworkers reported on 45 patients with 46 scaphoid fractures greater than 6 months after healing. There were good clinical outcomes in 83% of those with intrascaphoid angles less than 35 degrees, and posttraumatic arthritis in 22%. In contrast, in those with greater than 45 degrees of lateral intrascaphoid angulation, only 27% had good outcome, and 54% developed posttraumatic arthritis.

Nakamura and colleagues performed volar wedge bone grafting on seven symptomatic patients with scaphoid malunion, and all improved their symptoms.

How well did you know this?
1
Not at all
2
3
4
5
Perfectly
7
Q

A 55-year-old woman with a BRCA gene mutation elects to undergo bilateral mastectomy with reconstruction using a deep inferior epigastric perforator flap. BMI is 41 kg/m2. Physical examination shows both supra- and infraumbilical adiposity with excess skin and a mature cesarean delivery scar. This patient has the highest risk for which of the following early postoperative complications?

A) Abdominal wall bulge
B) Abdominal wall hernia
C) Delayed wound healing of donor site
D) Fat necrosis of the flap
E) Flap failure

A

The correct response is Option C.

The highest risk is for delayed wound healing of the donor site. Because of the patient’s morbid obesity and prior cesarean delivery scar, she has the highest risk for some form of wound breakdown or prolonged wound healing. This risk can be as high as 50 to 60% for morbidly obese patients. These trends are similar in patients following reconstruction with a transverse rectus abdominis musculocutaneous (TRAM) flap.

To reduce these risks, minimal undermining is recommended and only done if necessary. Techniques to preserve all cutaneous perforators will help reduce the risk associated with closure of the donor sites.

While morbid obesity can be associated with increased abdominal wall thickness, there is no correlation with the occurrence of abdominal wall bulge or hernia. The risks for these complications are less than 2%.

Patients with morbid obesity can have shorter operative times, but there is no correlation with overall flap failure, with rates reported to be less than 1%. This is also seen in pedicled and free TRAM flap reconstructions.

Rates of fat necrosis of the flap can be as high as 10 to 15% in patients undergoing reconstruction, but this risk is not affected by body habitus or body mass index and is lower than the risk for delayed wound healing.

How well did you know this?
1
Not at all
2
3
4
5
Perfectly
8
Q

A 45-year-old woman with systemic sclerosis (scleroderma) has severe Raynaud phenomenon. A photograph is shown. Periarterial injection of botulinum toxin type A is being considered for treatment in this patient. This treatment is believed to relieve vasospasm in Raynaud phenomenon by which of the following mechanisms?

A) Blocking fast sodium channels in axonal gap junctions
B) Increasing the activity of chronically down-regulated group C nerve fiber nociceptors
C) Inhibiting Rho/Rho kinase activity
D) Obstructing myofibroblast contractile activity in vascular smooth muscle
E) Promoting substance P secretion/receptor sensitivity

A

The correct response is Option C.

Several mechanisms have been proposed to explain the effect of botulinum toxin type A (Botox) to inhibit Raynaud phenomenon in patients with scleroderma. Studies have demonstrated inhibition of Rho/Rho kinase activity, inhibition of substance P secretion and receptor sensitivity, and decreasing the activity of chronically up-regulated C-fiber nociceptors all to occur in models of Raynaud phenomenon that responded to Botox treatment. Fast sodium channels conduct axonal signals AT in gap junctions, but have not been shown to be affected by Botox. Myofibroblasts may be involved in late fibrosis of scleroderma patients but do not exist within the vascular smooth muscle.

References

Fonseca C, Abraham D, Ponticos M. Neuronal regulators and vascular dysfunction in Raynaud’s phenomenon and systemic sclerosis. Curr Vasc Pharmacol. 2009;7(1) :34–39.

Iorio ML, Masden DL, Higgins JP. Botulinum toxin A treatment of Raynaud’s phenomenon: a review. Semin Arthritis Rheum. 2012 Feb;41(4):599-603.

Neumeister MW. The role of botulinum toxin in vasospastic disorders of the hand. Hand Clin. 2015 Feb;31(1):23-37.

Uppal L, Dhaliwal K, Butler PE. A prospective study of the use of botulinum toxin injections in the treatment of Raynaud’s syndrome associated with scleroderma. J Hand Surg Eur Vol. 2014 Oct;39(8):876-80.

How well did you know this?
1
Not at all
2
3
4
5
Perfectly
9
Q

The metacarpophalangeal (MCP) joint of the thumb is which of the following types of joint?

A) Ball-and-socket
B) Condyloid
C) Hinge
D) Pivot
E) Saddle

A

The correct response is Option C.

The metacarpophalangeal (MCP) joint of the thumb and interphalangeal joints of the index through little fingers are hinged joints and allow flexion and extension only. Lateral forces can disrupt the collateral ligaments, resulting in partial or full tears.

Condyloid joints allow flexion and extension, abduction and adduction, and circumduction, and they can be seen in the MCP joints of the index through little fingers and in wrist joints. Saddle joints allow flexion and extension, abduction and adduction, and circumduction, and they can be seen in the carpometacarpal joint on the thumb. Ball-and-socket joints allow flexion and extension, abduction and adduction, and internal and external rotation, and they can be seen in the shoulder and hip joints. Pivot joints allow rotation and are seen in the atlas and axis bones.

How well did you know this?
1
Not at all
2
3
4
5
Perfectly
10
Q

When considering dislocations of the thumb carpometacarpal (CMC) joint, which of the following is most correct regarding which vector of dislocation would occur with injury to the stabilizing ligament?

Injured LigamentVector of Dislocation

A)Dorsal intercarpalradial

B)Dorsoradialdorsal

C)Intermetacarpalulnar

D)Radiocarpaldorsal

A

The correct response is Option B.

The CMCJ is very important for hand function and plays a key role in pinch and grasp. The increased range of motion inherent to the thumb CMCJ is attributed to the anatomy of the joint. The biconcave saddle shaped articular surface of the CMCJ also provides some inherent stability. Motion allowed by the joint includes flexion, extension, adduction, abduction, circumduction. Stabilizing ligaments and joint capsule further reinforce the joint, thus thumb CMCJ dislocations are rare injuries. These injuries account for less than 1% of hand injuries.

There are five major stabilizing ligaments to the CMCJ: anterior (volar) oblique, ulnar collateral, intermetacarpal, dorsoradial, and dorsal (posterior) oblique. These ligaments are critical stabilizers during motion. The volar oblique ligament and dorsoradial ligaments are considered to be the most important resistive forces in dislocation in cadaver studies. Reports of traumatic thumb CMCJ dislocation have been in a dorsal vector. The volar oblique ligament was originally thought to be the critical resistive ligament; however, recent literature has supported the dorsal complex (includes the dorsoradial and posterior oblique ligaments) are the most critical for restraint of the joint, thus are injured in dorsal dislocations. Timely recognition is important for these injuries as immediate reduction and casting or splinting for 4 to 6 weeks may be adequate to prevent recurrence. However, these injuries are often missed on radiologic examination or may be persistently unstable. Closed reduction and Kirschner wire fixation may be adequate for treatment in persistently unstable injuries. Some authors advocate for open reduction and ligament reconstruction. Delayed treatment especially beyond three weeks will likely require open reduction and ligament reconstruction. These injuries are often missed on x-ray examination as they can be subtle especially in the setting of more obvious trauma. Inadequate treatment puts these patients at increased risk for subsequent posttraumatic osteoarthritis given the joint malalignment.

The radiocarpal and dorsal intercarpal ligaments are wrist stabilizer not thumb CMCJ stabilizers.

How well did you know this?
1
Not at all
2
3
4
5
Perfectly
11
Q

An otherwise healthy 37-year-old woman presents for delayed microsurgical breast reconstruction. Which of the following is associated with use of tamoxifen?

A) Hemodynamic instability
B) Impaired wound healing
C) Increased bleeding
D) Seroma formation
E) Thromboembolic events

A

The correct response is Option E.

Breast cancers that are estrogen receptor positive may be responsive to adjuvant chemotherapy with selective estrogen receptor modulators such as tamoxifen, which can reduce recurrence and mortality. Tamoxifen is associated with thromboembolic events, such as deep venous thrombosis and pulmonary embolism. This prothrombotic effect has been postulated to be secondary to the effect of tamoxifen on estrogen receptors that are abundant within vascular endothelium.

Tamoxifen has been shown to be associated with increased rates of total flap loss and decreased rates of flap salvage when taken within 28 days of microsurgical breast reconstruction, which represents two half-lives of the active metabolite of tamoxifen (N-desmethyl tamoxifen, t1/2=14 days). It has therefore been recommended that in patients undergoing microsurgical breast reconstruction, tamoxifen be held for at least 28 days preoperatively. Some authors have further advised holding the medication postoperatively in addition to preoperatively.

Tamoxifen is not associated with impaired wound healing, increased bleeding, hemodynamic instability, or seroma formation.

How well did you know this?
1
Not at all
2
3
4
5
Perfectly
12
Q

A 55-year-old woman is brought to the emergency department after sustaining mutilating injury to the hand during a motor vehicle collision. Examination shows the hand is unsalvageable. Disarticulation of the wrist is planned. Compared with transradial amputation, which of the following is the most likely benefit of this approach?

A) Better accommodation of a myoelectric unit
B) Better forearm pronation and supination
C) Decreased risk of neuroma formation
D) Decreased risk of prosthetic abandonment
E) More stable soft-tissue envelope

A

The correct response is Option B.

The choice of wrist disarticulation compared with transradial amputation is a controversial one. The primary benefit of the wrist level disarticulation is preservation of the distal radioulnar joint and consequential improvement in forearm rotation. Preservation of the metaphyseal flare of the radius may aid in prosthetic fit; however, the additional length associated with functional units such as myoelectrics may result in a limb length discrepancy. The prominence of the distal radius and ulna may predispose to pressure-related wound issues associated with prosthetic wear. Patients with wrist level disarticulation are more likely to abandon their prosthesis compared with transradial amputees.

References

Taylor CL. The biomechanics of control in upper-extremity prostheses. Artif Limbs. 1955 Sep;2(3):4-25.

Wright TW, Hagen AD, Wood MB. Prosthetic usage in major upper extremity amputations. J Hand Surg Am. 1995 Jul;20(4):619-22.

Rafael J. Diaz-Garcia and Paul S. Cederna. Major Limb Amputations and Prosthetics. In: Wolfe S, Pederson W, Hotchkiss R, eds. Green’s Operative Hand Surgery. 3rd ed. New York, NY: Churchill-Livingstone; 1993:1753-1762.

How well did you know this?
1
Not at all
2
3
4
5
Perfectly
13
Q

A 52-year-old man sustains an amputation of the index finger of his dominant right hand from a table saw. Physical examination shows a sharp amputation immediately distal to the flexor digitorum superficialis insertion. He does not smoke cigarettes. Which of the following factors is the most appropriate indication to perform a replantation?

A) Dominant hand
B) Index finger amputation
C) Level of amputation
D) Nonsmoking status
E) Patient age

A

The correct response is Option C.

The most appropriate indication to perform a replantation is the level of the amputation. Replantation of an amputation distal to the flexor digitorum superficialis is attempted because the function of the digit is improved with additional length to a normal proximal interphalangeal joint. An amputation in a child is an indication for replantation (adult age is not). Hand dominance is not a major variable in the determination of whether or not to perform a replantation. Replantation of single digits (including the index finger) at the proximal phalanx or proximal interphalangeal joint in adults often is not performed because the limited motion of the digit can inhibit overall hand function. An exception is any level amputation of the thumb, which is a major indication for replantation because the thumb provides 40 to 50% of hand function. Smoking status is not a major variable for the consideration of replantation.

References

Prucz RB, Friedrich JB. Upper extremity replantation: current concepts. Plast Reconstr Surg. 2014 Feb;133(2):333-42.

Jazayeri L, Klausner JQ, Chang J. Distal digital replantation. Plast Reconstr Surg. 2013 Nov;132(5):1207-17.

How well did you know this?
1
Not at all
2
3
4
5
Perfectly
14
Q

A 53-year-old man comes to the emergency department because of an avulsion degloving injury to the left nondominant thumb sustained 3 hours ago. The amputated part is not retrievable. Physical examination shows loss of skin from the interphalangeal joint distally on both volar and dorsal surfaces. The distal phalanx and flexor pollicis longus and extensor pollicis longus tendons are intact. He has no other associated injuries. Which of the following is the most appropriate method of reconstruction of the thumb?

A) Amputation revision at the mid-proximal phalanx
B) Great toe wraparound flap
C) Radial forearm osteocutaneous flap
D) Second toe-to-thumb transfer
E) Volar neurovascular advancement flap

A

The correct response is Option B.

Thumb reconstruction remains a difficult challenge for hand surgeons. Amputations of the skin distally may be covered with palmar advancement flaps; however, this technique is only limited to wounds less than 50% of the palmar surface of the thumb distal to the interphalangeal joint. In order to preserve length and function in more proximal amputations, either a regional or distant flap is required. The toe-to-thumb wraparound flap requires a microvascular anastomosis of digital vessels and nerves, providing excellent sensation and cosmetic results. The toe donor site can be covered with a skin graft in order to preserve length.

The volar neurovascular advancement flap would not adequately cover a defect this size. Amputation at the mid-proximal phalanx would result in a very short thumb with loss of function. The radial forearm flap may be utilized to cover the above defect; however, it would lack adequate sensation. Any osteocutaneous radial forearm flap would not be indicated since there is preservation of the bone. Similarly, a second toe-to-thumb transfer would not be indicated since there is preservation of bone in this patient.

References

Graham DJ, Venkatramani H, Sabapathy SR. Current Reconstruction Options for Traumatic Thumb Loss. J Hand Surg Am. 2016 Dec;41(12):1159-1169.

Del Piñal F, Pennazzato D, Urrutia E. Primary Thumb Reconstruction in a Mutilated Hand. Hand Clin. 2016 Nov;32(4):519-531.

How well did you know this?
1
Not at all
2
3
4
5
Perfectly
15
Q

A 25-year-old woman comes to the office because of a 1-week history of erythema and clear drainage from the right breast 6 weeks after undergoing bilateral augmentation mammaplasty. She is afebrile and her vital signs are within normal limits. The drainage from the breast is sent for cultures. Broad-spectrum antibiotics are administered, but no improvement is noted over the next 48 hours. Surgical debridement and explantation of the prostheses are performed. After 7 days, cultures grow Mycobacterium fortuitum. Which of the following is the most appropriate next step?

A) Administration of ciprofloxacin and trimethoprim-sulfamethoxazole for 6 weeks
B) Administration of ciprofloxacin and trimethoprim-sulfamethoxazole for 6 months
C) Administration of isoniazid, rifampicin, and pyrazinamide for 6 weeks
D) Administration of isoniazid, rifampicin, and pyrazinamide for 6 months
E) No antibiotic therapy is needed because the infected prostheses have been removed

A

The correct response is Option B.

The most appropriate next step in management is to initiate a 6-month course of ciprofloxacin and trimethoprim-sulfamethoxazole (Bactrim). Mycobacterium fortuitum is an atypical, nontuberculous mycobacterium (NTM), and it is one of the most common causes of NTM soft-tissue infections. It occurs most commonly in the presence of foreign bodies, such as breast prostheses. The incidence of these opportunistic infections has increased over the years. NTM infections can be more indolent and manifest weeks, or even months, following surgery. They occur most commonly with erythema, swelling, and clear drainage, although purulence may be seen. Fever may be absent. On surgical exploration, exuberant granulation tissue and turbid, odorless fluid are often noted. Routine Gram stains and cultures are usually negative. Therefore, it is imperative to request acid-fast bacilli staining and mycobacterial cultures if suspicion of NTM infection is high. Removal of the prosthesis and thorough debridement of the periprosthetic space, followed by long-term (3 to 6 months) antibiotic therapy, is required to treat this infection. Culture sensitivities should guide the antibiotic regimen, but ciprofloxacin, trimethoprim-sulfamethoxazole (Bactrim), clarithromycin, and doxycycline are used commonly for treatment. Reimplantation of the prosthesis should not be considered for a period of at least 6 months.

Isoniazid, rifampicin, and pyrazinamide are standard antibiotics used to treat tuberculosis caused by Mycobacterium tuberculosis, not atypical mycobacteria. Although removal of the affected prosthesis is required, long-term antibiotic therapy is an essential part of the treatment.

How well did you know this?
1
Not at all
2
3
4
5
Perfectly
16
Q

A 45-year-old carpenter presents with a six-month history of an ulceration of the ring fingertip and pain at rest. Digital brachial index is 0.45, and angiography demonstrates occlusion of the ulnar artery. The patient has tried three months of calcium channel blockers and aspirin without relief. Which of the following is the most appropriate treatment for this patient?

A) Amputation of the fingertip
B) Chemical sympathectomy
C) Reconstruction of the ulnar artery
D) Stellate ganglion block
E) Surgical sympathectomy

A

The correct response is Option C.

Conservative treatment includes smoking cessation, calcium channel blockers, anticoagulation therapy, stellate ganglion block, and behavior modification. Nonoperative management is generally considered first-line treatment, because most patients will have at least partial resolution of their symptoms. With that said, 70% of those treated nonoperatively had partial resolution of their symptoms, and only 12% had complete resolution. Of patients treated operatively, 42% had complete resolution of their symptoms and 42% had partial resolution.

For patients with evidence of more advanced disease such as digital ulceration, chronic resting pain, or conservative management failure, operative intervention may be considered. Preoperative noninvasive vascular studies can be used to determine which patients may require reconstruction versus simple excision and ligation. Studies have suggested that a digital brachial index less than 0.7 indicates reconstruction may be warranted. An index of less than 0.5 suggests critical ischemia, which may result in tissue loss.

Surgical options fall into two basic groups: resection of the involved arterial segment with ligation, and vascular reconstruction with or without interposed graft. Graft occlusion is reported in as high as 78% of patients. Despite a high percentage of occlusion, patients remained satisfied. Patients with occluded reconstructions did not experience worsening of symptoms in comparison with the patent reconstructions. Preoperative digital brachial index values, although informative as to the patient’s digital perfusion, do not mandate a particular operative intervention. The general treatment algorithm is to perform surgery on patients who have failed on medical management and local treatment to heal any digital soft tissues. A decision on ligation versus reconstruction can be made with the assistance of information gathered by preoperative angiography and noninvasive vascular studies, as well as intraoperative assessment of ulnar digital perfusion with temporary occlusion of the ulnar artery. Poor perfusion following temporary occlusion mandates reconstruction of the artery, whereas adequate perfusion, despite occlusion, can be treated with simple excision or ligation of the diseased ulnar artery segment.

References

Endress RD, Johnson CH, Bishop AT, Shin AY. Hypothenar hammer syndrome: long-term results of vascular reconstruction. J Hand Surg Am. 2015;40(4):660-665.e2.

Lifchez SD, Higgins JP. Long-term results of surgical treatment for hypothenar hammer syndrome. Plast Reconstr Surg. 2009;124(1):210-216.

Vartija L, Cheung K, Kaur M, Coroneos CJ, Thoma A. Ulnar hammer syndrome: a systematic review of the literature. Plast Reconstr Surg. 2013;132(5):1181-1191

How well did you know this?
1
Not at all
2
3
4
5
Perfectly
17
Q

A 7-year-old patient with a history of submucous cleft palate and persistent severe velopharyngeal insufficiency after Furlow palatoplasty undergoes video nasoendoscopy. An abnormal closure pattern is observed, with excellent movement of the velum and Passavant’s ridge but poor lateral wall motion. On the basis of these findings, which of the following is the most appropriate treatment for this patient’s velopharyngeal insufficiency?

A) Inferiorly based pharyngeal flap
B) Palatal lift appliance
C) Speech therapy
D) Sphincter pharyngoplasty
E) Superiorly based pharyngeal flap

A

The correct response is Option D.

With a “bow tie” pattern seen on the nasoendoscopy, the patient is an ideal candidate for sphincter pharyngoplasty, which will bring the lateral walls in more centrally, so the velum and pharynx can close off the velopharyngeal port.

Pharyngeal flap surgery is ideal for patients who have a large central gap or sagittal closure pattern caused by palatal hypotonia or shortened palatal length. Palatal soft tissue augmentation is ideal for patients with a very small central gap following adenoidectomy. A palatal lift appliance is typically used in patients with adequate palatal length, but inadequate motor function. The patient described in this scenario has excellent motor function of the velum, but poor lateral wall motion.

Speech therapy does not correct the anatomic deficiency in this patient population.

How well did you know this?
1
Not at all
2
3
4
5
Perfectly
18
Q

A patient comes to the office 6 months after undergoing bilateral vertical mastopexy because she is dissatisfied with her postoperative appearance. Height is 5 ft 5 in (165 cm). Physical examination shows the distance from nipple to sternal notch is 16 cm bilaterally, and the distance from nipple to inframammary fold is 14 cm bilaterally. Which of the following is the most appropriate next step in management?

A) Conversion to free nipple grafts
B) Placement of a dual-plane breast implant
C) Placement of a subglandular breast implant
D) Resection of excess skin at the level of the inframammary fold
E) Reassurance, massage, and observation

A

The correct response is Option D.

This case illustrates superior nipple malposition. The distance from nipple to inframammary (IMF) fold of 14 cm is much too long. The correct answer is to resect the lower pole skin at the IMF in order to move the nipple down. This would create a “T” scar and improve nipple position. Vertical mastopexies and reduction mammaplasties have a learning curve and much of this is predicting the nipple position postoperatively. The nipple should be designed lower on the breast than is done during marking a Wise pattern. At 6 months, it is unlikely the nipple position will change dramatically, so observation is not recommended. Addition of an implant will not help the nipple position. Conversion to free nipple grafts, while possible, will not lead to an aesthetic scar pattern.

How well did you know this?
1
Not at all
2
3
4
5
Perfectly
19
Q

A 24-year-old woman comes to the office to discuss augmentation mammaplasty. She is interested in subglandular implant placement and would like to discuss the risks of augmentation. Which of the following risks is more likely with smooth round silicone implants compared with textured anatomic silicone implants?

A) Anaplastic large cell lymphoma
B) Capsular contracture
C) Double capsule
D) Late seroma
E) Malrotation

A

The correct response is Option B.

Capsular contracture is more common in smooth round silicone implants than in textured implants. It is believed that the texturing of the implant is protective against significant capsule formation.

On the other hand, there are several increased risks associated with textured anatomic implants. These include increased risks of late seroma and breast implant-associated anaplastic large cell lymphoma (BIA-ALCL), although this is very rare. Double capsule is a complication more recently noted with the introduction of textured anatomic implants. Malrotation can only be seen in an anatomic textured implant, because smooth round implants are symmetric in shape. In addition, it can be difficult to differentiate between anatomic shaped and smooth round implants, with several studies showing their similar cosmetic outcomes.

How well did you know this?
1
Not at all
2
3
4
5
Perfectly
20
Q

A 55-year-old woman comes to the office for a second opinion because she is displeased with the results of a recent bilateral mastectomy and breast reconstruction with 800-mL high-profile silicone implants. A photograph is shown. BMI is 35 kg/m2. She repeatedly shows pictures of models with augmented breasts and says that she wants her breasts to be “perkier.” She requests augmentation/mastopexy. Which of the following is the most appropriate next step in management?

A) Augmentation/mastopexy
B) Implant exchange
C) Mastopexy
D) Reassurance
E) Referral to a psychiatrist

A

The correct response is Option D.

The most reasonable approach in this patient is to offer reassurance and reset her expectations. A patient with a BMI of 35 kg/m2 who undergoes mastectomy and implant reconstruction will never look like a model with augmented breasts. This patient clearly has misguided expectations. Any surgical intervention is unlikely to produce the result she is looking for, when in fact she has a very acceptable result as is. Referral of this patient to a psychiatrist will likely upset the patient and undermine her trust.

How well did you know this?
1
Not at all
2
3
4
5
Perfectly
21
Q

A 28-year-old woman, gravida 2, para 2, undergoes augmentation mammaplasty 1 year post partum. On postoperative day 3, the patient comes to the office because of impaired wound healing at the incision site. Physical examination shows white viscous discharge leaking from the edge of the wound consistent with galactorrhea. Which of the following is the most appropriate management?

A) Administration of bromocriptine
B) Administration of metoclopramide
C) Administration of trimethoprim-sulfamethoxazole
D) Application of negative pressure wound therapy
E) Debridement of the wound edges with wet-to-dry dressings

A

The correct response is Option A.

There are incidents of surgical procedures of the breast associated with galactorrhea leading to skin breakdown, nipple necrosis, and cellulitis. A dopamine agonist such as bromocriptine will cause decreased lactation in cases of galactorrhea/galactocele, thereby improving wound healing. Antibiotics such as sulfamethoxazole and trimethoprim (Bactrim) are generally not required, because the exudate is sterile. There is no need for debridement of the wound edges. Negative pressure wound therapy may increase lactation and galactorrhea, further impairing wound healing. Metoclopramide is a dopamine antagonist used for nausea and vomiting.

How well did you know this?
1
Not at all
2
3
4
5
Perfectly
22
Q

A 65-year-old woman comes to the office 1 month before a scheduled mastopexy. Annual mammography shows a 1.5-cm mass in the upper outer quadrant. Core needle biopsy is performed. Pathologic examination of excised tissue identifies papilloma without atypia. Which of the following is the most appropriate next step in management?

A) Bilateral breast sonography
B) Excisional biopsy of needle-localized area
C) Repeat annual mammography in 12 months
D) Repeat mammography at 6-month intervals for 1 year
E) Stereotactic vacuum-assisted biopsy

A

The correct response is Option B.

Percutaneous biopsy methods are commonly accepted for the initial evaluation of clinically occult breast lesions, although certain nonmalignant lesions pose dilemmas with respect to the most appropriate clinical management. Papillary lesions of the breast can either be benign or malignant, although differentiation is radiologically difficult. Moreover, it is difficult for pathologists to reliably distinguish among benign, atypical, and malignant papillary lesions on the limited fragmented tissue specimens they receive after needle sampling.

Previous studies have demonstrated high rates of ductal carcinoma in situ (11%) in patients diagnosed with benign papillomas by needle biopsy and who subsequently underwent a surgical excision, although conflicting data suggest an extremely decreased rate of malignancy when histology is benign on needle biopsy.

The management of benign papillary lesions is somewhat controversial. Although conservative follow-up with either yearly mammogram or short-interval follow-up may be appropriate for certain patients diagnosed with benign papilloma, certain features of this patient’s lesion make conservative follow-up inappropriate. Sonographic follow-up in a 65-year-old woman with mature breast parenchyma and a solid mammographically detected mass would not provide much additional information, and a repeat percutaneous biopsy, whether core needle or vacuum-assisted, would also not be effective. Given the size of the lesion and the age of the patient, surgical excision is warranted despite the lack of atypia on needle biopsy. Benign papillomas tend to be smaller than 1 cm and centrally located, whereas malignant lesions are more often greater than 1.5 cm and are peripherally located.

How well did you know this?
1
Not at all
2
3
4
5
Perfectly
23
Q

A 48-year-old woman undergoes excision of a 3-cm recurrent keloid of the presternal chest. Immediate reconstruction with a collagen-glycosaminoglycan scaffold dermal regeneration template is performed, followed by thin (0.008-in) epidermal autografting 21 days later. After it has healed completely, punch biopsy is performed. The absence of which of the following histologic features is most likely to indicate regenerated skin in this patient?

A) Capillary loops at the dermal-epidermal junction
B) Elastic fibers
C) Hair follicles
D) Neovascularization
E) Rete ridges

A

The correct response is Option C.

Regenerated skin is clearly quite different histologically from scar and, in fact, shares many characteristics with normal physiologic skin. Regenerated skin shows mechanical competence, vascularization, and heat and cold sensitivity. Furthermore, the dermal-epidermal junction shows formation of rete ridges and capillary loops. Regenerated skin displays elastic fibers and increased collagen fiber density in the reticular dermis, and it often exhibits nerve fiber regeneration as well. Regenerated skin, even when resurfaced with a split-thickness skin graft, however, does not have the dermal appendages such as hair follicles and sweat glands, that are present throughout normal skin.

How well did you know this?
1
Not at all
2
3
4
5
Perfectly
24
Q

A 52-year-old woman receives a diagnosis of invasive ductal carcinoma of the right breast. Which of the following details from this patient’s history is the strongest risk factor for this diagnosis?

A) Early first pregnancy (less than 30 years)
B) Early menarche (less than 12 years)
C) Early menopause (less than 55 years)
D) Multiple episodes of breast-feeding
E) Remote oral contraceptive use

A

The correct response is Option B.

Early menarche is the highest risk factor for breast cancer of the options listed. Late first pregnancy, late menopause, no breast-feeding, and recent oral contraceptive use are also risk factors for breast cancer but are not as high risk.

How well did you know this?
1
Not at all
2
3
4
5
Perfectly
25
Q

A 35-year-old woman comes to the office with her boyfriend for consultation regarding augmentation mammaplasty. She currently wears a size 34B brassiere and is considering having her brassiere size increased to a D cup. She says she is happy with the way she looks in clothes, but the boyfriend indicates he would like to see a little more cleavage when she is in a swimsuit. History includes liposuction of her lateral thighs 6 months ago by a local dermatologist; she was satisfied with the result. She has also had injection of botulinum toxin type A to the glabella 3 times in the last year. Which of the following is the best reason to refuse performing the procedure for this patient?

A) The patient may be being pushed into surgery
B) The patient may be a “surgiholic”
C) The patient may have body dysmorphic disorder
D) The patient may have a personality disorder
E) The patient may have unrealistic expectations

A

The correct response is Option A.

Most aesthetic surgeons and mental health professionals agree that patients who exhibit even mild signs of psychiatric problems are not good candidates for aesthetic surgery. Many patients present without obvious signs of problems and are unfortunately discovered when postoperative problems arise. However, there are certain groups of patients with easily identifiable characteristics that constitute a red flag: those who are pushed into surgery by others, those with whom you are incompatible, the ?surgiholic? with a long past surgical history, those facing marital or familial disapproval, those with body dysmorphic disorder, the overly demanding patient, and those with unrealistic expectations.

How well did you know this?
1
Not at all
2
3
4
5
Perfectly
26
Q

A 54-year-old woman with a history of augmentation mammaplasty with textured silicone implants has histologic confirmation of breast implant–associated anaplastic large cell lymphoma (BIA-ALCL). MRI and PET scans show no associated masses, with activity localized to the periprosthetic seroma. Which of the following is the most appropriate next step in management of this patient?

A) Anterior capsulectomy with removal of the implants bilaterally
B) Complete capsulectomy with removal of the implant on the affected side
C) Partial capsulectomy with replacement of the implant
D) Removal of the textured implant and replacement with a smooth implant
E) Sealing of the seroma cavity with fibrin glue

A

The correct response is Option B.

Breast implant–associated anaplastic large cell lymphoma (BIA-ALCL) is a rare peripheral T-cell lymphoma that has been increasingly recognized as a serious, albeit uncommon, complication associated with the use of textured breast implants. Since the initial case report in 1996, there have been continually increasing reported cases of this rare malignancy and according to the most recent data available, the lifetime risk of association between breast implants and BIA-ALCL is between 1 in 1000 to 1 in 30,000 with the ASPS recognizing nearly 200 cases in the US and nearly 500 cases worldwide.

BIA-ALCL patients typically present with a spontaneously occurring periprosthetic fluid collection or capsule-associated mass approximately 10 years following implantation of the breast implant. To date, all cases have had some association with a textured device. Initial workup includes ultrasound for evaluation of a periprosthetic fluid collection or mass. Periprosthetic fluid collections should undergo fine-needle aspiration in the clinic or ultrasound-guided aspiration by interventional radiology if there is concern for trauma to the implant while masses require tissue biopsy. Specimens should be sent for cytology with immunohistochemistry and flow cytometry for T-cell markers, specifically CD30 cell surface protein. A recent systematic review revealed that 66% of BIA-ALCL patients presented with isolated late-onset seroma while only 8% presented with an isolated new breast mass.

National Comprehensive Cancer Network (NCCN) guidelines for treatment of BIA-ALCL recommend complete removal of the lymphoma (fluid and/or mass), complete capsulectomy, and removal of the implant. More advanced disease may require chemotherapy, radiotherapy, and/or lymph node dissection. Although some surgeons advocate removal of the contralateral breast implant as approximately 4.6% of cases have demonstrated incidental lymphoma in the contralateral breast, this recommendation is controversial. The official NCCN guidelines for treatment only recommend consideration of contralateral breast implant removal but this is not mandated.

How well did you know this?
1
Not at all
2
3
4
5
Perfectly
27
Q

A 5-year-old male has a cerebrospinal fluid leak and a 3 x 3-cm area of wound dehiscence involving the posterior trunk following tethered cord repair. Which of the following is the most appropriate method to reconstruct the wound?

A) Gluteal muscle flap and skin advancement flap
B) Latissimus muscle turnover flap and skin advancement flap
C) Local fascial flap and skin advancement flap
D) Skin advancement flap
E) Split-thickness skin graft

A

The correct response is Option C.

The most appropriate method to reconstruct the wound is a local fascial flap and skin advancement flap. The major principle of tethered cord and myelomeningocele repair is to obtain a well-vascularized layer of soft tissue coverage between the dural and skin closures. The fascia overlying the paraspinous muscles can be turned over as flaps to cover the underlying dural repair. This vascularized soft tissue layer will minimize the risk of cerebrospinal fluid leak by reinforcing the dural repair. In addition, the fascial flaps will prevent contact with cutaneous bacteria and subsequent meningitis if either the dural repair or skin repair breaks down. A split-thickness skin graft over the dura would not adequately protect the spinal cord. Closing the skin directly over the dural repair using skin advancement flaps would place the child at risk for meningitis in the event of a cerebrospinal fluid leak or if wound breakdown occurred along the incision line of the widely undermined skin flaps. The use of a regional gluteal or latissimus muscle flap to cover the dural repair is unnecessary because local tissue (paraspinous muscle fascia) is available. Harvesting the gluteal or latissimus muscles also may cause significant donor site morbidity in a child already at risk for ambulatory problems from a neurological deficit.

How well did you know this?
1
Not at all
2
3
4
5
Perfectly
28
Q

A 50-year-old woman comes to the office 6 weeks after undergoing right mastectomy and immediate placement of a tissue expander. She reports swelling and redness of the right breast. A photograph is shown. Which of the following factors is most predictive of implant salvage failure in this patient?

A) Culture positive for Pseudomonas species
B) Elevated body mass index
C) Periprosthetic seroma
D) Presence of cellulitis
E) Previous irradiation

A

The correct response is Option A.

Immediate implant-based reconstruction has become increasingly popular over the past two decades, accounting for over 70% of all reconstructions in the United States. The benefits of immediate reconstruction are numerous, including decreased recovery/number of required procedures and increased patient psychological well-being and aesthetic outcome. However, the complication (seroma, mastectomy flap necrosis, loss of implant, and infection) rates after implant-based reconstruction remain relatively high. Infection rates in the reported literature range from 2.5 to 24%.

Historically, periprosthetic infection or implant exposure mandated immediate implant removal. However, numerous studies over the past several decades have demonstrated implant salvage rates of 37.3 to 73% depending on the methods employed. Several studies have looked at the predictive factors that increase the risk of a failed salvage attempt. Salvage was typically defined as administration of systemic antibiotics (oral or intravenous), removal of the infected implant, partial/total capsulectomy, pocket curettage, implant pocket irrigation with antibiotic solution, and placement of a new device.

Factors associated with implant salvage failure include an elevated white blood cell count, elevated temperature, deep-seated pocket infection (purulent periprosthetic fluid), and atypical pathogens such as methicillin-resistant Staphylococcus aureus (MRSA) and Pseudomonas species. Spear et al. showed that 93.9% of mild implant infections (localized cellulitis) could be salvaged compared with a 30% salvage rate in the severe infection group. Factors such as smoking, chemotherapy, previous irradiation, mastectomy skin necrosis, increased BMI, and use of acellular dermal matrix (ADM) have demonstrated increased rates of implant-related infections, but these factors have not been demonstrated to increase the risk of implant salvage failure.

How well did you know this?
1
Not at all
2
3
4
5
Perfectly
29
Q

A 47-year-old woman who underwent bilateral augmentation mammaplasty with silicone implants to treat mammary hypoplasia 17 years ago is evaluated because of worsening pain, firmness, and distortion of her breasts. Which of the following diagnostic evaluations is most sensitive for evaluating this patient’s silicone breast implants?

A) Breast thermography
B) CT scan
C) Mammography
D) MRI
E) Ultrasonography

A

The correct response is Option D.

MRI scan would be the most sensitive and specific method for detection of silent rupture of a silicone breast implant in this patient. Classic MRI findings indicating rupture include the linguini sign or the teardrop sign. Current FDA recommendations are to obtain MRI screening for silent rupture three years after placement of silicone implants and every two years after that.

CT scanning can show findings similar to those seen with MRI, but CT involves ionizing radiation, which can be harmful. CT has not been proven to be as sensitive as MRI in evaluating silicone breast implant rupture.

Ultrasonography is a less costly method of implant evaluation but this method is highly operator-dependent. In asymptomatic women, a subsequent MRI scan is generally needed to confirm a positive ultrasound screen.

Mammography is indicated for screening for breast cancer but not for implant rupture.

Breast thermography utilizes digital infrared imaging to evaluate metabolic activity and vascular circulation of the breast to look for suspicious signs of breast cancer. It is not effective in the evaluation of silicone breast implant rupture.

How well did you know this?
1
Not at all
2
3
4
5
Perfectly
30
Q

A 40-year-old woman is referred for reconstruction following mastectomy for a peripherally located ductal carcinoma in situ. A nipple-sparing mastectomy with immediate, single-stage prosthesis reconstruction with acellular dermal matrix is planned. Which of the following interventions is most appropriate to ensure that no residual cancer exists?

A) Chemotherapy
B) Ductal washing
C) Radiation therapy
D) Retroareolar frozen section
E) Sentinel lymph node evaluation

A

The correct response is Option D.

More plastic surgeons are performing reconstruction for women pursuing prophylactic mastectomy, which is requested quite frequently to avoid cancer recurrence and to achieve optimal aesthetic outcome. Exclusion criteria for nipple-sparing mastectomy include tumors greater than 3 cm, clinical invasion of the nipple-areola complex, tumors within 2 cm of the nipple, evidence of multicentric disease, positive intraoperative retroareolar frozen section, or nodal disease. If carcinoma is found in the retroareolar tissue, the nipple-areola complex must be removed. A patient who would require sentinel lymph node evaluation, radiation therapy, or chemotherapy would not be an appropriate candidate for nipple-sparing mastectomy. Annual mammography is recommended for any patient with a history of breast cancer and is not specific to the issue of nipple-sparing mastectomy. Ductal washing is not relevant for this pathology.

How well did you know this?
1
Not at all
2
3
4
5
Perfectly
31
Q

A 53-year-old woman is referred for consultation regarding breast reconstruction following mastectomy. Autologous breast reconstruction options are discussed. Which of the following is an advantage of the pedicled transverse rectus abdominis musculocutaneous (TRAM) flap over the free TRAM flap?

A) Decreased complications in patients with a history of smoking
B) Decreased incidence of complete flap loss
C) Decreased risk of abdominal hernia
D) Decreased risk of fat necrosis
E) Ease of insetting and flap shaping

A

The correct response is Option B.

There is considerable controversy regarding the merits of the pedicled TRAM, free TRAM, free deep inferior epigastric perforator (DIEP), and free superficial inferior epigastric perforator (SIEP) flap reconstructions. Reconstruction with each flap has distinct advantages and disadvantages. Currently, the literature does not clearly favor the use of one procedure over the others.

The pedicled TRAM flap is characterized by a shorter operative time and a decreased risk of complete flap loss when compared to the free flaps. However, the pedicled TRAM flap shows a higher incidence of fat necrosis with partial flap loss, as well as an increased length of stay. Free flaps offer ease of flap shaping and insetting, as well as a decreased risk of abdominal hernia and abdominal wall weakness. Free flaps are more suitable for patients who are diabetic, are overweight, or smoke cigarettes.

Deciding which particular procedure to perform requires assessment on a case-by-case basis and consideration of the surgeon’s level of comfort with the different techniques.

How well did you know this?
1
Not at all
2
3
4
5
Perfectly
32
Q

A 29-year-old man undergoes evaluation for nonunion of a scaphoid fracture. Reconstruction with a vascularized osseous flap is planned, and a medial femoral condyle flap is chosen. During harvest, the vascular pedicle for this flap runs between which of the following structures?

A) Anterior to the tensor fascia lata and posterior to the vastus lateralis
B) Anterior to the vastus medialis and anterior to the adductor tendon
C) Anterior to the vastus medialis and posterior to the rectus femoris
D) Posterior to the rectus femoris and anterior to the vastus lateralis
E) Posterior to the vastus medialis and anterior to the adductor tendon

A

The correct response is Option E.

The medial femoral condyle osseous free flap has become a useful option for reconstruction of bony defects in the extremities, particularly of the scaphoid waist and proximal pole. The vascular supply to this flap is from the descending geniculate artery in the distal medial aspect of the thigh. To explore and identify the pedicle for this flap, the vastus medialis is reflected anteriorly, and the adductor tendon is found posterior to the vessels. The rectus femoris is located anterior to the dissection for this flap.

How well did you know this?
1
Not at all
2
3
4
5
Perfectly
33
Q

A 42-year-old woman comes to the office because she is dissatisfied with an obvious step-off between the chest wall and the superior pole of the breast 6 months after she underwent immediate expander reconstruction of the left breast. No further adjuvant therapy was indicated. After full expansion, the tissue expander was removed, and a permanent smooth, round silicone prosthesis was placed. BMI today is 28 kg/m2. Examination shows a well-defined inframammary fold. The volume is matched to the contralateral breast. Which of the following is the most appropriate surgical procedure for correction of this patient’s deformity?

A) Exchange of the silicone prosthesis with a silicone prosthesis of a larger volume and dimension
B) Exchange of the silicone prosthesis with a saline prosthesis of the same volume and dimension
C) Fat grafting to the superior pole and chest wall
D) Placement of acellular dermal matrix as an inferolateral sling
E) Removal of the prosthesis and reconstruction with a deep inferior epigastric perforator flap

A

The correct response is Option C.

Fat grafting has been shown to successfully address acquired contour deformities in breast reconstruction. Changing the prosthesis to a saline prosthesis of the same size and dimension will not address the problem of the step-off between the prosthesis and the chest wall in this slender patient (BMI 20 kg/m2) which is due to decreased soft-tissue coverage in the superior pole. Likewise, increasing the size and dimensions of the silicone prosthesis would not correct this contour deformity and would also lose the symmetry with the contralateral breast. This patient, without excess lower abdominal skin and subcutaneous fat, is not a candidate for a deep inferior epigastric perforator flap. Placement of acellular dermal matrix as an inferolateral sling is typically performed in the initial stage of tissue expansion and reconstruction. It can help define the inframammary fold, which this patient does not need.

How well did you know this?
1
Not at all
2
3
4
5
Perfectly
34
Q

A 53-year-old woman comes to the office because of unilateral swelling of the breast 5 years after undergoing subglandular augmentation mammaplasty. A diagnosis of anaplastic large T-cell lymphoma (ALCL) is established. Which of the following is most likely to represent the progression of this patient’s disease when compared with a patient who has ALCL but no breast prostheses?

A) A more aggressive clinical course and a poorer prognosis
B) A more aggressive clinical course but a more favorable prognosis
C) A more indolent clinical course and a more favorable prognosis
D) A more indolent clinical course but a poorer prognosis
E) The same clinical course and prognosis

A

The correct response is Option C.

Anaplastic large T-cell lymphoma (ALCL) is a rare (1 per million) non-Hodgkin lymphoma that has been reported in women with and without breast prostheses. However, increasing case reports suggest an association with breast prostheses, although direct causation has not been established. ALCL associated with breast prostheses has malignant cells infiltrating the periprosthetic capsule or in the periprosthetic fluid collection. It is associated with both silicone- and saline-filled prostheses and seen in patients who have had prostheses for augmentation mammaplasty as well as breast reconstruction. Although the cytology is the same between ALCL associated with and without breast prostheses, ALCL that develops around prostheses tend to have an indolent clinical course and favorable prognosis when compared with systemic ALCL.

How well did you know this?
1
Not at all
2
3
4
5
Perfectly
35
Q

An otherwise healthy 40-year-old woman comes to the office for augmentation mammaplasty. Mammography 6 months ago showed no abnormalities. Family history is negative for breast cancer. She wants to know if silicone gel implants are safe and what she should do after the procedure to monitor the implant for evidence of rupture. According to the current federal guidelines, which of the following is the most appropriate recommendation to this patient regarding surveillance?

A) MRI 3 years after implantation and every 2 years thereafter
B) MRI every 10 years
C) MRI if symptoms such as chronic myalgia and fatigue develop
D) Yearly mammograms
E) Yearly MRI

A

The correct response is Option A.

Evidence-based data to confirm the validity of screening patients with silicone implants are lacking. In 2011, the FDA issued recommendations for physicians on the use of silicone gel-filled implants. Recommendations included providing copies of educational brochures, giving appropriate informed consent, maintaining medical vigilance, and reporting adverse events. It also suggested that patients undergoing augmentation mammaplasty get an MRI 3 years after implant placement and every 2 years thereafter. The purpose of these recommendations is not to replace routine cancer surveillance.

How well did you know this?
1
Not at all
2
3
4
5
Perfectly
36
Q

A 40-year-old woman presents with small, non-healing ulcers of the right index and middle fingertips. Medical history includes limited scleroderma diagnosed 5 years ago, chronic pain, and color changes of the fingers in cold temperatures. The patient’s symptoms have not improved with administration of nifedipine. Angiography shows diffuse vascular narrowing without any focal lesions. Which of the following is the most appropriate intervention for pain relief and ulcer healing in this patient?

A) Cervical sympathectomy
B) Continuous brachial plexus blockade
C) Digital bypass
D) Onabotulinum toxin A
E) Stellate ganglion block

A

The correct response is Option D.

This patient has Raynaud’s phenomenon associated with scleroderma. The pathophysiology of Raynaud’s is thought to be related to sympathetic hyperactivity, elevated plasma endothelin, increased peripheral alpha-2 receptors, and possibly abnormal platelet and red cell function. Botulinum toxin type A has been shown to improve digital perfusion on laser Doppler, decrease pain, and result in ulcer healing. In a series of 33 patients injected with 50 to 100 U of onabotulinum toxin A, all patients had ulcer healing by 60 days postinjection. Pain relief typically occurred within 5 to 10 minutes of injection and complication rates were low and limited to injection site reactions. A prospective, randomized, placebo-controlled trial showed patients with limited scleroderma and shorter duration of disease had the best response to onabotulinum toxin A.

Stellate ganglion blocks have been shown to have only variable success for Raynaud’s with only short-term symptom relief and no effect on ulcer healing. Stellate blocks may not disrupt all sympathetic input to the extremity. Brachial plexus blocks may help with perfusion temporarily but are advocated mainly in patients undergoing microvascular surgery. Their use is not recommended in this setting. Surgical bypass to the superficial palmar arch has been shown to increase blood flow to the hand and improve ulcer healing. However, bypass to the digital vessels would not be indicated as the distal target vessels are often diminutive without adequate flow.

References

Bello RJ, Cooney CM, Melamed E, et al. The Therapeutic Efficacy of Botulinum Toxin in Treating Scleroderma-Associated Raynaud’s Phenomenon: A Randomized, Double-Blind, Placebo-Controlled Clinical Trial. Arthritis Rheumatol. 2017 Aug;69(8):1661-1669.

Neumeister MW, Chambers CB, Herron MS, et al. Botox Therapy for Ischemic Digits. Plast Reconstr Surg. 2009 Jul;124(1):191-201.

Porter SB, Murray PM. Raynaud Phenomenon. J Hand Surg Am. 2013 Feb;38(2):375-377.

Neumeister MW. Botulinum Toxin Type A in the Treatment of Raynaud’s Phenomenon. J Hand Surg Am. 2010 Dec;35(12): 2085-2092.

How well did you know this?
1
Not at all
2
3
4
5
Perfectly
37
Q

Which of the following technical factors has the greatest favorable impact on the final appearance of a surgical scar?

A) Closing the wound in a single layer
B) Use of an absorbable suture
C) Use of topical cyanoacrylate
D) Retention suture
E) Wound-edge eversion

A

The correct response is Option E.

The two technical factors that increase the likelihood of a “good” scar are placement of sutures that will not leave permanent suture marks and wound-edge eversion. In wounds where the skin is brought precisely together, there is a tendency for the scar to widen. In wounds where the edges are everted or hypereverted in an exaggerated fashion, this tendency is minimized possibly by reducing the tension on the closure. While the most common method of closing a wound is with sutures, there is nothing necessarily superior about sutures or a specific type of suture. Staples, skin tapes, or wound adhesives are also useful in certain situations. Regardless of the method of closure or type of suture used, precise approximation of skin edges without tension is essential to ensure healing with minimal scarring. Simple interrupted suture is the gold standard for suturing wounds closed and everting the skin edges. Retention sutures tend to leave the most obvious and unsightly cross-hatching if they are not removed early. Wounds deeper than the skin are closed in layers. The key is to eliminate the dead space and provide a strong closure to prevent dehiscence and reduce tension. However, not all layers necessarily require separate closure.

How well did you know this?
1
Not at all
2
3
4
5
Perfectly
38
Q

A 42-year-old woman with a 3-cm invasive ductal carcinoma of the right breast is evaluated for breast reconstruction. She has not decided how she wants to manage her contralateral breast. Regarding eliciting a family history, which of the following cancers is associated with a mutation in a breast cancer-susceptibility gene?

A) Colon
B) Esophageal
C) Lung
D) Pancreatic
E) Thyroid

A

The correct response is Option D.

The breast cancer-susceptibility gene types 1 and 2 (BRCA1 and BRCA2) are tumor suppressor genes. Mutations in BRCA1 and BRCA2 are associated with hereditary breast and ovarian cancers. Additionally, they can be associated with increased risks of pancreatic and prostate cancer. Thyroid, lung, esophageal, and colon cancer are not associated with increased risks of BRCA1 and BRCA2 mutations.

How well did you know this?
1
Not at all
2
3
4
5
Perfectly
39
Q

A 22-year-old nulliparous woman is evaluated for improvement of breast shape and size. Examination shows bilateral hypoplastic breasts with constricted bases and herniation of breast parenchyma in the areolae. Tuberous breast deformity is diagnosed. Bilateral breast augmentation with smooth, round gel implants via periareolar incisions is planned. Which of the following maneuvers is most likely to decrease the risk for a “double-bubble” deformity?

A) Decreasing the areolar diameter
B) Lowering of the inframammary fold
C) Parenchymal scoring
D) Periareolar incision
E) Subpectoral placement of the implant

A

The correct response is Option C.

Common hallmarks of tuberous breast deformity include varying degrees of hypoplastic breast parenchyma, deficiencies of the inferior pole, herniation of the parenchyma in the areola, enlarged areolae, superior placement of the inframammary fold, and asymmetry. Surgical goals are to achieve symmetry, sufficient volume (especially in the hypoplastic areas), lowering of the inframammary fold, reduction of areolar tissue herniation, and correction of any ptosis. A double-bubble deformity can occur when the inframammary fold is not sufficiently obliterated. The risk for this is increased with superiorly displaced inframammary folds, as in tuberous breasts. Parenchymal scoring would both release any constricting bands to allow the lower pole tissue to spread over the implant as well as release the superiorly displaced inframammary fold. While decreasing the areolar diameter and lowering of the inframammary fold are goals for breast improvement, neither will treat a double-bubble deformity. A periareolar incision is often advocated in repair of tuberous breasts because of the ability to reduce the areola; it alone, however, will not prevent a double-bubble deformity. Subpectoral placement of implants increases the risk for double-bubble deformity while subglandular placement of implants decreases the risk. Many advocate a dual-plane approach to capitalize on increased upper pole coverage combined with the benefits of a subglandular relationship in the inferior pole.

How well did you know this?
1
Not at all
2
3
4
5
Perfectly
40
Q

A 30-year-old man presents to the emergency department with acute left wrist pain after falling 10 feet from a ladder. X-ray studies of the left wrist are shown. After failed closed reduction, the patient reports tingling that progresses to worsening and constant numbness of the left index and long fingers over the course of 6 hours. Which of the following urgent interventions is most appropriate?

A) Aspiration of the wrist
B) Carpal tunnel release
C) MRI of the wrist
D) Open reduction of the scaphoid
E) Repeat closed reduction

A

The correct response is Option B.

This patient has a type IV perilunate dislocation, or a true lunate dislocation. This represents a complete disruption of the ligamentous stabilizers about the lunate. These injuries are high energy and can be ligamentous only (lesser arc injuries) or include fractures (greater arc) and are then termed perilunate fracture dislocations. Mayfield et al described the stages of injury progressing from radial to ulnar in a type IV dislocation, including injury of the scapholunate ligament, disruption of the lunocapitate joint, injury of the lunotriquetral ligament, and dislocation of the lunate from its fossa at the radiocarpal joint volarly into the carpal tunnel.

On posteroanterior x-ray study of the wrist, there will be disruption of Gilula’s lines. On lateral x-ray study, a “spilled teacup” sign is seen.

Closed reduction with relaxation and traction is important, as the lunate needs to be relocated to its fossa to restore relative alignment of the wrist and to decompress the median nerve in the carpal tunnel. Surgical intervention can then be performed for open reduction of the joints and ligament repair after swelling has improved. However, progression in median nerve symptoms in the setting of successful or failed closed reduction is indicative of acute carpal tunnel syndrome and necessitates urgent surgical intervention.

Advanced imaging such as MRI is not required but may be helpful. Repeat closed reduction is likely to fail at this time, may worsen the swelling, and is unlikely to resolve the carpal tunnel symptoms. Open reduction of the scaphoid is not emergent, and the patient does not have a scaphoid fracture. Aspiration of the wrist will not resolve the inciting etiology of the patient’s carpal tunnel symptoms.

How well did you know this?
1
Not at all
2
3
4
5
Perfectly
41
Q

A 39-year-old man comes to the office 3 months after falling 10 feet from a ladder because of persistent radial-sided wrist pain, swelling, decreased grip strength, and a painful clicking in the wrist with moderate activity. Physical examination shows diffuse tenderness of the radial wrist and a painful “clunk” when palpating the scaphoid during radial deviation of the wrist. Initial x-ray studies showed no fracture or dislocation. Recent standard x-ray studies of the wrist show no fracture and normal carpal bone alignment. Which of the following is the most likely diagnosis?

A) de Quervain tenosynovitis
B) Dynamic scapholunate instability
C) Flexor carpi radialis tendinitis
D) Kienböck disease
E) Occult scaphoid fracture

A

The correct response is Option B.

This patient has dynamic scapholunate instability. These injuries can be difficult to diagnose and require a high index of suspicion. A normal x-ray study at 12 weeks in the setting of these clinical findings suggests there is a disruption of the scapholunate interosseous ligament (SLIL) that is symptomatic only with mechanical loading.

The SLIL is the primary stabilizer of the scapholunate joint, but it is surrounded by multiple secondary stabilizers consisting of the extrinsic wrist ligaments. Normal kinematic motion of the proximal carpal row is controlled by the tough interosseous ligaments. The dorsal component of the SLIL is the primary restraint to distraction, torsion, and translational forces. Disruption of the dorsal SLIL alone will result in changes in wrist mechanics, but the presence of the intact secondary stabilizing ligaments will prevent changes seen on a normal static x-ray study, such as scapholunate dissociation or an increased scapholunate angle.

Stress view x-ray studies, such as the clenched-pencil view, should be obtained when dynamic instability is suspected in the setting of a normal static x-ray study series. These results can be compared with the contralateral normal side. Non-contrast MRI is an advanced imaging modality averaging 71% sensitivity, 88% specificity, and 84% accuracy for SLIL tears. There is improved accuracy with 3.0T MRI machines. Wrist arthroscopy is the gold standard for diagnosis and can be combined with therapeutic procedures such as debridement or thermal shrinkage.

An occult scaphoid fracture should be visible at 12 weeks following the injury. Bone resorption at the fracture site makes the fracture line generally visible within 14 days. If suspicion remains for an occult scaphoid fracture at 2 weeks, additional imaging such as MRI or CT scan is indicated. At 3 months following the injury, any fracture present should be visible and treated as a non-union of the scaphoid.

De Quervain tenosynovitis is defined as tendinitis of the first dorsal extensor compartment. This condition generally presents with pain and tenderness over the radial styloid with a positive Finkelstein test. Tenderness of the carpal bones and carpal bone instability such as a painful “clunk” would not be present. The condition is most associated with repetitive use and not acute trauma.

Kienböck disease involves collapse of the lunate due to vascular insufficiency and avascular necrosis. Etiology is unknown but may involve a combination of anatomic factors and trauma. Early symptoms are similar to a wrist sprain but involve more global wrist pain, loss of dorsiflexion, and tenderness of the dorsal wrist over the lunate. Early stage I disease can have normal x-ray studies but will often show signs of a lunate fracture. Later stage disease shows sclerosis and ultimately fracture or collapse of the lunate.

Flexor carpi radialis (FCR) tendinitis is not a common diagnosis. It presents with wrist pain, crepitus, and point tenderness over the FCR at the wrist flexion crease with flexion and radial deviation. Although it is a cause of radial-sided wrist pain, findings of carpal bone instability on examination are not present.

How well did you know this?
1
Not at all
2
3
4
5
Perfectly
42
Q

An 8-year-old girl is brought to the office because of severe, worsening pain as well as finger swelling and numbness three days after she underwent cast placement for a fracture of the left forearm. After removal of the cast, her pain continues and is worsened by passive wrist motion. Which of the following is the most appropriate next step in assessment of this patient’s condition?

A) Angiography
B) CT scan
C) Duplex ultrasound
D) Electromyography
E) Manometry

A

The correct response is Option E.

The most appropriate next test is manometry. The patient is exhibiting signs of compartment syndrome after swelling due to fracture under a tight restrictive cast. Signs and symptoms of compartment syndrome include pain with passive stretch, increased pressure on palpation, paresthesia, paralysis, pallor, and pulselessness.

Early recognition and treatment are necessary to prevent permanent damage. The pressure within the muscles increases, preventing blood flow to the area and capillary exchange of nutrients. Fasciotomy is recommended if compartment pressure exceeds 30 mmHg, or if the difference between intracompartmental pressure and diastolic blood pressure is less than 30 mmHg. Without treatment, ischemic necrosis to the muscles can result, leading to Volkmann ischemic contracture and causing permanent disability. Scarring and shortening of the muscles can occur, with resultant contracted intrinsic minus appearance of the hand.

Compartment pressures can be measured by handheld manometer (Stryker pen), or needle manometer method (Whitesides) with an arterial line setup. Operative fasciotomy is indicated to release the compartment pressures and prevent tissue loss and muscle necrosis in cases of compartment syndrome. Loss of pulse typically occurs later in the spectrum of findings.

Angiography would be useful in evaluating vasculature and blood flow. Typically pain with passive stretch does not occur in cases of arterial insufficiency.

Duplex ultrasound can evaluate the presence of deep venous thrombosis, which can be a source of pain and swelling. This can occur through compression of the antecubital region, but in this case, the symptomatology would prompt measurement of compartment pressures and urgent fasciotomy.

Electromyography can be used to evaluate nerve function but would not be the next appropriate measure.

CT scan can provide detailed imaging but would not be indicated in this situation and would delay treatment.

References

Hughes T. Compartment Syndrome and Volkmann’s Contracture. In: Trumble T, Rayan G, Budoff, J, et al. Principles of Hand Surgery and Therapy, 2nd ed. Philadelphia, PA: Saunders Elsevier; 2010;8:154-166.

Kistler JM, Ilyas AM, Thoder JJ. Forearm Compartment Syndrome: Evaluation and Management. Hand Clin. 2018;34:53-60.

Leversedge FJ. Compartment Syndromes. In: Hammert W, Calfee RP, Bozentka DJ, et al, eds. ASSH Manual of Hand Surgery. Philadelphia, PA: Lippincott Williams & Wilkins; 2010;27:483-492.

Stevanovic MV, Sharpe F. Compartment Syndrome and Volkmann Ischemic Contracture. In: Wolfe SW, Hotchkiss RN, Pedersen WC, et al. Green’s Operative Hand Surgery, 7th ed. Philadelphia, PA: Elsevier Churchill Livingstone; 2016;51:1763-1787.

How well did you know this?
1
Not at all
2
3
4
5
Perfectly
43
Q

Which of the following cell types is most associated with the chronic inflammation that leads to breast implant–associated anaplastic large cell lymphoma?

A) B-cells
B) Monocytes
C) Neutrophils
D) Red blood cells
E) T-cells

A

The correct response is Option E.

Evidence suggests that chronic inflammation is the stimulus responsible for the development of breast implant–associated anaplastic large cell lymphoma (ALCL) and T-cells are the predominant cell type responding to this antigenic stimulus. B-cells have been implicated in orthopedic implant lymphomas. The other cell types are involved in inflammation, but they are not associated with breast implant-associated ALCL.

How well did you know this?
1
Not at all
2
3
4
5
Perfectly
44
Q

An 11-year-old boy is brought to the office with an acute injury of the left small finger. A lateral x-ray study is shown. Which of the following is the most appropriate description of this patient’s injury?

A) Displaced Salter Harris fracture of the middle phalanx
B) Displaced Salter Harris fracture of the proximal phalanx
C) Nondisplaced Salter Harris fracture of the distal phalanx
D) Nondisplaced Salter Harris fracture of the middle phalanx
E) Nondisplaced Salter Harris fracture of the proximal phalanx

A

The correct response is Option A.

This is a displaced growth plate fracture of the small finger middle phalanx. There is a 90% displacement of the metaphysis relative to epiphysis. Although there is no obvious involvement of the metaphysis and, thus, the injury could be interpreted as a Salter Harris I fracture, minor concurrent involvement of some portion of the metaphysis (making it technically a Salter Harris II fracture) cannot be excluded and is quite common. The proximal and distal phalanges of the small finger are not injured.

The mnemonic “SALTER” may be used to recall Salter–Harris fracture types:

Type I: S (Slipped). The fracture occurs through the cartilage of the growth plate (physis) with an incidence of 5-7%.

Type II: A (Above). The fracture occurs above the physis, through the metaphysis. This is the most common type, occurring in 75% of patients.

Type III: L (Lower). The fracture occurs below the physis into the epiphysis. This occurs in 7-10%.

Type IV: TE (Through Everything). The fracture occurs through everything which includes the metaphysis, physis, and epiphysis. This type occurs in 10%.

Type V: R (Rammed or crushed). The growth plate (physis) has been crushed. This occurs in <1%.

How well did you know this?
1
Not at all
2
3
4
5
Perfectly
45
Q

A 7-year-old boy presents with a chief complaint of multiple wide and thin scars from skin lesion excisions. The patient’s mother reports a history of late walking, hypermobile joints, and easy bruising. On the basis of these complaints, which of the following is the most likely diagnosis?

A) Capillary fragility syndrome
B) Cutis laxa
C) Ehlers-Danlos syndrome
D) Marfan syndrome
E) Osteogenesis imperfecta

A
How well did you know this?
1
Not at all
2
3
4
5
Perfectly
46
Q

Following a skin-sparing mastectomy, a 39-year-old woman undergoes deep inferior epigastric perforator (DIEP) flap breast reconstruction. To augment flap sensation, the anterior sensory branch of the fourth intercostal nerve is coapted to which of the following nerves within the DIEP flap?

A) Genitofemoral
B) Iliohypogastric
C) Ilioinguinal
D) Intercostal
E) Lateral femoral cutaneous

A

The correct response is Option D.

The third, fourth, and fifth intercostal nerves are responsible for innervation of the majority of the breast. The anterior branch of the fourth intercostal nerve provides most erogenous sensation to the nipple. Sensation to the lower abdomen arises from segmental cutaneous branches of the intercostal nerve, which travel through the rectus abdominis muscle. T10 provides sensation to the dermatome, including the periumbilical region, and is most commonly used. The iliohypogastric nerve provides sensation to the lateral gluteal region. The ilioinguinal nerve provides sensation to the upper medial thigh. The genitofemoral nerve provides sensation to the upper anterior thigh and mons pubis. The lateral femoral cutaneous nerve provides innervation to the lateral thigh and is not used for this purpose.

How well did you know this?
1
Not at all
2
3
4
5
Perfectly
47
Q

A 34-year-old woman with a history of grade I breast ptosis who is 6 years status post-augmentation mammaplasty with subglandular gel-filled implants returns to the clinic. Physical examination shows normal-appearing breasts, but there is mild firmness on palpation. Which of the following Baker grades best describes these findings?

A) Grade I
B) Grade II
C) Grade III
D) Grade IV

A

The correct response is Option B.

Many classification systems have been used to evaluate the severity of breast capsular contracture, which occurs when the peri-implant capsule undergoes fibrotic change. The most widely employed assessment tool remains the Baker grading system, which takes into account patient signs and symptoms. According to the Baker classification, only the highest degrees of contractures (grades III and IV) require surgical treatment. The descriptors for each grade are listed here:

Grade I: the breast is soft and appears normal in size and shape
Grade II: the breast is a little firm and appears normal
Grade III: the breast is firm and appears abnormal
Grade IV: the breast is firm, appears abnormal, and is painful

Studies note decreased relative risk for Baker grade III to IV capsular contracture in patients who undergo primary breast augmentation through an inframammary fold incision, subpectoral pocket placement, and textured implants. There is an increased relative risk for capsular contracture when patients undergo a periareolar or axillary incision and subglandular placement of smooth implants.

How well did you know this?
1
Not at all
2
3
4
5
Perfectly
48
Q

A 35-year-old BRCA-positive woman undergoes bilateral prophylactic nipple-sparing mastectomy via lateral radial incisions with periareolar extensions. She undergoes direct-to-implant reconstruction with an acellular dermal matrix inferior lateral sling. Mastectomy specimens weigh 435 g each, and placement of 450 mL smooth, round, high-profile implants is performed. Postoperatively, 25% partial nipple areolar complex necrosis in both breasts is noted. Which of the following most likely contributed to the necrosis?

A) Bilateral surgery
B) Mastectomy incision
C) Use of acellular dermal matrix
D) Volume of the implants
E) Weight of the mastectomy specimen

A

The correct response is Option B.

Nipple-sparing mastectomies are becoming more prevalent with the rise of prophylactic mastectomies and increased comfort with performing it in presence of in situ and invasive carcinoma. While universal standards for patient selection do not exist, there is consensus that general recommendations include no inflammatory breast cancers, no pathologic nipple discharge, no Paget’s disease, and for many, a 2-cm distance from the tumor to the nipple. Nipple-sparing mastectomy incisions have been described as radial lateral, lateral inframammary fold, and peri-areolar. Because the entire skin envelope is preserved, direct-to-implant reconstruction is possible with inferolateral support from acellular dermal matrices or other scaffolds. Areolar involvement in mastectomy incisions is associated with increased rates of nipple necrosis. The weight of mastectomy specimen, volume of implants, use of acellular dermal matrices or laterality of surgery have not been shown to be associated with nipple necrosis.

How well did you know this?
1
Not at all
2
3
4
5
Perfectly
49
Q

The postoperative CT scan shown is obtained to evaluate a wound dehiscence in a patient who underwent left-sided unilateral reduction mammaplasty for asymmetry six weeks ago. Which of the following upper extremity deformities is most likely to be found in this patient?

A) Contralateral radial club hand
B) Contralateral “pouce flottant” thumb
C) Ipsilateral brachysyndactyly
D) Ipsilateral radial head subluxation
E) Ipsilateral type IV Wassel preaxial polydactyly

A

The correct response is Option C.

Assessment of the chest CT shows right-sided absence of the pectoralis major and minor muscles and breast hypoplasia. The patient suffers from Poland syndrome, which is a congenital disorder of unknown etiology with the prevailing theory being hypoplasia of the subclavian artery or its branches during the sixth week of embryogenesis. Variability exists in physical findings with the most common being: anterior axillary fold and pectoralis major sternal head absence, breast gland thinning, rib and cartilage hypoplasia, and ipsilateral brachysyndactyly. After local wound care and antibiotic therapy, the patient had resolution of her symptoms.

A type IV Wassel preaxial polydactyly is the most common congenital thumb duplication but is not associated with Poland syndrome. Radial club hand is more common than ulnar club hand, but has no association with Poland syndrome. Both are congenital hand deformities, but unrelated to the pathological condition mentioned. Radial head subluxation is also known as “nursemaid’s elbow.” Nursemaid’s elbow is a common injury of early childhood. It is sometimes referred to as “pulled elbow” because it occurs when a child’s elbow is pulled and partially dislocates. There is no connection between Poland syndrome and increased incidence of radial head subluxation. Type IV Manske modification of the Blauth classification thumbs (the “pouce flottant” thumb) have rudimentary elements and are attached to the hand by a small skin bridge. These thumb anomalies are not associated with Poland syndrome.

How well did you know this?
1
Not at all
2
3
4
5
Perfectly
50
Q

A 51-year-old woman is scheduled to undergo needle aponeurotomy for Dupuytren disease of the small finger. A photograph is shown. The addition of lipografting after needle aponeurotomy is most likely to decrease the rate and severity of recurrence in this patient by which of the following mechanisms?

A) Decreasing the proximity of residual cord tissue to the skin
B) Increasing the density of myofibroblast cell-to-cell contact
C) Increasing the density of the residual cord tissue
D) Inhibiting myofibroblast proliferation
E) Providing stem cells to promote collagen production

A

The correct response is Option D.

Fat grafting (also called lipofilling) has shown promise as a means to improve outcomes after percutaneous needle aponeurotomy for Dupuytren disease. It is believed to work by several mechanisms:

Reducing the density of cell-to-cell myofibroblast contact

Inhibiting myofibroblast proliferation via adipose-derived stem cells

Acting as an interposed tissue graft

Providing passing over the cords to replace native subdermal fat displaced by the nodules and cords

A randomized prospective trial by Kan and colleagues showed that aponeurotomy with lipofilling showed equivalent results at one year out from treatment with a much faster recovery compared with limited fasciectomy.

References

Hovius SER, Kan HJ, Smit X, et al. Extensive percutaneous aponeurotomy and lipografting: a new treatment for Dupuytren’s disease. Plast Reconstr Surg. 2011.128:221-8.

Kan HJ, Selles RW, van Nieuwenhoven CA, et al. Percutaneous aponeurotomy and lipofilling (PALF) versus limited fasciectomy in patients with primary Dupuytren’s contracture: a prospective, randomized, controlled trial. Plast Reconstr Surg. 2016. 137:1800-12.

Murphy A, Lalonde DH, Eaton C, et al. Minimally invasive options in Dupuytren’s contracture: aponeurotomy, enzymes, stretching, and fat grafting. Plast Reconstr Surg. 2014. 134:822e-829e.

Verhoekz JSN, Mudera V, Walbeehm ET, Hovius SER. Adipose-derived stem cells inhibit the contractile myofibroblast in Dupuytren’s disease. Plast Reconstr Surg. 2013.132:1139-48.

How well did you know this?
1
Not at all
2
3
4
5
Perfectly
51
Q

A healthy 45-year-old woman with a history of breast malignancy underwent bilateral mastectomy and reconstruction with tissue expanders followed by exchange for cohesive silicone gel implants eight years ago with routine postoperative MRI surveillance. She comes to the office to report pain and tightness in the right breast that has gradually increased over the past month. On examination, temperature is 36.8°C (98.2°F), blood pressure is 112/76 mmHg, and heart rate is 68 bpm. The right breast appears fuller than the left breast; otherwise, the right implant is in a symmetric position with the left side. The skin is otherwise normal in appearance, and there is no tenderness on palpation. Which of the following is the most appropriate next step in management?

A) MRI of the right breast to assess the integrity of the implant
B) One week of an oral antibiotic and prednisone taper
C) Operative exploration, culture, and replacement of implant
D) Referral of the patient back to her medical and surgical oncologists
E) Ultrasound of the right breast and fine-needle aspiration of any fluid

A

The correct response is Option E.

Patients that present with a late seroma should be evaluated for possible Breast implant associated Anaplastic Large Cell Lymphoma (BI-ALCL). A late seroma is usually accepted as occurring 1 year following surgery: however there are cases of BI-ALCL seroma that have presented as early as 4 months.

The first step in evaluation for BI-ALCL is an ultrasound followed by fine needle aspiration is indicated. The fluid requires evaluation beyond routine cell cytology. Immunohistochemistry test for CD30 was the most commonly positive marker for BI-ALCL. Immunohistochemistry stains specific antigens in cells by binding to this antigen in an antibody/antigen reaction. The specific stain can then be seen under light microscopy. CD30 antibody labels anaplastic large cell lymphoma cells. CD30 is a transmembrane cytokine receptor belonging to the tumor necrosis factor receptor family and characteristically stains ALCL cells.

MRI for implant integrity and referral to her Oncologist may be needed but it is not the most appropriate next step. BIA-ALCL needs to be ruled out. Immediate operative exploration is not indicated before fluid aspiration and immunohistochemistry evaluation. Antibiotics and prednisone is not indicated in this patient without evidence of infection or inflammation (red breast syndrome).

How well did you know this?
1
Not at all
2
3
4
5
Perfectly
52
Q

A 23-year-old woman comes to the office because of a hypertrophic scar after undergoing abdominoplasty 3 months ago. A multimodal approach to improving the appearance of the scar is planned. Which of the following therapies is supported by the highest quality evidence in this patient?

A) Allium cepa extract
B) Fat injection
C) Microneedling
D) Silicone gel
E) Vitamin E

A

The correct response is Option D.

Silicone gel has demonstrated efficacy in improving hypertrophic scars in a number of studies and is supported by level I evidence. Vitamin E, fat injection, allium cepa extract and microneedling are supported by lesser quality studies in a recent comprehensive review of the literature.

How well did you know this?
1
Not at all
2
3
4
5
Perfectly
53
Q

A 23-year-old African-American man presents with a raised thickened scar on his anterior chest that he complains is pruritic and unattractive. It was removed by another provider 4 years earlier and has slowly recurred over the past year. On examination, the lesion extends beyond the initial borders of the scar and is firm and hyper-pigmented. On review of his prior pathology report, which of the following histologic characteristics is most likely?

A) Greater ratio of type III to type I collagen
B) Multitude of myofibroblasts and smooth muscle actin
C) Parallel collagen bundles
D) Thick, wavy, and randomly oriented collagen fibers

A

The correct response is Option D.

In patients with abnormal or excessive scar tissue formation, treatment and prognosis will be driven by the correct diagnosis of a keloid versus a hypertrophic scar. This patient presents with a recurrent keloid of the chest. His clinical history supports this diagnosis by recurrence after resection, growth extending beyond the original border of the lesion, late recurrence after several years, and continued growth over several years without regression or improvement. Hypertrophic scars are less likely to recur, contained within the original boundaries of the lesion, often regress somewhat within a year, and recur earlier in the postoperative period if they are to recur. Both hypertrophic scars and keloid scars can be pruritic.

Pathologic analysis of keloids reveals more type I collagen than type III collagen, similar to normal skin. Hypertrophic scars will exhibit increased type III collagen and pro-fibrotic collagen cross-linking. Keloid growth is thought to be impacted by cell-signaling between keratinocytes and fibroblasts, but hypertrophic scar production requires an abundance of myofibroblasts expressing smooth muscle actin. While hypertrophic scars have parallel collagen fibrils and bundles, keloids are characterized histologically by thick, randomly oriented collagen fibrils that are not organized into bundles.

How well did you know this?
1
Not at all
2
3
4
5
Perfectly
54
Q

Which of the following is most commonly associated with decreased incidence of capsular contracture?

A) Formation of biofilm
B) Placement of textured silicone device
C) Subglandular placement of the implant
D) Use of a periareolar incision
E) Use of a postoperative surgical brassiere

A

The correct response is Option B.

The rest of the options have been shown to increase the incidence of capsular contracture. Textured silicone implants, inframammary incisions, and submuscular implant placement have been shown to decrease the incidence of capsular contracture. The use of a surgical brassiere postoperatively has not been shown to decrease incidence of capsular contracture as well.

How well did you know this?
1
Not at all
2
3
4
5
Perfectly
55
Q

Which of the following structures contributes to the formation of the tragus?

A) First branchial arch
B) First branchial cleft
C) Second branchial arch
D) Second branchial cleft

A

The correct response is Option A.

The first branchial arch contributes to the formation of the tragus and anterior helix.

The first branchial cleft is incorrect. It gives rise to the external auditory canal.

The second branchial arch is incorrect. It contributes to the formation of the majority of the external ear – the antitragus, remainder of the helix, antihelix, and crura all arise from the second branchial arch.

The second branchial cleft is incorrect. It is typically obliterated during development, but may persist in the form of a second branchial cleft cyst.

References

Adams A, Mankad K, Offiah C, Childs L. Branchial Cleft Anomalies: A Pictorial Review of Embryological Development and Spectrum of Imaging Findings. Insights Imaging. 2016 Feb; 7(1): 69-76.

Schmidt R, Conrad D, Field E, O’Reilly R. Management of First Branchial Arch Anomalies via a Cartilage-Splitting Technique. Otolaryngol Head and Neck Surg. 2015 Jun;152(6):1149-51.

How well did you know this?
1
Not at all
2
3
4
5
Perfectly
56
Q

A 35-year-old woman comes to the office to discuss a recent diagnosis of breast cancer. Recent mammography showed diffuse microcalcifications throughout the breast, and needle biopsy showed infiltrating ductal carcinoma. On physical examination, some retraction of the skin in the lower outer quadrant of the breast is noted. She wears a size 36C brassiere. The patient reports that she is currently considering whether to have lumpectomy and radiation therapy or mastectomy. Which of the following features of this clinical scenario is a CONTRAINDICATION to breast conservation therapy?

A) Breast size
B) Mammographic findings
C) Patient age
D) Skin retraction on physical examination of the breast
E) Tumor pathology

A

The correct response is Option B.

Breast conservation therapy (BCT) refers to breast conserving therapy followed by moderate-dose radiation to eradicate microscopic residual disease. The goal is to provide the equivalent survival of mastectomy while maintaining a cosmetically acceptable appearance with a low rate of recurrence.

When considering breast conservation therapy or mastectomy, the needs and desires of each patient should be addressed. Age alone is not a contraindication to BCT, but overall health and comorbidities should be considered. Histologic subtype and pathology are not contraindications to BCT as long as the tumor is not diffuse and can be safely excised with negative margins. Similarly, breast size needs to be considered along with tumor size and location, but “small” or “large” breasts are not indications or contraindications. While retraction of the skin, nipple, or breast parenchyma is not an absolute contraindication to BCT, as long as negative margins can be safely removed, the cosmetic impact of their involvement should be considered.

There are few absolute contraindications to BCT, but they include:

Multicentric disease with two or more tumors in separate quadrants of the breast such that they cannot be encompassed in a single excision

Diffuse malignant microcalcifications on mammography

A history of prior radiation in the same breast or chest wall

Pregnancy

Persistently positive margins despite re-excision

How well did you know this?
1
Not at all
2
3
4
5
Perfectly
57
Q

Which of the following is the most common complication associated with “donut” mastopexy?

A) Boxy breast shape
B) Increased distance from nipple to inframammary fold
C) Loss of nipple sensation
D) Nipple necrosis
E) Widening of the areola

A

The correct response is Option E.

A common complication of the “donut” (circumareolar) mastopexy is widening of the areola. This can be minimized by using a Gore-Tex suture placed using the “wagon-wheel” technique and limiting the amount of skin resected to a 2:1 ratio of outside diameter to areolar diameter.

Boxy breast shape is associated with Wise pattern mastopexy. Nipple necrosis is associated with combined augmentation and mastopexy. Increased distance from the nipple to the inframammary fold is associated with vertical mastopexies in which the height of the medial and lateral pillars is too tall. Loss of nipple sensitivity is unusual because there is no parenchymal resection.

How well did you know this?
1
Not at all
2
3
4
5
Perfectly
58
Q

A 32-year-old woman comes to the office for consultation regarding augmentation mammaplasty. She is concerned about the potential complications with the use of silicone gel prostheses within the first 5 years postoperatively. Which of the following is the most commonly reported complication of the implantation of cohesive silicone gel breast prostheses?

A)Capsular contracture
B)Granuloma
C)Hematoma
D)Infection
E)Rupture

A

The correct response is Option A.

Cohesive silicone gel is a breast prosthesis option that has been approved by the FDA since 2006. Cohesive gel prostheses have also been called ?gummy bear? prostheses. They maintain their shape because of the increased cross-linking within the silicone gel.

A study by Cunningham followed 1008 patients and 1898 cohesive gel prostheses. Rupture rate was 1.1% for aesthetics and 3.8% for reconstructive procedures. Capsular contracture rates (Baker III/IV) were 9.8/13.7%, and infection was 1.6/6.1%, respectively. Thus, capsular contracture was the most common of the listed complications. The reported incidence of hematoma is approximately 2%.

It should be noted that complications occur more commonly in primary reconstruction as compared to primary augmentation. These findings are important in the preoperative counseling of patients.

How well did you know this?
1
Not at all
2
3
4
5
Perfectly
59
Q

A 2-year-old boy is evaluated because of a soft, nontender, noncompressible glabellar mass that has progressively grown since birth. A photograph is shown. Which of the following is the most appropriate next step in management before scheduling surgery?

A) Corticosteroid injections
B) MRI
C) Plain film x-ray study
D) Propranolol trial
E) Observation

A

The correct response is Option B.

The differential diagnosis for a lesion in this location with the findings described include dermoid cyst, hemangioma, and encephalocele. Osteoma, which is a benign bony tumor, is unlikely because of patient age and examination findings. The noncompressible quality of the lesion makes hemangioma and encephalocele less likely. Propranolol therapy after 12 months of age is unlikely to help, even if the lesion is a hemangioma. If the lesion is a hemangioma, then observation would be appropriate, but because the lesion is still growing, this diagnosis is questionable. Corticosteroid injections are only moderately helpful in treating a hemangioma, but they are contraindicated for dermoid cysts and encephalocele. Diagnosis is the next step with a goal of ruling out intracranial communication, as it will impact the surgical approach. MRI is the best option. Plain x-rays films would not provide adequate information for management.

References

Al-Muhaylib A, Alkatan HM, Al-Faky YH, Alsuhaibani AH. Periorbital lesions misdiagnosed as dermoid cysts. J AAPOS. 2017;21(6):509-511.

Rezaei E, Shams Hojjati Y. Misdiagnosed extranasal mass: report of a 2-year old child with maltreated rare nasal neuroglial heterotopia. World J Plast Surg. 2019;8(1):122-124.

Van Wyhe RD, Chamata ES, Hollier LH. Midline craniofacial masses in children. Semin Plast Surg. 2016;30(4):176-180.

Vincent J, Baker P, Grischkan J, Fernandez Faith E. Subcutaneous midline nasal mass in an infant due to an intramuscular lipoma. Pediatr Dermatol. 2017;34(3):e135-e136.

How well did you know this?
1
Not at all
2
3
4
5
Perfectly
60
Q

An active 73-year-old woman comes to the office because of Eaton Stage IV arthritis of the carpometacarpal joint of the dominant thumb (pantrapezial arthritis with carpometacarpal [CMC] joint subluxation). She says she has severe pain when she tries to grip something, such as open a door or twist off the top of a jar. Which of the following is the most predictable procedure to decrease pain and improve hand function in this patient?

A) CMC fusion
B) Metacarpal osteotomy
C) Trapezial hemi-resection and tendon interposition
D) Trapezial resection and silicone implantation
E) Trapezial resection, ligament reconstruction, and tendon interposition

A

The correct response is Option E.

Thumb basilar joint arthritis is a common debilitating problem. The prevalence in postmenopausal women has been estimated at 33%, although many patients with radiographic evidence of arthritis remain asymptomatic. It more often occurs in the dominant hand. The extent of arthritis and joint deformity dictates the best treatment choice. The most widely used classification is that of Eaton and is based on radiographic findings. Stage I has normal joint contours but possible joint widening due to effusion. Although most patients respond to splinting, anti-inflammatory medications, trapezial hemi-resection, and metacarpal osteotomy have been advocated in very symptomatic patients.

Stage II shows slight trapeziometacarpal (TM) joint narrowing and minimal sclerosis of the articular surface. The indications for operative treatment are more concrete, and surgical options are largely the same as Stage I, with the addition of CMC fusion as an option in a laborer.

Stage III presents as TM joint narrowing with cystic or sclerotic changes in the articular surface. There is variable dorsal subluxation of the TM joint, and adduction contracture may occur. There can be early signs of scaphotrapezial (ST) joint arthritis. If the ST joint is in relatively good condition, some authors still advocate trapezial-sparing procedures such as hemi-resection. Nevertheless, most advocate trapeziectomy with or without ligament reconstruction/tendon interposition (LRTI). There is some evidence that ligament reconstruction preserves the joint space better than no reconstruction, but provides no better clinical outcome and has a higher complication rate. Trapeziectomy ± LRTI provides excellent pain relief and improved function, especially in lower demand patients.

In Stage IV, the TM and ST joints are completely destroyed. In these patients, LRTI is the preferred treatment. Some authors report good early results in selected patients with implant arthroplasty; however, there is a moderately high rate (up to 40%) of instability, dislocation, and implant breakage. The use of silicone as a spacer has fallen into disuse due to the risk of chronic tissue inflammation and resultant bone resorption.

How well did you know this?
1
Not at all
2
3
4
5
Perfectly
61
Q

A 21-year-old woman comes to the office for consultation regarding a palpable lump in the left breast that she first noticed 4 months ago. Phyllodes tumor of the breast is diagnosed. Which of the following is the most appropriate management at this time?

A ) Observation and incisional biopsy

B ) Neoadjuvant chemotherapy

C ) Local excision and annual surveillance

D ) Wide local excision and radiation therapy

E ) Mastectomy with immediate reconstruction

A

The correct response is Option C.

Phyllodes tumors of the breast were once called cystosarcoma phyllodes because of the fleshy nature of the tumor. Because most tumors are benign, the name may be misleading. Thus, the favored terminology is now phyllodes tumor.

Phyllodes tumor is the most commonly occurring nonepithelial neoplasm of the breast but represents only about 1% of tumors in the breast. It has a sharply demarcated, smooth texture and is typically freely movable. It is a relatively large tumor; the average size is 5 cm. However, lesions greater than 30 cm have been reported. Approximately 90% of phyllodes tumors are benign and approximately 10% are malignant.

In considering excision, the tumor-to-breast ratio should be small enough to allow for segmental excision of the tumor and possible reduction mammaplasty technique (eg, inferior pedicle for a superior tumor or a superior pedicle for an inferior lesion). Annual surveillance and follow-up is recommended because the pathologic appearance does not always predict the clinical behavior.

How well did you know this?
1
Not at all
2
3
4
5
Perfectly
62
Q

A 36-year-old woman comes to the office for consultation regarding augmentation mammaplasty. She wears a size 34B brassiere and wants the size increased to a full C cup. Height is 5 ft 6 in (168 cm) and weight is 126 lb (57 kg). She feels her breasts are reasonable in appearance but has been encouraged by her husband, from whom she is separated, to seek enhancement. The risks of the surgery, including loss of nipple-areola sensation and the need for prosthesis maintenance over time, are discussed. She opts to proceed with surgery, and 375-mL saline breast prostheses are placed subpectorally through inframammary fold incisions. Which of the following is most likely to cause patient dissatisfaction after the procedure?

A ) Continued separation from her husband

B ) Deflation of the breast prostheses

C ) Hypertrophy of the breast scars

D ) Inability to breast-feed

E ) Inadequate breast size

A

The correct response is Option A.

Thorough patient evaluation before surgery, including screening, discussion of risks and complications, and the need for realistic expectations, is necessary to optimize patient satisfaction after surgery. This is especially true of aesthetic surgery.

Despite these efforts, patient dissatisfaction occurs and can be extremely difficult to manage. Patient dissatisfaction is usually associated with failures in communication and patient selection criteria. Determining which patients are unsuitable for operation is a skill acquired with experience. General guidelines include patients who (1) have unrealistic expectations, (2) are excessively demanding, (3) have dissatisfaction with a previous surgical procedure, (4) are psychologically unstable, and (5) have a minimal deformity.

In the scenario described, the patient has an unrealistic expectation that the surgery might save her marriage. Because of her motivation for surgery, she is unlikely to be happy, despite a very good result, unless the expectation of reconciliation has been fulfilled.

The other options are possible causes of postoperative dissatisfaction; however, preoperative counseling and education of the potential complications allow for enhanced acceptance if they do occur.

How well did you know this?
1
Not at all
2
3
4
5
Perfectly
63
Q

A 59-year-old woman comes for evaluation because of a 7-month history of pain over the radial aspect of the right wrist that is aggravated with forceful gripping. She denies any history of trauma to the hand or wrist. On physical examination, there is tenderness to palpation over the right anatomic snuffbox and thenar eminence. Axial load and shifting of the basal joint does not result in crepitance or pain. Resisted thumb extension at the metacarpophalangeal joint level is not painful. An x-ray study of the wrist is shown. Which of the following is the most appropriate operative management?

A) Arthrodesis of the scaphotrapeziotrapezoid joint
B) Arthrodesis of the trapeziometacarpal joint
C) Hemi-resection of the distal trapezium and tendon interposition
D) Release of the first dorsal compartment
E) Release of the second dorsal compartment

A

The correct response is Option A.

Scaphotrapeziotrapezoid (STT) arthritis can often be misdiagnosed on initial presentation as basal joint arthritis. The keys to differentiating the two sites of pain include physical examination, which shows tenderness more proximal than the basal joint and absence of pain with a grind maneuver, coupled with imaging showing osteoarthritic degeneration at the STT rather than the trapeziometacarpal level. Treatment for the arthritic pain can consist of resection arthroplasty or arthrodesis. Both techniques can provide good relief of symptoms. Resection arthroplasty often is used when the scapho-trapezoid articulation is relatively preserved. Regardless of technique chosen, the surgeon should address both the scapho-trapezial and the scapho-trapezoid joints during the procedure.

Release of the first dorsal compartment would address de Quervain tenosynovitis, which could present with pain over the anatomic snuffbox. On examination, however, the patient would typically demonstrate pain with the “resisted Hitchhiker” maneuver (resisted extensor pollicis brevis function at the metacarpophalangeal level). Given the negative findings on examination and the STT arthritis noted on imaging, this patient would not likely respond to treatment directed at the first dorsal compartment.

Second dorsal compartment tenosynovitis can present with distal forearm and wrist pain. The location of the pain is typically more proximal in the forearm and localized to the intersection between the muscle bellies of the first compartment tendons and the radial wrist extensors. This patient’s pain is localized to the STT region rather than the second dorsal compartment.

Hemiresection of the distal trapezium and tendon interposition has been used successfully in the management of trapezio-metacarpal arthritis (basal joint arthritis). In this patient, the location of the pain, absence of pain with a “grinding” type maneuver, and the imaging showing preservation of the basal joint argue against directing treatment at the basal joint itself.

For the same reasons that hemiresection of the distal trapezium is a poor choice for this patient, arthrodesis of the basal joint addresses the wrong site of arthritis. X-ray study and physical examination both indicate STT arthritis as the etiology of the patient’s pain.

How well did you know this?
1
Not at all
2
3
4
5
Perfectly
64
Q

A 27-year-old woman who underwent augmentation mammaplasty with 325-mL textured prostheses one year ago comes to the clinic because her breasts look asymmetric and feel hard. Physical examination shows firm asymmetric breasts with palpable capsules. No pain, signs of skin infection, hematoma, or seroma are observed. Hypertrophic scars are seen on the inframammary fold of both breasts. Which of the following factors is the most likely cause of capsular contracture in this patient?

A) Implant size
B) Patient history of hypertrophic scarring
C) Subclinical infection with biofilm formation
D) Submuscular positioning of the implants
E) Textured implants

A

The correct response is Option C.

On the basis of her clinical presentation, this patient is experiencing Baker Grade III capsular contracture. Capsular contracture is the most common complication after breast implant placement. This is a multifactorial complication; however, only subclinical infection with biofilm formation has a clear correlation with a higher degree of capsular contracture.

Implant size is not directly associated with an increased risk for clinically significant capsular contracture, and it has been established that textured implants are associated with a decreased risk. There is no clear evidence of a relation between a patient’s tendency to scar and an increased risk for capsular contracture. It is accepted that submuscular placement leads to lower rates of capsular contracture than the subglandular technique.

How well did you know this?
1
Not at all
2
3
4
5
Perfectly
65
Q

A 50-year-old man comes to the emergency department after sustaining amputation of the right long finger involving an avulsion mechanism. The patient is taken to surgery for replantation. During surgery, extensive vascular injury is seen, and an approximately 2-cm vascular gap of the digital arteries and veins results following excision of injured vessels. Which of the following interventions is most likely to increase the probability of functional digit replantation?

A) Bone shortening
B) Medicinal leech therapy
C) Postoperative warming
D) Systemic heparin
E) Vein grafts

A

The correct response is Option E.

In patients who sustain digital amputation as a result of an avulsion mechanism, there is often an extensive zone of injury that precludes primary vascular anastomosis. Vein grafts permit vascular anastomosis outside of the zone of injury.

Bone shortening can sometimes allow excision of the injured vasculature and primary anastomosis. However, in this case, bone shortening is unlikely to make up for a 2-cm vascular gap.

Longer vascular gaps can be addressed with vein grafts. Despite the fact that vein grafts involve an additional anastomosis per vessel compared to primary anastomosis, they have been found to exhibit similar rates of thrombosis and replantation survival.

Medicinal leech therapy can help address venous congestion, but is typically considered when venous congestion occurs after attempt at surgical replantation, or if no suitable veins can be found for anastomosis.

While postoperative warming and systemic heparin are often used adjunctively in patients undergoing replantation, they have not been demonstrated to increase the likelihood of survival of the replanted part, and would most likely not have as significant an effect as restoring perfusion to the amputated part using vein grafts.

References

Hyza P, Vesely J, Drazan L, et al. Primary vein grafting in treatment of ring avulsion injuries: a 5-year prospective study. Ann Plast Surg. 2007 Aug;59(2):163-7.

Prucz RB, Friedrich JB. Upper extremity replantation: current concepts. Plast Reconstr Surg. 2014 Feb;133(2):333-42.

Yan H, Jackson WD, Songcharoen S, et al. Vein grafting in fingertip replantations. Microsurgery. 2009;29(4):275-81.

How well did you know this?
1
Not at all
2
3
4
5
Perfectly
66
Q

A 43-year-old Caucasian woman is referred to the office because of a mass on her right breast that has been rapidly growing for 8 weeks. Physical examination shows a 4.5-cm, freely movable mass in the right breast. No axillary adenopathy or nipple discharge is noted. Which of the following is the most likely nonepithelial neoplasm in this patient?

A) Fibroadenoma
B) Hamartoma
C) Lipoma
D) Phyllodes tumor
E) Primary breast lymphoma

A

The correct response is Option D.

Primary breast lymphoma is rare; it constitutes less than 0.6% of all breast malignancies. Phyllodes tumors represent about 1% of tumors in the breast; they are the most commonly occurring nonepithelial neoplasm of the breast. The tumor has a smooth, sharply demarcated texture and is generally freely movable. It is a relatively large tumor, with an average size of 5 cm. A hamartoma is the most common benign tumor of the lung. Hamartomas of the breast are benign tumors composed primarily of dense fibrous tissues with variable amounts of fat and associated ducts. Eighty-five percent of phyllodes tumors are benign, and 15% are malignant. There is no race predilection, but phyllodes tumors occur almost exclusively in women. In young women under age 25 years, asymmetric, tender, and fibrocystic tissues usually point to a fibroadenoma or circumscribed fibrocystic mass. Lipoma of the breast causes diagnostic and therapeutic uncertainty. Clinically, it may be difficult to distinguish a lipoma from other conditions. Fine-needle aspiration cytology is often not helpful. Both mammography and ultrasonography results are often negative. MRI may be overread, but it is useful as a diagnostic tool. This neoplasm can be seen in men and women.

How well did you know this?
1
Not at all
2
3
4
5
Perfectly
67
Q

A 47-year-old woman is referred by her primary care physician to evaluate a suspected intracapsular rupture of her prosthesis on the left identified during routine mammography. She underwent primary augmentation mammaplasty with subglandular placement of single-lumen silicone breast prostheses in 1990. Physical examination shows a smaller breast on the left. An MRI is requested. Which of the following findings on MRI is most likely to confirm the diagnosis?

A ) Double wall sign

B ) Linguine sign

C ) Multiple echogenic lines

D ) Reverse double-lumen sign

E ) Snowstorm sign

A

The correct response is Option B.

MRI, mammography, ultrasonography, and CT scanning have all been used to diagnose silicone breast prosthesis rupture.

Although each modality has specific strengths and weaknesses that may make a particular modality the study of choice for an individual patient, MRI of silicone breast prostheses reports the highest sensitivity and specificity for detection of silicone prosthesis rupture.

Of the options listed, only the linguine sign is consistent with intracapsular silicone prosthesis rupture and represents the prosthesis shell floating in free silicone gel.

The double wall sign, also known as Rigler sign, is a radiographic sign of pneumoperitoneum.

Snowstorm sign and echogenic lines may be seen on ultrasound examination.

Water suppression or a reverse double-lumen sign would not be expected findings in a single-lumen device but may have a role in double-lumen devices.

How well did you know this?
1
Not at all
2
3
4
5
Perfectly
68
Q

A 36-year-old man comes to the office because of a 2-week history of pain of the right wrist after a fall on his outstretched hand. X-ray studies are shown. If this injury is left untreated, which of the following joint surfaces is most likely to develop arthritis first?

A) Capitolunate
B) Lunotriquetral
C) Radiolunate
D) Radioscaphoid
E) Scaphocapitate

A

The correct response is Option D.

The most likely joint surface to develop arthritis is the radioscaphoid joint. This patient shows evidence of scapholunate ligament tear. There is evidence of widening of the scapholunate interval and increase in the scapholunate angle.

The scapholunate angle is calculated by measuring the angle between a line drawn perpendicular to the distal surface of the lunate and along the axis of the scaphoid on the lateral view. The normal scapholunate angle varies from 30 to 60 degrees. A tear in the scapholunate ligament results in volar flexion of the scaphoid bone and dorsiflexion of the lunate, with a resultant increase in the angle.

If a scapholunate ligament tear is left untreated, a degenerative pattern of changes result. This is known as scapholunate advanced collapse (SLAC) wrist. Over time there is separation of the scaphoid and lunate bones and descent of the capitate into the intervening space.

With scapholunate ligament tears, arthritis occurs in a predictable sequence. This initially begins in the radioscaphoid joint, followed by the scaphocapitate joint and the capitolunate joint. The radiolunate joint is typically spared until advanced stages. The lunotriquetral ligament is intact and arthritis does not occur in this area with SLAC wrist.

How well did you know this?
1
Not at all
2
3
4
5
Perfectly
69
Q

A 62-year-old woman with a history of Stage III breast cancer is scheduled for delayed autologous breast reconstruction from the abdominal donor site. She has no other medical problems. BMI is 30 kg/m2. Her mother had a lower extremity deep venous thrombosis in the past. Caprini risk assessment score is 9. Which of the following is the most appropriate method of postoperative VTE risk reduction?

A) Aspirin therapy
B) Early ambulation after surgery
C) Low-molecular-weight heparin therapy
D) Sequential compression device use
E) No VTE prevention is indicated

A

The correct response is Option C.

Venous thromboembolism (VTE) is a disorder with short-term mortality and long-term morbidity. Plastic and reconstructive surgery patients are known to be at high risk for VTE after surgery. Symptomatic VTE occurs with high frequency after post-bariatric body contouring (7.7%), abdominoplasty (5%), and breast or upper body contouring (2.9%). To fully identify VTE risk in surgical patients, individualized patient assessment is advocated. The Caprini risk assessment model (RAM) is a useful and effective tool to stratify surgical patients for VTE risk. For patients with high Caprini scores, a significantly greater likelihood of VTE events is observed. Approximately 11% of patients with Caprini score >8 will have a VTE within 60 days after surgery.

Based upon recommendations from the ASPS VTE Task Force, patients undergoing elective plastic and reconstructive surgical procedures who have Caprini RAM score of 7 or more should have VTE risk reduction strategies employed, such as limiting operating room times, weight reduction, discontinuation of hormone replacement therapy, and early postoperative mobilization. Patients undergoing major plastic and reconstructive operative procedures performed during general anesthesia that last longer than 60 minutes should receive VTE prevention. For patients with Caprini score of 3 to 6, the use of postoperative low-molecular-weight heparin (LMWH) or unfractionated heparin (UH) should be considered. For patients with Caprini score of 3 or more, use of mechanical prophylaxis throughout the duration of chemical prophylaxis for non-ambulatory patients should be considered. For patients with Caprini score of 7 or more, the use of extended LMWH postoperative prophylaxis should be strongly considered.

Aspirin does not decrease the risk of VTE and may increase the risk of perioperative complications.

How well did you know this?
1
Not at all
2
3
4
5
Perfectly
70
Q

A 32-year-old man comes to the emergency department after being hit in the right eye. Examination shows enophthalmos, hyphema, and numbness over the cheek. There is no diplopia. CT scan shows a large orbital floor fracture with herniation of contents into the maxillary sinus. Which of the following findings requires urgent management?

A) Cheek numbness
B) Enophthalmos
C) Hyphema
D) Maxillary sinusitis
E) Orbital floor fracture

A

The correct response is Option C.

Hyphema is marked by presence of blood in the anterior chamber and is an emergent concern. It can lead to permanent damage to the vision. All the other options are urgent concerns but can be addressed after the hyphema is treated.

How well did you know this?
1
Not at all
2
3
4
5
Perfectly
71
Q

A newborn female presents with a large intraoral mass arising from alveolar mucosa of the lower jaw that does not cause any airway obstruction. Photographs are shown. Which of the following is the most likely pathology of the lesion?

A) Congenital epulis
B) Hemangiopericytoma
C) Odontogenic keratocyst
D) Rhabdomyosarcoma
E) Teratoma

The correct response is A.

A

Congenital epulis is a rare, benign tumor of the oral cavity that is found in newborns. They are considered a form of granular cell tumor that can lead to mechanical obstruction, resulting in respiratory distress or difficulty eating. Surgical excision is the treatment of choice and recurrence after excision is rare. The female-to-male ratio is 10:1. It is observed three times more frequently on the maxilla than the mandible. They are solitary in most cases, but can be large and multiple.

Teratomas and rhabdomyosarcomas of the mandible are even more rare and are usually seen in the older patient population; they are not seen in the neonatal population.

Hemangiopericytomas are rare, vascular neoplasms that originate from vascular pericytes. They can occur anywhere in the body, including the mandible. They are slow-growing and present in the older patient population. The likelihood of presentation in a neonate is exceedingly low.

Odontogenic keratocysts are rare and benign, but locally aggressive lesions of the posterior mandible. They most commonly present in the third decade of life. They make up 19% of jaw cysts.

References

Goldblum JR, Folpe AL, Weiss SW, eds. Enzinger and Weiss’s Soft Tissue Tumors. 6th ed. Philadelphia, PA: Elsevier Saunders; 2014:845.

Husain AN, Stocker JT, Dehner LP, eds. Stocker and Dehner’s Pediatric Pathology. 4th ed. Philadelphia, PA: Wolters Kluwer; 2016:1027.

How well did you know this?
1
Not at all
2
3
4
5
Perfectly
72
Q

Which of the following innervates the nipple-areola complex?

A ) Intercostal

B ) Lateral pectoral

C ) Long thoracic

D ) Supraclavicular

E ) Thoracodorsal

A

The correct response is Option C.

Two different studies show that prosthesis volume relative to the skin envelope has a potential adverse effect on the sensation of the nipple-areola complex. The type of prosthesis and the pocket placement have no influence on sensitivity. No difference was found between the periareolar and inframammary incision approaches. Other factors, such as a previous history of breast-feeding or minor complications, were not associated with sensory alterations.

How well did you know this?
1
Not at all
2
3
4
5
Perfectly
73
Q

In a patient undergoing reconstructive cranioplasty, an increased rate of complications is most likely if which of the following is present?

A) Frontal location
B) Occipital location
C) Parietal location
D) Sphenoidal location
E) Temporal location

A

The correct response is Option A.

Early decompressive craniectomy is a life-saving maneuver for certain traumatic brain injuries and can be performed far forward in the theater of war. Patients treated with decompressive craniectomy for combat injuries are a unique understudied population. Outcome of treatment of this patient cohort has been previously reported using a standardized cranial defect treatment protocol using custom alloplast implants. Two subgroups of patients (large endocranial dead space and frontal orbital bar injuries) were identified as often having higher rates of complications than other cranial reconstruction cohorts.

How well did you know this?
1
Not at all
2
3
4
5
Perfectly
74
Q

An otherwise healthy 38-year-old woman undergoes prophylactic bilateral mastectomy and immediate reconstruction with deep inferior epigastric artery perforator (DIEP) free flaps. Intraoperatively, the left DIEP flap appears congested before the conclusion of the case. The left deep inferior epigastric artery and vein (DIEA and DIEV) were anastomosed to the proximal internal mammary vessels. The vascular pedicle is evaluated and each anastomosis appears patent and not kinked; however, the venous congestion persists. Which of the following is the most appropriate management?

A) Anastomose the superficial inferior epigastric vein to an internal mammary vessel perforator
B) Convert to left prosthetic reconstruction
C) Infuse tissue plasminogen activator (tPA) to the DIEA
D) Initiate leech therapy
E) Revise the DIEV anastomosis to the retrograde internal mammary vessel limb

A

The correct response is Option A.

Venous drainage of the lower abdominal skin and subcutaneous tissue occurs primarily through the superficial venous system and secondarily through the deep venous system, with perforating veins interconnecting the two systems. These communicating veins have been identified on computed tomography angiography in approximately 90% of abdominal walls in vivo. The majority of the remaining 10% of patients likely have communicating veins that are too small to visualize or are absent. In these cases of anatomical superficial venous system dominance, venous drainage is dependent on the superficial venous system. A recently published 2012 article by Sbitany et al. demonstrated that the incidence of intraoperative venous congestion secondary to persistent superficial venous system dominance was 0.9% in 1201 muscle-sparing transverse rectus abdominis musculocutaneous and deep inferior epigastric artery perforator free flaps. A free flap that becomes congested after reperfusion in the operating room should be assessed immediately for possible etiologies including twisting, kinking, tension, or vasospasm of the vascular pedicle. If a technical problem is ruled out and the venous anastomosis remains patent, obligatory enhancement of venous drainage with the superficial venous system is necessary to salvage the free flap rather than revision of the original anastamosis. Various methods include an anastomosis of the superficial inferior epigastric vein (SIEV) to the DIEV system or any chest wall vein, including the retrograde limb of the internal mammary vessel, the branch of the internal mammary vessel, or the thoracodorsal system. This requires preemptive planning and sparing of the superficial epigastric vein or SIEV during the dissection of the flap. A vein graft can be utilized if additional length is necessary. Another option is to substitute the DIEV anastomosis with the SIEV.

Tissue plasminogen activator (tPA) would not be indicated in this scenario, as it is used as a thrombolytic and there is no evidence of vascular thrombosis. Revising the DIEV anastomosis would be moot because it is patent and the deep system is being drained. Leech therapy is useful for venous congestion, but primarily as an adjunct after potential surgical etiologies have been addressed. Sacrificing the free flap without first attempting salvage is not warranted, and using a prosthetic would be possible only if prior patient consent were obtained

How well did you know this?
1
Not at all
2
3
4
5
Perfectly
75
Q

In embryologic breast development, which of the following best describes the formation of the mammary ridge?

A) Starts at the fifth or sixth week of fetal development, when buds of mesoderm grow into the overlying ectodermal skin layer
B) Starts at the fifth or sixth week of fetal development, when outgrowths from the ectodermal skin layer penetrate into the mesoderm
C) Starts at the seventh or eighth week of fetal development, when buds of mesoderm grow into the overlying ectodermal skin layer
D) Starts at the seventh or eighth week of fetal development, when outgrowths from the ectodermal skin layer penetrate into the mesoderm
E) Starts at the third or fourth week of fetal development, when buds of mesoderm grow into the overlying ectodermal skin layer

A

The correct response is Option B.

Muntan, et al. described breast development as starting at the fifth or sixth week of development, when outgrowths from the ectodermal skin layer penetrate into the underlying mesoderm, forming the mammary ridge or milk line. The ectodermal thickenings along the mammary line regress between gestational months 2 and 4, except for two of them in the region of the third and fourth ribs. The ectoderm keeps on extending into the underlying mesoderm at the fifth month, and a branching network forms what will eventually become the lactiferous system. The supportive connective and adipose tissue of the breast develops from the surrounding mesenchyme.

How well did you know this?
1
Not at all
2
3
4
5
Perfectly
76
Q

A 65-year-old woman with a history of left mastectomy for breast cancer 10 years ago undergoes biopsy of a suspicious lesion of the right breast found on a recent mammogram. Examination of the biopsy specimen confirms a right breast carcinoma. This lesion most likely originated from which of the following structures?

A) Adipose tissue
B) Areolar skin
C) Lactiferous duct
D) Lymph node
E) Pectoralis major muscle

A

The correct response is Option C.

Women who were previously treated for breast cancer are at increased risk for development of a metachronous lesion of the contralateral breast. Cancers of the breast are typically adenocarcinomas, arising from the glandular tissue such as the ducts or lobules. Paget disease of the breast would involve the areolar skin but is fairly uncommon. Sarcomas arising from the connective tissue (such as adipose or muscle) are also rare. Breast adenocarcinomas do not originate from lymphatic tissue.

How well did you know this?
1
Not at all
2
3
4
5
Perfectly
77
Q

Which of the following mechanisms is most likely to inhibit normal wound healing in a patient who smokes cigarettes?

A) DNA strand breaks and helical cross-linking
B) Increased cosubstrate for enzymes involved in collagen production
C) Increased platelet aggregation
D) Increased tissue oxygen delivery
E) Nicotine-induced vasodilation

A

The correct response is Option C.

The detrimental effects of smoking on wound healing are due primarily to nicotine, carbon monoxide, and hydrogen cyanide. One of the effects of nicotine is increased platelet aggregation due to enhanced adhesiveness of the platelets themselves. This leads to thrombus formation and decreased oxygen delivery. Nicotine does not produce vasodilation, but rather vasoconstriction. Both of these effects can lead to local tissue ischemia, which inhibits the normal wound healing process.

One of the major mechanisms by which ionizing radiation inhibits wound healing is production of DNA strand breaks and helical cross-linking, but smoking is not significantly involved.

Vitamin C is the vitamin which plays the greatest role in wound healing. It is required as a cosubstrate for hydroxylase enzymes, which are involved in the production of collagen. Vitamin C deficiency has long been known to inhibit wound healing (scurvy). However, supplemental vitamin C in the nondeficient patient has not been shown conclusively to produce any beneficial wound-healing effects.

How well did you know this?
1
Not at all
2
3
4
5
Perfectly
78
Q

Which of the following comorbidities is associated with the highest risk of digital replant failure?

A) Alcohol abuse
B) Chronic obstructive pulmonary disease
C) Diabetes mellitus
D) Psychotic disorders
E) Tobacco use

A

The correct response is Option D.

In a study looking at all amputation injuries and digital replantations captured by the National Inpatient Sample from 2001 to 2012, the comorbid conditions associated with the highest risk of replant failure were psychotic disorders, peripheral vascular disease, and electrolyte imbalances. The risk of replant failure increased 79% in a patient with a psychotic disorder. Alcohol abuse increased the risk of replant failure by 16%, tobacco use by 7%, diabetes by 3%, and chronic obstructive pulmonary disease by 1%. Interestingly, age in and of itself was not associated with a higher chance of replant failure in this and other studies.

References

Dec W. A meta-analysis of success rates for digit replantation. Tech Hand Up Extrem Surg. 2006 Sep;10(3):124-129.

Hustedt JW, Chung A, Bohl DD, et al. Evaluating the effect of comorbidities on the success, risk, and cost of digital replantation. J Hand Surg Am. 2016 Dec;41(12):1145-1152.

Nazerani S, Motamedi MH, Ebadi MR, Nazerani T, Bidarmaghz B. Experience with distal finger replantation: a 20-year retrospective study from a major trauma center. Tech Hand Up Extrem Surg. 2011 Sep;15(3):144-50.

How well did you know this?
1
Not at all
2
3
4
5
Perfectly
79
Q

An 80-year-old man sustains an extravasation injury to the dorsum of the arm secondary to administration of a dopamine infusion. Which of the following findings is an indication for a surgical intervention in this patient?

A) Blanching of the skin
B) Blistering
C) Erythema
D) Induration
E) Persistent pain

A

The correct response is Option E.

The indications for surgery in an extravasation injury include full-thickness skin necrosis, chronic ulceration, and persistent pain. Whereas blistering indicates a partial-thickness skin loss, it is alone not an indication for surgery. Erythema, induration, and poor capillary refill (blanching) are signs of extravasation injury but are not indications for an operative intervention.

References

Al-Benna S, O’Boyle C, Holley J. Extravasation injuries in Adults. ISRN Dermatol. 2013 May 8;2013:856541.

Scuderi N, Onesti MG. Antitumor agents: Extravasation, management, and surgical treatment. Ann Plast Surg. 1994 Jan;32(1):39-44.

How well did you know this?
1
Not at all
2
3
4
5
Perfectly
80
Q

A 43-year-old woman would like to discuss plans for breast reconstruction after her upcoming unilateral mastectomy. Postoperative radiation therapy is planned. Which of the following is the most likely benefit of tissue expander–based breast reconstruction compared with immediate autologous breast reconstruction using this patient’s abdominal tissue?

A) Better symmetry
B) Improved postoperative sensation
C) A larger, more ptotic breast reconstruction
D) Lower risk of complications from radiation
E) Preservation of the patient’s options for final reconstruction

A

The correct response is Option E.

Immediate breast reconstruction with tissue expanders followed by reconstruction of choice preserves the patient’s skin envelope and keeps open options for definitive final reconstruction of choice whether with an implant or autologous tissue.

Tissue expander-based reconstruction is associated with a higher complication rate in the setting of radiation therapy but preserves abdominal and back tissue as options for autologous reconstruction. Implant-based reconstruction does not provide the advantages of improved symmetry, sensation, or breast ptosis.

How well did you know this?
1
Not at all
2
3
4
5
Perfectly
81
Q

A 41-year-old woman comes to the office because of an invasive ductal carcinoma of the left breast. On mammography, the tumor is 3 cm from the nipple and measures 4 cm. A left-sided lateral periareolar scar extending from the 12 o’clock to the 3 o’clock position from a previous biopsy is noted. The patient wishes to undergo a nipple-sparing mastectomy. Which of the following findings places the patient at greatest oncologic risk, including risk for de novo or recurrent cancer or inadequate surgical margins, with this procedure?

A) Distance of tumor to nipple
B) Patient age
C) Presence of the periareolar scar
D) Size of tumor
E) Type of tumor

A

The correct response is Option D.

As surgical approaches to breast cancer treatment have evolved, nipple-sparing mastectomy (NSM) has emerged as an alternative to other approaches. It was initially used for prophylactic mastectomies, and patients reported increased satisfaction and body image with nipple-areola complex (NAC) preservation. The role of NSM has been expanded to therapeutic mastectomy, and with that there has been increased research in the oncologic safety of this approach.

Studies have evaluated therapeutic NSM in the context of invasive ductal carcinoma, invasive lobular carcinoma, and ductal carcinoma in situ. The type of cancer does not appear to be associated with the oncologic safety of NSM. Several studies have demonstrated an inverse association between NAC involvement and distance of the tumor from the nipple. While these studies have varied in their distance cutoffs, nipple involvement is reported to be over 50% when the tumor-nipple distance is less than 2 cm, as noted in one study. There is a direct correlation between tumor size and NAC involvement—the same study cited data that when the tumor was greater than 4 cm, the likelihood of nipple involvement was greater than 50%.

One published screening algorithm for plastic surgery includes tumor size less than 3 cm, and tumor location greater than 2 cm from the nipple as criteria for NSM candidacy.

A periareolar scar, if large, may compromise the blood supply to the NAC. Acceptable incisions for NSM, however, include a periareolar incision of 25 to 50%.

How well did you know this?
1
Not at all
2
3
4
5
Perfectly
82
Q

A 52-year-old woman undergoes immediate unilateral breast reconstruction with a free deep inferior epigastric flap. Near completion of the procedure, the flap skin paddle is noted to have venous congestion. On exploration of the pedicle, the anastomosis between the vena comitans and the internal mammary vessel appears patent. Which of the following preventive measures would most likely have averted this issue?

A) Administration of heparin immediately before release of vessel clamps and flap revascularization
B) Anastomosis of the veins using sutures instead of a venous coupling device
C) Preservation and anastomosis of the superficial inferior epigastric vein
D) Routine anastomosis of two venae comitantes per flap
E) Use of near-infrared fluorescence imaging to assess flap blood flow

A

The correct response is Option C.

Preservation of the superficial inferior epigastric veins (SIEV) during flap harvest is a useful preventive measure in microsurgical free tissue transfer operations. These veins can serve as important lifeboats to augment venous outflow in the setting of venous congestion. Typically, if a free flap demonstrates venous congestion, the inset should be taken down and the pedicle, recipient vessels, and anastomoses interrogated. Simple issues, such as mechanical compression or twisting of the vein should be ruled out. Next, the SIEV should be inspected. If it is engorged, it is likely that the flap is reliant on superficial outflow, and this vein should be connected to a recipient vessel to augment the venous outflow of the flap. Options for recipient veins include an anterograde branch on the pedicle vena comitans, or in a retrograde fashion to the vena comitans that was not used in the initial set of anastomoses.

Near-infrared fluorescence imaging technology can assist with flap design and may be useful for assessing the arterial inflow of the flap, but it has not been shown to correlate with flap loss or venous complications.

The use of one or two veins in microsurgical free tissue transfer is a topic that has been debated for several years. While some studies indicate that the use of two venous connections may decrease the velocity of blood flow across the anatomosis, there are no data to support differences in flap outcomes or thrombotic events. Therefore, the routine use of a second vein is largely up to surgeon preference.

Heparin may be a useful adjunct when thrombosis of the arterial or venous anastomosis is excised and a revision of the anastomosis is performed. Without evidence of thrombosis, it is unlikely to have any added benefit.

For venous anastomoses, the use of venous coupling devices has not been associated with patency issues or increased thrombosis rates. Therefore, a hand-sewn anastomosis is not likely to prevent the issue presented in this question.

How well did you know this?
1
Not at all
2
3
4
5
Perfectly
83
Q

A 44-year-old previously healthy woman comes to the clinic because of a 2-week history of a painless mass in the left breast. She initially felt this mass while taking a shower. Her mother was diagnosed with fibrocystic changes. The patient denies alcohol consumption and smoking cigarettes. Examination of the left breast shows a 5-cm mobile, painless mass in the left upper external quadrant without nipple discharge, skin retractions, or color changes. Examination of a specimen obtained on biopsy discloses a phyllodes tumor, and surgical excision of the lesion is planned. Which of the following is the most important factor to prevent local recurrence after surgery?

A) Adjuvant radiotherapy
B) Concurrent axillary node dissection
C) Postoperative chemotherapy
D) Surgical margins less than or equal to 0.5 cm
E) Wide surgical margins

A

The correct response is Option E.

In a young woman who has no history of breast cancer, presents with a painless mass, and has a mammogram suggestive of fibroadenoma but a core needle biopsy showing stromal hypercellularity with atypical spindle cells and a high mitotic rate, a phyllodes tumor must be suspected.

Phyllodes tumors are uncommon fibroepithelial breast tumors that behave like benign fibroadenomas, although they have a high propensity to recur locally. More aggressive tumors can metastasize distantly. Surgery is the preferred treatment for this condition. In this context, surgical margins greater than or equal to 1 cm have been associated with a lower recurrence rate in borderline and malignant tumors.

Axillary lymph node involvement is rare. Wide local excision or mastectomy with appropriate margins is the preferred clinical intervention.

Based on limited data, the role of systemic chemotherapy in phyllodes tumors is limited. Patients with benign or borderline phyllodes tumors are usually cured with surgery and should not be offered chemotherapy unless they develop unresectable metastases.

Local recurrence rate is higher after excision with narrower margins than broader ones. The efficacy of postoperative adjuvant radiotherapy for a breast phyllodes tumor is not clear. In clinical practice, the utilization of adjuvant radiotherapy for a phyllodes tumor appears to be modest.

How well did you know this?
1
Not at all
2
3
4
5
Perfectly
84
Q

A 42-year-old woman comes to the office for treatment after receiving a diagnosis of cancer of the right breast. She has decided to undergo mastectomy of the right breast. Which of the following is a relative CONTRAINDICATION to nipple-sparing mastectomy?

A) Comedo-type breast tumor
B) Invasive lobular carcinoma
C) Subareolar tumor
D) Tumor location 3 cm from the nipple
E) Tumor size of 2.5 cm

A

The correct response is Option C.

A relative contraindication to nipple-sparing mastectomy is a centrally located tumor. Although various authors have employed different distance criteria, it is generally accepted that patients whose tumors are within 2 cm of the nipple are not candidates for nipple-sparing mastectomy.

Nipple-sparing mastectomy is an appropriate option for high-risk patients undergoing prophylactic mastectomy and for patients diagnosed with breast cancer who meet certain criteria. Those criteria are: tumor size of 3 cm or less, at least 2 cm from the nipple, not multicentric, and with clinically negative nodes.

Comedo carcinoma of the breast is a type of ductal carcinoma in situ. It is considered to be an early stage of breast cancer, is confined to the ducts, and usually does not spread beyond. It is not a contraindication to nipple-sparing mastectomy.

Invasive lobular carcinoma originates from the breast lobules, may form a thickening of the breast tissue rather than a discrete mass, and is often bilateral. As long as it meets the above criteria, it is not a contraindication to nipple-sparing mastectomy.

Inflammatory breast cancer, Paget disease, and tumors infiltrating the skin are also not candidates for skin-sparing or nipple-sparing mastectomy, according to several authors.

In more recent studies, a tumor size of 3 cm or less appears to result in no increase in local or regional recurrence in nipple-sparing mastectomy compared with alternative surgical approaches. A tumor of 2.5 cm is not a contraindication to nipple-sparing mastectomy.

How well did you know this?
1
Not at all
2
3
4
5
Perfectly
85
Q

An 18-year-old woman comes to the office for evaluation of her breasts. Photographs of the patient are shown. Which of the following statements most accurately describes the anatomy of this patient’s breasts?

A) The areola is normal size although the breast is small
B) The breast tissue is uniformly distributed throughout the breast pocket
C) The inframammary fold is elevated
D) The skin envelope has greater laxity than in a normal breast
E) The underlying musculature is underdeveloped

A

The correct response is Option C.

The tuberous breast deformity results in a protruding, oblong shape that resembles a tuberous root plant (Latin derivation tuber = to swell). The features noted in the tuberous breast deformity include a constricted breast base, decreased breast parenchyma, abnormal elevation of the inframammary fold, a decreased skin envelope, and herniation of the breast parenchyma through the central breast and into the areola. The areola is large and lacks firm underlying structure, thus allowing the breast tissue to protrude through this path of least resistance. The deformity is also often referred to as a tubular breast, constricted breast, doughnut breast, nipple breast, breast with narrow base, dome nipple, and snoopy dog breast.

The overall etiologic factors leading to the full expression of the constricted breast deformity are still largely unknown and likely involve a delicate balance of anatomic and endocrinologic forces. A constricting fibrous ring at the level of the areola periphery, representing probably a thickening of the superficial fascia coupled with the normally absent fascial layer in the NAC, has been proposed as a likely cause. The ring is composed of dense fibrous tissue made of large concentrations of collagen and elastic fibers arranged longitudinally. It is usually denser at the lower part of the breast and does not allow the developing breast parenchyma to expand during puberty. It has been suggested that a thickening of the superficial fascia combined with the absence of a superficial fascial layer under the NAC is the underlying anatomic/histopathologic cause of the deformity. The cause of the thickened fascia is unknown, although at least one study by Klinger, Caviggioli, et al. demonstrated altered collagen in both disposition and quantity. The same study excluded amyloid deposition as a component of the fibrosis.

The areola in the tuberous breast still contains the normal muscular structures that result in areolar changes with stimulation and temperature changes, although the tissue beneath the areola may be thinned.

How well did you know this?
1
Not at all
2
3
4
5
Perfectly
86
Q

A 46-year-old woman who is 5 ft 7 in (170 cm) tall and weighs 135 lbs (61 kg) is evaluated one year following bilateral nipple-sparing mastectomy and immediate reconstruction with placement of 350-mL smooth, round silicone gel implants beneath the pectoralis major muscle. Since the surgery, she has experienced hyperdynamic deformity of her breasts. On physical examination, the breast reconstruction appears natural, and there is significant movement of the breasts when the patient flexes her chest. Which of the following is the most appropriate management for this patient?

A) Inject botulinum toxin into the pectoralis major muscle
B) Inject triamcinolone-40 into the areas of tenderness using ultrasound guidance
C) Move the implants to the prepectoral plane and cover them fully with acellular dermal matrix
D) Perform a breast MRI to assess for rupture of the implants
E) Refer the patient to a physical therapist for range of motion, massage, and ultrasound treatments

A

The correct response is Option C.

This patient is experiencing significant movement because her implants were placed beneath the pectoralis major muscles. While reconstruction options are limited in this otherwise healthy and very thin patient who is not a good candidate for fat grafting or pedicled or free tissue transfer, placing implants over the pectoralis major muscles and covering the implants fully with acellular dermal matrix would be the most appropriate method of reconstructing her breasts and addressing her concerns.

Physical therapy and muscle relaxants are unlikely to produce long-term improvement. An MRI would likely be nondiagnostic, and even if her implants were ruptured, change to a prepectoral plane is still indicated. Botulinum toxin type A is likely not as effective for long-term significant improvement as reoperation. Triamcinolone would not be effective for hyperdynamic deformity.

How well did you know this?
1
Not at all
2
3
4
5
Perfectly
87
Q

A 33-year-old woman comes to the office for consultation because she is dissatisfied with the “sagging” appearance of her breasts. Examination shows grade II ptosis and loss of fullness in the upper pole. A vertical mastopexy is planned. The most common medial innervation to the nipple-areola complex is the anterior cutaneous branches of which of the following intercostal nerves?

A) Second and third
B) Third and fourth
C) Fourth and fifth
D) Fifth and sixth
E) Sixth and seventh

A

The correct response is Option B.

The most common medial innervation of the nipple-areola complex is mainly 57% from the anterior cutaneous branches of the third and fourth intercostal nerves. The third intercostal nerve accounts for 21.4%. They always reach the areolar edge between 8 and 11 o’clock on the left and 1 and 4 o’clock on the right. The nerve innervation to the nipple-areola complex is important in planning different incisions around the areola in both reduction mammaplasty and mastopexy.

How well did you know this?
1
Not at all
2
3
4
5
Perfectly
88
Q

A 45-year-old woman with a T2 N0 M0 invasive ductal carcinoma in the inferior pole of the left breast is scheduled to undergo segmental mastectomy followed by radiation therapy. She currently wears a size 36E brassiere and wants to have any cup size from a C to D. After segmental resection of the tumor, which of the following procedures is most likely to yield the best cosmetic result in this patient?

A) Bilateral reduction mammaplasty
B) Primary closure of the left breast and reduction mammaplasty of the right breast
C) Reconstruction of the left breast with a latissimus dorsi musculocutaneous flap and reduction mammaplasty of the right breast
D) Reconstruction of the left breast with a transverse rectus abdominis musculocutaneous flap and mastopexy of the right breast
E) A single-stage breast reconstruction with placement of a silicone prosthesis in the left breast and reduction mammaplasty of the right breast

A

The correct response is Option A.

In a patient with large breasts, in whom a partial mastectomy is required, reduction mammaplasty is an appropriate management. This procedure will potentially relieve symptoms of macromastia, reduce the amount of breast tissue present in both breasts, and offer the best aesthetic outcome. Implantation of a prosthesis in a breast that will undergo radiation therapy increases the risk for complications and would likely lead to a less symmetrical result. The latissimus dorsi or transverse rectus abdominis musculocutaneous (TRAM) flaps could be used for immediate partial reconstruction, but they are ideal for delayed reconstruction of partial mastectomy defects. Reduction mammaplasty does not preclude future reconstruction options, but a latissimus flap reconstruction would. With a T2 tumor, a significant portion of the lower pole of the breast is removed to obtain clear margins. Even though the patient described has moderate-to-large breasts, there is a high likelihood that she will develop a deformity of the left breast and asymmetry with the right breast if no reconstruction is performed.

How well did you know this?
1
Not at all
2
3
4
5
Perfectly
89
Q

A 20-year-old man is brought to the emergency department after sustaining a stab wound to the neck during a violent assault. Physical examination shows an expanding neck hematoma and stridor. Intraoperative exploration shows a deep laceration to the anterior lateral neck at the level of thyroid cartilage and profuse extravasation of blood from the carotid sheath. According to anatomical zone-based classification of penetrating neck injuries, which of the following zones is involved?

A) Zone 1
B) Zone 2
C) Zone 3
D) Zone 4

A

The correct response is Option B.

“Penetrating neck injury represents 5-10% of all trauma cases. It is important for clinicians to be familiar with management principles, as mortality rates can be as high as 10%.”

Penetrating neck injury describes trauma to the neck that has breached the platysma muscle. The most common mechanism of injury worldwide is a stab wound from violent assault, followed by gunshot wounds, self harm, road traffic accidents, and other high velocity objects. The neck is a complex anatomical region containing important vascular, aerodigestive, and neurological structures that are relatively unprotected. Arterial injury occurs in approximately 25% of penetrating neck injuries; carotid artery involvement is seen in approximately 80% and vertebral artery in 43%.

Hard signs indicating immediate explorative surgery in penetrating neck injury:

Shock

Pulsatile bleeding or expanding hematoma

Audible bruit or palpable thrill

Airway compromise

Wound bubbling

Subcutaneous emphysema

Stridor

Hoarseness

Difficulty or pain when swallowing secretions

Neurological deficits

The assessment and management of penetrating trauma to the neck has traditionally centered on the anatomical zone-based classification first described by Monson et al. in 1969

Zone 1 extends from clavicles to cricoid, zone 2 from cricoid to angle of mandible, and zone 3 from angle of mandible to skull base.

There are only 3 zones in penetrating neck injuries.

References

Monson DO, Saletta JD, Freeark RJ. Carotid vertebral trauma. J Trauma. 1969;9(12):987-999.

Nowicki JL, Stew B, Ooi E. Penetrating neck injuries: a guide to evaluation and management. Ann R Coll Surg Engl. 2018;100(1):6-11.

Saito N, Hito R, Burke PA, Sakai O. Imaging of penetrating injuries of the head and neck: current practice at a level I trauma center in the United States. Keio J Med. 2014;63(2):23-33.

Vishwanatha B, Sagayaraj A, Huddar SG, Kumar P, Datta RK. Penetrating neck injuries. Indian J Otolaryngol Head Neck Surg. 2007;59(3):221-224.

How well did you know this?
1
Not at all
2
3
4
5
Perfectly
90
Q

A 47-year-old woman comes to the office after sustaining an injury to the left wrist after falling on her outstretched hand. Examination shows pain of the radial aspect of the left wrist and anatomical snuffbox. Scaphoid fracture is suspected. When obtaining posterior-anterior x-ray studies, which of the following is the optimal positioning of the wrist for evaluation of the entire scaphoid?

A) Wrist in 20 degrees of radial deviation, 20 degrees of wrist extension
B) Wrist in 20 degrees of radial deviation, 20 degrees of wrist flexion
C) Wrist in 20 degrees of ulnar deviation, 20 degrees of wrist extension
D) Wrist in 20 degrees of ulnar deviation, 20 degrees of wrist flexion
E) Wrist in neutral radial/ulnar position, neutral flexion/extension

A

The correct response is Option C.

Scaphoid fractures are the most common carpal fracture and frequently occur after a fall onto an extended and radially deviated wrist. Initial workup often involves plain x-ray studies, which have a sensitivity of approximately 85%. The optimal position of the wrist when imaging scaphoid fractures includes ulnar deviation and wrist extension, which allows for evaluation of the long axis of the scaphoid. CT scan or MRI may be used as additional imaging if plain x-ray studies do not demonstrate a fracture, yet there is high clinical suspicion.

How well did you know this?
1
Not at all
2
3
4
5
Perfectly
91
Q

A 40-year-old woman is evaluated in the emergency department after she fell on her outstretched hand while playing tennis. Examination shows tenderness of the wrist. After the scaphoid, which of the following carpal bones is most likely fractured in this patient?

A) Capitate
B) Hamate
C) Lunate
D) Pisiform
E) Triquetral

A

The correct response is Option E.

The triquetral is the second most commonly fractured carpal bone. Most triquetral fractures are dorsal ridge fractures that appear as avulsion fractures on lateral view wrist x-ray studies.

The most common carpal bone fracture incidences in order of frequency are scaphoid, triquetral, trapezium, lunate, and hamate.

How well did you know this?
1
Not at all
2
3
4
5
Perfectly
92
Q

A 27-year-old man sustained multiple facial fractures when he was involved in a motorcycle collision. On arrival to the emergency department, blood pressure is 80/50 mmHg and heart rate is 150 bpm. Significant retropharyngeal bleeding is noted. Trauma workup reveals no other injuries. CT angiography shows active bleeding from the right maxillary artery. Angioembolization is planned and massive transfusion protocol is initiated. Which of the following is the most appropriate intravenous resuscitation in this patient?

A) Fresh frozen plasma (FFP) and packed red blood cells (pRBC) in a 1:1 ratio; discontinuation of crystalloids
B) FFP and pRBC in a 1:1 ratio; crystalloids via rapid transfuser (max rate)
C) FFP and pRBC in a 1:4 ratio; crystalloids at 125 cc/h
D) FFP and pRBC in a 1:4 ratio; discontinuation of crystalloids
E) FFP and pRBC in a 4:1 ratio; crystalloids via rapid transfuser (max rate)

A

The correct response is Option A.

For initiation of a massive transfusion protocol, transfusing fresh frozen plasma (FFP) and packed red blood cells (pRBC) at a 1:1 ratio and discontinuing intravenous crystalloids is the most appropriate next step in patient management.

Massive Transfusion Protocol guidelines have been set forth by the American College of Surgeons through its Trauma Quality Improvement Program (TQIP). Recommendations for initiating a massive transfusion protocol include:

Beginning universal blood product infusion rather than crystalloid or colloid solutions,

Transfusing universal pRBC and FFP in a ratio between 1:1 and 1:2 (FFP:pRBC),

Transfusing one single donor apheresis or random donor platelet pool for each six units of pRBC.

It is also suggested to deliver PRBC and FFP by a rapid transfuser and through a blood warmer, and that the initial rate of transfusion should restore perfusion while allowing for “permissive hypotension” until the operation or angioembolization to stop the bleeding begins.

How well did you know this?
1
Not at all
2
3
4
5
Perfectly
93
Q

A 28-year-old woman comes to discuss primary augmentation mammaplasty options and is deciding between form-stable shaped implants and less cohesive round silicone gel implants. She inquires about the benefits of each type of implant. Compared with smooth round silicone gel implants, highly cohesive form-stable gel implants have a decreased incidence of which of the following?

A) Capsular contracture
B) Implant malposition
C) Infection
D) Seroma

A

The correct response is Option A.

Form-stable silicone gel implants are fifth-generation, shaped, and textured implants that have additional cross-linking between molecules. They are purported to have several advantages over other round saline and silicone gel implants because they retain their shape and decrease the incidence of folding and rippling. This has translated into significantly lower capsular contracture rates.

However, they do have some disadvantages. Because they are shaped and maintaining orientation is critical, they have a higher incidence of malposition. They are also more prone to seroma formation, which may be associated with their textured surface.

Infection and resorption rates remain similar.

How well did you know this?
1
Not at all
2
3
4
5
Perfectly
94
Q

A 22-year-old man who has consumed alcoholic beverages punches a concrete wall with both hands and sustains multiple metacarpal fractures. Which of the following will cause the most significant long-term hand impairment in this patient?

A ) Fifth metacarpal neck fracture with 40-degree angulation
B ) Fourth metacarpal neck fracture with 35-degree angulation
C ) Fourth metacarpal shaft fracture with 10-degree angulation
D ) Third metacarpal neck fracture with 25-degree angulation
E ) Third metacarpal shaft fracture with 0-degree angulation

A

The correct response is Option D.

Angulation is better compensated for in the ring and little fingers. The carpometacarpal (CMC) joints of these digits have 20 to 30 degrees of mobility in the sagittal plane. Angulation deformities in the little finger up to 40 to 70 degrees have been followed and found to have no functional impairment.

However, because of the lack of compensatory CMC motion in the index and long metacarpal neck fractures, there is universal agreement that residual angulation greater than 10 to 15 degrees should not be accepted.

Metacarpal shaft fractures generally require reduction for angulation greater than 30 degrees in the little finger, 20 degrees in the ring finger, and any angulation in the long and index fingers.

How well did you know this?
1
Not at all
2
3
4
5
Perfectly
95
Q

A 36-year-old woman with genetic susceptibility to breast cancer is scheduled to undergo bilateral prophylactic mastectomy. She has elected to proceed with immediate single-stage reconstruction using permanent silicone implants and acellular dermal matrix. Which of the following is the most sensitive method to detect implant rupture in this patient?

A) CT scan
B) Mammography
C) MRI
D) Physical examination
E) Ultrasonography

A

The correct response is Option C.

Implant rupture is one of the most common reasons for implant removal. While implant rupture can be associated with symptoms such as capsular contracture, it is often completely asymptomatic or “silent.” For this reason, screening imaging to detect such ruptures has been recommended.

According to the 2011 Update on the Safety of Silicone Gel-Filled Breast Implants published by the U.S. Food and Drug Administration, it is currently recommended that women with silicone implants get their first MRI 3 years after they receive the implants and every 2 years thereafter to detect silent ruptures.

Since the rate of rupture increases the longer an implant is in place, not screening is unacceptable.

Mammograms can detect extracapsular silicone when an implant ruptures, but they do not detect intracapsular ruptures.

The accuracy of ultrasound largely depends on the skill of the ultrasound technologist, the type of equipment used, and the experience of the interpreting physician. Furthermore, ultrasound is limited in its ability to detect ruptures in the back wall of the implant and in the breast tissue behind it.

CT scans can detect intracapsular silicone gel-filled breast implant rupture, but they are limited in their ability to detect extracapsular ruptures.

How well did you know this?
1
Not at all
2
3
4
5
Perfectly
96
Q

A 48-year-old woman, gravida 3, para 3, who wears a size 36B bra comes to the physician for evaluation of breast ptosis. BMI is 24 kg/m2. Physical examination shows the distance from sternal notch to nipple is 28 cm, and there is grade 2 breast ptosis with skin laxity. A combined augmentation/mastopexy is planned. Which of the following is the biggest risk of combining the procedures rather than staging them?

A) Hematoma
B) Need for revision procedure
C) Nipple-areola complex necrosis
D) Seroma
E) Transection of lateral intercostal nerves

A

The correct response is Option B.

Combining an augmentation with a mastopexy has long been considered risky because the surgeon is addressing two opposing forces during the same operation: the ptosis and volume, for which the placement of additional weight may exacerbate ptosis. Studies have shown, however, that the two operations can safely be combined. During the planning, particularly for severe ptosis, the surgeon must be careful not to overresect skin that will be critical for closure over an implant.

Compared with staged procedures, mastopexy-augmentation has a higher rate of need for revision procedures. Patients should be counseled about the potential need for revisions.

Seroma and hematoma are not increased when combining the procedures, and nipple-areola complex necrosis is a function of pedicle size and patient-specific factors such as obesity and tobacco use, rather than the combination of procedures. Similarly, transection of intercostal nerves is associated more closely with pedicle type than with combining procedures.

How well did you know this?
1
Not at all
2
3
4
5
Perfectly
97
Q

The use of routine systemic antibiotic prophylaxis is indicated in which of the following procedures?

A) Abdominoplasty
B) Carpal tunnel release
C) Excision of squamous cell carcinoma of the skin
D) Reduction mammaplasty
E) Rhytidectomy

A

The correct response is Option D.

Systemic antibiotic prophylaxis is recommended in clean breast surgery. Studies have shown that the use of antibiotic prophylaxis in patients undergoing breast surgery (with or without implant) significantly reduces the risk of surgical site infections. The benefit from routine antibiotic prophylaxis is greater in individuals receiving tissue expanders or breast implants for reconstruction, but patients undergoing breast augmentation or reduction mammaplasty also benefit from antibiotic prophylaxis. With the exception of cosmetic breast surgery, clean operations have not been shown to benefit from routine antibiotic prophylaxis. Therefore, the use of routine antibiotic prophylaxis is not indicated in clean surgical cases of the hand (carpal tunnel release), skin (squamous cell carcinoma of the skin), head and neck, or abdominoplasty. It is indicated in contaminated surgery of the hand or face.

How well did you know this?
1
Not at all
2
3
4
5
Perfectly
98
Q

A 24-year-old man is scheduled to undergo reconstruction for avascular necrosis of the proximal pole of the scaphoid with a free osteochondral bone flap. Which of the following arteries is the most commonly encountered vascular pedicle for the medial femoral condyle free bone flap?

A) Anterior tibial recurrent
B) Descending genicular
C) Popliteal
D) Saphenous
E) Superficial femoral

A

The correct response is Option B.

The medial femoral condyle free bone (corticocancellous) flap has been shown to be an excellent option for treatment of complicated degenerative bone pathology in the wrist, particularly scaphoid avascular necrosis. The Mayo group has also shown improved outcomes for scaphoid nonunion with humpback deformity compared with pedicled flaps from the distal radius. The same group has shown more consistent presence of supply from the medial superior genicular artery, which can be used in cases where the descending genicular artery is insufficient, although the pedicle length of the medial superior genicular artery is generally shorter. More recent anatomic analysis has shown this vessel can supply flaps up to 11 cm in length.

The (superficial) femoral and popliteal arteries are larger, regional vessels, with the superficial femoral artery being the immediate source vessel for the descending genicular artery. The popliteal artery is the source vessel for the medial superior genicular artery. The anterior tibial recurrent artery is distal and lateral, lying over the lateral aspect of the tibial plateau, and does not supply the medial femoral condyle. The saphenous artery has been described as a branch of the superficial femoral supplying the skin paddle overlying the medial femoral condyle but does not supply the bone.

How well did you know this?
1
Not at all
2
3
4
5
Perfectly
99
Q

A 46-year-old woman with ductal carcinoma in situ is scheduled to undergo right mastectomy. Immediate reconstruction with a tissue expander and acellular dermal matrix (ADM) is planned. Which of the following is an expected outcome with use of ADM?

A) Decreased formation of seroma
B) Decreased incidence of hematoma
C) Decreased risk of infection
D) Increased capsule contracture
E) Increased initial fill of the expander

A

The correct response is Option E.

According to Sbitany, et al, acellular dermal matrix (ADM) allows for a greater initial fill of saline. This potentially improves cosmetic outcome, as it better capitalizes on preserved mastectomy skin for reconstruction. Sbitany, et al, concluded that ADM-assisted prosthesis breast reconstruction has a safety profile no worse than that of complete submuscular coverage but offers the benefit of fewer expansions and the potential for more predictable secondary revisions.

ADM has enhanced prosthesis-based reconstruction and remains useful in immediate prosthetic breast reconstruction. However, it has been found to have higher rates of postoperative seroma and infection. It has also been reported to decrease capsule contracture.

How well did you know this?
1
Not at all
2
3
4
5
Perfectly
100
Q

A 27-year-old woman comes to the office for evaluation of bilateral hand pain. The patient reports worsening pain when she retrieves items from the freezer and says that in the winter she experiences pain in her fingers unless she wears electric, heated gloves and on occasion her fingers will turn white and blue. Medical history includes no personal or family history of joint or skin problems. Physical examination shows the patient’s fingers are warm, and wrist pulses are palpable. Which of the following is the most appropriate initial management of this patient’s symptoms?

A) Botulinum toxin type A injection
B) Oral nifedipine
C) Temperature biofeedback
D) Thoracoscopic sympathectomy
E) Topical nitroglycerin

A

The correct response is Option B.

Patients presenting with Raynaud syndrome fall into two classic categories: primary (traditionally referred to as Raynaud disease) and secondary (Raynaud phenomenon, associated with an underlying condition, most commonly involving an autoimmune process). In this woman presenting without an underlying etiology for her vasospastic symptoms, primary treatment should be directed at managing the vasospasm. Although a plethora of interventions have been tried, recent reviews show the calcium-channel blockers, such as nifedipine, to be the optimal first-line intervention.

Temperature biofeedback has shown variable effect in multiple small trials, and, consequently, it is not recommended as a primary intervention for vasospasm.

Topical nitrates can assist with vasodilation in the digits, making them an occasional adjunct treatment for symptoms not completely managed by calcium channel blockers. In isolation, topical nitrates have been ineffective for managing Raynaud syndrome.

Multiple small trials have demonstrated successful relief of pain and digital ulcers in a mixed group of both primary and secondary Raynaud syndrome with injection of botulinum toxin around the digital vessels in the palm. The cost and risk of temporary paralysis to intrinsic muscles, however, renders this a second-line treatment for refractory pain or nonhealing ulcers. Treatment of digital vasospasm is still considered an “off-label” use of botulinum toxin and may not be covered by insurance.

Surgical sympathectomy, either proximally through a thoracoscopic approach or peri-arterially in the wrist and hand, represents the most aggressive treatment and would typically be reserved for patients with nonhealing wounds or chronic ischemic changes. These procedures are gradually being supplanted by injection of botulinum toxin type A.

References

Butendieck RR, Murray PM. Raynaud Disease. J Hand Surg Am. 2014 39 (1):121-4.

Wigley FM, Flavahan NA. Raynaud’s Phenomenon. N Engl J Med. 2016 Aug 11;375(6):556-65.

Valdovinos ST, Landry GL. Raynaud Syndrome. Tech Vasc Interv Radiol. 2014 Dec;17(4):241-6.

How well did you know this?
1
Not at all
2
3
4
5
Perfectly
101
Q

A 77-year-old man undergoes wide local excision of a melanoma on the posterior shoulder. Concomitant sentinel lymph node biopsy is positive for metastasis. Completion axillary lymph node dissection and adjuvant radiation therapy to the axilla are performed. Metastatic workup shows no abnormalities. Postoperatively, the patient develops chronically draining seroma and open wound to the axilla. He is referred for management of the radiation wound after it fails to improve with conservative local wound care. Physical examination shows a 2-cm open wound to the axilla, which tunnels 5 cm into the apex and is surrounded by extensive fibrosis and erythema. Which of the following interventions is most likely to result in a definitive closure?

A) Administration of 30 hyperbaric oxygen treatments at 2.4 ATA
B) Application of negative pressure wound therapy
C) Excision of the wound cavity and full-thickness skin graft reconstruction
D) Excision of the wound edges and application of acellular dermal matrix
E) Wide excision of the irradiated soft-tissues and repair with a thoracodorsal artery perforator flap

A

The correct response is Option E.

Radiation damage produces a hypoxic, hypovascular, and hypocellular environment that can lead to delayed healing and even ulcer formation. Principles of management once a chronic radiation wound has developed include aggressive surgical removal of the entire zone of injury, which is sometimes larger than initially anticipated, and repair with well-vascularized tissue. A thoracodorsal artery perforator flap will provide sufficient healthy and supple soft tissue to cover the entire axilla. The thoracodorsal pedicle should be intact as it is not regularly divided during axillary lymph node dissection.

Negative pressure wound therapy would be contraindicated in this wound because of the potential for exposure of blood vessels in the axilla as well as for sponge retention in the tunneled wound and is, therefore, incorrect.

Excision of the wound edges only may be inadequate treatment of the zone of injury, and it is not the best choice. Furthermore, acellular dermal allograft plays no role, aside from possibly a temporizing measure, in the treatment of radiation ulcers.

Hyperbaric oxygen therapy is indicated for soft tissue radionecrosis. This delivers increased levels of oxygen to the hypoxic, irradiated wound and establishes a steeper oxygen gradient between the wound and surrounding tissues. The cellular response to this gradient stimulates neovascularization and can improve or even heal some wounds. However, with this large, cavitary wound, hyperbaric oxygen therapy, as a single-line treatment, would be unlikely to heal the wound completely.

Excision of the wound cavity and full-thickness skin graft reconstruction is not the best option for two reasons. First, the excision may lead to exposure of neurovascular structures that should not be covered with a skin graft. Second, risk of partial or total loss of a full-thickness skin graft may be unacceptably high in an irradiated wound bed.

How well did you know this?
1
Not at all
2
3
4
5
Perfectly
102
Q

A 45-year-old woman comes to the office 10 years after undergoing subglandular implantation of textured silicone implants for augmentation mammaplasty. Physical examination shows swelling of the left breast. She is concerned about cancer. Increased incidence of which of the following malignancies is associated with breast implants?

A) Acute myeloid leukemia
B) Anaplastic large cell lymphoma
C) Angiosarcoma
D) Infiltrating ductal carcinoma
E) Malignant fibrous histiocytoma

A

The correct response is Option B.

Several reports have suggested an association between breast implants and anaplastic large cell lymphoma (ALCL), which is an extremely rare malignancy. In these cases, ALCL has usually occurred several years after implantation as swelling or a mass around the implant and is often associated with a periprosthetic seroma. Treatments have included capsulectomy with implant removal and chemotherapy and/or radiation therapy, though there is no defined consensus regimen. Despite evidence of an increased risk of ALCL in breast implant patients, the absolute risk remains extremely low.

Several large epidemiologic studies have demonstrated a similar or lower incidence of breast cancer (infiltrating ductal carcinoma) among patients who have undergone prosthetic augmentation mammaplasty surgery compared with those who have not. Most cases of ALCL have been in textured implants.

Angiosarcoma and malignant fibrous histiocytoma are two sarcomas that may arise in the breast. Angiosarcoma may be caused by radiation therapy for breast cancer. Neither of these sarcomas has been associated with breast implants.

Acute myeloid leukemia may be associated with radiation treatment to the breast but has not been associated with breast implants.

How well did you know this?
1
Not at all
2
3
4
5
Perfectly
103
Q

A 28-year-old postpartum woman comes to the office for evaluation of breast asymmetry with pain and enlargement of the right breast for 2 months. Medical history includes augmentation mammaplasty 4 years ago. She denies fever or chills. She was previously breast-feeding but stopped this 1 month ago. Physical examination shows the right breast is significantly larger than the left breast. A well-healed peri-areolar incision is present and no evidence of infection is noted. Ultrasound shows a complex cyst, which yields 150 cc of milky fluid. A drain is placed. The most appropriate next step is administration of which of the following medications?

A) Bromocriptine
B) Cephalexin
C) Fluconazole
D) Prolactin
E) Trimethoprim-sulfamethoxazole

A

The correct response is Option A.

This postpartum patient is presenting with a symptomatic galactocele after breast-feeding. Galactoceles are benign breast cysts containing milk. They typically occur in women of childbearing age in the setting of active lactation, recent pregnancy, or the use of hormonal medications such as oral contraceptives. The galactocele is thought to occur from ductal obstruction. Although the presence of a breast implant and the respective pocket placement is unknown to have an effect on the development of galactoceles, there is some thought that peri-areolar incisions may contribute to the ductal obstruction. There are, however, documented cases of post-augmentation galactocele without peri-areolar incisions.

Treatment for a galactocele is typically medical with the initiation of oral bromocriptine. Bromocriptine is a dopamine receptor agonist and causes inhibition of prolactin secretion, which is the primary hormone responsible for milk production. Dosage is titrated to effect. Incision and drainage of the cyst, particularly in the setting of implants, is often performed as well to rule out the possibility of infection.

Cephalexin and trimethoprim-sulfamethoxazole are antibiotics and are not indicated in this case because there is no active infection. Fluconazole is indicated for the treatment of fungal infections. Prolactin would actually stimulate milk production and would worsen the patient’s symptoms.

How well did you know this?
1
Not at all
2
3
4
5
Perfectly
104
Q

A 40-year-old man comes to the office for evaluation of a nonhealing scaphoid fracture. History includes bone grafting for avascular necrosis of the proximal pole, but there is evidence of nonunion. Carpal collapse and humpback deformity are also noted. Reconstruction with a vascularized medial femoral condyle flap is scheduled. Which of the following arteries provides the blood supply to this flap?

A) Descending genicular artery
B) Lateral circumflex femoral artery
C) Medial circumflex femoral artery
D) Peroneal artery
E) Profunda femoris artery

A

The correct response is Option A.

The descending genicular artery supplies the medial femoral condyle flap. The descending genicular artery is a branch of the superficial femoral artery. This corticoperiosteal flap has been used with increasing frequency for small bony defects and the treatment of nonunion. A cutaneous component can also be harvested based on a saphenous artery branch.

The anterolateral thigh flap is based off the lateral femoral circumflex artery perforators. The gracilis muscle flap is based off vessels from the medial circumflex femoral artery; perforator flaps can also be designed based off this vascular system. The profunda femoris supplies the posterior thigh flap. The peroneal artery provides the vascular supply to the fibula flap.

How well did you know this?
1
Not at all
2
3
4
5
Perfectly
105
Q

The stylopharyngeus muscle is innervated by which of the following nerves?

A) Facial (VII)
B) Glossopharyngeal (IX)
C) Hypoglossal (XII)
D) Trigeminal (V)
E) Vagus (X)

A

The correct response is Option B.

The pharyngeal muscles are all innervated by the vagus (X) nerve, except the stylopharyngeus muscle, which is innervated by the glossopharyngeal nerve (IX).

The trigeminal nerve (V) is responsible for facial and oral sensation. The maxillary branch (V2) is responsible for sensation of the upper teeth, upper lip, hard palate, cheeks, and nasopharyngeal mucosa. The mandibular branch (V2) provides sensory fibers for the lower teeth, lower mucosa of the mouth and the anterior two-thirds of the tongue. The facial nerve (VII) provides motor innervation of the muscles of facial expression and the posterior bellies of the stylohyoid and digastric muscles. The vagus nerve (X) provides motor innervation to all of the pharyngeal muscles except the stylopharyngeus muscle. The hypoglossal nerve (XII) provides motor innervation to the intrinsic and extrinsic tongue muscles and also provides motor innervation to the geniohyoid muscle through the ansa cervicalis.

References

Costa MMB. Neural control of swallowing. Arq Gastroenterol. 2018 Nov;55 Suppl 1(Suppl 1):61-75.

Netter FM. Glossopharyngeal nerve: schema [print]. In: Netter FM. Atlas of Human Anatomy. 5th ed. Philadelphia, PA: Saunders Elsevier; 2010:plate 119.

How well did you know this?
1
Not at all
2
3
4
5
Perfectly
106
Q

A 54-year-old woman undergoes bilateral immediate tissue expander–based breast reconstruction. BMI is 36 kg/m2. On postoperative day 10, examination shows bilateral breast erythema; empiric antibiotics are started. In addition to Staphylococcus species, which of the following bacteria should be treated as the next most likely pathogen?

A) Bacteroides fragilis
B) Enterococcus faecalis
C) Mycobacterium marinum
D) Pasteurella multocida
E) Pseudomonas aeruginosa

A

The correct response is Option E.

After Staphylococcus aureus and S. epidermidis, Pseudomonas aeruginosa is among the the next most common sources of breast infections. It is common in hospitalized or immunocompromised patients, as well as patients with foreign body devices such as catheters or implants. It is a gram negative rod, and common antibiotic treatments include advanced β-lactams (piperacillin, ceftazidime), carbapenems, quinolones, and aminoglycosides. Dual coverage is often recommended in severe infections. In the case of breast implant infections, if there is not rapid improvement on antibiotic therapy, or if significant systemic symptoms develop (vital sign instability, high white blood cell count, fever, renal impairment), then surgical washout and device removal is mandatory. In patients without systemic symptoms, wash out and new implant placement can be an option in carefully selected and counseled patients. Other breast pathogens include Escherichia coli, Propionibacterium, and Corynebacterium.

More than 300,000 breast implant procedures are performed each year in the United States. In reconstructive cases, the infection rate averages 6% and the explantation rate 3% (range, 1.5 to 8%). Preventative measures include proper patient selection, preoperative MRSA management when carriers are suspected, routine presurgery chlorhexidine washes, proper antibiotic timing presurgery and continuation of antibiotics in implant reconstruction cases for at least 24 hours (though the optimal treatment duration has not yet been determined).

None of the other bacteria listed are common in breast infections, though all are common pathogens. Bacteroides are anaerobic gram-negative rods that are common in gut flora and feces.

Enterococcus faecalis is a frequent cause of nosocomial infection, with a high prevalence of multi-drug resistance. It is a gram-positive coccus, and is not commonly seen in breast surgery patients as it primarily colonizes the digestive tract.

Mycobaterium marinum is a rare pathogenic cause of hand infections from injuries that occur in aquatic environments.

Pasteurella multocida is a frequent cause of animal bite infections, particularly from cats and dogs.

How well did you know this?
1
Not at all
2
3
4
5
Perfectly
107
Q

When performing immediate breast reconstruction, it is important to reconstruct the lateral inframammary fold. This is because the oncologic extirpation of the breast is carried out to which of the following anatomic landmarks?

A) Anterior edge of the latissimus dorsi muscle
B) Anterior edge of the serratus muscle
C) Lateral edge of the pectoralis major muscle
D) Lateral edge of the pectoralis minor muscle
E) Posterior edge of the serratus muscle

A

The correct response is Option A.

For modified radical and simple mastectomies, the landmarks of dissection are: superiorly to the clavicle, medially to the sternum, inferiorly to the inframammary fold, and laterally to the border of the latissimus dorsi muscle. The pectoralis major muscle fascia is resected with the specimen.

The recreation of the inframammary fold is important for shaping in breast reconstruction and care must be taken to evaluate and repair both the inferior and lateral components of the inframammary fold.

How well did you know this?
1
Not at all
2
3
4
5
Perfectly
108
Q

A 28-year-old man is brought to the emergency department after sustaining injury during a motor vehicle collision. Cranialization of the frontal sinus is planned. Which of the following best describes the components of cranialization?

A) Removal of the anterior table, reconstruction of the posterior table with a titanium plate, and closure of the dura
B) Removal of the posterior table, sinus mucosa, and closure of the sinonasal tract
C) Repair of both the posterior and anterior tables with bioabsorbable plates, and obliteration of the frontal sinus
D) Repair of the anterior table and obliteration of the frontal sinus
E) Repair of the posterior table with bioabsorbable plates, removal of the sinus mucosa, and closure of the dura

A

The correct response is Option B.

Cranialization involves removal of the posterior table (not repair), closure of the dura, sinonasal tract, and obliteration of the sinus mucosa. Management of the anterior table is as indicated.

Surgical repair of the anterior table is indicated if there is nasofrontal duct involvement, or, in the absence of nasofrontal duct involvement (such as a minimally displaced anterior table), patient desire for a better aesthetic outcome. If there is nasofrontal duct involvement, the nasofrontal duct and frontal sinus can be obliterated (repair of the anterior table and obliteration of the frontal sinus).

Bioabsorbable or titanium plates can be used to fixate the fractured anterior table. It is not used for the posterior table.

How well did you know this?
1
Not at all
2
3
4
5
Perfectly
109
Q

A 49-year-old woman is evaluated because of a traumatic laceration of the right lower eyelid and cheek. Physical examination shows difficulty with eyelid closure, voluntary squinting, and animation. Which of the following branches of the facial nerve is most likely injured?

A) Buccal
B) Cervical
C) Marginal mandibular
D) Temporal
E) Zygomatic

A

E) Zygomatic

The correct response is Option E.

Anatomically, the orbicularis oculi muscle is divided into three segments: pretarsal, preseptal, and orbital. However, functionally, the orbicularis oculi muscle is divided into the medial inner canthal orbicularis and the extracanthal orbicularis. The medial inner canthal orbicularis is responsible for blinking, lower lid tone, and the pumping mechanism of the lacrimal system. Innervation to the inner canthal orbicularis is from the buccal branches of the facial nerve. The zygomatic branch of the facial nerve innervates the extracanthal orbicularis, which controls eyelid closure, voluntary squinting, and animation. The temporal, marginal mandibular, and cervical branches do not provide innervation to the orbicularis oculi muscle.

How well did you know this?
1
Not at all
2
3
4
5
Perfectly
110
Q

A 68-year-old woman comes to the office for a delayed breast reconstruction. She had right breast cancer and a mastectomy followed by chemotherapy and radiation therapy 1 year ago. BMI is 35 kg/m2. Medical history includes well-controlled type 2 diabetes mellitus, a previous cesarean section through a low transverse incision, and an open cholecystectomy through a subcostal incision. The patient requests autologous reconstruction, but the surgeon is not comfortable performing a free flap. Which of the following is the most appropriate method for reconstruction?

A) Bipedicled transverse rectus abdominis musculocutaneous (TRAM) flap
B) Contralateral pedicled TRAM flap
C) Ipsilateral pedicled TRAM flap
D) Surgical delay procedure followed by contralateral pedicled TRAM flap
E) Surgical delay procedure followed by ipsilateral pedicled TRAM flap

A

The correct response is Option D.

In this obese patient with right breast cancer and a previous subcostal incision, a delay procedure with a contralateral transverse rectus abdominis musculocutaneous (TRAM) flap is the most appropriate method for reconstruction. Although a contralateral TRAM flap can be performed without a delay procedure, it has been shown that the addition of a delay procedure decreases risks of ischemia to the flap. In addition, a delay procedure may also reduce risks of abdominal wall complications. In this patient, the subcostal incision excludes right-sided pedicled flap reconstruction, so an ipsilateral pedicled TRAM would not be the right choice, nor would a bipedicled TRAM flap.

Although there is controversy on which patients should have a delay procedure, the use of this technique has usually been limited to high-risk patients and to those requiring large amounts of tissue. Multiple reports have shown that obese patients undergoing a delay can decrease the risks of tissue related ischemia. In a paper by Wang et al., the delay procedure was performed at least 14 days prior to the reconstruction; however, other studies have shown improvements at 7 days. The procedure described consists of ligation of both deep inferior epigastric arteries and veins bilaterally accessed from an inferior flap incision. This can also be done laparoscopically. Some authors advocate more extensive incisions and elevating portions of the flap; however, there is little data to show that this is effective or necessary.

How well did you know this?
1
Not at all
2
3
4
5
Perfectly
111
Q

A 30-year-old man is scheduled to undergo transhumeral amputation after unsalvageable brachial artery occlusion. A photograph is shown. Use of targeted muscle reinnervation may allow improved functional recovery by which of the following means?

A) Better bulk and durability by preventing denervation atrophy of muscles at the amputation stump
B) Better prosthesis control by input from median and ulnar nerve signals
C) Better sensory detection in the prosthesis by positioning amputated nerve stumps closer to the skin closure
D) More precise control of an osseointegrated body-powered prosthesis
E) Preservation of greater bony length in the amputation stump

A

The correct response is Option B.

A body-powered prosthesis uses motion of remaining joints, such as the gleno-humeral and scapulo-thoracic joints, to control an upper extremity prosthesis.

Targeted muscle reinnervation (TMR) would not affect function of a body-powered prosthesis. TMR positions amputated nerve stumps well within the remaining muscle and far from the cutaneous closure. Current prosthetics are not yet able to detect sensation and transmit this to the patient. Having nerve stumps near the amputation closure site increases the risk for neuroma pain.

TMR has not been shown to decrease denervation atrophy of residual upper extremity muscles. TMR has gained increasing acceptance in the treatment of patients who have undergone or will undergo upper extremity amputation. Resected nerves, such as the median and ulnar nerves, can be coapted to nerve branches to remaining muscles, such as the pectoralis and deltoid. Transcutaneous EMG detectors are positioned over these reinnervation sites to detect nerve signal, which a myoelectric prosthesis can then use to better control distal joints.

TMR does not affect the amount of bony length that can be preserved in an amputation.

References

Dumanian GA, Ko JH, O’Shaughnessy KD, et al. Targeted reinnervation for transhumeral amputees: current surgical technique and update on results. Plast Reconstr Surg. 2009 Sep;124(3):863-9.

Souza JM, Cheesborough JE, Ko JH, et al. Targeted muscle reinnervation: a novel approach to postamputation neuroma pain. Clin Orthop Relat Res. 2014 Oct;472(10):2984-90.

Hijjawi JB, Kuiken TA, Lipschutz RD, et al. Improved myoelectric prosthesis control accomplished using multiple nerve transfers. Plast Reconstr Surg. 2006 Dec;118(7):1573-8.

Solarz MK ,Thoder JJ, Rehman S. Management of Major Traumatic Upper Extremity Amputations. Orthop Clin of North Am. 2016; 47(1):127–36

How well did you know this?
1
Not at all
2
3
4
5
Perfectly
112
Q

A 27-year-old woman is evaluated because of pain 2 weeks after undergoing subglandular augmentation mammaplasty. She has no history of fever, chills, or drainage. Physical examination discloses a painful, tender cord in the inframammary region of the left breast. Which of the following is the most appropriate next step in management?

A) Administration of an antibiotic
B) Administration of an anticoagulant
C) Administration of an anti-inflammatory agent
D) Duplex ultrasonography
E) Removal of the implant

A

The correct response is Option C.

Mondor disease of the breast is a benign, self-limiting thrombophlebitis of the inframammary veins. Clinically, Mondor disease usually occurs 2 to 3 weeks postoperatively as a painful, tender cord within the superficial veins of the thoracoepigastric system. Management is observation and includes the use of warm, moist dressings and anti-inflammatory agents for symptomatic relief. The use of anticoagulation, antibiotics, or steroids is not indicated. Implant removal is not indicated in the absence of infection. Duplex ultrasonography is not required for management.

How well did you know this?
1
Not at all
2
3
4
5
Perfectly
113
Q

Nipple-sparing mastectomy is CONTRAINDICATED in which of the following women considering mastectomy and immediate breast reconstruction?

A) A 21-year-old with BRCA1 mutation and grade I breast ptosis
B) A 31-year-old with BRCA2 mutation; subglandular augmentation mammaplasty 1 year ago
C) A 45-year-old with unilateral breast cancer and need for postoperative radiation therapy
D) A 50-year-old with unilateral breast cancer in the tail of Spence
E) A 61-year-old with unilateral breast cancer; tumor-to-nipple distance of 1 cm

A

The correct response is Option E.

Nipple-sparing mastectomy is increasingly prevalent owing to perceived improvement in reconstructive outcome and patient satisfaction. Prevalence of nipple ischemia, which can vary in severity from incomplete, partial-thickness epidermolysis to total nipple loss, ranges from 2 to 60% depending on the definition used. A recent pooled analysis found a cumulative prevalence of 7%. Most cases respond to local wound care; reoperation for total nipple loss is relatively infrequent.

Patient selection remains paramount to successful nipple preservation. Typically, thin, non-smoking patients with small, non-ptotic breasts are considered ideal candidates. Severe macromastia and ptosis may not only increase risk of poor nipple vascularity, but also contribute to nipple malposition. Oncologic determinants of the safety of nipple preservation should also be considered. Noninflammatory cancers located in excess of 2 cm from the nipple can generally be safely extirpated without removal of the nipple. Usually, an intraoperative frozen section biopsy of the retroareolar tissue (“doughnut”) is performed after excision of the main mastectomy specimen to demonstrate nipple margins free of tumor. Patients undergoing prophylactic mastectomy in the setting of high genetic predisposition are also considered good candidates. Successful nipple preservation has also been described in patients with a history of reduction mammaplasty or mastopexy.

Successful nipple-sparing mastectomy with implant reconstruction has been described recently in patients who require postoperative radiation, albeit with higher risk of capsular contracture and nipple malposition. Prior augmentation in the subglandular or subpectoral positions does not contraindicate nipple-sparing mastectomy; rather, the latter group may present opportunities for direct-to-implant reconstruction. Location of a breast cancer in the tail of Spence (axillary extension of breast tissue) should not contraindicate nipple-sparing mastectomy, as it is likely to be far enough from the nipple to safely spare it.

How well did you know this?
1
Not at all
2
3
4
5
Perfectly
114
Q

In women with breast ptosis, which of the following is an advantage of performing combined one-stage augmentation mammaplasty with mastopexy compared with mastopexy alone?

A) Better predictability of cosmetic outcome
B) Decreased complication rates
C) Decreased revision rates
D) Improved upper pole projection
E) Lower operative costs

A

The correct response is Option D.

Early reports have raised concerns about the safety of combined augmentation mammaplasty with mastopexy surgeries. However, in patients who wish to correct their breast ptosis these two procedures are often combined to a one-stage surgery and can show favorable outcomes. Nevertheless, plastic surgeons advocate that these cases should only be performed by experienced physicians.

Reasons are that the overall aesthetic results are harder to predict in one-stage augmentation/mastopexy procedures compared to mastopexy alone or even the two-stage augmentation mammaplasty followed by mastopexy. Both complication and revision rates are highest in the one-stage approach that combines augmentation mammaplasty with mastopexy. Longer operative time and the need for implants naturally increase operative costs.

The advantage of the simultaneous insertion of implants is the improved superior pole projection that cannot be achieved by mastopexy alone.

How well did you know this?
1
Not at all
2
3
4
5
Perfectly
115
Q

A 34-year-old woman is scheduled to undergo bilateral prophylactic mastectomy because of a strong family history of breast cancer. Reconstruction with free flaps from the upper inner thigh is planned. Which of the following is the pedicle to these flaps?

A) Ascending branch of the lateral femoral circumflex artery
B) Descending branch of the lateral femoral circumflex artery
C) Medial femoral circumflex artery
D) Superficial circumflex iliac artery
E) Superficial femoral artery

A

The correct response is Option C.

The transverse upper gracilis (TUG) flap is a free flap that can be used in breast reconstruction. It is particularly useful in cases where the abdominal donor site is not available or not desired. The flap incorporates the skin and subcutaneous tissues of the upper inner thigh, in the region of the medial thigh lift. The TUG flap is based on the gracilis vessels and perforators from the descending branch of the medial femoral circumflex artery, which forms the dominant pedicle. This arises from the profunda femoris artery. Modifications of this flap have been proposed to increase the volume available for free tissue transfer.

The ascending branch of the lateral femoral circumflex artery provides blood supply to the tensor fascia lata flap.

The descending branch of the lateral femoral circumflex artery forms the pedicle for the anterolateral thigh flap.

The superficial circumflex iliac artery forms the pedicle for the groin flap.

Branches from the superficial femoral artery provide minor pedicles to the distal portion of the gracilis muscle, but are less important and distal to the area of the TUG flap.

How well did you know this?
1
Not at all
2
3
4
5
Perfectly
116
Q

Which of the following best represents the likelihood that a patient with a frontal sinus fracture would have a concurrent intracranial injury?

A) 1%
B) 15%
C) 30%
D) 55%
E) 90%

A

The correct response is Option D.

In an acute trauma setting, the recognition of mild traumatic brain injury (mTBI) is a diagnostic challenge as there are often competing diagnoses that take immediate priority. Furthermore, within this cohort, patients with craniofacial fractures have been shown to be at risk for delayed or missed diagnosis for all degrees of TBI, although with a higher likelihood of missed or delayed diagnosis for mTBI compared with moderate to severe TBI. Previously, it was hypothesized that facial fractures buffered the forces transmitted during blunt head trauma, thereby protecting intracranial structures. This conceptual framework has since been questioned as evidence has mounted that individuals with facial fractures are at increased risk for head injury. The biomechanics resulting in different types of facial fractures and the amount of force required to fracture the different components of the facial bony structure have been well described. The nasal bone has the lowest tolerance for fracture at 25 to 75 lbs, while the frontal bone has the highest tolerance at 800 to 1600 lbs. Recent studies have proposed that craniofacial fractures can serve as clinical markers for brain injury and Mulligan et al. suggest that the prevalence of overall head and cervical spine injuries in the setting of facial fractures is high enough to warrant a change in current protocols.

In this context, the prevalence of mTBI and moderate to severe TBI in patients with isolated facial fractures in the National Trauma Databank (NTDB) was evaluated, and further characterized the association of isolated facial fractures with different degrees of TBI in patients with mild, moderate, and severe TBI. Facial fractures can serve as objective clinical markers for the potential presence of mTBI and moderate to severe TBI in trauma patients. As mTBI patients have been shown to benefit from simple, easy-to-administer educational interventions, trauma patients with facial fractures may benefit from automatically receiving education about mTBI and TBI recovery, given the clinically meaningful prevalence of mTBI and TBI in this population. As one moves up the craniofacial skeleton, the forces are transmitted more reliably to the intracranial space. Therefore, a frontal sinus fracture is at extremely high risk (usually a 45 to 65% chance) of having an associated intracranial injury.

How well did you know this?
1
Not at all
2
3
4
5
Perfectly
117
Q

A 30-year-old man is evaluated for a hand injury after punching a wall. X-ray study shows a fracture of the fifth metacarpal neck. Which of the following findings is most likely to require surgical intervention?

A) Angulation
B) Callus formation
C) Impaction
D) Malrotation
E) Shortening

A

The correct response is Option D.

Malrotation causes scissoring of the affected digit, which will adversely affect neighboring digits. This will affect activities of daily living and is an indication for operative intervention. Other indications for operative intervention include displaced intra-articular fractures, severe soft-tissue injury, unstable open fractures, segmental bone loss, and multiple fractures. Angulation is usually dorsal tip–oriented because of intrinsic and extrinsic muscle pull. A good rule of thumb is the 10, 20, 30, 40 for digits two, three, four, and five. The fourth and fifth digits have carpometacarpal joint mobility and can tolerate larger angulation. The fifth digit metacarpal may even tolerate up to 70 degrees of angulation as long as there is no extensor lag. Impaction can lead to shortening and/or angulation, which is tolerated more than rotational deformities. Similarly, shortening is well tolerated as long as there is no extensor lag. Nonoperative complications include aesthetic with loss of knuckle and possible pain in the palm from the metacarpal head. Callus formation is indicative of an old, healed fracture.

How well did you know this?
1
Not at all
2
3
4
5
Perfectly
118
Q

An otherwise healthy, nonsmoking 30-year-old mechanic has the long, ring, and little fingers amputated sharply through Zone II of the right hand. The amputated digits are stored appropriately, and he is rushed to surgery within 2 hours of the accident. Which of the following sequences is the best method of replantation?

A) Digit by digit: bone, tendons, arteries, nerves, veins
B) Digit by digit: bone, tendons, arteries, veins, nerves
C) Structure by structure: bone, nerves, tendons, arteries, veins
D) Structure by structure: bone, tendons, arteries, nerves, veins
E) Structure by structure: tendons, bones, veins, arteries, nerves

A

The correct response is Option D.

The most efficient sequence to perform the replantation is structure by structure: bone, tendons, arteries, nerves, and then veins. It has been shown that the time to complete the procedure is significantly shorter if the same anatomic structure on each severed digit is fixed before repairing the next structures, as opposed to completing all aspects of the replantation one digit at a time. With respect to the sequence of repair of the severed structures, the general thought is to have a stable construct prior to starting the delicate microscopic repairs. However, the technical sequence used by microsurgeons varies greatly.

The only consistent agreement is starting with bony shortening and fixation. The traditional sequence that follows is extensor and flexor tendon repair, and then vessel/nerve repair. However, individual surgeon preference and patient circumstances dictate the usual sequence thereafter. Some surgeons like to start dorsally and complete the extensor tendon, venous, and skin repair first, and then complete the volar structures next. On the volar side, some surgeons repair the tendon first, followed by the artery and nerve, while others fix the artery and nerve first, followed by the tendon. There are those who believe that the nerve is better repaired in a bloodless field, so that should be done first. Others feel that repairing the vein first reduces blood loss and keeps a bloodless field more reliably for better vision. In patients who present with long ischemia time, it may be beneficial to anastomose the artery first, because this provides the advantages of earlier revascularization and allows easier detection of the most functional veins by their spurting backflow. In short, any of these sequences is fine, as long as it follows the bony fixation.

The other options are incorrect sequences for the above reasons.

References

Maricevich M, Carlsen B, Mardini S, Moran S. Upper extremity and digital replantation. Hand (NY). 2011; Dec; 6(4):356–363.

Prucz RB, Friedrich JB. Upper Extremity Replantation: Current Concepts. Plast Recon Surg. 2014;133(2):333-342.

Walaszek I, Zyluk A. Long term follow-up after finger replantation. J Hand Surg Eur Vol. 2008;33(1):59–64.

How well did you know this?
1
Not at all
2
3
4
5
Perfectly
119
Q

Breast implant–associated anaplastic large cell lymphoma is most closely associated with which of the following implant characteristics?

A) High-profile dimensions
B) Saline filling
C) Silicone gel filling
D) Smooth shell
E) Textured shell

A

The correct response is Option E.

The overwhelming majority of reported cases of breast implant–associated anaplastic large cell lymphoma (BIA-ALCL) have been associated with textured surface implants.

Anaplastic large cell lymphoma in association with breast implants is a rare occurrence; however, when it does appear, the course is usually less aggressive with a better prognosis than when it is unrelated to breast implants. Recent studies suggest that the breast implant shell causes a chronic T-cell stimulation. This reaction is thought to be caused by an interaction of textured surface characteristics and associated biofilm.

BIA-ALCL associated with smooth shell implants has occurred; however, it is disproportionately rare.

ALCL is seen with both silicone- and saline-filled implants. These numbers are highly influenced by the specific popularity of each implant. Specific implant dimensions, be it projection or width, are not uniquely associated with ALCL.

How well did you know this?
1
Not at all
2
3
4
5
Perfectly
120
Q

A 35-year-old woman comes to the office for consultation because she is dissatisfied with the appearance of her “deflated” and “saggy” breasts. Augmentation/mastopexy is planned. Compared with placement of the implant in the subglandular position, placement of the implant in the subpectoral space will preserve blood supply to the breast tissue and skin through which of the following arteries?

A) Internal thoracic
B) Lateral thoracic
C) Superficial superior epigastric
D) Thoracoacromial
E) Thoracodorsal

A

The correct response is Option D.

The perfusion of the nipple-areola complex is a major concern during breast procedures involving periareolar and intraparenchymal incisions. The nipple-areola complex has a very rich and overlapping perfusion through multiple sources. This fact allows the design of various pedicles to carry the nipple and areola with different techniques. The blood supply through the internal thoracic vessels reaches the breast, nipple, and areola through the intercostal perforators, which may be divided during both subpectoral and subglandular implant placement.

The location of the implant deep or superficial to the pectoralis muscle will not change the perfusion through the superficial epigastric vessels. The same is true for the blood supply through the lateral thoracic vessels. However, the flow through the thoracoacromial vessels to the breast parenchyma will be preserved by placement of the implant deep to the pectoralis muscle. Creation of a subglandular pocket above the muscle will interrupt the collaterals from the thoracoacromial vessels through the muscle to the parenchyma.

The thoracodorsal artery is not a major source of blood supply to the breast and the position of the implant will not affect it.

How well did you know this?
1
Not at all
2
3
4
5
Perfectly
121
Q

In augmentation mammaplasty, which of the following is the ideal upper pole to lower pole anatomic ratio?

A) 25:75
B) 35:65
C) 45:55
D) 50:50
E) 55:45

A

The correct response is Option C.

Studies have demonstrated the ideal anatomical characteristics of the breast to include: an upward pointing nipple, a straight or mildly concave upper pole slope, smooth lower pole convexity and fuller lower pole compared to upper pole. Breasts with an upper pole-to-lower pole ratio of 45:55 were identified as defining the ideal breast. The ratio was defined ideal by respondents including women, men, plastic surgeons, and individuals of ethnic diversity.

How well did you know this?
1
Not at all
2
3
4
5
Perfectly
122
Q

A 23-year-old woman undergoes augmentation mammaplasty with round, smooth silicone implants placed in the dual-plane position. Postoperatively, unilateral erythema and warmth are noted, and they slowly resolve over 10 days of oral antibiotic treatment. The patient asks what this might mean for future satisfaction with the outcomes. Which of the following is the most likely sequela of this patient’s clinical course?

A) Breast gland ptosis
B) Capsular contracture
C) Double-bubble appearance
D) Implant rupture
E) Nipple numbness

A

The correct response is Option B.

One of the most often mentioned potential risk factors for capsular contracture is biofilm, and this may be related to bacterial contamination. History of infection is unlikely to impact nipple sensation, implant rupture, true breast gland ptosis, or effacement of the inframammary fold with downward descent of the implant.

How well did you know this?
1
Not at all
2
3
4
5
Perfectly
123
Q

A 55-year-old postmenopausal woman desires improvement in the appearance of her breasts. Change in which of the following levels of hormones is most likely responsible for postmenopausal involution of breast tissue?

A) Estrogen
B) Growth hormone
C) Oxytocin
D) Prolactin
E) Testosterone

A

The correct response is Option A.

Estrogen is the primary hormone in promoting the development of the breast epithelium and ductal tissue. Progesterone acts in combination with estrogen to regulate breast development. With the onset of menopause, there is a decrease in the secretion of estrogen and progesterone. As a result of the decrease in the circulating levels of these hormones, the breast undergoes regression and atrophy of the glandular elements.

Oxytocin and prolactin are hormones involved in the physiology of lactation. Growth hormone and testosterone may have an effect on breast tissue, but they are not primary factors in the physiology of the female breast.

How well did you know this?
1
Not at all
2
3
4
5
Perfectly
124
Q

A 28-year-old woman is evaluated for micromastia. During consultation, she reports that her best friend underwent breast augmentation that was complicated by painful capsular contracture. Which of the following measures is most likely to prevent this complication in this patient?

A) Initiation of implant massage on postoperative day 5
B) Placement of a closed suction drain for prevention of postoperative hematoma
C) Use of a surgical support bra postoperatively for 2 weeks
D) Use of a subglandular, smooth, round implant via periareolar incision
E) Use of a subpectoral, textured implant via inframammary incision

A

The correct response is Option E.

Capsular contracture occurs when there is fibrosis of the peri-implant capsule. The severity is typically described by the Baker Grade classification.
Grade 1: the breast is soft and appears normal in size and shape
Grade 2: the breast is a little firm and appears normal
Grade 3: the breast is firm and appears abnormal
Grade 4: the breast is firm, appears abnormal, and is painful

Studies have shown a decreased relative risk for Baker grade 3-4 capsular contracture in primary breast augmentation associated with inframammary fold incision, textured implants, and subpectoral placement. The relative risk for capsular contracture was increased with periareolar or axillary incision, smooth implants, and subglandular placement. There is no evidence that wearing a support bra or implant massage will decrease the risk for capsular contracture. While hematoma is linked to capsular contracture, the presence of a drain does not prevent hematoma.

How well did you know this?
1
Not at all
2
3
4
5
Perfectly
125
Q

A 40-year-old man and his 80-year-old father are assaulted. They both have facial fractures. The older victim is more likely to have which of the following?

A) Decreased chance of noncraniofacial injuries
B) Higher mortality
C) Less severe injuries
D) Mandibular body fracture
E) Shorter hospital stay

A

The correct response is Option B.

In recent years many publications focused on craniofacial injury in the elderly as not only the mode of trauma differs compared with the younger population, but also the associated injuries and morbidities. In general, most related comorbidities in patients older than 60 to 65 (depending on the study) versus those younger are worse, including: longer hospital stays, need for assistance upon discharge, more severe injuries, likely to have noncraniofacial injuries like limb and spine fractures, and, of greatest concern, a much higher death rate. In a recent article though, Mundinger et al, showed that panfacial and mandible fractures were more common in the nongeriatric population, whereas mid face, orbital, and condylar fractures were more common in those older than 60 years of age.

How well did you know this?
1
Not at all
2
3
4
5
Perfectly
126
Q

A 28-year-old woman desires augmentation mammaplasty with silicone implants. Physical examination shows tuberous breast deformity with an elevated inframammary crease. Sternal notch to nipple distance is 21 cm bilaterally. Nipple to inframammary crease distance is 3.5 cm bilaterally. Periareolar mastopexy with 350-mL silicone implants is planned. Which of the following operative plans will most effectively minimize the likelihood of a double-bubble deformity?

A) Lower the inframammary crease by 3 cm
B) Perform radial release of the lower pole breast fascia
C) Place implants in subparenchymal pocket
D) Reinforce the inframammary crease with acellular dermal matrix
E) Use highly cohesive gel implants

A

The correct response is Option B.

The tuberous breast is a developmental deformity characterized by a constricted inframammary fold, short nipple to inframammary crease distance, and both horizontal and vertical deficiencies. The pathophysiology of the tuberous breast predisposes the patient to develop a double-bubble deformity. In this patient, the inframammary crease must be lowered to accommodate the implant and improve the vertical skin deficiency. Radial release of the lower pole breast fascia is done with either a cautery or a knife. Multiple radial incisions are made, thereby allowing the tight crease to expand and decrease the chance for a double-bubble deformity.

Lowering the crease is necessary but will increase the chances of a double-bubble deformity. Subparenchymal implant placement and use of highly cohesive gel implants may help but are not the essential procedures required. The use of acellular dermal matrix can help secure the position of the inframammary crease in a patient who develops a double-bubble deformity secondary to an inferior migration of the implant below the inframammary crease. This does not apply in the patient described.

How well did you know this?
1
Not at all
2
3
4
5
Perfectly
127
Q

A 44-year-old woman presents in evaluation for breast reconstruction with biopsy-proven left breast-infiltrating ductal carcinoma after routine mammography discovered a 7-cm lesion. She has been referred to medical oncology and genetic testing is pending. Her past medical history is significant for hypertension and scleroderma. On examination, she has grade I ptosis and wears a size 34A brassiere. During the consultation, the patient reports a strong preference for lumpectomy and oncoplastic reconstruction over total mastectomy. Which of the following is most likely to increase this patient’s chances of qualifying for breast-conserving therapy?

A) Active scleroderma
B) BRCA-1 gene mutation
C) Multicentric tumor
D) Preoperative chemotherapy
E) Small-sized breasts

A

The correct response is Option D.

Preoperative chemotherapy could increase this patient’s chances of qualifying for locoregional treatment (partial mastectomy or lumpectomy). Studies have shown that breast conservation rates are improved with preoperative systemic therapy, which can also render inoperable tumors resectable. Other potential benefits of preoperative (neoadjuvant) chemotherapy include providing important prognostic information based on response to therapy, minimizing the extent of axillary surgery, and allowing time for genetic testing and reconstructive planning prior to surgery. A small-sized breast would likely provide insufficient uninvolved breast tissue for breast-conserving therapy after resection of a large (7 cm) mass. The same applies to multicentric tumors.

Whole breast irradiation is strongly recommended after lumpectomy, with studies showing a favorable effect in reducing the 10-year risk of recurrence (19% versus 35%) and the 15-year risk of breast cancer death (21% versus 25%). Therefore, patients with (relative) contraindications to radiation therapy, such as lupus or scleroderma (connective tissue disease involving the skin), should ordinarily be offered total mastectomy, particularly if this resolves the need for radiation therapy. While radiation therapy would likely still be considered for this particular patient even after total mastectomy (tumor size greater than 5 cm), the diagnosis of scleroderma itself does not increase her chances of qualifying for breast conservation surgery. BRCA-1 gene mutation and other genetic predispositions to breast cancer are relative contraindications for breast-conserving therapy. These patients may be considered for prophylactic bilateral mastectomy for risk reduction.

How well did you know this?
1
Not at all
2
3
4
5
Perfectly
128
Q

A 6-year-old boy presents with a supracondylar fracture sustained during a fall on an outstretched hand. A splint with the elbow flexed less than 90 degrees is placed. The patient is screaming in pain. Examination shows the affected hand has a 3-second capillary refill. Which of the following is the most appropriate next step in management?

A) Closed reduction
B) Continued observation and application of ice packs
C) Elevation of the arm
D) Exploration of the brachial artery
E) Replacement of the current splint with an elbow extension splint

A

The correct response is Option A.

Supracondylar fractures are one of the most common traumatic fractures seen in children. It occurs most commonly in children 5 to 7 years of age with similar male and female incidence. The mechanism is usually from a fall onto an outstretched hand. The fracture can lead to severe forearm edema, then ischemia leading to Volkmann’s contracture.

Immobilization would be long arm casting with the elbow flexed at less than 90 degrees. Arm elevation would decrease tissue perfusion and would therefore be contraindicated. Immediate bedside closed reduction by gentle traction and elbow flexion to 20 to 40 degrees would be indicated in this case as a next step. If the closed reduction is unsuccessful or ischemia persists after reduction or recurs, urgent operative closed reduction with percutaneous pinning is required. Pins are placed to prevent recurrence. Brachial artery exploration could be required if ischemia has not resolved even after successful reduction, but not initially.

How well did you know this?
1
Not at all
2
3
4
5
Perfectly
129
Q

A 56-year-old man is evaluated because of gynecomastia. Physical examination shows mild, diffuse breast enlargement with no visible inframammary fold or ptosis. Which of the following is the most appropriate surgical correction of this patient’s condition?

A) En bloc resection of skin and breast tissue with free nipple grafting
B) Open excision of breast tissue with mastopexy
C) Subcutaneous mastectomy with nipple preservation
D) Suction-assisted lipectomy
E) Superior periareolar excision with skin excision

A

The correct response is Option D.

The treatment of gynecomastia is based on the degree of breast enlargement and the extent of ptosis that is noted on examination. Grade 1 gynecomastia is minimal breast hypertrophy without ptosis. Grade II gynecomastia is moderate hypertrophy without ptosis. Grade III gynecomastia is severe hypertrophy with moderate ptosis. Grade IV gynecomastia is severe hypertrophy with severe ptosis. The treatment of mild to moderate gynecomastia without ptosis is suction-assisted lipectomy. Direct periareolar excision with skin excision and subcutaneous mastectomy are not indicated for gynecomastia without ptosis. Mastopexy and free nipple grafting techniques are indicated for gynecomastia with severe ptosis.

How well did you know this?
1
Not at all
2
3
4
5
Perfectly
130
Q

A 23-year-old man is brought to the emergency department after twisting the long finger of his dominant right hand while playing basketball. Physical examination shows dorsal instability of the proximal interphalangeal (PIP) joint. X-ray study shows a volar buttress fracture involving 40% of the articular surface of the base of the middle phalanx. X-ray joint reduction is attained by passively flexing the PIP joint to 30 degrees. Which of the following is the most appropriate management? A ) Dynamic force-coupler external fixation
B ) Extension block splinting
C ) Hemi-hamate reconstruction of the volar buttress
D ) Open reduction and internal fixation of the fracture fragments
E ) Volar plate arthroplasty

A

The correct response is Option B.

If the PIP fracture/dislocation can be maintained stable with 30 degrees of flexion, then this is suitable to treat by extension block splinting. This will be the case with an approximate 40% volar articular fracture of the base of the middle phalanx. There is a risk of late flexion contracture if greater than 30 degrees of flexion is required to maintain PIP joint stability. More complex injuries may be treated with force-coupler dynamic splinting or with hemi-hamate reconstruction. For a larger fracture fragment with greater instability, open reduction and internal fixation may be required. Volar plate arthroplasty may be suitable with more chronic injuries and is limited to 60% of the articular surface.

How well did you know this?
1
Not at all
2
3
4
5
Perfectly
131
Q

A 54-year-old man comes to the office because of an injury to the long finger of the dominant right hand sustained when it was pinched in a machine at work. Physical examination shows a 1.3-cm loss of pulp tissue with no exposed bone. To preserve function and sensation in the digit, which of the following is the most appropriate intervention?

A) Amputation at the distal interphalangeal joint
B) Cross-finger flap
C) Moist dressings
D) Thenar flap
E) Volar V-to-Y advancement flap

A

The correct response is Option C.

Fingertip injuries are one of the most common problems encountered in hand surgery. The long finger is the most common finger involved. The patient’s age, occupation, and compliance with treatment should be considered when determining treatment. When possible, if the patient has no exposed bone or only a small area of exposed bone, treatment with dressing changes offers excellent results. There is no donor site morbidity, scarring is often minimal, and return of sensation is generally excellent. Patients, however, need to be cautioned that a prolonged period of dressing changes is required, often lasting 3 to 6 weeks.

Amputation at the distal interphalangeal joint would result in loss of function of the profundus tendon and grip weakness. Neurovascular island flaps and V-to-Y advancement flaps offer excellent closure options when digital length needs to be preserved and there is significant exposure of bone. However, with these flaps there is a donor defect and decreased sensation. Care must be taken when using a cross-finger flap or thenar flap in older patients to avoid contractures and stiffness of the digits.

References

Panattoni JB, De Ona IR, Ahmed MM. Reconstruction of fingertip injuries: surgical tips and avoiding complications. J Hand Surg Am. 2015 May;40(5):1016-24.

Krauss EM, Lalonde DH. Secondary healing of fingertip amputations: a review. Hand (N Y). 2014 Sep;9(3):282-8

How well did you know this?
1
Not at all
2
3
4
5
Perfectly
132
Q

A 40-year-old woman who underwent a subglandular augmentation mammaplasty with smooth round silicone breast implants 5 years ago returns to the office for evaluation of an increasingly firm left breast. Surgical revision of the left breast is planned. Which of the following measures is most likely to decrease the recurrence of the symptoms?

A) Conversion to a new plane or pocket
B) Performing a total capsulectomy
C) Using botulinum toxin type A in and around the implant pocket
D) Using fat grafting in and around the implant pocket

A

The correct response is Option A.

Site change and implant exchange are the only factors that have consistently been shown to decrease recurrence of capsular contracture, although other factors including use of a textured implant and fat grafting used are in augmentation mammoplasty revision. Botulinum toxin type A has been described for prevention of capsular contracture however; no consensus that these treatments decrease recurrence of capsular contracture exists.

Furthermore, there are no data to support performing total versus partial capsulectomy, or even the superiority of capsulectomy over capsulotomy.

How well did you know this?
1
Not at all
2
3
4
5
Perfectly
133
Q

A 62-year-old woman undergoes breast reconstruction using autologous tissue from the abdomen. Intraoperatively, use of a perforator flap is found to be impossible because of multiple small nondominant perforators. Conversion to a delayed pedicled transverse rectus abdominis muscle flap is planned. Ligating which of the following vessels in this stage will best facilitate future viability of the tissue transferred in the next stage?

A) Deep inferior epigastric
B) Hypogastric
C) Internal mammary
D) Superficial inferior epigastric
E) Superior epigastric

A

The correct response is Option A.

Pedicled transverse rectus abdominis muscle (TRAM) flaps are based on the superior epigastric system, which is often less robust than the deep inferior epigastric system. Therefore, surgical delay by ligation of the deep inferior epigastric system may facilitate overall viability of the transferred tissue. Ligation of the superior epigastric system would make a pedicled TRAM flap unlikely to survive. The internal mammary ligation may also interrupt blood supply to the superior epigastric system, and even if the tissue is fed through collaterals, it would not strengthen the flap. Division of the superficial inferior epigastric system might also help, but it is not as critical as ligation of the deep inferior epigastric system. The hypogastric system does not have a direct impact on the pedicled TRAM tissues.

How well did you know this?
1
Not at all
2
3
4
5
Perfectly
134
Q

A 36-year-old woman, gravida 3, para 3, comes to the physician because she desires larger breasts. She has breast-fed three children. Physical examination shows grade 3 ptosis and loss of superior pole volume. The distance from nipple to sternal notch is 26 cm. Result of upper pole pinch test is 1.5 cm. A dual-plane augmentation/mastopexy is planned. Which of the following is the strongest indicator for subpectoral placement of the implant in this patient?

A) Concurrent mastopexy
B) Grade 3 ptosis
C) Loss of superior pole volume
D) Nipple to sternal notch distance of 26 cm
E) Pinch test result of 1.5 cm

A

The correct response is Option E.

A dual-plane approach is subpectoral in the superior pole and subglandular in the inferior pole. This is to afford more subcutaneous coverage in the superior pole. Tebbetts recommends pinching the skin and subcutaneous tissues of the superior pole for a “pinch test.” For thickness less than 2 cm, he recommends a dual-plane placement for adequate soft tissue coverage. This is not affected by the grade of ptosis, need for mastopexy, history of loss of superior pole volume, or nipple to notch distance.

How well did you know this?
1
Not at all
2
3
4
5
Perfectly
135
Q

A 52-year-old man presents with a chronic ulcer of the lower extremity. Current medications include prednisone for management of rheumatoid arthritis. In addition to standard local wound care, which of the following treatments is most appropriate?

A) Folate
B) Hyperbaric oxygen therapy (HBOT)
C) Long-acting insulin
D) Vitamin A
E) Vitamin C

A

The correct response is Option D.

Malnutrition is a well-established risk factor for the development of chronic wounds. Vitamin A has been shown in multiple studies to offset the detrimental effects of corticosteroids on wound healing.

Appropriate glucose management is critical to the treatment of diabetic ulcers, but insulin would not be indicated in the absence of uncontrolled diabetes mellitus.

Vitamin C is required as a cosubstrate for enzymes involved in collagen production, and its supplementation is recommended for the nutritionally deficient. However, vitamin C has not been shown to be of any benefit to wound healing in the setting of chronic corticosteroid therapy per se.

Hyperbaric oxygen therapy delivers oxygen to tissues by both hemoglobin-dependent transport and vastly increased dissolved oxygen content in blood when a patient breathes 100% oxygen at pressures well above atmospheric level. This improves wound healing by multiple cellular mechanisms in select wounds. However, benefits in treatment of chronic corticosteroid use have not been demonstrated.

Elevated serum homocysteine has been associated in multiple studies with impaired wound healing and increased risk of coronary and cerebrovascular disease due to its enhancement of clotting pathways. Folate supplementation is often used to treat hyperhomocysteinemia, but conclusive benefit in chronic wounds is not as well-established.

How well did you know this?
1
Not at all
2
3
4
5
Perfectly
136
Q

A 32-year-old man comes to the emergency department after a motorcycle collision. Examination and x-ray studies show an isolated injury to the left wrist consistent with a perilunate dislocation. In perilunate dislocations, dislocation of which of the following is the initial injury that leads to lunate dislocation?

A) Dorsal carpal ligaments
B) Lunocapitate junction
C) Lunotriquetral ligaments
D) Scapholunate ligament
E) Triangular fibrocartilage complex

A

The correct response is Option D.

All the other answers are incorrect due to incorrect sequence of force transmission across the wrist. Furthermore B and E are wrong due to incorrect mechanism, as well.

Wagner and Mayfield conducted classic studies on carpal dynamics and anatomy to determine the progression of stresses across the wrist in severe hyperextension injuries. They determined that there is a reliable and predictable pattern to these injuries, which is described as Progressive Perilunate Instability (PLI). There are four stages of PLI, corresponding to the degree of stress applied in the injury. The mildest form is the isolated scapholunate dissociation: PLI stage 1. As the forces continue in an ulnar and distal direction, the distal row and scaphoid progress dorsally, and the capitate separates from the lunate: PLI stage 2. As the force continues in an ulnar direction, the lunotriquetral ligaments separate, and if the lunate is still in place, this is the full Midcarpal Dislocation: PLI stage 3. Finally, in the most severe cases, the dorsally dislocated capitate will dislodge the lunate and push it volarly, creating the true lunate dislocation: PLI stage 4.

How well did you know this?
1
Not at all
2
3
4
5
Perfectly
137
Q

A 47-year-old woman, gravida 3, para 3, is evaluated for improvement of breast appearance. She breast-fed all three of her children for 1 year each. Examination shows the distance from nipple to sternal notch is 27 cm bilaterally; decreased superior pole volume, and striae are also noted. There is Grade 3 ptosis bilaterally. The pinch of the superior pole soft tissue is 1 cm. Which of the following procedures is most likely to improve superior pole volume and breast shape in this patient?

A) Dual-plane implant augmentation
B) Mastopexy with dual-plane implant augmentation
C) Mastopexy with subglandular implant augmentation
D) Subglandular implant augmentation
E) Vertical mastopexy

A

The correct response is Option B.

Goals of improvement would be upper pole fullness and a coned, rounded breast, with raising the nipple. Because the superior pole thickness is less than 2 cm, a subglandular implant is not recommended. A dual-plane implant would not address the ptosis and would likely leave persistent ptosis. Vertical mastopexy alone would require some modification to address the excess vertical skin with some element of horizontal inferior excision. This would not address the lack of upper pole volume in the long term. The striae indicate poor tissue strength. Staged implant placement would have the fewest risks.

How well did you know this?
1
Not at all
2
3
4
5
Perfectly
138
Q

A 10-year-old girl is brought to the office by her mother because of difficulty using her hand. Medical history includes supracondylar fracture 6 months ago treated with a closed reduction and casting. The fingers of the affected hand are held in the intrinsic minus position. Volkmann ischemic contracture following the fracture is suspected. Which of the following muscles is LEAST likely to be affected by Volkmann contracture?

A) Brachioradialis
B) Flexor digitorum profundus
C) Flexor digitorum superficialis
D) Flexor pollicis longus
E) Pronator teres

A

The correct response is Option A.

Volkmann ischemic contracture results from forearm muscle shortening and fibrosis as a result of ischemia of forearm muscles during increased compartment pressures. Common reasons for increased compartment pressures include gunshot wounds and fractures, particularly supracondylar pediatric fractures. The radial artery is superficially located, whereas the ulnar artery is deeply positioned, traversing deep to the pronator teres muscles. The ulnar artery becomes the common interosseous artery, which divides immediately into anterior and posterior interosseous branches. The muscles dependent on this deep circulatory pattern are more likely to be affected by ischemia during increased compartment pressures. Flexor muscles commonly involved in this process are the flexor digitorum superficialis, flexor digitorum profundus, flexor pollicis longus, and pronator teres. The brachioradialis is not typically affected due to its more superficial radial artery circulation. Patients with significant functional loss may require surgical procedures such as a free gracilis functioning muscle transfer.

References

Fischer JP, Elliott RM, Kozin SH, Levin LS. Free function muscle transfers for upper extremity reconstruction: a review of indications, techniques, and outcomes. J Hand Surg Am. 2013;38(12):2485-2490.

Harris IE. Supracondylar fractures of the humerus in children. Orthopedics. 1992;15(7):811-817.

Stevanovic M, Sharpe F. Management of established Volkmann’s contracture of the forearm in children. Hand Clin. 2006;22(1):99-111.

Zuker RM, Bezuhly M, Manktelow RT. Selective fascicular coaptation of free functioning gracilis transfer for restoration of independent thumb and finger flexion following Volkmann ischemic contracture.J Reconstr Microsurg. 2011 Sep. 27(7):439-44.

Brahmamdam P1, Plummer M, Modrall JG, Megison SM, Clagett GP, Valentine RJ. Hand ischemia associated with elbow trauma in children. J Vasc Surg. 2011 Sep;54(3):773-8.

How well did you know this?
1
Not at all
2
3
4
5
Perfectly
139
Q

A 63-year-old man undergoes microdermabrasion for scar irregularity following treatment of facial skin cancer. Which of the following intraoperative findings signals the endpoint of treatment?

A) Cobblestoned, yellow adiposity
B) Confluent patches of bleeding lakes on a yellowish background
C) Minimal dermis with visible subdermal plexus
D) Pinpoint, punctate bleeding on a white background
E) Thinned but present epidermis

A

The correct response is Option D.

When performing microdermabrasion, the most appropriate endpoint is removal of soft tissue into the papillary dermis. The appropriate level in the papillary dermis is visualized as pinpoint bleeding in a white dermal background. Epithelial removal would be insufficient. Removal of tissue into the reticular dermis, either superficial or deep, increases the risk for scarring. The superficial reticular dermis demonstrates a yellowish white coloration and would reinforce the need to discontinue treatment before deeper injury occurs. Furthermore, dermal excisional depth would manifest as confluent red bleeding in a yellowish background of dermis or nearly complete dermal removal with a visible subdermal vascular plexus. If a full-thickness skin removal is performed (into the subcutaneous adipose layer), significant deformity could result. Preservation of adnexal structures to allow reepithelialization is a key tenet in determining the appropriate depth of treatment. Posttreatment changes include thickened epidermis and increased elastin and collagen.

How well did you know this?
1
Not at all
2
3
4
5
Perfectly
140
Q

A 45-year-old woman who underwent bilateral breast augmentation mammaplasty returns to the office after a motor vehicle accident with deployment of airbags. Physical examination shows point tenderness over the chest with an obvious “seat belt” sign along the left breast. The immediate diagnostic workup of the implants shows no rupture. Six months later, the patient returns with distortion of the left breast over the implant with a cleft formation. Which of the following is the most appropriate next step in assessing the integrity of the implants?

A) Chest x-ray study
B) CT scan with intravenous contrast
C) Mammography
D) MRI
E) Ultrasound

A

The correct response is Option D.

The patient has a late presentation of seat belt syndrome, which requires an MRI to assess the integrity of the breast implants. Patients with seat belt syndrome may present with a cleft or a mass. It is important to rule out invasive ductal carcinoma located in the line of the diagonal contracture. An intracapsular seroma can form gradually over time as well. Reconstructive options include unilateral capsulectomy and implant exchange.

Chest x-ray study can be used in a more immediate setting to rule out any bony injury to the chest wall. Mammography is an appropriate choice in patients who have a palpable mass in an initial assessment; however, this does not rule out implant rupture. In the event that there is an expanding breast in the immediate setting, CT scan with intravenous contrast can be used to rule out possible arterial extravasation or pneumothorax. Ultrasound can be used to assess implant or capsule rupture, but MRI is the most definitive investigative study.

How well did you know this?
1
Not at all
2
3
4
5
Perfectly
141
Q

A 52-year-old woman undergoes autologous breast reconstruction with unilateral deep inferior epigastric perforator (DIEP) flaps. According to the Hartrampf model of perfusion zones, if the lateral row perforator vessels are used, in which chronological order will the flap zones be perfused?

A) I – II – III – IV
B) I – III – II – IV
C) II – I – III – IV
D) II – I – IV – III
E) IV – III – II – I

A

The correct response is Option B.

In medial perforator-based flaps, the zones are perfused in the order I – II – III – IV (A) as shown in the image. In lateral perforator-based flaps, however, the zones are perfused in the order I – III – II – IV (B).

How well did you know this?
1
Not at all
2
3
4
5
Perfectly
142
Q

After a nipple-sparing mastectomy, which of the following branches of an intercostal nerve predominantly provides remaining sensation to the nipple-areola complex?

A) Anterior branch of the fifth
B) Anterior branch of the fourth
C) Lateral branch of the fifth
D) Lateral branch of the fourth
E) Lateral branch of the third

A

The correct response is Option B.

The cutaneous innervation of the female breast is derived medially from the anterior cutaneous branches of the first to sixth intercostal nerves and laterally from the lateral cutaneous branches of the second to seventh intercostal nerves. The nipple-areola complex is physiologically innervated by the lateral and anterior branches of the third to fifth intercostal nerves. The fourth intercostal nerve has further shown to be most consistent in various anatomical studies.

However, the anterior branches take a superficial course within the subcutaneous tissues of the medial breast while the lateral branches take a deep course within the pectoral fascia and reach the nipple via the breast parenchyma. Therefore, the lateral branches are most likely resected during mastectomy and contribute little to the postoperative innervation of the nipple-areola complex.

How well did you know this?
1
Not at all
2
3
4
5
Perfectly
143
Q

A 23-year-old woman with micromastia and bilaterally tuberous breasts comes to the office to discuss augmentation mammaplasty and improving the overall appearance of her breasts. Physical examination shows bilateral mildly ptotic breasts with glandular tissue herniating through the nipple-areola complex. The lower pole appears mildly deficient in the lower medial and lateral quadrants, and the distance from the nipple to the inframammary crease is 5.5 cm on stretch. Which of the following is the most appropriate operative approach for this patient?

A) Implant placement with circumareolar mastopexy
B) Implant placement without mastopexy
C) Implant placement with vertical mastopexy
D) Implant placement with Wise-pattern mastopexy
E) Two-stage reconstruction with tissue expander, followed by placement of a permanent implant

A

The correct response is Option A.

The tuberous breast deformity was first described by Aston and Rees in 1976. While most of the surgical approaches listed, with the exception of implants together with Wise-pattern mastopexy, have been described for the spectrum of tuberous breast deformities, the key is to select the right procedure for the right patient. In this case, a mild form of the deformity is described. Implant placement alone, even with parenchymal scoring and lowering of the inframammary crease, is unlikely to correct the deformity of the nipple-areola complex. In cases of severe ptosis, vertical mastopexy may be used but would be unnecessary in this patient with mild ptosis. In severely deficient cases, a two-stage approach with tissue expansion may be necessary, but it would be over-operating in this mildly deficient patient. Recently, fat grafting has also been advocated for this procedure.

In the case described, which is a common presentation, a periareolar approach is typically used to place the implant in a dual-plane configuration. Subglandular placement is also described. The inframammary crease is commonly adjusted downward. Radial scoring of the parenchyma and a circumareolar mastopexy are typically performed.

In the recent review by Kolker and Collins, 92% of tuberous breast patients had a one-stage procedure. Ninety-six percent of these were treated with implant placement and circumareolar mastopexy, combined with inframammary crease adjustment and radial scoring of the parenchyma.

How well did you know this?
1
Not at all
2
3
4
5
Perfectly
144
Q

A 40-year-old woman comes to the office for consultation on an augmentation mastopexy 2 years after giving birth to her second child. She is back to her pre-pregnancy weight. Physical examination shows involutional changes contributing to a deflated appearance of the breasts. This appearance is most likely due to a histologic decrease in which of the following?

A) Area composed of stromal matrix
B) Number of differentiated lobules
C) Thickness of dermis
D) Thickness of pectoralis muscle
E) Volume of adipose tissue

A

The correct response is Option B.

Postpartum involutional changes can manifest clinically as breasts that appear deflated, commonly due to a loss of volume and skin that has been stretched. On a histologic level, these clinical manifestations occur due to a decrease in the number and area of differentiated lobules that were enlarged and specialized for milk production. As this occurs, it is hypothesized that the lobular area is then replaced by stromal matrix and eventually fat. Involutional changes do not refer to changes in the dermis, pectoralis muscle or chest wall structures.

How well did you know this?
1
Not at all
2
3
4
5
Perfectly
145
Q

A 19-year-old woman with a medical history significant for Poland syndrome and a BMI of 19 kg/m2 undergoes first-stage breast reconstruction with a tissue expander that is complicated by extrusion and infection 40 days after implantation. A photograph is shown. Attempts at implant salvage are made. The presence of which of the following factors is most likely to lead to decreased salvage rates?

A) BMI of 19 kg/m2
B) Culture-positive Staphylococcus sp
C) Hemoglobin A1c of 6.5%
D) Prepectoral placement of the device
E) Use of acellular dermal matrix

A

The correct response is Option B.

It has been shown that successful breast device salvage in breast reconstruction is possible if caught early. However, there are associated factors with failure, including culture-positive Staphylococcus (epidermidis or aureus), as demonstrated by several studies. Other associated risk factors for failure include obesity, poorly-controlled diabetes, smoking, history of radiation therapy, postoperative seroma, and early contamination of the implant with biofilm formation. Therefore, prompt and aggressive intervention is warranted in these situations where the device is threatened by either infection and/or exposure. This includes both surgical and antimicrobial options.

In a 2017 study, prepectoral and subpectoral placement demonstrated comparable complications. Acellular dermal matrix did not increase failure rates.

How well did you know this?
1
Not at all
2
3
4
5
Perfectly
146
Q

An 11-year-old boy is brought to the emergency department after sustaining an injury to the tip of the left long finger while playing baseball. The long fingertip is held in a flexed position. The proximal nail plate is slightly visible superficial to the eponychial fold, and a small subungual hematoma is noted. An x-ray study is shown. Which of the following is the most appropriate management?

A) Closed reduction and distal interphalangeal joint extension splinting for 6 to 8 weeks
B) Closed reduction, removal of nail plate, and percutaneous pinning
C) Open reduction, repair of nail bed, and Kirschner wire fixation
D) Repair of zone I flexor tendon avulsion
E) Suture repair of lacerations and observation for 6 weeks

A

The correct response is Option C.

This patient has a Seymour fracture, or an open physeal/juxta-epiphyseal fracture of the distal phalanx. These injuries present as mallet-like injuries, but they are open fractures by definition. The treatment of choice is open reduction, debridement of the fracture site, repair of associated nail bed laceration, and Kirschner wire fixation of the fracture across the distal interphalangeal (DIP) joint. The injury is secondary to hyperflexion and is essentially a Salter I or Salter II fracture of the distal phalanx. The flexed posture occurs because the terminal extensor tendon inserts on the proximal dorsal epiphysis, while the flexor digitorum profundus inserts on the metaphysis of the bone distal to the fracture site. These injuries can often be mistaken for mallet injuries or DIP dislocations. Often a flap of nail bed matrix becomes interposed between the fracture fragments, preventing closed reduction. This must be repaired. It is recommended to not discard the nail plate, because it helps maintain bone reduction. Dorsal physeal widening and flexion of the distal fracture fragment is seen on lateral x-ray studies.

Observation alone for a Seymour fracture, mallet finger, or DIP dislocation is inappropriate.

Closed reduction and DIP extension splinting is generally the treatment of choice for mallet-type injuries. These can be purely soft-tissue mallet fingers or osseous mallet fractures. Initial treatment should include closed reduction by extension or hyperextension and splinting that isolates the DIP joint in extension. Generally, the splint is worn for 6 to 8 weeks continuously, and then for a period of time at night as indicated. Compliance may be an issue with the pediatric or adolescent population, and the patient should be seen regularly to assess splint fit and skin integrity.

Closed reduction and pinning may be necessary for osseous mallet fingers with associated volar subluxation of the distal phalanx. Several techniques have been described, but this is not appropriate for a Seymour fracture. The nail bed tissue must be reduced from the fracture site to achieve anatomic reduction in this case. Also, closed treatment has an unacceptable incidence of infection, residual finger deformity from incomplete reduction, and nail deformity.

A zone I flexor tendon avulsion, or jersey finger, would present with the DIP joint held in extension. This is opposite to the presentation of this patient. In that case, the patient would require open exploration and reinsertion of the profundus tendon to the distal phalanx via bone tunnels or suture anchor.

How well did you know this?
1
Not at all
2
3
4
5
Perfectly
147
Q

A 25-year-old man is brought to the emergency department after falling down a staircase and landing on his outstretched hand. X-ray studies show a perilunate dislocation. The scapholunate and which of the following ligaments must be ruptured for this dislocation to occur?

A ) Dorsal intercarpal

B ) Dorsal radiotriquetral

C ) Lunotriquetral

D ) Radioscaphocapitate

E ) Ulnotriquetral

A

The correct response is Option C.

Perilunate dislocation is the most common form of carpal dislocation. There is disruption between the ligamentous connections of the lunate and other carpal bones and radius. In the scenario described, rupture of the scapholunate and lunotriquetral ligaments is the most likely cause of the dislocation. These ligaments are usually repaired, followed by open reduction and internal fixation.

The dorsal intercarpal, dorsal radiotriquetral, radioscaphocapitate, and ulnotriquetral ligaments are not appropriate choices because they do not connect to the lunate.

How well did you know this?
1
Not at all
2
3
4
5
Perfectly
148
Q

A 36-year-old woman is evaluated because of spontaneous galactorrhea 6 days after undergoing augmentation mammaplasty. Which of the following factors most likely contributed to this outcome?

A) Inframammary placement of the incision
B) Subglandular versus dual-plane position of the device
C) Surgical interruption of the intercostal nerves
D) Use of silicone versus saline breast implants

A

The correct response is Option C.

Although no one knows exactly what leads to postoperative galactorrhea, it is observed to occur more often in parous women and theorized to occur due to a combination of factors which simulate suckling or change in the innervation of the chest wall and nipple-areola complex. This would include increased tissue pressure related to the implant placement and interruption of intercostal nerves. No relationship has been identified between incision placement (peri-areolar, inframammary, transaxillary, or even peri-thelial) and postoperative galactorrhea. Similarly no relationship has been identified between device positioning (dual-plane, subglandular, and submuscular) and postoperative galactorrhea. Again, no relationship has been observed in implant type, saline versus silicone, and postoperative galactorrhea.

How well did you know this?
1
Not at all
2
3
4
5
Perfectly
149
Q

A 38-year-old woman reports decreased areola sensitivity after undergoing mastopexy. Intraoperative injury to which of the following nerves is the most likely cause of this patient’s reduced sensitivity?

A) Intercostobrachial nerve
B) Lateral cutaneous branch of the fourth intercostal nerve
C) Lateral cutaneous branch of the sixth intercostal nerve
D) Medial cutaneous branch of the fifth intercostal nerve
E) Medial cutaneous branch of the third intercostal nerve

A

The correct response is Option B.

The lateral cutaneous branch of the fourth intercostal nerve is most commonly responsible for nipple and areola sensitivity. The other intercostal nerve branches listed do contribute to breast sensitivity but are less often thought to be the primary innervation to the nipple and areola. The intercostobrachial nerve supplies innervation to the upper medial arm.

How well did you know this?
1
Not at all
2
3
4
5
Perfectly
150
Q

In women undergoing prosthetic breast reconstruction complicated by an expander/implant infection, which of the following is the most common gram-negative bacteria isolated from cultures?

A) Escherichia coli
B) Klebsiella
C) Proteus
D) Pseudomonas
E) Serratia

A

The correct response is Option D.

Tissue expander/implant-based breast reconstruction remains the most common form of reconstruction after mastectomy. One of the most potentially devastating complications of this form of breast reconstruction is an implant infection with need for removal of the expander/implant. The mean reported incidence of implant infection after breast reconstruction is 8%, with a range of 1 to 35%. When cultures are obtained, the most common causative bacteria on microbiology examination are gram-positive organisms (41 to 83%), specifically, Staphylococcus species (56 to 76.5%). Gram-negative bacteria accounted for 15.3 to 28.6%, with Pseudomonas (10.7 to 14%) being the most common gram-negative bacteria present on microbiology examination.

How well did you know this?
1
Not at all
2
3
4
5
Perfectly
151
Q

A 28-year-old woman comes to the office to discuss augmentation mammaplasty. She is interested in silicone implants, specifically highly cohesive gel shaped implants. Which of the following is the most likely result of increasing the cross-linking in these implants?

A) Decreased risk of gel fracture
B) Decreased risk of shell delamination
C) Improved form stability
D) Increased risk of folds
E) Softer implants

A

The correct response is Option C.

Increasing the cross-linking in a highly cohesive gel shaped silicone implant improves form stability. This allows for the creation of shaped implant designs that persist despite position or external forces on the implant.

The current, fifth-generation silicone breast implants derive their cohesiveness from the cross-linking of the silicone. Increasing the amount of cross-linking leads to an increase in cohesiveness and a firmer implant. This may lead to less rippling and folding because of resistance to collapse; however, recent MRI studies have shown folds and distortions are still possible. Increasing cohesiveness, however, does have some disadvantages with potential risks for gel fracture and delamination of the implant shell.

How well did you know this?
1
Not at all
2
3
4
5
Perfectly
152
Q

A 24-year-old woman comes to the office 8 months after undergoing a circumareolar mastopexy/augmentation. She is concerned because her areolas are now asymmetric. They were symmetric preoperatively. Physical examination shows that the right areola diameter is 7 cm and the left areola diameter is 4 cm. The most likely cause of this asymmetry is a failure of which of the following?

A)Breast pillar approximation
B)Periareolar de-epithelialization
C)Prosthesis pocket
D)Purse-string suture
E)Skin envelope tailor tacking

A

The correct response is Option D.

The most likely cause of nipple-areola asymmetry in the patient described is failure in the purse-string suture. Periareolar mastopexy/augmentation has been plagued with inconsistent control of the nipple-areola complex diameter. This mastopexy technique creates concentric resection of periareolar epithelium to elevate the nipple-areola complex and reduce the skin envelope. The etiology of this areola-spreading is the tension of the closure intrinsic to the technique. Use of a permanent suture for the purse-string helps limit the postoperative spreading of the areolar diameter. Introduction of the interlocking polytetrafluoroethylene (GORE-TEX) suture has allowed improved control of areolar shape and diameter. If one of the purse-string sutures breaks or pulls through its dermal attachments, that areola will be subject to the forces of tension and expand in diameter. In the patient described, operative correction involves either replacing the purse-string on the widened side or removing the purse-string on the smaller diameter areola.

Periareolar de-epithelialization is the cause of the tension and is an essential part of the procedure. In patients who are significantly asymmetric, tension of the areolas will also be asymmetric; however, a permanent purse-string suture is crucial in these cases.

Prosthesis pocket and parenchyma shaping sutures will not have the impact on areolar diameter that is described in this scenario.

Envelope tailor tacking relates to final adjustments in periareolar de-epithelialization.

How well did you know this?
1
Not at all
2
3
4
5
Perfectly
153
Q

An otherwise healthy 54-year-old perimenopausal woman is scheduled for a mastectomy for biopsy-proven right-sided grade 2 ductal carcinoma. According to the National Comprehensive Cancer Network (NCCN) guidelines, postmastectomy radiation therapy will be the standard of care for this patient if she has which of the following surgical outcomes?

A) 1-cm surgical margins, four positive axillary lymph nodes
B) 1-cm surgical margins, one positive axillary sentinel node
C) 1-mm surgical margins, no positive axillary nodes
D) 5-mm surgical margins, no positive axillary nodes
E) 5-mm surgical margins, three positive axillary nodes

A

The correct response is Option A.

Traditionally, the need for radiation therapy has been a contraindication for implant-based reconstruction, and autologous reconstruction is the conservative gold standard for women with advanced cancer needing postmastectomy radiation. More recently, there have been reports of successful implant based reconstruction in the setting of postmastectomy radiation that have similar complication profiles and good oncologic outcomes compared with autologous reconstruction. Protocols vary between those that radiate the expander and then expand, and those that expand and then radiate the permanent implant. Being able to anticipate which patient will require postmastectomy radiation is essential for joint decision making about breast reconstruction with the patient prior to her mastectomy.

By National Comprehensive Cancer Network (NCCN) guidelines, relative indications for postmastectomy radiation therapy include: positive sentinel node with unknown status of other axillary nodes, one to three positive nodes on permanent histology, and close surgical margins (less than 5 mm). Postmastectomy radiation is recommended as the standard of care in the situations of positive surgical margins with the inability to get clear margins and four or more positive lymph nodes.

How well did you know this?
1
Not at all
2
3
4
5
Perfectly
154
Q

Which of the following characteristics is correlated with increased risk of nipple-areola complex necrosis in nipple-sparing mastectomies with immediate reconstruction?

A) Autologous tissue reconstruction
B) Direct to implant reconstruction
C) Patient age
D) Periareolar incision
E) Small breast size

A

The correct response is Option D.

Nipple-sparing mastectomies (NSMs) are becoming more common for both therapeutic and prophylactic mastectomies. Nipple-areola complex (NAC) necrosis can imperil reconstructive efforts, as well as negatively affect patients emotionally. It is important to maximize perfusion to the mastectomy skin flaps and NAC while still performing an oncologically sound procedure. There are multiple different incisions for performing NSM. Periareolar, inframammary-fold, radial, and vertical incisions are the most common. Periareolar incisions are associated with an increased risk of NAC necrosis in NSMs. Type of reconstruction, small breast size, and patient age have not been shown to be linked to increased rates of NAC necrosis.

How well did you know this?
1
Not at all
2
3
4
5
Perfectly
155
Q

A 35-year-old woman with tuberous breast deformity is scheduled to undergo augmentation/mastopexy. A smooth, round, cohesive gel implant will be used. This patient is at higher risk for which of the following complications when compared with augmentation/mastopexy performed on a patient without a tuberous breast?

A) Capsule contracture
B) Double bubble
C) Hematoma
D) Nipple-areola depigmentation
E) Rippling

A

The correct response is Option B.

The classic features of a tuberous breast deformity include a constricted base with a high inframammary crease and herniation of breast parenchyma into the nipple-areola complex producing a large-diameter areola. Variable extent of micromastia is associated as well as breast asymmetry. When a patient has a high and tight inframammary crease, this crease must be released to accommodate an implant and allow correction of the deformity. If this native crease does not fully expand, then a double bubble will occur. Over time, the lower pole skin stretches in response to the implant and this double bubble often improves spontaneously. The incidence of capsule contracture, hematoma, nipple-areola depigmentation, and rippling should be similar to a patient who undergoes periareolar augmentation/mastopexy without a tuberous breast.

How well did you know this?
1
Not at all
2
3
4
5
Perfectly
156
Q

A 60-year-old woman comes to the office because of a 2-year history of disabling pain in the carpometacarpal joint of the thumb of the nondominant left hand. Physical examination shows swelling and tenderness. Grind test results are positive. X-ray study shows osteoarthritis with subluxation of the joint. Which of the following types of biomaterial is most appropriate for arthroplasty?

A ) Expanded polytetrafluoroethylene (GORE-TEX)

B ) Polypropylene (Marlex)

C ) Polyurethaneurea (Artelon)

D ) Porcine dermal collagen xenograft (Permacol)

E ) Silicone trapezial implant arthroplasty

A

The correct response is Option C.

Polyurethaneurea (Artelon) implants typically biodegrade by hydrolysis and are described as causing minimal giant cell and foreign body reaction. Although long-term studies are still pending, they currently appear to be the best biomaterial for this application.

The use of silicone trapezial implants was, at one time, a common procedure, but the long-term results of silicone arthroplasty wear and deformation led to multinucleated giant cell reactions with silicone granulomas and synovitis. This resulted in about a 25% failure rate with this modality.

Similarly, a study by Greenberg, et al, showed an incidence of 80% osteolysis and a high failure rate with GORE-TEX implants. A biopsy specimen of one retrieved implant showed giant cell reactions. Marlex implants also showed foreign body reactions and synovitis. A study by Belcher, et al, on Permacol implants was terminated prematurely because of adverse reactions to the implant in 6 of 13 patients, with significant pain and evidence of multinucleated giant cells.

How well did you know this?
1
Not at all
2
3
4
5
Perfectly
157
Q

Which of the following is the arterial supply of the flap for digital tip reconstruction shown?

A) Distally based from the contralateral digital artery
B) Dorsal perforating vessels
C) Perineural perforating vessels
D) Proximally based from the ipsilateral digital artery

A

The correct response is Option A.

A reverse homodigital island flap is shown. It is a distally based flap that is useful in the repair of fingertip injuries. Arterial inflow is based upon the contralateral digital artery in the crossing ladder vessels of the palmar digital arch that lie just dorsal to the volar plate at each joint. The flap requires sacrifice of the ipsilateral digital artery and care must be taken to preserve the digital nerve during elevation of the skin paddle. Typically, the donor site is grafted.

References

Regmi S, Gu JX, Zhang NC, Liu HJ. A Systematic Review of Outcomes and Complications of Primary Fingertip Reconstruction Using Reverse-Flow Homodigital Island Flaps. Aesthetic Plast Surg. 2016 Apr;40(2):277-83.

Foucher G and Khouri RK: Digital reconstruction with island flaps. Clin Plast Surg. 1997 Jan;24(1):1-32.

158
Q

A 39-year-old woman with a history of fibrocystic breast lesions comes to the office for consultation. She has no family history of breast cancer. Results of routine mammograms have been negative; she has never undergone biopsy. Which of the following is the most appropriate recommendation for this patient regarding managing her risk of breast cancer?

A) Continue to schedule routine mammograms
B) Refer for mastectomy
C) Schedule core needle biopsy
D) Schedule fine-needle aspiration
E) Schedule genetic testing

A

The correct response is Option A.

Studies have shown the fibrocystic changes alone in the breast are not directly linked to an increased risk of breast cancer, so there is no indication for fine-needle aspiration or core biopsy. Cancer risk increases in benign breast disease with increased proliferation and atypical hyperplasia. Even with no family history of breast cancer, it is recommended that the patient continue routine mammograms. Fibrocystic breast disease has not been linked in the literature to an increased risk of mutations of the BRCA genes; therefore, genetic testing is unnecessary.

159
Q

A 37-year-old man who works as a laborer sustains a saw injury to the non-dominant left hand. X-ray studies are shown. Which of the following is the most appropriate functional option for reconstruction of this thumb defect?

A) Metacarpal lengthening
B) Osteoplastic reconstruction
C) Pollicization
D) Prosthesis
E) Toe transfer

A

The correct answer is Option C.

The x-ray study shows a carpometacarpal-level amputation of the thumb. Toe transfer, osteoplastic reconstruction, and metacarpal lengthening require part or most of the thumb metacarpal to be present. A thumb prosthesis would have limited functionality and be insensate.

Pollicization would potentially allow a sensate, functional index finger to accomplish some of the functions of the thumb. It is, however, not without its drawbacks because it is technically demanding and has a high likelihood of requiring secondary procedures. For a carpometacarpal-level amputation, pollicization provides the best option for function.

References

Graham D, Bhardwaj P, Sabapathy SR. Secondary thumb reconstruction in a mutilated hand. Hand Clin. 2016 Nov;32(4):533-547.

Muzaffar AR, Chao JJ, Friedrich JB. Posttraumatic thumb reconstruction. Plast Reconstr Surg. 2005 Oct;116(5):103e-122e.

Pet MA, Ko JH, Vedder NB. Reconstruction of the traumatized thumb. Plast Reconstr Surg. 2014 Dec;134(6):1235-1245.

Graham DJ, Venkatramani H, Sabapathy SR. Current reconstruction options for traumatic thumb loss. J Hand Surg Am. 2016 Dec;41(12):1159-1169.

160
Q

Improvement in which of the following is an advantage of nipple-areola complex reconstruction?

A) Breast feeding ability
B) Breast mound shape
C) Reactivity to touch
D) Satisfaction with reconstruction
E) Sexual sensation

A

The correct response is Option D.

Satisfaction with reconstruction, quality of life, and feeling of completeness with reconstruction have all been shown to be linked to nipple and/or areola reconstruction. The reconstruction does not restore nipple function; therefore, breast feeding, sexual sensation, and reactivity to touch are not accomplished with nipple and/or areola reconstruction. Often the nipple reconstruction, if using local tissue techniques, may actually flatten the anterior aspect of the breast and is not thought to positively impact the breast mound shape.

161
Q

A 26-year-old woman comes to the office for consultation regarding right mammary hypoplasia and a superiorly displaced nipple-areola complex. Examination shows a depressed right chest wall. The pectoralis major muscle is anatomically normal. Which of the following is the most likely diagnosis?

A ) Anterior thoracic hypoplasia

B ) Pectus carinatum

C ) Pectus excavatum

D ) Poland syndrome

E ) Sternal cleft

A

The correct response is Option A.

The most likely diagnosis in this patient is anterior thoracic hypoplasia. Anterior thoracic hypoplasia is a syndrome composed of an anterior chest wall depression resulting from posteriorly displaced ribs, hypoplasia of the ipsilateral breast, and a superiorly displaced nipple-areola complex. The sternum is in normal position, and the pectoralis major muscle is normal.

Pectus excavatum is the most common congenital chest wall abnormality in which the ribs and sternum form abnormally, resulting in a concave anterior chest wall. Typically, the lower third of the sternum is involved. In the most severe form, pectus excavatum can present with the sternum adjacent to the vertebral bodies associated with cardiopulmonary abnormalities. In contrast, pectus carinatum is a chest wall deformity in which the sternum and ribs are forced anteriorly, creating the appearance of a €œpigeon €™s chest. € Pectus excavatum and carinatum have sternal involvement, but they do not involve changes in the development of the breast.

Poland syndrome is a congenital anomaly characterized by a number of unilateral findings. The classic features of Poland syndrome include absence of the sternal head of the pectoralis major, hypoplasia and/or aplasia of the breast or nipple, deficiency of subcutaneous fat and axillary hair, abnormalities of the rib cage, and upper extremity anomalies. In its simplest form, Poland syndrome may present with only mild hypoplasia of the breast and lateral displacement of the nipple. Complex presentations of Poland syndrome include hypoplasia or aplasia of the chest wall musculature (serratus, external oblique, pectoralis minor, and latissimus dorsi muscles) or total absence of the anterolateral ribs with herniation of the lung.

Sternal cleft is a rare congenital defect of the anterior chest wall resulting from a failure of midline fusion of the sternum. Depending on the degree of clefting, there are complete and incomplete forms. The sternal cleft is clinically significant because of the potential for the lack of protection to the heart and great vessels. Sternal clefts are not associated with aplasia or hypoplasia of the breast.

162
Q

A 50-year-old woman presents to the clinic to discuss breast reconstruction after bilateral mastectomy. She is interested in free tissue transfer. She has a diagnosis of systemic lupus erythematosus treated with chronic steroid therapy and wants to know if she is an appropriate candidate for free flap reconstruction. Which of the following statements best describes the surgical risks for this patient with lupus compared with the general population?

A) Higher rate of free flap failure
B) Higher risk of a thromboembolic event
C) Similar rate of hernias after abdominally based free flaps
D) Similar rates of infection

A

The correct response is Option B.

The statement which best describes the surgical risks for a patient with lupus undergoing free tissue transfer for breast reconstruction is that the patient has a higher risk of a thromboembolic event than the average patient.

The rate of free flap failure in patients with lupus is similar to patients without lupus. Chronic steroid use increases the risk of wound healing complications in patients with lupus, rather than increases the risk of free flap failure. Additionally, patients with lupus have an increased risk of abdominal wall bulge and hernia after abdominally based free flaps compared with the average population. Chronic steroid use also suppresses the immune system, predisposing patients treated with steroids to increased rates of infection compared to patients not taking steroids.

163
Q

A 5-year-old boy presents to the emergency department 4 hours after he sustained an amputation of his left index finger when it was slammed in a door. The parents brought the amputated digit in a plastic bag on ice. The amputation is at the level of the mid proximal phalanx. Which of the following is the most important reason to attempt replantation?

A) The amputation is proximal to the flexor digitorum superficialis insertion
B) The cold ischemia time is less than 6 hours
C) It is the index finger
D) It is a single-digit amputation
E) The patient is a child

A

The correct response is Option E.

Digital replantation should almost always be attempted in a child, except when the amputated part is severely crushed or there are other life-threatening injuries that preclude surgery. Replantation in children is technically more challenging due to the smaller size of the vessels. However, functional outcomes are more superior than in adults. The replanted parts have better sensory return and can have normal growth. Amputations through joints also exhibit remarkable joint remodeling.

A single digit amputation, especially proximal to the flexor digitorum superficialis (FDS) insertion is considered a contraindication to replantation. Digit replantations proximal to the FDS insertion have a poor range of motion as compared to amputations distal to the FDS insertion. This is, thus, an important landmark when making decisions about amputation versus replantation. Multiple digit amputations are an indication for replantation as the functioning deficit with loss of multiple digits is great. The thumb is responsible for 40% of the function of the hand and should always be replanted, if possible. Even if it is stiff and insensate, a replanted thumb will act as a post for opposition.

Index finger amputations at or proximal to the proximal interphalangeal joint are considered by many to be an indication for amputation. A stiff and painful index finger is likely to be excluded by the patient; amputation will result in better global hand function.

Digits tolerate longer ischemia times than more proximal level amputations, due to absence of muscle. Amputated digits tolerate warm ischemia times of 6 to 12 hours and cold ischemia times of 12 to 24 hours. Digital replantation has been reported with warm ischemia time of 33 hours and cold ischemia time of 94 hours. Cold ischemia time is thus not a major consideration in the decision-making process for amputation versus replantation.

References

Barbary S, Dap F, Dautel G. Finger replantation: surgical technique and indications. Chir Main. 2013;32(6):363-372.

Pederson WC. Replantation. Plast Reconstr Surg. 2001;107(3):823-841.

Prucz RB, Friedrich JB. Upper extremity replantation: current concepts. Plast Reconstr Surg. 2014;133(2):333-342.

164
Q

A 36-year-old woman with invasive ductal carcinoma of the left breast comes to the office to discuss options for immediate breast reconstruction. Examination shows that the tumor is positive for estrogen receptor (ER) and progesterone receptor (PR) expression. Family history is negative for breast and ovarian cancer, and genetic testing shows no abnormalities. Bilateral mastectomy is planned. Which of the following outcomes is most likely in this patient following contralateral prophylactic mastectomy of the right breast?

A) Decreased number of anticipated overall operations
B) Decreased risk of recurrence of the known cancer
C) Increased cure rate for her known cancer
D) Increased possibility of future breast-feeding
E) Increased risk of surgical site complications

A

The correct response is Option E.

Contralateral prophylactic mastectomy (CPM) has become increasingly common in recent years. Women who are at higher than average risk of developing breast cancer may have a clear oncologic benefit from CPM in terms of risk reduction. This would include women who are carriers of high-risk genetic mutations such as BRCA1 and BRCA2, women with a strong family history of breast cancer, and young women with high risk of aggressive tumors. However, for women with average risk of contralateral breast cancer (2 to 6% over 10 years), studies have not supported an oncologic benefit to CPM, attitudes of the lay public and media attention notwithstanding.

Since the surgeries of each breast, involving both mastectomy and reconstruction, are largely (although not completely) independent of one another, the risk of surgical complications to one side or the other is nearly additive, resulting in an approximate doubling of those risks.

CPM has not been shown to improve the cure rate for the known cancer treated with therapeutic mastectomy.

CPM has not been shown to reduce risk of recurrence for the known cancer on the primary side.

CPM would not be expected to decrease the number of anticipated operations. Given the increased likelihood of complications, one may reasonably expect the same number of operations or greater (for treatment of complications), but not fewer.

Removal of both breasts eliminates the ability to breast-feed in the future.

165
Q

A 43-year-old woman comes to the office for consultation regarding augmentation mammaplasty. She has never had any lumps or nipple discharge from her breasts, and has no family history of breast cancer. After discussion, she chooses saline prostheses. She is concerned about breast cancer and inquires about screening. Which of the following screening studies is most appropriate for this patient after augmentation?

A) CT scan
B) Mammography
C) MRI
D) Positron emission tomography
E) Ultrasonography

A

The correct response is Option B.

Current recommendations for breast cancer screening in women with augmentation mammaplasty include mammography with Eklund views. In the Eklund technique, the prosthesis is pushed back against the chest wall, and the breast tissue is pulled forward and around the prosthesis. The use of this technique increases the sensitivity of mammography for breast cancer. Breast prostheses may affect the visualization of breast tissue, and it has been suggested that diagnostic mammography be obtained instead of screening mammography, even for the asymptomatic patient.

CT scanning has been studied for the evaluation of the breast but is not routinely used as a tool for breast imaging. MRI is recommended for the evaluation of a ruptured silicone prosthesis. The technique has high sensitivity, but lower specificity and high cost. It is not recommended as a screening tool for breast cancer in the general population at this time, but it may play a role in the high-risk patient.

Positron emission tomography is not used as a screening test for breast cancer. It is often used as an adjunct in patients diagnosed with breast cancer to determine if the cancer has spread to the lymph nodes or other parts of the body. Ultrasonography may be used for screening but is not recommended because it is very operator dependent. It will often be used as an adjunct to mammography in screening or if a suspected lesion is found.

166
Q

A 71-year-old woman elects to undergo surgery for basal thumb osteoarthritis. In addition to carpal tunnel syndrome and presence of pathology at the scaphotrapeziotrapezoid (STT) joint, assessment for which of the following additional concomitant conditions is most appropriate during the operative planning for this patient?

A) Lunotriquetral dissociation
B) Radioscaphoid arthritis
C) Scapholunate dissociation
D) Thumb interphalangeal arthritis
E) Thumb metacarpophalangeal hyperextension

A

The correct response is Option E.

Operative planning for surgical treatment of basal thumb osteoarthritis requires not only careful history, physical examination, and radiographic examination of the basal thumb joint, but also the scaphotrapeziotrapezoid (STT) joint, the carpal tunnel, and the thumb metacarpophalangeal (MP) joint. Persistent arthritic symptoms following treatment of the basal thumb joint are often due to unrecognized STT arthritis, and many patients will have carpal tunnel syndrome concomitant with basal thumb arthritis; thus, it is important to evaluate for these pathologies to avoid persistent symptoms following surgery. The MP joint must be evaluated for collapse, or hyperextension, particularly with pinch prior to operative treatment. Failure to correct MP hyperextension, particularly that beyond 30 degrees, may lead to persistent pain and progressive collapse of the thumb.

The scapholunate, lunotriquetral, thumb interphalangeal, and radioscaphoid joints are not associated with basal thumb arthritis or its treatment.

167
Q

A 42-year-old woman who underwent mastectomy of the right breast 6 months ago is evaluated for delayed autologous breast reconstruction with free tissue transfer. Which of the following medications should be discontinued preoperatively if she is routinely taking it?

A) Diltiazem
B) Fluoxetine
C) Metoprolol
D) Multivitamin
E) Tamoxifen

A

The correct response is Option E.

It is well known that tamoxifen can increase the risk of thrombembolic events. In a retrospective study at MD Anderson Cancer Center, it was shown that patients who received the drug close to the procedure had a significantly higher rate of complications. It is recommended that the patient stop tamoxifen at least 28 days before surgery.

Other listed medications do not have a direct effect on thrombotic complications.

168
Q

An otherwise healthy 38-year-old woman who is a smoker is considering implant-based breast reconstruction following mastectomy. She has been counseled about likely use of acellular dermal matrix when the intermediate tissue expander is placed and wants to further understand why the matrix will be used in her body. The patient should be advised that use of acellular dermal matrix is associated with a decreased risk for which of the following?

A) Cancer recurrence
B) Capsular contracture
C) Seroma
D) Skin flap necrosis
E) Smoking-related complications

A

The correct response is Option B.

Acellular dermal matrix (ADM) use has been increasing over time with tissue expander or implant-based breast reconstruction following mastectomies. Many potential advantages and disadvantages have been studied, and some of the data are contradictory. However, the consensus of the literature indicates that ADMs are associated with decreased capsular contracture rates. There is literature to suggest that seroma rates are increased or remain stable, not decreased, with ADMs. ADMs have not been shown to decrease independent patient risk factors for complications such as tobacco use or to decrease cancer recurrence rates. ADMs also do not appear to improve vascularity of the tissue overlying them when initially placed.

169
Q

A 33-year-old woman with no family history of breast cancer undergoes bilateral augmentation mammaplasty with 300 mL of autologous fat per breast. Six months later, she has onset of pain in the right breast. Mammography shows linear clustered microcalcifications in the lower inner quadrant of the right breast, small lipid cysts bilaterally with scattered dystrophic rod-like calcifications in the upper outer quadrants bilaterally, and heterogeneity of the pectoral muscles. Which of the following is the most appropriate next step in management?

A) Baseline mammography between ages 35 and 40 and yearly thereafter
B) Core needle biopsy of the bilateral upper outer quadrants
C) Core needle biopsy of the right lower inner quadrant
D) Repeat mammography at 6 months and 12 months
E) Repeat mammography in 1 year

A

The correct response is Option C.

Augmentation mammaplasty with autologous fat transfer has become an increasingly popular option for patients desiring modest volumetric improvement. Despite its popularity, there is still some concern regarding its safety and efficacy. ASPS offered guidelines on fat grafting for reconstructive procedures of the breast in 2009. However, caution is recommended in the setting of cosmetic procedures because the impact on radiologic changes in follow-up is still uncertain to date.

Fat necrosis is a nonspecific histologic finding most commonly resulting from surgery, trauma, or radiation therapy. It is common after fat transfer procedures, though often is clinically occult, and detected through follow-up mammography. The mammographic images of fat necrosis range from lipid cysts to findings that are suspected for malignancy such as clustered microcalcifications or spiculated masses. The most frequent mammographic finding in the breast parenchyma after augmentation mammaplasty with fat transfer is bilateral scattered microcalcifications followed by radiolucent oil cysts with or without microcalcification. Microcalcifications represent an evolution in the mammographic appearance of fat necrosis and are usually not present in early postoperative screening, but rather are a relatively late finding that is present months to years after the inciting trauma.

It is imperative that radiologists distinguish between benign and suspected microcalcifications in order to minimize the number of postoperative biopsies and frequent follow-up imaging. Although round, spherical, punctuate, and diffusely scattered calcifications are typical of benign processes, cluster, branching microcalcifications can be indicative of a malignant process and should be worked up. For this 33-year-old patient with no baseline mammography and a suspected lesion within 6 months of the procedure, routine or short-interval mammographic screening is not appropriate. A biopsy of the suspected area is required, and this patient should undergo a core needle biopsy of the clustered microcalcifications of the right breast, while the more benign-appearing calcifications within the upper outer quadrants can be observed.

170
Q

An otherwise healthy 25-year-old woman is evaluated and scheduled for augmentation mammaplasty with silicone gel implants. Which of the following is most accurate regarding breast implant-associated anaplastic large cell lymphoma (BIA-ALCL)?

A) All late breast implant-associated seromas should be evaluated
B) BIA-ALCL is most often associated with an aggressive clinical course
C) BIA-ALCL is most often associated with smooth implants
D) It is not necessary to include BIA-ALCL in a standard breast augmentation/reconstruction consent
E) Knowledge about BIA-ALCL’s cause is based on strong evidence-based studies

A

The correct response is Option A.

Due to the potential critical relevance of breast implant-associated anaplastic large cell lymphoma (BIA-ALCL) and its tendency to present as a late seroma, correct diagnostic pathways should be carried out on all late breast implant-associated seromas to include cytologic examination and, if indicated further, fine-needle aspiration, flow cytometry, and immunohistochemistry for CD30. BIA-ALCL is a critical outcome in implant-associated breast augmentation/reconstruction. It is most commonly confined to peri-implant seroma fluid and follows a nonaggressive course amenable to implant and capsule removal, although there are aggressive variants. Discussion about this condition should be part of the consenting process for all breast implant cases. Finally, to this point evidence on cause is based on very low-evidence studies. ALCL is most often associated with textured implants.

171
Q

A 37-year-old woman comes to the office for consultation regarding left breast reconstruction after mastectomy to treat breast cancer. Chemotherapy and radiation therapy are planned postoperatively. She wears a size 34D brassiere. Height is 5 ft 6 in (168 cm), and weight is 160 lb (73 kg). BMI is 25.8 kg/m2. She does not want abdominal scars. Autologous reconstruction with a transverse musculocutaneous gracilis (TMG) flap is planned. Which of the following is a disadvantage of using a TMG flap for this patient’s reconstruction?

A) Difficulty of flap harvest
B) High risk of donor site morbidity
C) High risk of fat necrosis
D) Small flap size
E) Variable vascular anatomy

A

The correct response is Option D.

For breast reconstruction, the TMG flap offers a valuable alternative therapy compared with other standard flaps from the lower abdomen, such as the transverse rectus abdominis musculocutaneous (TRAM) or DIEP flaps. The amount of tissue that can be transferred, however, is limited. The largest flap harvested in one large series weighed 420 g and measured 30 × 10 cm. The flap usually offers enough volume to reconstruct small- to mid-sized breasts. The major advantage of this flap as compared with other flaps, such as the gluteal flap or the perforator latissimus flap, is its constant vascular anatomy. Flap perfusion is always reliable, with low rates of fat necrosis and tissue similar in consistency with breast tissue. The donor scar is inconspicuous. The incision is comparable to incisions for a thigh lift and is well hidden. A distortion of the labia majora is typically not observed. However, as in thigh lifts, a lowering of the scar remains a possible problem. Functional donor-site morbidity after TMG flap harvest itself is low. After having clinically established the transverse approach for routine procedures, harvesting one flap is rapid and usually takes no longer than 30 minutes.

172
Q

A 28-year-old man is brought to the emergency room because of pain, swelling, and deformity of the right small finger and a laceration over the finger after being involved in a motor vehicle collision. Physical examination shows a 7-mm laceration over the fifth metacarpal shaft. X-ray studies confirm a significantly displaced transverse metacarpal shaft fracture of the right small finger. Which of the following is the most appropriate management?

A) Closed reduction of the fracture and splint immobilization followed by 7 to 10 days of outpatient oral antibiotic therapy
B) Closed reduction of the fracture followed by splint immobilization only
C) Emergent debridement in the operating room followed by external fixation of the fracture
D) Inpatient intravenous antibiotic therapy and debridement in the operating room within 48 hours
E) Irrigation of the wound in the emergency room, splint immobilization, 24 hours of oral antibiotic therapy, and elective operative fixation of the fracture

A

The correct response is Option E.

This patient has a displaced Grade I open fracture of the fifth metacarpal shaft. Appropriate management includes irrigation and debridement of the wound in the emergency room, splint immobilization, and a brief course of antibiotic therapy. This allows the fracture to be managed as if it were a closed injury. Transverse shaft fractures typically are unstable and require some type of elective fixation beyond splint immobilization.

Open fractures usually are classified according to the Gustilo-Anderson scale, although outcomes of upper extremity open fractures do not necessarily correlate to this system. Grade I injuries are defined as fractures with open wounds <1 cm. Grade II fractures have open wounds measuring between 1 and 10 cm. Grade IIIA fractures have wounds >10 cm with comminution but adequate soft-tissue coverage. Grade IIIB fractures have wounds >10 cm with extensive periosteal stripping that requires soft-tissue reconstruction. Grade IIIC fractures have large wounds with associated vascular injury.

In upper extremity fractures, studies have not correlated time to operative debridement or the administration of antibiotic therapy with ultimate outcomes in terms of infection, malunion, or osteomyelitis. Factors that have been shown to correlate to poor outcomes in large-scale studies have been high injury severity score, Gustilo grade III, and fractures of the tibia/fibula. Shorter duration of antibiotic administration for open fractures has not been shown to result in a higher rate of infection. The current recommendation from the American Society of Orthopedic surgeons is a 3-day regimen of antibiotics for Grade I/II injuries and 5 days for Grade III.

Grade I injuries have only a minor soft-tissue component and there is no role for emergent surgery, hospital admission, or the need for provisional external fixation.

Closed reduction and splint immobilization are not appropriate for unstable fractures as definitive management in a healthy, young patient. In addition, there is no role for prolonged courses of antibiotic therapy. The fracture can be treated with percutaneous pin fixation or plate fixation depending on the preference of the surgeon.

173
Q

A 35-year-old woman, gravida 2, para 2, seeks implant-based augmentation mammaplasty. She breastfed both her children. Which of the following is the most common complication of this procedure?

A) Early implant rupture
B) Hematoma
C) Infection
D) Lifetime need for reoperation
E) Seroma

A

The correct response is Option D.

Augmentation mammaplasty is known to have high rates of complications including reoperation. Infection, seroma, hematoma, and early implant rupture are rare in elective, cosmetic augmentation mammaplasty.

174
Q

In grade II ptosis of the breast, which of the following best describes the position of the nipple?

A) At the apex of the breast mound
B) At the lowest contour of the breast
C) At the transposed inframammary fold
D) Between the inframammary fold and the lowest contour of the breast
E) On the posterior aspect of the breast as it rests on the chest wall

A

The correct response is Option D.

The classic Regnault definition of breast ptosis classifications are as follows:

Grade I: Nipple at the level of the inframammary fold
Grade II: Nipple between the level of the inframammary fold and the lowest contour of the breast
Grade III: Nipple at the lowest contour of the breast

175
Q

A 48-year-old woman undergoes immediate unilateral breast reconstruction with a free deep inferior epigastric artery perforator (DIEP) flap. At the conclusion of the procedure, the flap skin paddle is noted to have venous congestion. Upon reexploration, the venous anastomosis appears patent with venous outflow detected by handheld pencil Doppler evaluation, but the flap continues to have venous congestion with brisk capillary refill. Which of the following is the most appropriate next step in management?

A) Apply leeches postoperatively
B) Loosely re-inset the flap and monitor closely
C) Perform a second venous anastomosis using the superficial inferior epigastric vein
D) Perform a second venous anastomosis using the vena comitans
E) Revise the venous anastomosis using a hand-sewn technique

A

The correct response is Option C.

Preservation of the superficial inferior epigastric veins (SIEV) during flap harvest is a useful preventive measure in microsurgical free tissue transfer operations. These veins can serve as important lifeboats to augment venous outflow in the setting of venous congestion. Typically, if a free flap demonstrates venous congestion, the inset should be taken down and the pedicle, recipient vessels, and anastomoses should be interrogated. Simple issues, such as mechanical compression or twisting of the vein, should be ruled out. Next, the SIEV should be inspected. If engorged, the flap is likely reliant on superficial outflow, and this vein should be connected to a recipient vessel to augment the venous outflow of the flap. Options for recipient veins include an anterograde branch on the pedicle vena comitans, or in a retrograde fashion to the vena comitans that was not used in the initial set of anastomoses.

In this case scenario, the flap continued to demonstrate venous congestion intraoperatively. This makes it unlikely that tension or pressure from the inset of the flap was causing the venous outflow obstruction. Furthermore, leech therapy is not indicated for a free flap with global venous congestion.

The venous coupling device is safe and effective for the anastomosis of veins in DIEP flap surgery. It has not been associated with patency rates that are different from hand-sewn anastomoses. The coupling device, however, has been shown to reduce the microsurgery time.

The use of one or two veins in microsurgical free tissue transfer is a topic that has been debated for several years. While some studies indicate that the use of two venous connections may reduce the velocity of blood flow across the anastomosis, there is not sufficient data to support differences in flap outcomes or thrombotic events. Therefore, the routine use of a second vein is largely up to surgeon preference.

176
Q

A 24-year-old man comes to the office because of a scaphoid wrist nonunion with apex dorsal angulation and proximal pole avascular necrosis. A free tissue transfer from the lower extremity is planned. A branch of which of the following arteries supplies the most appropriate flap for this patient?

A) Dorsalis pedis
B) Genicular
C) Lateral femoral circumflex
D) Medial sural
E) Peroneal

A

The correct response is Option B.

The descending genicular artery is the arterial pedicle for the medial femoral condyle free vascularized osseous corticoperiosteal free flap, or free vascularized bone graft. Scaphoid nonunions with a humpback deformity, carpal collapse, and proximal pole osteonecrosis are difficult to treat. Vascularized bone grafts have been shown to have nearly 2× the union rate of traditional nonvascularized bone grafts. Vascularized corticocancellous bone has the potential to revascularize necrotic bone and can provide structural support for fractures with loss of height of the scaphoid. Studies have shown superior union rates for the medial femoral condyle vascularized bone graft versus pedicled grafts from the distal radius. Anatomical studies show no clinical loss of stability of the femur after flap harvest.

The peroneal artery is the blood supply of the fibular osseous or osteocutaneous free flap. It is generally reserved for head and neck reconstruction and larger defects of the extremities.

The descending branch of the lateral femoral circumflex artery supplies the anterolateral thigh free flap. The dorsalis pedis artery supplies the dorsalis pedis fasciocutaneous flap. The medial sural artery and its perforators supply the medial gastrocnemius muscle, and musculocutaneous and fasciocutaneous flaps. All of these flaps are used for soft-tissue defects alone and are not appropriate for reconstruction of bony defects.

177
Q

A 17-year-old boy comes to the office because of ongoing pain of the right hand after he punched a wall 5 days ago. Physical examination demonstrates tenderness of the fifth carpometacarpal joint. Posteroanterior, oblique, and lateral x-ray studies taken at an urgent care facility were read as negative by the radiologist. Which of the following additional radiographic views is most likely to help confirm this patient’s diagnosis?

A) Anteroposterior with 30 degrees of pronation from full supination
B) Carpal tunnel view with wrist in full extension
C) Clenched fist lateral in neutral forearm position
D) Lateral with 15 degrees of supination from neutral forearm position
E) Posteroanterior with 45 degrees of supination from full pronation

A

The correct response is Option A.

Injuries to the fifth carpometacarpal joint, including subluxation, dislocation, and fracture-dislocation, are often missed with standard two-view and three-view hand images. Two views have been suggested to help detect this subtle injury:

Anteroposterior view with forearm pronated 30 degrees from full supination. This view shows more clearly the profile of the articulation between the hamate and fifth metacarpal base. (This is similar to the “reverse oblique” view, which is typically done in 45 degrees of pronation and might also be useful.)

Lateral with 30 degrees of pronation. This view is especially helpful for detecting subluxation of the metacarpal dorsally off of the hamate. In some cases, CT scan may be warranted if plain films are inconclusive.

178
Q

A 42-year-old woman is evaluated because of an invasive cancer of the right breast. Which of the following best approximates the likelihood that this patient’s cancer is associated with the BRCA1 or BRCA2 genes?

A) 1%
B) 10%
C) 25%
D) 40%
E) 75%

A

The correct response is Option B.

Among familial breast cancers, 5 to 10% are considered to be hereditary. These familial breast cancers are linked to specific mutations on a cancer susceptibility gene. The breast cancer susceptibility genes (BRCA) belong to a class of genes known as tumor suppressors. In normal cells, BRCA1 and BRCA2 genes stabilize the DNA and prevent uncontrolled cell growth.

A woman’s lifetime risk of developing breast and/or ovarian cancer is greatly increased if she inherits a mutation on BRCA1 or BRCA2 genes. BRCA1- and BRCA2-related breast cancers occur in younger women and are often associated with estrogen receptor-negative tumors.

179
Q

A 53-year-old woman comes to the office because of ulcerated tissue 6 weeks after undergoing radiation therapy for breast cancer. She underwent mastectomy 1 year ago. Analysis of the radiated tissue is most likely to show an increase in which of the following?

A ) Acute inflammatory response

B ) Cytokines and growth factors

C ) Neutrophil function

D ) Tissue oxygenation

E ) Vessel thrombosis

A

The correct response is Option E.

Analysis of radiated tissue will show increased vessel thrombosis. Ionizing radiation directly damages the genome or injures the DNA through free radical production. In acute radiation injury, the skin becomes erythematous and edematous with the dilation of fine blood vessels, endothelial edema, and lymphatic obliteration. Although perfusion of the skin assessed with fluorescein injection appears normal, tissue oxygenation is inadequate. Healing is impaired with slowed fibroblast proliferation and impairment of the acute inflammatory response. Fibroblast defects are the main problem in the inhibited healing of chronic radiation injury. Phagocytosis and bacteriocidal metabolic functions in neutrophils are also impaired. This effect increases after therapy, which cannot be a direct effect of radiation on the neutrophils because their lifespan is too short. The local wound environment and irradiated tissue do not prime the neutrophils with the appropriate cytokines and growth factors needed for activation. This may lead to an increased incidence of postoperative infections in patients with previous radiation.

Recent studies have shown promise in the treatment of radiation tissue damage with lipoaspirate transplantation. Adipose-derived stem cells have been hypothesized to target damaged areas, release angiogenic factors, form new vessels, and increase tissue oxygenation.

180
Q

A patient with a history of breast cancer undergoes nipple-sparing mastectomy of the right breast with immediate implant-based reconstruction. Ten months after surgery, the patient starts to recover sensitivity at the nipple. Which of the following nerves is most likely providing sensitivity to the nipple-areola complex in this patient?

A) Anterior branch of the fourth intercostal nerve
B) Anterior branch of the second intercostal nerve
C) Lateral branch of the fifth intercostal nerve
D) Lateral branch of the fourth intercostal nerve
E) Lateral branch of the second intercostal nerve
F) Lateral branch of the third intercostal nerve

A

The correct response is Option A.

The nipple and areola of the breast are innervated by both the anterior and lateral cutaneous branches of the third, fourth, or fifth intercostal nerves. The anterior and lateral cutaneous branches of the second and sixth intercostal nerves innervate breast skin only.

In anatomical studies conducted in female cadavers, the fourth intercostal nerve’s lateral cutaneous branch supplied the nipples in 93% dissected breasts. The third and fifth intercostal lateral branches were found to innervate the nipple alone in 3.6%. However, the fourth intercostal lateral branch penetrates the deep fascia in the midaxillary line, takes an inferomedial course to reach the midclavicular line, and continues through the glandular tissue towards the posterior surface of the nipple. Thus, when a mastectomy is performed, this lateral branch is the most likely one to be dissected.

On the other hand, the anterior cutaneous branches take a superficial course, as they run in the subcutaneous tissue close to the skin and reach the nipple from the lateral side. According to this, the anterior branch of the fourth intercostal nerve is most likely providing sensitivity to the nipple-areolar complex after nipple-sparing mastectomy.

181
Q

Which of the following is most likely an open fracture?

A) Barton
B) Colles
C) Epiphyseal
D) Salter-Harris III
E) Seymour

A

The correct response is Option E.

The Seymour fracture in children displaces through the epiphysis with the nail matrix interposed between the fragments. It is always an open fracture.

The Salter-Harris classification is the most commonly used method to describe the five most frequent patterns of pediatric fractures involving the physis. The classification helps explain the mechanism of injury and anticipate the consequences of the fracture upon subsequent growth. A Salter-Harris III fracture is epiphyseal but is not necessarily open.

Colles and Barton fractures are seen in the distal radius.

182
Q

Which of the following is a risk factor for hormone-sensitive breast cancer?

A) Breast-feeding
B) Early age at first pregnancy
C) Early menopause
D) Late menarche
E) Post-menopausal obesity

A

The correct response is Option E.

Post-menopausal obesity is associated with increased adipose production of estrogen, which can increase the risk for hormone-sensitive breast cancer. Other options (late menarche, early menopause, and breast feeding) decrease the number of menstrual cycles, and therefore may decrease the risk of breast cancer. Early age at first pregnancy is also associated with decreased risk for hormone-sensitive breast cancers.

183
Q

A 35-year-old woman comes for consultation regarding breast prosthesis removal because she is concerned about her risk of cancer. Specifically, she has read about anaplastic large cell lymphoma in women with breast prostheses. She underwent augmentation mammaplasty with saline breast prostheses 5 years ago. Physical examination shows absence of contracture and satisfactory position. Which of the following is the most appropriate next step in management?

A) Complete blood cell count
B) Evaluation by a hematologist
C) MRI of the breasts
D) Prosthesis removal
E) Reassurance

A

The correct response is Option E.

The US Food and Drug Administration (FDA) searched its adverse event reporting systems for reports received between January 1, 1995 and December 1, 2010, including information submitted by manufacturers as part of their required post-approval studies. This search identified 17 reports of possible anaplastic large cell lymphoma (ALCL) in women with breast prostheses. Although ALCL is extremely rare, the FDA believes that women with breast prostheses may have a very small but increased risk of developing this disease in the scar capsule adjacent to the prosthesis. Based on available information, it is not possible to confirm with statistical certainty that breast protheses cause ALCL. Currently, it is not possible to identify a type of prosthesis (silicone gel versus saline) or a reason for implantation (reconstruction versus aesthetic augmentation) associated with a smaller or greater risk.

When ALCL occurs, it has been most often identified in patients undergoing prosthesis revision procedures for late-onset, persistent seroma. Because it is so rare and most often identified in patients with late onset of symptoms such as pain, lumps, swelling, or asymmetry, it is unlikely that increased screening of asymptomatic patients would change their clinical outcomes. The FDA does not recommend prophylactic breast prosthesis removal in patients without symptoms or other abnormalities.

A patient with suspected ALCL should be referred to an appropriate specialist for evaluation. When testing for ALCL, fresh seroma fluid and representative portions of the capsule should be collected and sent for pathology tests to rule out ALCL. Diagnostic evaluation should include cytologic evaluation of seroma fluid with Wright-Giemsa–stained smears and cell block immunohistochemistry testing for cluster of differentiation and anaplastic lymphoma kinase markers. Any confirmed cases of ALCL in women with breast prostheses must be reported to the FDA.

184
Q

An otherwise healthy 16-year-old girl comes to the office because of a painless mass in the left breast. Physical examination of the left breast discloses a circumscribed firm, rubbery, 3-cm mass without overlying skin changes, and no axillary lymphadenopathy. Results of a pregnancy test are negative. Which of the following is the most likely diagnosis?

A) Common fibroadenoma
B) Giant fibroadenoma
C) Lactating adenoma
D) Phyllodes tumor
E) Tubular adenoma

A

The correct response is Option A.

A common fibroadenoma is the most likely diagnosis of this patient. Common fibroadenoma is the most common breast tumor in adolescent females and present between the ages of 14 and 16.

Juvenile fibroadenoma is a variant of fibroadenoma and is usually seen in adolescents and young adults. It is associated with a normal stromal/epithelial balance, which distinguishes it from phyllodes tumor, and has both stromal and epithelial hyperplasia. In addition, juvenile fibroadenomas are characterized by rapid growth.

A giant fibroadenoma is a clinical diagnosis, rather than a pathologic diagnosis. It is characterized by its size, usually greater than 5 cm.

Complex fibroadenoma is characterized by fibrocystic changes on glandular tissue with underlying features of common fibroadenoma on pathologic analysis.

A tubular adenoma has glandular proliferation on pathologic analysis, and while it is a subtype of fibroadenoma, it is not as common as common fibroadenoma. A lactating adenoma, similarly, is defined by the presence of secretory hyperplasia of lobules on pathologic analysis. Lactating adenomas are so defined because of the histologic presence of secretory hyperplasia, and they commonly occur in pregnant or lactating women. Many lactating adenomas will spontaneously regress.

A phyllodes tumor is typically a large, rapidly growing lesion and can be either benign or malignant. It is rare in adolescents, but when found, is usually aggressive. Phyllodes tumor is related to fibroadenoma and is distinct from other forms of breast cancer. Treatment is wide local excision.

185
Q

A 12-month-old boy presents for evaluation of a soft, fixed lateral brow mass that has been enlarging since it was first noted at 2 months of age. A photograph is shown. The parents report that the mass does not change in size during crying or activity, nor did it change in size during recurrent pink eye infections. Which of the following is the most likely diagnosis?

A) Branchial cleft cyst
B) Dermoid cyst
C) Hemangioma
D) Lacrimal gland
E) Lymphatic malformation

A

The correct response is Option B.

Dermoid cyst is the correct answer and the most common cause of a lateral brow mass in an infant. If it were a hemangioma, its size would change during crying or any activity that increases blood flow. Lacrimal gland ptosis is unlikely in a child but can be seen in the adult population, and can be ruled out if the mass is above the orbital rim, as seen in the photograph. The mass is not in the distribution of any of the different types of branchial cleft cysts. Lymphatic malformations typically change in size during infections.

References

Al-Muhaylib A, Alkatan HM, Al-Faky YH, Alsuhaibani AH. Periorbital lesions misdiagnosed as dermoid cysts. J AAPOS. 2017;21(6):509-511.

Rezaei E, Shams Hojjati Y. Misdiagnosed extranasal mass: report of a 2-year old child with maltreated rare nasal neuroglial heterotopia. World J Plast Surg. 2019;8(1):122-124.

Van Wyhe RD, Chamata ES, Hollier LH. Midline craniofacial masses in children. Semin Plast Surg. 2016;30(4):176-180.

Vincent J, Baker P, Grischkan J, Fernandez Faith E. Subcutaneous midline nasal mass in an infant due to an intramuscular lipoma. Pediatr Dermatol. 2017;34(3):e135-e136.

186
Q

A 53-year-old woman comes to discuss breast reconstruction after undergoing left mastectomy for ductal carcinoma in situ. She does not require chemotherapy or radiation therapy and does not want surgery on the unaffected breast. She is obese but otherwise healthy with a large, ptotic right breast. Which of the following breast reconstruction techniques is most likely to result in the greatest long-term patient satisfaction?

A) Autologous breast reconstruction with microvascular free tissue transfer from the abdomen
B) Extended latissimus dorsi musculocutaneous flap reconstruction without an implant
C) Immediate reconstruction with cohesive, anatomically shaped silicone gel implant
D) Immediate tissue expander followed by implant-based reconstruction
E) Latissimus dorsi musculocutaneous flap reconstruction with immediate placement of a permanent implant

A

The correct response is Option A.

Multiple studies have supported improved patient satisfaction with autologous breast reconstruction in the setting of a unilateral reconstruction. The clinical scenario involves an obese patient with a large, ptotic breast who does not desire a surgical procedure for symmetry and is the ideal candidate for an autologous reconstruction. Microvascular free tissue transfer of abdominal tissue for breast reconstruction demonstrates improved reliability and decreased fat necrosis compared with pedicled flap reconstruction. Implant-based reconstruction is less likely to provide adequate symmetry in this patient. An extended latissimus dorsi musculocutaneous flap–based reconstruction alone is unlikely to provide enough volume for symmetry and a traditional latissimus flap with implant is unlikely to provide adequate ptosis for symmetry.

187
Q

A French woman, who underwent placement of Poly Implant Prothèse (PIP) gel implants in 2009, comes to the office for consultation because she had heard that the implants were filled with a nonmedical grade silicone. She reports that she has not had any problems with the implants, but would like to know the implications of retaining the implants and whether she should have them removed. This patient should be told that she is at increased risk for which of the following complications if she retains the implants?

A) Breast cancer
B) Cytotoxicity
C) Heavy metal poisoning
D) Implant rupture
E) Siloxane poisoning

A

The correct response is Option D.

The final report, in conjunction with the Department of Health in Australia, has shown a 2 to 6 times increased rupture rate in Poly Implant Prothèse (PIP) implants, which is detectable within 5 years of implantation. Increased levels of siloxane have been detected, but are not considered a health risk. No organic impurities have been detected and platinum levels are decreased in PIP gel compared with medical grade silicone. There is no increased breast cancer risk and no evidence of cytotoxicity. In the light of the increased rupture rate and the nonmedical grade nature of PIP silicone gel, the following recommendations were made:

all providers of breast implant surgery should contact any women who have or may have PIP implants, if they have not already done so, and offer them a specialist consultation and any appropriate investigation to determine if the implants are still intact;

if the original provider is unable or unwilling to do this, a woman should seek referral through her general practitioner to an appropriate specialist;

if there is any sign of rupture, she should be offered an explantation;

if the implants still appear to be intact, she should be offered the opportunity to discuss with her specialist the best way forward;

if, in the light of this advice a woman decides with her specialist that, in her individual circumstances, she wishes to have her implants removed, her health care provider should support her in carrying out this surgery. Where her original provider is unable or unwilling to help, the NHS will remove, but not normally replace, the implants;

if a woman decides not to seek early explantation, she should be offered annual follow up in line with the advice issued by the specialty surgical associations in January 2012. Women who make this choice should be encouraged to consult their doctor if they notice any signs of tenderness or pain, or swollen lymph glands in or around their breasts or armpits, which may indicate a rupture. At the first signs of rupture, they should be offered removal of the implants.

188
Q

A 40-year-old woman comes to the office because of firmness of the right breast. Twenty years ago, she underwent augmentation mammaplasty with smooth silicone prostheses placed in subglandular pockets. Which of the following is the most appropriate management?

A)Injection of corticosteroids
B)Treatment with zafirlukast (Accolate)
C)Closed capsulotomy
D)Open capsulotomy
E)Total capsulectomy

A

The correct response is Option E.

In the patient described with a capsular contracture, the most appropriate option is open capsulectomy. As opposed to open capsulotomy, open capsulectomy removes the entire capsule. Leaving the capsule behind in open capsulotomy can contribute to late seromas. Scar tissue left behind during an open capsulotomy may also prevent the prosthesis and breast from obtaining a natural shape.

Closed capsulotomy is no longer advised for breast prostheses because of the risk of rupturing the prosthesis during the procedure. Open capsulotomy and open capsulectomy with replacement of the prosthesis in the subglandular plane will continue to be associated with higher capsular contracture rates than submuscular or dual-plane placement. These are options for the patient as long as she understands the trade-offs of keeping the prosthesis in this plane.

Zafirlukast (Accolate) is a leukotriene receptor antagonist that is used as a bronchodilator in the management of asthma. The evidence supporting its use in capsular contracture is anecdotal. It is not approved by the US Food and Drug Administration (FDA) for use in capsular contracture; therefore, its use in the scenario described would be considered an ?off-label? indication. As such, zafirlukast cannot be recommended for the routine treatment of capsular contracture

189
Q

An otherwise healthy 26-year-old woman undergoes zone 2 wide-awake flexor tendon repair of the right index finger. A solution of 1% lidocaine with 1:100,000 epinephrine is injected into the hand and digit. After surgical repair of the flexor digitorum profundus (FDP) tendon, the patient’s finger is still pale without capillary refill. Administration of which of the following classes of drug is most likely to reverse the effects of epinephrine in this patient?

A) Alpha-adrenergic receptor activator
B) Alpha-adrenergic receptor blocker
C) Beta-adrenergic receptor blocker
D) Potassium channel activator
E) Sodium channel blocker

A

The correct response is Option B.

The medication that is used to reverse the effects of epinephrine is phentolamine, which is an alpha-adrenergic receptor blocker. The wide-awake Hand Surgery is well described by Donald Lalonde and utilizes the effects of local anesthesia to perform a wide variety of hand-surgical procedures without general anesthesia.

An alpha-adrenergic receptor activator, such as epinephrine, could increase vasoconstriction and worsen the scenario, as could a beta-adrenergic receptor blocker. Sodium channel blockers and potassium channel activators are not indicated for reversal of epinephrine effect.

References

Higgins A, Lalonde DH, Bell M, McKee D, Lalonde JF. Avoiding flexor tendon repair rupture with intraoperative total active movement examination. Plast Reconstr Surg. 2010 Sep;126(3):941-945.

Lalonde D, Bell M, Benoit P, Sparkes G, Denkler K, Chang P. A multicenter prospective study of 3,110 consecutive cases of elective epinephrine use in the fingers and hand: the Dalhousie project clinical phase. J Hand Surg Am. 2005 Sep;30(5):1061-1067.

Lalonde DH. Wide-awake flexor tendon repair. Plast Reconstr Surg. 2009 Feb;123(2):623-625.

Nodwell T, Lalonde D. How long does it take phentolamine to reverse adrenaline-induced vasoconstriction in the finger and hand? A prospective, randomized, blinded study: the Dalhousie project experimental phase. Can J Plast Surg. 2003 Winter;11(4):187-190.

190
Q

A 52-year-old woman with cancer of the right breast undergoes mastectomy and axillary node dissection, complicated by mastectomy flap necrosis requiring skin grafting. She completes adjuvant chemoradiation. One year later, she comes to the office with a fungating mass growing through the skin graft. Imaging demonstrates involvement of the fourth and fifth ribs with an anticipated skeletal defect of 4 × 4 cm. A photograph is shown. Which of the following is the most appropriate treatment for the skeletal reconstruction?

A) High-density porous polyethylene
B) Methyl methacrylate with mesh
C) 2.4-mm Titanium plate
D) No skeletal reconstruction
E) Vascularized rib

A

The correct response is Option D.

The principles of management of this recurrent right breast cancer include radical resection of all involved tissues (including ribs) and reconstruction with well vascularized flaps. In this case, a right latissimus muscle flap and skin graft was used for reconstruction. No alloplastic material was placed or skeletal thoracic cage reconstruction performed. This is common in these types of patients, because excessive fibrosis caused by the radiation to the chest wall prevents loss of respiratory efficiency through paradoxical motion which otherwise occurs in patients who have more than four ribs involved or a defect larger than 5 cm.

191
Q

A 35-year-old woman presents with a fixed adduction contracture of the first web space that has not improved with splinting and hand therapy for 4 months. Medical history includes a crush injury with complex laceration to the first web and dorsal hand and index finger five months ago. A photograph is shown. Which of the following is the most appropriate plan for reconstruction of the first web space contracture in this patient?

A) First dorsal metacarpal artery flap
B) Flexor carpi ulnaris flap
C) Posterior interosseous artery flap
D) Thenar flap
E) Split-thickness skin grafting

A

The correct response is Option C.

Contracture of the first web space may be secondary to cutaneous scarring, skin deficiency, fibrosis of the fascia and thenar muscles, or joint contractures. Mild contractures may be isolated to the skin; however, deeper structures are most likely involved as the contracture becomes more severe. It is important to understand the mechanism of injury, length of time the contracture has been present, and any prior treatments.

Reconstruction of the first web space involves complete contracture release and resurfacing with adequate vascularized tissue. The dissection should be carried out palmarly and dorsally with release of the palmar fascia and adductor aponeurosis as needed. Intrinsic muscle and joint contractures should be addressed at this time, and a trapeziectomy may be needed to restore carpometacarpal (CMC) motion.

This patient has a severe contracture that likely involves multiple structures given her history of deep lacerations and bony injury. This requires resurfacing with thin, pliable vascularized tissue. In this setting, the posterior interosseous artery (PIA) flap is the best choice. This flap is outside the zone of injury and provides an adequate amount of vascularized tissue for resurfacing of the web space. The PIA runs between the extensor carpi ulnaris and extensor digit quinti and forms an anastomosis with the anterior interosseous artery 2cm proximal to the distal radioulnar joint.

Skin grafting alone, either split-thickness or full-thickness, should be avoided because of the inherent tendency for secondary contracture. Skin grafts may be combined with local flaps such as a 4-flap or 5-flap z-plasty in mild to moderate contractures.

Tissue flaps from the dorsum of the hand such as the first dorsal metacarpal artery fasciocutaneous flap or dorsal hand transposition flap may be good options in some patients with small- to moderate-sized skin deficits. However, this patient sustained trauma to the dorsal hand with dorsal skin lacerations. This makes a random-pattern transposition flap unreliable. The defect in question is also too large to be completely resurfaced with a first dorsal metacarpal artery (FDMA) flap. The flexor carpi ulnaris flap is useful for elbow coverage as a turn-over flap but will not reach the hand.

References

Moody L, Galvez M, Chang J. Reconstruction of First Web Space Contractures. J Hand Surg Am. 2015 Sept;40:1892-95.

Cavadas P et al. The Simplified Posterior Interosseous Flap. J Hand Surg Am. 2016 Sep;41(9):e303-7.

192
Q

A 42-year-old woman with Grade 3 ptosis of the breasts is scheduled to undergo augmentation mammaplasty and mastopexy. Which of the following operative decisions is most likely to have an adverse effect on the outcome of the procedure?

A ) Augmentation mammaplasty and use of vertical mastopexy technique

B ) Augmentation mammaplasty and use of a Wise-pattern mastopexy technique

C ) Mastopexy and placement of 450-mL saline prostheses in a dual-plane pocket

D ) Mastopexy and placement of 200-mL silicone prostheses in a subpectoral pocket

E ) Performance of the operation in two stages

A

The correct response is Option C.

Augmentation mammaplasty and mastopexy is a complex procedure that can increase the risks and difficulties beyond those of each one performed independently. A mastopexy is designed to raise the nipple-areola complex and reshape the breast by resecting skin and tightening the parenchyma. In direct opposition to this shaping, an augmentation enlarges the volume of the breast and expands the skin envelope. Further, mastopexy techniques involve elevation of flaps that require adequate vascularity, while prosthesis placement devascularizes the breast and puts direct pressure on the remaining circulation.

The larger the prosthesis, the greater the adverse effect on vascularity. This can lead to early problems with nipple-areola complex loss, skin flap loss, prosthesis infection and exposure, and resultant deformities.

Larger prostheses are also associated with long-term complications of soft-tissue attenuation. This results in tissue thinning, stretching, atrophy, rippling, and recurrent ptosis. Despite conflicting studies, prosthesis size of 350 mL is considered the crossover to large prostheses.

Despite these risks, most patients want to have both operations performed simultaneously. If these patients are accepted, it is the surgeon €™s responsibility to minimize complications.

Some surgeons prefer to perform augmentation and mastopexy in two separate operations to control the result and reduce the complication rate.

Placement of 200-mL silicone prostheses in a subpectoral pocket is less likely to cause problems because of their modest size.

Vertical mastopexy and Wise-pattern techniques are both acceptable procedures that can be applied to patients with Grade 3 ptosis.

193
Q

A 58-year-old woman with moderate ptosis is evaluated for mastopexy. According to Regnault classification, which of the following best describes the location of the nipple-areola complex in type II breast ptosis?

A) 1 to 3 cm inferior to the inframammary fold
B) 4 cm inferior to the inframammary fold
C) 6 cm inferior to the inframammary fold
D) At or 1 cm inferior to the inframammary fold
E) Superior to the inframammary fold

A

The correct response is Option A.

Regnault classification of breast ptosis, based on the position of the nipple-areola complex (NAC) relative to the inframammary fold (IMF):

The type of mastopexy performed will depend on the degree of breast ptosis. Breast ptosis is graded using Regnault classification. Type I can be treated with a crescent mastopexy, when the degree of nipple-areola complex elevation does not exceed 1 cm. Type I or II ptosis can be treated with a periareolar mastopexy, when the distance of nipple-areola complex elevation ranges from 1 to 2 cm. Type II and III ptosis is amenable to the inverted-T technique, where the horizontal incision will reduce the distance from the nipple-areola complex to the inframammary fold, while the vertical incision will reduce the base diameter.

194
Q

Which of the following is most commonly associated with alloplastic breast reconstruction in the setting of adjuvant radiation therapy?

A) Decreased rates of implant rupture
B) Decreased rates of seroma
C) Increased patient satisfaction
D) Increased risk for capsular contracture

A

The correct response is Option D.

Alloplastic breast reconstruction increases the risk for capsular contracture in the setting of adjuvant radiation therapy. It can also increase the risks for seroma, wound healing complications, and infections. Radiation would not decrease the rates of implant rupture and would not increase patient satisfaction.

195
Q

Which of the following structures is formed from the same branchial arch as the vagus (X) nerve?

A) Inferior parathyroid glands
B) Lesser horn of hyoid
C) Levator veli palatini
D) Maxillary artery
E) Styloid process

A

The correct response is Option C.

Each of the six branchial arches gives rise to a branch of the aortic arch, a cranial nerve, muscular structures, and skeletal structures. The fourth branchial arch gives rise to the right proximal subclavian artery, the aortic arch, the vagus (X) nerve, the superior laryngeal nerve, the intrinsic muscles of the levator veli palatini, cricothyroid muscles, laryngeal cartilages, and the superior parathyroid glands.

The styloid process is derived from the second branchial arch, along with the stapedial and hyoid arteries, the facial (VII) nerve, the muscles of facial expression, the stapes, the lesser horn of the hyoid bone, and the crypts of the palatine tonsils.

The maxillary artery is derived from the first branchial arch, along with the trigeminal (V) nerve, the muscles of mastication, anterior belly of the digastric muscle, tensor tympani, tensor veli palatini, mylohyoid, mandible, incus and malleus, maxilla, vomer, zygoma, and temporal bone.

The inferior parathyroid glands are derived from the third branchial arch, along with the common carotid artery, internal carotid artery, glossopharyngeal (IX) nerve, stylopharyngeus muscle, greater horn of the hyoid bone, and thymus.

The lesser horn of the hyoid bone is derived from the second branchial arch.

References

Adams A, Mankad K, Offiah C, Childs L. Branchial Cleft Anomalies: A Pictorial Review of Embryological Development and Spectrum of Imaging Findings. Insights Imaging. 2016 Feb; 7(1):69-76.

Cohen M. Malformations of the Craniofacial Region: Evolutionary, Embryologic, Genetic, and Clinical Perspectives. Am J Med Genet A. 2002 Dec; 115(4):245-68.

196
Q

A 28-year-old man sustains acute wrist extension during a fall on an outstretched arm. Examination shows snuffbox tenderness. A scaphoid fracture is suspected. Which of the following imaging studies should be performed first to identify this patient’s injury?

A) Bone scan
B) CT scan
C) MRI
D) Plain x-ray studies
E) Ultrasonography

A

The correct response is Option D.

The correct answer is plain x-rays. Negative x-rays in scaphoid fractures are up to 30%. Cost effectiveness of obtaining x-rays first is shown by the positive finding in 70%. The predictive value of clinical examination is 13-69% with an average of 21%. Depending on clinical suspicion and whether the patient needs to avoid immobilization if the absence of fracture can be confirmed, additional imaging studies may be obtained.

For MRI, the estimated sensitivity is 97.7% and the specificity is 99.8% with 96% accuracy. For a CT scan, estimated sensitivity is 85.2 to 94% and the specificity is 96 to 99.5% with 98% accuracy. Bone scintigraphy is 96 to 97.8% and 89 to 93.5%, respectively, with 93% accuracy. For follow-up x-ray studies, 91.1 and 99.8%, respectively. MRI is therefore the best test for ruling in scaphoid fractures where the other tests are better at ruling out scaphoid fractures. Cost effectiveness of MRI for patients with suspicion for scaphoid fracture with negative x-rays is shown by getting patients out of unnecessary splints sooner.

197
Q

A 46-year-old woman with cancer of the right breast comes to the office to discuss a right mastectomy with immediate implant reconstruction and a symmetry procedure for the contralateral breast. Physical examination shows bilateral Grade 3 ptosis with volume symmetry. The patient currently wears a size 34C brassiere and desires to remain the same size. Which of the following is the most appropriate procedure for the contralateral breast?

A) Augmentation mammaplasty
B) Fat injections
C) Mastopexy
D) Reduction mammaplasty

A

The correct response is Option C.

Contralateral breast procedures are frequently necessary to achieve symmetry following mastectomy and reconstruction. Options to achieve symmetry include mastopexy, reduction mammaplasty, or augmentation mammaplasty combined with mastopexy. Volume symmetry can be achieved through reduction mammaplasty or fat injections. Positional asymmetry of the contralateral breast and the nipple-areola complex may require correction with mastopexy. Augmentation mammaplasty with mastopexy may be indicated for the correction of volume and positional asymmetry. The most appropriate contralateral procedure in a patient with symmetric volumes of the breast and ptosis of the contralateral breast is mastopexy.

198
Q

A 46-year-old man presents with a midline 18-cm-wide ventral hernia 1 year after undergoing midline exploratory laparotomy for a bowel resection and right end ileostomy. Medical history includes significant weight loss through diet and exercise. His weight has been stable for 2 years. BMI is 29 kg/m2. He undergoes bilateral component separation with biologic mesh bridged between the rectus muscles and concomitant panniculectomy. Which of the following clinical characteristics will most likely increase the likelihood of hernia recurrence?

A) BMI greater than 24.9 kg/m2
B) Bridged biologic mesh hernia closure
C) Concomitant panniculectomy
D) Presence of an end ileostomy
E) Prior abdominal surgery

A

The correct response is Option B.

The patient presents after significant weight loss with a wide midline ventral hernia, right end ileostomy through his rectus muscle, and an abdominal pannus. Given the 18-cm waist of the hernia defect, he is being counseled that only a bridged repair with a biologic mesh will be possible rather than total muscular coverage for the midline defect. Hernia recurrence is a major problem for patients and can be associated with specific characteristics. When the technique of bilateral component separation and inlay biologic mesh repair is being performed, the most important predictor of recurrence is whether the rectus muscle and fascia will be able to be closed at midline, creating a total submuscular repair, or whether the mesh will be bridged. A bridged repair is associated with a 33% chance of recurrence at 3 years compared to 6.2% for total muscle coverage with fascial closure at midline.

With a BMI of 29 kg/m2, the patient remains overweight despite his prior stable weight loss. Surgical site occurrences are increased in the overweight patient with a 26.4% incidence versus 14.9% in patients with BMI less than 24.9 kg/m2. Similarly, skin dehiscence is significantly increased in the overweight patient (19.3% versus 7.2%), while hernia recurrence rates are not statistically significant (11.4% versus 7.7%). Concomitant panniculectomy was associated with an increase in surgical site occurrences and skin dehiscence, but hernia recurrence rates were not affected.

Similarly, patients with existing ileostomies or stomas complicated by parastomal hernias do have a significantly increased surgical site occurrence rate (34.1% with parastomal and midline hernia versus 18.7% with midline hernia only) but hernia recurrence rates are not affected. Prior abdominal surgery will be in the clinical history of all incisional hernia patients.

199
Q

A 12-year-old boy with a thyroglossal duct cyst undergoes a Sistrunk procedure. Which of the following structures are resected during this procedure?

A) Cyst and cyst tract only
B) Cyst, cyst tract, and middle third of the cricothyroid cartilage
C) Cyst, cyst tract, and middle third of the hyoid bone
D) Cyst, cyst tract, and middle third of the thyroid cartilage
E) Cyst, cyst tract, and the pyramidal lobe of the thyroid

A

The correct response is Option C.

The Sistrunk procedure is the operation of choice for thyroglossal duct cysts. This operation involves resection of the cyst, the cyst tract, and the middle third of the hyoid bone. In the Sistrunk procedure, the thyroid cartilage is not removed, nor is the cricothyroid cartilage. If, upon exploration, the distal tract is found to be in communication with the pyramidal lobe of the thyroid, then the communication should be excised. Despite this, resection of the pyramidal lobe of the thyroid is not a standard component of the Sistrunk procedure.

References

LaRiviere CA, Waldhausen JH. Congenital cervical cysts, sinuses, and fistulae in pediatric surgery. Surg Clin North Am. 2012;92(3):583-597.

Povey HG, Selvachandran H, Peters RT, Jones MO. Management of suspected thyroglossal duct cysts. J Pediatr Surg. 2018;53(2):281-282.

200
Q

A 20-year-old rugby player is evaluated for an acute thumb injury 4 hours after falling onto his outstretched hand while being tackled. X-ray study is shown. Surgical intervention is planned. Which of the following is the most appropriate force to apply to the thumb metacarpal during reduction?

A) Abduction
B) Axial compression
C) Flexion
D) Supination

A

The correct response is Option A.

In a Bennett fracture, the smaller volar-ulnar fragment is retained by the anterior oblique (beak) ligament, while the abductor pollicis longus, thumb extensor tendons, and the adductor pollicis combine to distract the base of the larger shaft fragment radially, dorsally, and proximally. These distracting forces create joint incongruity, which is a relative indication for fracture reduction. These forces must be countered to reduce the fracture, thus, requiring axial distraction, pronation, and abduction of the metacarpal shaft, while simultaneously applying external pressure at the radial base of the metacarpal.

Axial compression will worsen proximal migration of the metacarpal shaft. Extension will worsen dorsal displacement, and supination will further distract the volar surfaces of the fracture fragments. Application of these forces will not promote fracture reduction.

201
Q

A 54-year-old woman with breast cancer undergoes a skin-sparing mastectomy with tissue expander reconstruction. Adjuvant chemotherapy and subsequent radiation therapy have been recommended. Chemotherapy proceeds during tissue expansion. Radiation may be performed either before or after the implant exchange procedure. When compared with radiating the permanent implant, radiating the tissue expander is most likely to increase the risk of which of the following?

A) Cancer recurrence
B) Capsular contracture
C) Device rupture
D) Explantation
E) Radiation dermatitis

A

The correct response is Option D.

Cordeiro et al. updated the largest series of women undergoing two-stage implant breast reconstruction who require postmastectomy radiation. The authors found that radiating the tissue expander, as opposed to the permanent implant, increased the rate of reconstructive failure by 46%. However, aesthetic results were better and capsular contracture was less frequent. There were no differences in patient-reported outcomes. This study confirmed the earlier findings of Nava et al. regarding reconstruction failure (explantation). There are no known differences in cancer recurrence between the two approaches discussed.

202
Q

A 32-year-old woman is scheduled to undergo augmentation mammaplasty with highly cohesive, anatomically shaped, silicone-filled breast implants. She asks the surgeon about postoperative monitoring for implant rupture. This patient should be counseled that, according to FDA recommendations, postoperative monitoring for rupture most appropriately includes which of the following?

A) Manual examination 3 years postoperatively, then annually thereafter
B) MRI screening 2 years postoperatively, then every 3 years thereafter
C) MRI screening 3 years postoperatively, then every 2 years thereafter
D) Ultrasonography screening 2 years postoperatively, then every 3 years thereafter
E) Ultrasonography screening 3 years postoperatively, then every 2 years thereafter

A

The correct response is Option C.

Diagnosis of rupture is difficult by physical examination alone, which is why the majority of ruptures are silent. Subsequent MRI screening for silent rupture is recommended initially 3 years postoperatively, then every 2 years thereafter.

Highly cohesive, anatomically shaped, silicone-filled breast implants combine the “gummy bear” silicone with an anatomical shape, in which inferior pole projection is higher than the superior pole projection. In studies of Allergan’s Natrelle 410 breast implants (the “Pivotal Study,” the 410 Swedish MRI study, and the 410 European MRI study) approximately 3 in 100 women had silent ruptures.

Cohesive gel is still subject to rupture, because rupture occurs when the shell fails. In cohesive implants, however, as opposed to noncohesive implants, the rupture rarely becomes extracapsular.

203
Q

A 59-year-old woman presents with an infected sternal nonunion after coronary artery bypass grafting 4 weeks ago. After debridement of the wound, five sternal plates and bilateral pectoralis flaps are placed. Postoperatively, the patient becomes hypotensive, tachycardic, and confused. Jugular distention is noted. Oxygen saturation is 100% on nasal cannula. Which of the following is the most appropriate initial step in management?

A) Auscultation
B) Chest x-ray
C) ECG
D) Ultrasonography of the heart
E) Return the patient to the operating room

A

The correct response is Option A.

On auscultation a muffled heart sound and pericardial friction rub is heard and would direct the clinician to decompress tamponade.

Patient is demonstrating Beck’s triad and has reason for possible cardiac tamponade.

Immediate chest x-ray can be ordered to help rule out pneumothorax, but with normal oxygenation, the chance of a pneumothorax is lower on the differential, and there are other better initial diagnostic and therapeutic steps.

ECG can help support the diagnosis of pericardial effusion, but this is not diagnostic and is only used as an adjunct.

Ultrasonography of the heart can confirm the existence of pericardial effusion, as well as allow needle drainage for immediate treatment. However, this would be performed after auscultation.

204
Q

Which of the following sequelae is more likely to result from the use of textured silicone gel prostheses rather than smooth silicone gel prostheses?

A ) Capsular contracture

B ) Hematoma

C ) Malposition

D ) Rippling

E ) Rupture

A

The correct response is Option D.

The use of textured prostheses is associated with a significant rate of rippling when compared with smooth prostheses. One study reported over a two-fold increase. Visible rippling can be minimized with subpectoral implantation as well as by limiting the use of these prostheses to patients with more native breast tissue. Rippling is more pronounced with saline-filled prostheses.

Rippling occurs when the breast skin and soft tissue are thin. This rippling will worsen with time because of the skin stretching and thinning. The key to treatment is to thicken the breast skin or change the prosthesis characteristics. Overinflation of saline prostheses is thought to minimize rippling; however, one recent study did not show any difference in the incidence of rippling in underfilled saline prostheses. Surgical treatment for rippling is usually incomplete. Dermal grafts have been used with some success to thicken the rippled breast skin. Changing a saline prosthesis to a cohesive silicone gel prosthesis will also improve rippling. Various flaps can also be used to reinforce the thinned breast skin.

Textured surface prostheses are superior to smooth prostheses in decreasing capsular contracture. However, this advantage is minimal when using saline prostheses in a subpectoral pocket.

The incidence of hematoma formation is similar for both types of prostheses.

Rupture rates for textured gel and saline gel are similar; however, textured saline prostheses have a higher rate of deflation than smooth saline prostheses.

Malposition rates are not higher with the use of textured prostheses.

205
Q

A 65-year-old man who wears glasses sustained a massive injury to the left side of the face causing a ruptured globe with total loss of the upper and lower eyelids. Which of the following is the best aesthetic option to recommend?

A) Eye patch
B) Hemifacial prosthesis
C) Ocular prosthesis
D) Orbital prosthesis

A

The correct response is Option D.

In this case the patient has had severe orbital trauma with loss of lids and globe. Natural-looking and functional total-lid reconstruction is challenging. Lids would be needed to support an ocular prosthesis. An orbital prosthesis would likely provide this patient a comfortable and aesthetically satisfactory prosthesis. Eyeglasses can help mask the seam of the prosthesis. The hemifacial prosthesis is larger than necessary for this patient and has unnatural seams. An eye patch would not improve symmetry or be reconstructive.

206
Q

Hyperbaric oxygen therapy (HBOT) is most appropriate for a patient with which of the following conditions?

A) Acute osteomyelitis of the tibia
B) Anaerobic necrotizing soft-tissue infection
C) Chemical burn because of lye exposure
D) Stevens-Johnson syndrome
E) Wagner grade 2 diabetic foot ulcer

A

The correct response is Option B.

Hyperbaric oxygen therapy (HBOT) is an accepted adjunct to surgical debridement, appropriate antibiotic therapy, and indicated critical care measures for necrotizing soft-tissue infections such as necrotizing fasciitis and Fournier gangrene. The increased oxygen delivery of HBOT improves leukocyte function and can enhance penetration of certain antibiotics such as aminoglycosides. The clinical effects include slowing of the progress of the infection and decreased risk of both amputation and mortality.

There is not adequate evidence to justify HBOT in diabetic foot ulcers (DFUs) with Wagner grade 2 (extension to bone, tendon, or capsule) or less. However, there is moderate evidence to suggest benefit in DFUs with Wagner grade 3 (deep ulcer with osteomyelitis or abscess) or greater.

HBOT may be indicated in the treatment of chronic osteomyelitis but not in the acute setting. It plays no role in the treatment of Stevens Johnson syndrome, and may be beneficial in acute thermal burns but is not indicated for chemical burns.

The complete list of approved indications for HBOT, as determined by the Undersea and Hyperbaric Medical Society and the U.S. Food and Drug Administration, includes the following:

Air or gas embolism

Carbon monoxide poisoning

Clostridial myositis and myonecrosis (gas gangrene)

Crush injury, compartment syndrome, and other acute traumatic ischemias

Decompression sickness

Arterial insufficiency

Severe anemia

Intracranial abscess

Necrotizing soft-tissue infections

Refractory osteomyelitis

Delayed radiation injury (soft tissue and bony necrosis)

Compromised grafts and flaps

Acute thermal burn injury

Idiopathic sudden sensorineural hearing loss

207
Q

A 20-year-old woman comes to the office for evaluation of chronic pain of the right wrist, which is alleviated by nonsteroidal anti-inflammatory drugs (NSAIDs). Osteoid osteoma is suspected on x-ray. Which of the following imaging studies is most likely to confirm the suspected diagnosis?

A) Bone scan
B) CT scan
C) Laser fluorescence angiography
D) Magnetic resonance arthrography
E) Ultrasonography

A

The correct response is Option B.

Osteoid osteoma is a benign bone tumor that arises from osteoblasts; the principal symptom is focal pain at the site of the lesion.

Multiple studies suggest that CT is the best imaging technique for detection of this tumor. Specifically, CT is best at depicting the nidus, the radiolucent area typical of this tumor type. Within the radiolucent nidus, a central area of high attenuation is often seen, representing mineralized osteoid.

As ultrasound waves do not adequately penetrate bone, this intracortical lesion would not easily be detected by this technique.

Although magnetic resonance (MR) has been used to detect these lesions, it is not as sensitive at detecting the nidus as CT scanning. This is because the nidus, especially if it is small, will have signal similar to cortical bone on MR. Although an arthrogram may detect an intra-articular osteoid osteoma, this is a more unusual entity.

A bone scan may show the lesion, but it is nonspecific and will not confirm the diagnosis.

Laser fluorescence angiography has gained popularity for assessing the perfusion of soft tissues (skin, flaps, etc.), but this technique will not help assess bone or tissues of significant depth, nor can it reliably distinguish tumor from other tissue.

208
Q

A 35-year-old woman comes to the office for consultation regarding augmentation mammaplasty. A preoperative mammogram is most indicated if the patient’s history includes which of the following?

A ) A grandmother diagnosed with breast cancer at age 73 years

B ) A mother diagnosed with breast cancer at age 45 years

C ) Personal history of breast cysts

D ) Personal history of fibroadenoma

E ) A sister diagnosed with ovarian cancer

A

The correct response is Option B.

Among the risk factors for breast cancer, family history is the most significant. It can be divided into two broad categories: familial breast cancer, which most likely results from changes in multiple low penetrance genes coupled with environmental influences, and hereditary breast cancer, which results in high penetrance mutation in a single gene.

Familial breast cancer is relatively common and conveys a modest elevation in risk compared with genetic breast cancer, which is rare but associated with high risk.

A family history of breast cancer has been demonstrated to increase the risk of breast cancer in multiple studies. Breast cancer in a first-degree relative increases the risk of breast cancer, and that risk decreases as the age of the affected relative increases (ie, it is a 2.3 relative risk factor if the affected relative is under 50 years of age; it is 1.8 if she is over 50). Individuals whose first-degree relatives have bilateral breast cancer have an increased risk of 5.5 times the normal population.

209
Q

A 33-year-old woman comes to the office with a 6-cm rapidly growing tumor of the left breast. She wears a size 36C brassiere. The tumor has a bluish hue and skin veins are dilated. A phyllodes tumor is diagnosed, and surgical excision is planned. Which of the following is the most appropriate surgical procedure to treat this patient?

A) Excision with 1-cm margin
B) Excision with 2-cm margin
C) Excisional biopsy
D) Modified radical mastectomy
E) Radical mastectomy

A

The correct response is Option A.

Phyllodes are large benign tumors that occur primarily in the perimenopausal patient. Previously, they were referred to as cystosarcoma phyllodes, a term coined in 1838 because the tumors are fleshy and have a gross leaf-like intracanalicular growth pattern. However, this is a misnomer because these tumors do not behave like sarcomas and are rarely malignant. The histologic characteristics that separate fibroadenomas from phyllodes tumors are not well defined and have been somewhat controversial. Nevertheless, phyllodes tumors typically are large fibroadenomas that histologically have more stromal cellularity than that seen in the typical fibroadenoma. The classification of benign versus malignant phyllodes tumors is not sharply delineated, and the term borderline lesion may be more appropriate. Borderline lesions have more mitoses per high-power field and moderate nuclear pleomorphism. They have a tendency to recur after local excision but do not demonstrate true malignant behavior. When metastases of a phyllodes tumor have been reported, there have been obvious sarcomatous elements such as liposarcoma or rhabdomyosarcoma in the lesion.

The surgical treatment of phyllodes tumors has recently been redefined. In the past, simple or radical mastectomies were recommended for the treatment of large phyllodes tumors. Currently, most surgeons perform more conservative surgery. Several clinical studies have recommended the excision of tumors with 1-cm clear margins or mastectomy if breast conservation is impossible.

210
Q

A 65-year-old woman comes to the office because of pain at the base of the right thumb. Which of the following is the most appropriate plain x-ray study view for visualizing thumb basal joint subluxation?

A) Bett view
B) Brewerton view
C) Eaton stress view
D) Roberts view
E) Standard lateral view of the thumb

A

The correct response is Option C.

The Eaton stress view is done with the radial borders of the thumb distal phalanges pressed together. It is a posteroanterior view and assesses laxity of the basal joint as demonstrated by subluxation of the thumb metacarpal on the trapezium.

The Brewerton view is taken with the fingers flat on the x-ray plate with the metacarpophalangeal joints flexed 65 degrees beam angled from a point 15 degrees to the ulnar side of the hand. It shows the metacarpal head and is useful for demonstrating degenerative disease or occult fractures.

Bett (or Gedda) view is characterized as a true lateral view of the trapeziometacarpal joint, perpendicular to the plane of the hand. It is performed as a posteroanterior view, with the hand pronated 30 degrees and the axis of the imaging tube angled 25 degrees distally. The view isolates the trapeziometacarpal joint and is useful for evaluating metacarpal base fractures (Bennett’s fracture).

Roberts view is done with the wrist hyper-pronated and the dorsum of the thumb flat on the plate with an AP view. It is used to evaluate degeneration of the trapeziometacarpal joint but does not show subluxation as the stress view does.

211
Q

A 35-year-old woman with a Stage T2 infiltrating ductal carcinoma is scheduled to undergo a skin-sparing, right total mastectomy and a nipple-sparing, left prophylactic mastectomy. The possibility of adjuvant radiation therapy to the right breast depends on the final surgical pathology. The patient has a history of smoking. BMI is 28 kg/m2. She wears a brassiere with a D cup and would like the postoperative result to be of a similar size. Which of the following immediate bilateral reconstructive techniques is most appropriate for this patient?

A) Abdominal-based free flaps
B) Gluteal-based free flaps
C) Latissimus dorsi myocutaneous flaps and silicone implants
D) Silicone implants and acellular dermal matrix
E) Tissue expanders and acellular dermal matrix

A

The correct response is Option E.

For this patient in whom postoperative radiation therapy is possible, the best first-stage, immediate reconstructive approach is placement of tissue expanders with acellular dermal matrix. The outcome of immediate autologous flap reconstruction may be compromised if subjected to adjuvant radiation therapy and is best delayed until after such treatment has been rendered. Although successful, cost-effective outcomes are possible with a single-stage, direct-to-implant approach, this patient has risk factors for early revision and implant failure due to her large breasts and history of smoking.

212
Q

Which of the following is the most accurate location of the elliptical skin island of a profunda artery perforator (PAP) flap?

A) Anteromedial thigh with the superior border within the groin crease
B) Inferior buttock with the inferior border within the gluteal fold
C) Lateral hip superior to the iliac crest
D) Middle buttock, from the posterior superior iliac spine to the apex of the greater trochanter
E) Posteromedial thigh with the superior border within the gluteal fold

A

The correct response is Option E.

According to the literature, the skin island of the profunda artery perforator (PAP) flap is inferior to the gluteal fold.1,2 An ellipse of the inferior buttock with the inferior border within the gluteal fold describes the skin island of the inferior gluteal artery perforator (IGAP) free flap.3 An ellipse of the anteromedial thigh with the superior border within the gluteal fold describes the transverse upper gracilis (TUG) flap. An ellipse of the middle buttock, from the posterior superior iliac spine to the apex of the greater trochanter, describes the superior gluteal artery perforator (SGAP) flap.3 An ellipse of the lateral hip superior to the iliac crest describes the Rubens or lateral hip flap.4 The only option that correctly identifies the skin island for the PAP flap is an ellipse of the posteromedial thigh with the superior border within the gluteal fold. The superior marking is within or just below the gluteal fold and the inferior marking is roughly 7 cm below the superior marking. The flap is an ellipse so the scar does not extend outside of the gluteal fold.1,2

213
Q

A 23-year-old woman comes to the office for evaluation of bilateral ear keloids. She reports that the keloids developed after she had her ears pierced 5 years ago. She has not had previous surgery for this problem. Which of the following is the most likely recurrence rate after surgical excision with injection of a corticosteroid?

A) 5%
B) 15%
C) 35%
D) 50%
E) 75%

A

The correct response is Option B.

Earlobe keloid formation after piercing is reported to affect approximately 2.5% of the population. Various adjuvant therapeutic modalities, including radiation therapy, intralesional corticosteroids, interferon, 5-fluorouracil, topical silicone, and pressure devices, are used to decrease recurrence rate after surgical excision. A recent meta-analysis looked at the recurrence rate of keloid formation after surgical excision with the use of radiation therapy and intralesional corticosteroids. Recurrence rate after excision with radiation therapy was found to be 14%. The recurrence rate after excision with intralesional corticosteroids was 15.4%. Although radiation therapy had an overall reduced recurrence rate, it was associated with higher cost and more significant potential complications. Five cases of carcinogenesis after radiation therapy have been reported. The main disadvantage of corticosteroid injections was found in most studies to be the pain of injection. Adjuvant corticosteroid injections in conjunction with surgery were performed preoperatively, intraoperatively, or postoperatively in various studies.

214
Q

A 32-year-old woman is evaluated 4 years after undergoing bilateral augmentation mammaplasty with 375-mL, textured, shaped gel implants. She is concerned because over the past 3 months her left breast has grown one cup size larger than her right. On examination, there is no erythema or palpable mass. The left breast is much larger and firmer than the right. The patient started taking an oral contraceptive 6 months ago. Which of the following is the most likely diagnosis?

A) Breast implant–associated anaplastic large cell lymphoma
B) Capsular contracture
C) Double capsule
D) Drug-induced breast hypertrophy
E) Phyllodes tumor

A

The correct response is Option C.

This patient presents with a late seroma (more than 1 year after surgery) following augmentation mammaplasty with textured implants. The most common reason for this late seroma is a benign process related to the textured implant. A double capsule forms when the textured implant surface breaks away from its attachment to the breast parenchyma and forms a double capsule, which can then fill with blood or fluid. This phenomenon can be caused by a known trauma or in the course of daily living.

The possible etiologies for any late seroma include trauma, infection, inflammation, and malignancy. The first step in evaluation is ultrasound and aspiration of the fluid. The fluid should be evaluated for tumor markers (flow cytometry, CD30 T-cell surface protein) and sent for cytology and bacteriology.

This patient could have breast implant–associated anaplastic large cell lymphoma (BIA-ALCL). BIA-ALCL is rare, and it is highly unlikely that this patient has this lymphoma. Nonetheless, it must be ruled out. All patients presenting with a late seroma (more than 1 year after surgery) need to be evaluated for tumor markers. If the patient tests positive, full oncologic evaluation is needed.

Capsular contracture can cause hardening and deformity of the breast; however, it will not cause breast enlargement. A phyllodes tumor presents as a localized breast mass that can grow rapidly. This patient has no palpable masses.

Oral contraceptives can cause unilateral breast enlargement, but this patient is presenting with massive breast enlargement 6 months after starting birth control pills.

215
Q

A 30-year-old man presents with a large open wound to the right thigh. The proposed treatment plan after debridement and establishing a clean wound is to use negative pressure wound therapy (NPWT). Which of the following is the main direct mechanism for wound healing by this method?

A) Improvement in tissue auto-debridement
B) Increase in collagen synthesis
C) Reduction in wound bacterial load
D) Removal of interstitial fluid leading to increased blood flow

A

The correct response is Option D.

Based on the original studies by Moryk, it was hypothesized that the beneficial wound-healing effects of negative pressure wound therapy (NPWT) was a combination of a fluid-based mechanism and a mechanical stress mechanism. The fluid-based mechanism involves the removal of excess interstitial fluid from the wound bed by the vacuum, which results in the interstitial pressure decreasing below the capillary filling pressures, thus allowing “re-opening” of these wound bed capillaries. This leads to improved blood flow within the wound, allowing for granulation tissue formation. The mechanical strain mechanism is created by micro-strain forces created by the vacuum on the cells within the wound. Cellular deformation leads to numerous molecular changes, including activation of the vascular endothelial cell growth factor (VEGF) pathway, which enhance angiogenesis.

Collagen synthesis is not directly affected by NPWT. There is equivocal evidence for whether there is a positive or negative effect of NPWT on wound bacterial loads.

NPWT does not auto-debride wounds. It is important when using NPWT to ensure adequate mechanical debridement of nonviable tissues from the wound bed prior to initiating NPWT.

216
Q

A 58-year-old woman develops full-thickness dermal necrosis in a 4 × 4 × 2-cm area of her lower breast following reduction mammaplasty. After debridement to healthy tissue, she starts daily wound packing with a calcium alginate fiber dressing. The main advantage of calcium alginate versus saline gauze dressings is a decrease in which of the following?

A) Dressing change frequency
B) Healing time
C) Keloid scarring
D) Treatment cost
E) Wound infection rate

A

The correct response is Option A.

An effective dressing should aid in surface debridement, absorb wound exudate, and maintain a moist healing environment. Normal saline wet-to-dry gauze dressings have been a mainstay of wound management for generations because they are easy to perform, widely available, and inexpensive. They are best changed 2 to 3 times daily to remove exudative material because they can quickly become saturated. Some of the wound healing byproducts, such as metalloproteinases and elastase, can slow down wound healing and result in chronic wounds. By wicking away these potentially harmful agents, more absorptive wound dressings can help simplify care. Alternatives to conventional saline wet-to-dry gauze dressing materials include hydrogels, hydrocolloids, foams, alginates, and negative pressure dressings. They are more expensive than traditional saline-gauze dressings but are typically far more absorptive, allowing for less frequent dressing changes. Daily dressing changes versus two to three times a day are far more convenient for patients and may ultimately save total treatment costs by allowing for fewer nursing visits or allowing for outpatient care. Many studies show no difference in healing times, though some studies suggest a mild benefit in diabetic foot ulcers. Current recommendations call for additional studies, as evidence of faster healing times is lacking. No studies show lower infection or scarring.

Calcium alginates are fibers made of brown seaweed fibers, and they can hold more than ten times their weight in fluid. Some manufacturers claim that they are able to deactivate metalloproteinases and stimulate healing, although in vitro data are lacking. They are a comfortable and effective alternative to saline wet-to-dry dressings, albeit at a higher product cost.

217
Q

A 45-year-old woman comes to the office for consultation regarding mastectomy and immediate breast reconstruction because of recurrent right-sided breast cancer. She underwent breast-conserving therapy and radiation therapy 5 years ago. BMI is 23 kg/m2. The patient is otherwise healthy, and she does not smoke cigarettes. On examination, the breast is soft with obvious fibrosis. Use of which of the following has the highest risk for reconstructive failure in this patient?

A) Deep inferior epigastric artery perforator (DIEP) flap
B) Free transverse rectus abdominis musculocutaneous (TRAM) flap
C) Latissimus flap with prosthesis
D) Pedicled TRAM flap
E) Tissue expander and prosthesis

A

The correct response is Option E.

Pre-reconstruction radiation is typically seen in two groups of patients: those who underwent mastectomy without reconstruction followed by radiation therapy, or those who underwent breast-conserving therapy and radiation with recurrence or new cancer. Although these patients may present with a reasonable skin envelope, complication rates associated with tissue expander/prosthesis have been reported as high as 70%, with a 40% rate of failure or conversion to flap. Additionally, aesthetic outcomes in most patients who completed expander/prosthesis reconstruction after previous radiation therapy were deemed good or very good compared with the majority of non-irradiated patients who achieved excellent results. Tissue expander/prosthesis can be considered as an option for patients with a history of previous irradiation who wish to avoid the scars and recovery of flap-type reconstructions; however, they should be counseled of the high risks of complications.

The correct response is Option E.

Pre-reconstruction radiation is typically seen in two groups of patients: those who underwent mastectomy without reconstruction followed by radiation therapy, or those who underwent breast-conserving therapy and radiation with recurrence or new cancer. Although these patients may present with a reasonable skin envelope, complication rates associated with tissue expander/prosthesis have been reported as high as 70%, with a 40% rate of failure or conversion to flap. Additionally, aesthetic outcomes in most patients who completed expander/prosthesis reconstruction after previous radiation therapy were deemed good or very good compared with the majority of non-irradiated patients who achieved excellent results. Tissue expander/prosthesis can be considered as an option for patients with a history of previous irradiation who wish to avoid the scars and recovery of flap-type reconstructions; however, they should be counseled of the high risks of complications.

218
Q

An otherwise healthy 52-year-old woman is scheduled to undergo bilateral breast reconstruction with abdominal perforator flaps. She does not smoke cigarettes. Which of the following is the most appropriate antibiotic prophylaxis regimen?

A) Preoperative antibiotics
B) Preoperative chlorhexidine bath
C) Preoperative and intraoperative antibiotics
D) Preoperative, intraoperative, and postoperative antibiotics for 7 days
E) Preoperative, intraoperative, and postoperative antibiotics until all drains are removed

A

The correct response is Option C.

For a healthy patient undergoing a lengthy autologous breast reconstruction, the most appropriate regimen of antibiotic prophylaxis consists of a preoperative dose of intravenous antibiotic and intraoperative antibiotics to ?maintain therapeutic levels during operation and, at most, a few hours after closure? (Mangram, et al). Although preoperative chlorhexidine bathing is supported by some studies and strong theoretical rationale, it is not nearly as effective as antibiotic prophylaxis on its own. A single dose of preoperative antibiotic intravenously is appropriate for short procedures; however, therapeutic levels of antibiotic should be maintained throughout the lengthy surgery described. Continued administration of prophylactic antibiotics for 7 days following surgery or until drains are removed has not been proven to reduce the incidence of surgical site infections. This regimen also promotes drug resistance and adverse patient reactions.

219
Q

A 46-year-old woman undergoes a fleur-de-lis abdominoplasty following successful gastric bypass surgery. She has a 94-lb (43-kg) weight loss and current BMI is 28 kg/m2. To ensure the best outcome for wound healing, which of the following is the recommended postoperative daily intake of protein for this patient?

A) 20 to 30 g/day
B) 40 to 50 g/day
C) 60 to 70 g/day
D) 80 to 90 g/day
E) More than 100 g/day

A

The correct response is Option C.

Nutritional status in postbariatric subjects is essential in achieving successful healing of surgical wounds. Anatomical changes to the gastrointestinal tract following bariatric surgery can exacerbate nutritional deficiencies and inadequacies. Reduced protein intake has been associated with significantly lower healing rates in massive weight loss patients. It is recommended that postbariatric patients consume 60 to 70 g/day of protein 2 to 4 weeks prior to surgery and for 1 to 2 months postoperatively.

220
Q

During embryological development, which of the following structures is derived from the same pharyngeal arch as the antihelix?

A) Mandible
B) Maxilla
C) Mylohyoid
D) Stapes
E) Tensor veli palatini

A

The correct response is Option D.

The pharyngeal arches are created during embryological development by the migration of the neural crest cells and the surrounding pharyngeal endoderm and mesoderm. The first four pharyngeal arches are the most prominent and have unique nervous, arterial, muscular, and bony components. The muscular components of the first pharyngeal arch are innervated by the trigeminal nerve (cranial nerve V) and include the muscles of mastication, as well as the anterior digastric, mylohyoid, tensor tympani, and tensor veli palatini. The maxillary artery supplies blood to the components of the first pharyngeal arch. The greater wing of the sphenoid, as well as the incus, the malleus, maxilla, zygomatic, mandible, and temporal bones comprise the bony components of the first pharyngeal arch. The second pharyngeal arch is innervated by the facial nerve (cranial nerve VII) and receives blood supply from the stapedial artery. The bony components of the second pharyngeal arch include the stapes, styloid process, stylohyoid ligament, lesser horn, and upper body of the hyoid. The muscular components of the second pharyngeal arch include the muscles of facial expression, as well as the posterior digastric, stylohyoid, and stapedius.

During embryological development of the external ear, the antihelix, antitragus, and lobule are derived from the three posterior hillocks of the second pharyngeal arch, while the tragus, the root of the helix, and the superior helix are derived from the three anterior hillocks of the first pharyngeal arch.

References

Afshar M, Brugmann SA, Helms JA. Embryology of the craniofacial complex. In Rodriguez ED, Losee JE, Neligan PC, eds. Craniofacial, Head and Neck Surgery, and Pediatric Plastic Surgery. 3rd ed. London: Elsevier Saunders, 2013. Plastic Surgery; vol 3.

Sadler TW. Langman’s Medical Embryology. Baltimore, MD: Lippincott Williams & Wilkins; 2012.

221
Q

A 53-year-old man comes to the office because of a 2-year history of increasing pain of the radial aspect of the right wrist. X-ray studies show scapholunate disruption and arthritis. Which of the following joints is most likely to be affected first by the arthritic degeneration in this patient?

A) Capitolunate
B) Radiolunate
C) Radioscaphoid
D) Radioulnar
E) Scaphotrapezio

A

The correct response is Option C.

This patient has early-stage scapholunate advanced collapse (SLAC) wrist. SLAC wrist is generally categorized by the Watson classification, which is a descriptive classification but also helps determine management options.

The Watson classification is as follows:

Stage I: Arthritis between scaphoid and radial styloid
Stage II: Arthritis between scaphoid and entire scaphoid facet of the radius
Stage III: Arthritis between capitate and lunate

While original Watson classification describes preservation of radiolunate joint in all stages of SLAC wrist, subsequent description by other surgeons of pancarpal arthritis (stage IV) observed rare cases in which the radiolunate joint is affected. The radioulnar joint is not affected by SLAC wrist.

222
Q

A 58-year-old woman undergoes removal of round 280-cc silicone gel implants she has had for over 30 years. New silicone gel implants measuring 10 cm in width by 12 cm in height with a 5-cm projection are placed. Compared with her original gel implants, the new implants are more likely to have a higher rate of which of the following complications?

A) Contracture
B) Infection
C) Rippling
D) Rotation
E) Rupture

A

The correct response is Option D.

Breast implant technology has evolved greatly since implants were introduced in the 1960s. Increased cross-linking of silicone polymers (polydimethylsiloxanes) results in a more stable, cohesive form and closer shell-gel interactions. Advantages of these more “form-stable” implants include lower rates of rippling and rupture. They allow for the creation of shaped implants that offer clear advantages for certain patients, such as those seeking a natural upper pole shape transition, and those with wider or taller breast shapes.

The biggest drawback of shaped implants is the need to place them in a precise surgical pocket lest they rotate, causing deformity and potentially requiring reoperation. As long as surgeons follow sound surgical principles of dissecting an appropriate pocket limited to the approximate width of the implant, malrotation rates are low, typically in the 1.5% range. In one study, half of patients with implant rotation improved with manual repositioning and taping for 3 to 6 weeks, while the other half required reoperation.

Infection rates do not vary among implant types.

Shaped implants have textured shells, which have been shown to have lower rates of capsule contracture, particularly in the subglandular position.

Implant rupture rates are also lower in new generation implants, in the 0.7% per year range.

Visible rippling rates are more common in thinner consistency implants, such as saline and older silicone devices.

223
Q

A 56-year-old woman is evaluated 6 hours after undergoing bilateral breast reconstruction with a deep inferior epigastric perforator (DIEP) flap. Doppler examination shows strong arterial signals in both flaps. The right breast appears bluish with a capillary refill time of 1 second compared to 3 seconds on the left side. Which of the following is the most appropriate next step?

A) Administration of tissue plasminogen activator
B) Application of leeches
C) Application of nitroglycerin ointment
D) Exploration in the operating room
E) Observation

A

The correct response is Option D.

The patient described has evidence of venous congestion. The reported incidence of venous congestion in free tissue breast reconstruction ranges from 2 to 20%. Causes include venous thrombosis, inadequate perforator selection, and superficial venous system dominance with lack of sufficient communication to the deep system. Signs of venous compromise include the following: cyanotic/blue color, brisker than normal capillary refill, increased tissue turgor, cooler temperature compared to normal skin (greater than 2 degrees), rapid bleeding of dark blood with pinprick, and absence of continuous venous Doppler signal. The most appropriate course of action in this scenario is emergent exploration in the operating room to assess the vascular pedicle for thrombosis, compression from hematoma, kinking, or superficial system dominance. Flap salvage rate is directly tied to timing of exploration, with higher salvage rates in flaps explored within 6 hours of identification of compromise.

Early recognition and rapid exploration of compromised flaps are the most important factors predicting flap salvage, so observation would be unacceptable. Tissue plasminogen activator is useful if diffuse clotting is suspected within the flap, but should only be given locally within the flap. Leeches can be a useful adjunct postoperatively after employing the other maneuvers described above, but would not resolve the underlying problem in this case. Application of topical nitroglycerin can improve venous congestion in random skin flaps, but has no role in the management of acute microvascular thrombosis.

224
Q

An 86-year-old farmer is brought to the emergency department because of a large dorsal wound of his nondominant left hand sustained when his hand was caught in a flail mower. After debridement is performed, examination shows normal volar structures, including nerve and tendon function, and loss of dorsal skin and tendons. Medical history includes myocardial infarction 1 month ago. An x-ray study and photograph are shown. Which of the following is the most appropriate method of reconstruction for this patient?

A) Bilaminate neodermis (Integra) and skin grafting with delayed bone grafting
B) External fixator and posterior interosseous artery flap
C) Finger fillet flaps of index and middle finger
D) Free anterior lateral thigh flap with secondary bone grafting
E) Pedicle radial forearm flap with secondary bone grafting

A

When caring for patients with mangling hand injuries, it is imperative to consider all aspects of the patient’s history and future goals. This patient would be at risk for cardiac complications if a longer procedure such as a free flap were chosen. The amount of bone loss in the index and middle metacarpals is also problematic and would most likely require multiple procedures. Just placing an external fixation and covering the wound with a local flap is also possible but will require several procedures. Bone grafting while receiving bilaminate neodermis (Integra) and skin grafts is not recommended because of the lack of subcutaneous tissue and poor blood supply. The most expeditious method of covering this patient’s wounds in one procedure is finger fillet flaps of the injured digits. Finger fillet flaps can cover a large area for reconstruction as shown.

References

Leunam S, Prugsawan K, Kosiyatrakul A ,et al. Neural anatomy of the anterolateral thigh flap. J Hand Microsurg. 2015;7:49-54.

Lachia RD. Evidence-Based medicine: management of acute lower extremity trauma. Plast Reconstr Surg. 2017;139:287-301e.

225
Q

Oral contraceptive use increases the incidence of which of the following histologic risk factors for breast cancer?

A) Apocrine metaplasia
B) Atypical hyperplasia
C) Intraductal hyperplasia
D) Intraductal papilloma
E) Sclerosing adenosis

A

The correct response is Option B.

Patients in whom an otherwise benign breast biopsy shows atypical hyperplasia have a 4.5- to 5-fold increased risk for developing breast cancer. Proliferative breast disease without atypia increases the risk 1.5- to 2-fold. Oral contraceptives have shown to decrease the occurrence of all proliferative forms of benign breast disease without atypia, such as intraductal hyperplasia, intraductal papilloma, and sclerosing adenosis. Apocrine metaplasia is a non-proliferative histologic change and carries no increased risk for breast cancer. Of the choices, only atypical hyperplasia is increased with oral contraceptive use.

226
Q

A 24-month-old infant is evaluated because of a mass on the lower lip that her parents first noticed 1 month ago. Physical examination shows a mucosal lesion of the right lower lip that is round, clear, translucent, soft, and measures approximately 0.5 cm in diameter. Which of the following is the most appropriate next step in management?

A) CT scan of the face without contrast
B) Excisional biopsy
C) Fine-needle aspiration
D) MRI of the face with contrast
E) Ultrasonography of the lesion

A

The correct response is Option B.

The lesion described is a mucocele, which is a type of cyst likely caused by minor salivary gland mucin seepage. They are most frequently on the lower lip mucosa away from the midline, but they can present anywhere there is oral mucosa. A minority of these resolve on their own, therefore, the majority of the time an excisional biopsy is recommended.

Imaging is not indicated for this type of lesion. Fine-needle aspiration is not indicated.

References

Glickman A, Karlis V. Pediatric benign soft tissue oral and maxillofacial pathology. Oral Maxillofac Surg Clin North Am. 2016 Feb;28(1):1-10.

Mínguez-Martinez I, Bonet-Coloma C, Ata-Ali-Mahmud J, Carrillo-García C, Peñarrocha-Diago M, Peñarrocha-Diago M. Clinical characteristics, treatment, and evolution of 89 mucoceles in children. J Oral Maxillofac Surg. 2010 Oct;68(10):2468-2471.

Yuhan BT, Svider PF, Mutchnick S, Sheyn A. Benign and malignant oral lesions in children and adolescents: an organized approach to diagnosis and management. Pediatr Clin North Am. 2018 Oct;65(5):1033-1050.

227
Q

A 56-year-old woman with a history of systemic sclerosis (scleroderma) is evaluated for intractable pain and progressive ulceration to the right index and middle fingers despite medical management. Duplex ultrasonography shows no identifiable vascular occlusion in the affected digits. Which of the following is the most appropriate surgical management?

A) Interposition bypass grafting
B) Intra-arterial TPA
C) Sympathectomy
D) Thrombectomy
E) Venous arterialization

A

The correct response is Option C.

For patients who have patent arterial inflow on imaging, spasm is likely to be responsible for their ischemia. Spasm is most common in those with autoimmune disease. Digital sympathectomy involves stripping the adventitia from the radial, ulnar, and digital arteries in an effort to decrease sympathetic input that is the presumed cause of pathologic vasoconstriction. Vascular occlusion with a satisfactory distal target may require an interposition bypass. Occlusion without a distal target for bypass may require venous arterialization. In the absence of evidence of occlusion, there is no indication for thrombolytic therapy.

References

Thibaudeau S, Serebrakian AT, Gerety PA, Levin LS. An Algorithmic Approach to the Surgical Treatment of Chronic Ischemia of the Hand: A Systematic Review of the Literature. Plast Reconstr Surg. 2016 May;137(5):818e-28e.

Cappelli L, Wigley FM. Management of Raynaud Phenomenon and Digital Ulcers in Scleroderma. Rheum Dis Clin North Am. 2015 Aug;41(3):419-38.

Flatt AE. Digital artery sympathectomy. J Hand Surg Am. 1980 Nov;5(6):550-6.

228
Q

A 23-year-old woman comes to the office for consultation regarding surgical correction of a tuberous breast deformity. On physical examination, which of the following characteristics is most likely in this patient?

A) Absence of the sternal head of the pectoralis muscle
B) Effacement of the inframammary fold
C) Grade III ptosis of the nipple-areola complex
D) Herniation of breast tissue into the nipple-areola complex
E) Macromastia

A

The correct response is Option D.

Physical examination of a tuberous breast would show herniation of the nipple-areola complex. A constricted inframammary fold, rather than an effaced inframammary fold, is often associated with tuberous breast deformity. Macromastia and/or grade III ptosis of the nipple-areola complex are not standard components of tuberous breast deformity. Absence of the sternal head of the pectoralis muscle is a characteristic feature of Poland syndrome.

229
Q

A 3-year-old child with pectus excavatum deformity is evaluated for surgical correction of the chest wall. The child has experienced mild respiratory insufficiency. Which of the following is the optimal timing of treatment for this patient?

A) Surgical correction between ages 2 and 5
B) Surgical correction between ages 6 and 12
C) Surgical correction between ages 13 and 17
D) Surgical correction at skeletal maturity

A

The correct response is Option B.

Pectus excavatum is the most common congenital chest wall deformity, occurring in approximately 1 in 400 live births. The condition is more common in males, and there is a positive family history in 30 to 40% of patients. The etiology is thought to be multifactorial and associated with increased incidence of congenital cardiac abnormalities, connective tissue disorders (e.g., Marfan and Ehlers-Danlos syndromes), and scoliosis. Treatment options have shifted from the traditional open technique involving sternal osteotomy and resection of abnormal costal cartilage to minimally invasive options such as the Nuss procedure and minimally invasive technique for repair of excavatum (MIRPE), which utilizes thoracoscopy and placement of intrathoracic retrosternal support bars to reposition the sternum and allow gradual remodeling over a period of 2 to 4 years. The ideal timing of repair is mid-adolescence, usually between ages 6 and 12.

230
Q

Which of the following is the most common cause of litigation in cosmetic breast surgery?

A) Assault
B) Failure to diagnose or treat an injury related to the procedure
C) Lack of informed consent
D) Negligence
E) Retained surgical instrument

A

The correct response is Option D.

Plastic surgery faces one of the highest proportions of malpractice claims compared with other medical specialties. A number of studies have revealed that breast-related surgeries account for 37% of overall claims against plastic surgeons. The most common cause of action is negligence, related either to lack of appropriate knowledge or skill or to failing to meet the standard of care. The second most common cause of action is lack of informed consent. Lack of informed consent results from the failure of the physician to thoroughly discuss the risks associated with surgery and the options of alternative therapies. Other causes of action include failure to diagnose or treat injury related to the procedure, retained surgical materials, assault, and distortion of physician’s credentials.

231
Q

A 60-year-old woman is evaluated in the emergency department after she fell on her outstretched hand while playing tennis. Examination shows tenderness in the dorsal and volar aspects of the wrist. Which of the following bones was most likely fractured in this patient?

A) Capitate neck
B) Distal radius
C) Lunate body
D) Scaphoid wrist
E) Triquetral ridge

A

The correct response is Option B.

Distal radius fractures usually occur in adults older than 40 years and are more common in women than in men due to the higher incidence of osteoporosis in women. The most common mechanism is a fall on an outstretched hand.

After distal radius fracture, the next most common fracture of the wrist is scaphoid, followed by triquetrum, trapezium, and lunate.

232
Q

A 41-year-old woman presents with right breast lobular carcinoma in situ (LCIS) involving a 1-cm area with no palpable axillary nodes. According to the TNM staging system, which of the following is this patient’s T classification?

A) Tx
B) Tis
C) T0
D) T1a
E) None; there is no TNM staging for LCIS

A

The correct response is Option E.

Lobular carcinoma in situ (LCIS) has been removed from the staging classification system in the 8th edition and is no longer included in the pathologic tumor in situ (pTis) category. LCIS is treated as a benign entity with an associated risk for developing carcinoma in the future but not as a malignancy capable of metastases. There is a small subset of LCIS that has high-grade nuclear features and may exhibit central necrosis. This subset has been referred to as pleomorphic LCIS and has histologic features that partially overlap the features of ductal carcinoma in situ (DCIS), including the potential to develop calcifications detectable by mammography. The expert panel debated whether to include this variant of LCIS in the pTis category; however, there are insufficient data in the literature regarding outcomes and reproducible diagnostic criteria for this LCIS variant. Cases exhibiting DCIS and LCIS are classified as pTis (DCIS).

233
Q

A 54-year-old woman with BRCA mutation is scheduled for a bilateral nipple-sparing mastectomy. Which of the following would lead to the highest risk for postoperative nipple necrosis?

A) Grade 2 breast ptosis
B) Periareolar mastectomy incision
C) Previous excisional breast scar
D) Tumor greater than 1.5 cm
E) Use of smooth round gel implants

A

The correct response is Option B.

Nipple-sparing mastectomy and direct to implant reconstruction is becoming increasingly popular in the setting of prophylactic mastectomies. The criteria for nipple-sparing mastectomies have been increased to tumors not involving the nipple-areola complex, with some surgeons requiring a distance of 2 cm from the nipple and others espousing just a clean surgical margin at the nipple. Nipple-sparing mastectomies do not require recreating the breast envelope and therefore direct to implant reconstruction is possible. Increased risk for incidence of nipple necrosis in the setting of direct to implant reconstruction is associated with mastectomy incision involving the areola. The use of smooth round gel implants, grade 2 ptosis, tumor size, or previous breast biopsy scars are not associated with nipple necrosis.

234
Q

A 52-year-old woman with breast cancer undergoes right mastectomy and reconstruction with a free transverse rectus abdominis musculocutaneous (TRAM) flap. The procedure is uneventful. In the recovery room, the patient’s husband says that she has been smoking one pack of cigarettes daily up to the day of surgery. Which of the following postoperative complications is most likely to occur?

A) Flap hematoma
B) Mastectomy skin loss
C) Microvascular thrombosis
D) Partial flap loss
E) Superficial infection

A

The correct response is Option B.

Patients who smoke cigarettes and who undergo breast reconstruction with a free flap have a higher rate of mastectomy skin loss. In addition, they have a high rate of donor site abdominal flap necrosis, umbilical necrosis, and hernias. There is no increase in microsurgical complications, flap-related complications (partial flap loss or fat necrosis), infections, or hematomas.

Current recommendations are for patients to stop smoking at least 4 weeks prior to breast reconstruction. Patients who quit smoking prior to this period have a lower risk of perioperative complications when compared to active smokers.

235
Q

A 49-year-old woman is scheduled to undergo subglandular augmentation mammaplasty with silicone prostheses. During the preoperative discussion, the patient asks about postoperative complications with silicone versus saline prostheses. Which of the following is a disadvantage of using silicone in this patient?

A) Their rupture results in an obvious decrease in breast size
B) They are more likely to result in invasive breast cancer
C) They can obscure breast tissue on mammagraphy
D) They may show more rippling

A

The correct response is Option C.

Silicone prostheses are radiopaque on mammography. Therefore, when placed in the subglandular position, a small percentage of breast tissue is obscured on mammography. Breast prostheses made completely of or in part with silicone have not been shown to cause a delay in detection of breast cancer. Women with breast prostheses are not more likely to develop breast cancer. Women with breast prostheses who have developed breast cancer are not diagnosed at a more advanced stage and do not have a worse prognosis or survival when compared with women without prostheses. Silicone prostheses are less likely to show superior pole rippling when compared with saline prostheses. If a saline prosthesis ruptures, the saline tends to become absorbed by the body, resulting in an obvious decrease in breast size after a few days. When silicone prostheses rupture, the silicone may remain intracapsular. These ruptures may change the breast shape slightly but usually do not change the size and are often subclinical.

236
Q

Which of the following factors is most likely to lead to a patient undergoing breast reconstruction after mastectomy?

A) BRCA1/BRCA2 status
B) Insurance coverage through Medicaid
C) Living close to a reconstructive surgeon
D) Patient age
E) Residence in the southeastern United States

A

The correct response is Option C.

Several large population-based studies in different regions of the United States show that provision of breast reconstruction is not uniform among all mastectomy patients. The two biggest factors are geographic distance from reconstructive surgeons and insurance status. Patients who live farther away from providers of reconstruction and those uninsured or with Medicaid have a much lower incidence of reconstruction compared to the overall cohorts. Most studies also show BRCA positive and younger patients tend to have a higher incidence of bilateral reconstruction.

237
Q

A 35-year-old woman is evaluated for long-term follow-up 9 years after undergoing bilateral augmentation mammaplasty for hypomastia by another surgeon. The mammaplasty was performed with 450-mL smooth, round, silicone subglandular implants. The patient recently found out she is BRCA2 positive and underwent MRI of the breasts as part of a surveillance study. The MRI showed a “linguine sign” in the right breast. Which of the following findings on physical examination is most consistent with the diagnosis associated with the “linguine sign”?

A) The right breast has more rippling than the left breast
B) The right breast is not significantly different from the left breast
C) The right breast is significantly larger than the left breast
D) The right breast is significantly smaller than the left breast

A

The correct response is Option B.

The right breast is not significantly different from the left breast. The linguine sign describes multiple low-signal curvilinear lines on MRI that correlated to the collapsed implant shell. It is an indication of intracapsular rupture. Physical examination alone is not specific or sensitive enough to diagnose all cases of intracapsular rupture. Ultrasound and/or MRI is recommended. The physical examination finding of one breast that is smaller, firmer, and higher than the other is indicative of capsular contracture. MRI is not a sensitive predictor of capsular contracture. A right breast that is significantly larger than the left breast would indicate a late seroma and a workup for breast implant-associated anaplastic large-cell lymphoma would be indicated. A right breast that is significantly smaller than the left breast would be indicative of a ruptured saline implant. Increased rippling is not expected with an intracapsular rupture.

238
Q

Use of negative pressure wound therapy with a sponge dressing is likely to result in an adverse outcome in which of the following clinical scenarios?

A) A hand wound with exposed tendons
B) A lower extremity wound with exposed femoral vessels
C) A Stage IV sacral pressure sore
D) As a bolster over a split thickness skin graft on the arm
E) Over a closed surgical incision after hernia repair

A

The correct response is Option B.

Negative pressure wound therapy (NPWT) has dramatically improved our ability to manage complicated and complex wounds. Advantages of NPWT include decreased healing time, simplified wound care with less frequent dressing changes, and promotion of healthy granulation tissue. However, there are several contraindications to the use of NPWT. The presence of exposed vessels is an absolute contraindication to its use; arterial rupture has been reported with multiple fatalities. In addition to placement over exposed vessels, significant bleeding has also been noted with use of NPWT in groin and sternal wounds, in patients on blood-thinning medications, and during removal of well-integrated foam pieces. Other complications include infections due to retained pieces of foam.

In addition to exposed vessels, NPWT should not be used in the setting of active, uncontrolled infection, malignant wounds, wounds with unexplored and/or nonenteric fistulas, and poorly debrided wounds.

Although NPWT may not be the definitive wound management in the other scenarios listed, there is no contraindication for its use.

239
Q

The capsules from patients with breast implant–associated anaplastic large-cell lymphoma (ALCL) have significant presence of which of the following bacteria?

A) Escherichia coli
B) Ralstonia pickettii
C) Staphylococcus aureus
D) Pseudomonas aeruginosa
E) Serratia marcescens

A

The correct response is Option B.

Most concerning in the past two decades is the incidence of breast implant–associated anaplastic large-cell lymphoma (ALCL). This entity was first diagnosed and associated with breast implants in 1997, and is almost only associated with a history of textured implants and/or tissue expanders. The most common presentation of these patients is late seroma, with some patients presenting with mass, tumor erosion, or lymph node metastasis. A recent review of the world literature on this entity include the following: (1) 173 cases were documented, (2) no cases were found in patients with documented smooth devices only (although this remains controversial, as the data in many cases are incomplete), (3) there may be an associated genetic predisposition as suggested for cutaneous T-cell lymphoma, and (4) the cause is likely multifactorial.

Bacterial biofilm is thought to be an inciting factor for the development of both breast-implant related ALCL and Non-Tumor related capsule contractures. The capsules from patients with tumor had significant presence of Gram-negative bacteria (Ralstonia species) compared to nontumor capsules (Staphylococcus species). Such data may support the bacterial induction model, as there are also other types of implant-associated lymphomas.

240
Q

A 55-year-old woman who underwent augmentation mammaplasty with retro-pectoral smooth saline implants 18 years ago comes to the office because she is dissatisfied with her breast shape. Physical examination shows glandular ptosis hanging off the implants. She has Grade I capsules. Which of the following is the most appropriate procedure to correct this patient’s deformity?

A) Implant exchange alone
B) Implant exchange with capsulectomy
C) Implant exchange with mastopexy
D) Implant exchange with suture plication of the expanded inferior pocket
E) Site change to subglandular placement

A

The correct response is Option C.

The described patient has a “snoopy nose deformity” or “waterfall breast deformity,” with the ptotic breast hanging off of the implant. There is no pocket expansion. The implants have stayed in their original position while the native breast tissue has become ptotic with time and gravity. This is not superior malposition due to capsular contracture; both breasts are soft. Correction of this problem is best performed with an appropriately chosen form of mastopexy. In this case, replacement of the implants would also be performed because of their age.

Implant exchange alone would not correct the ptotic breast. Capsulectomy is not indicated, since the breasts are soft, and no capsule is noted clinically. Similarly, suture plication of the pocket is not required, since the inframammary fold is in the correct position, and no second fold is seen. Site change would not correct the patient’s grade III ptosis, but it may be used to correct this problem in cases without significant ptosis.

241
Q

An otherwise healthy 35-year-old man sustains an amputation of the right thumb while using a cutting saw. Assuming appropriate cooling of the amputated part is performed immediately, successful replantation of the digit could be realistically achieved if performed within which of the following maximum time frames?

A) 2 hours
B) 4 hours
C) 6 hours
D) 12 hours
E) 24 hours

A

The correct response is Option E.

The reasonable maximum time frame for replantation of an appropriately cooled and stored digit (referred to as cold ischemia time) is 24 hours. However, the warm ischemia time for digital replantation is 12 hours. These times are further reduced when replanting more proximally amputated limbs, such as an arm or leg, because of the presence of ischemia-sensitive muscle. The recommended maximum cold ischemia time to replantation in these major replants is 12 hours; the warm ischemia time is 6 hours. Despite these recommended time frames, successful replantations performed many hours after the amputations have been reported. In 1986, May et al. reported a successful digit replantation after 39 hours of cold ischemia, the seventh of a seven-finger replant. Then, in 1988, Wei et al. reported successful digital replantations after 84, 86, and 94 hours of cold ischemia. Whenever there is more muscle mass in the replanted limb, ischemia time becomes more critical. In these clinical scenarios, immediate shunting should be considered especially when the ischemia time is nearing the time frames described above.

The other options are incorrect because they are within the ideal maximum time frame.

References

Prucz RB, Friedrich JB. Upper extremity replantation: current concepts. Plast Reconstr Surg. 2014 Feb;133(2):333-42.

May JW. Digit replantation with full survival after 28 hours of cold ischemia. Plast Reconstr Surg. 1981 Apr. 67(4):566.

Chiu HY, Chen MT. Revascularization of digits after thirty-three hours of warm ischemia time: a case report. J Hand Surg Am. 1984 Jan. 9A(1):63-7.

May JW Jr, Hergrueter CA, Hansen RH. Seven-digit replantation: digit survival after 39 hours of cold ischemia. Plast Reconstr Surg. 1986 Oct. 78(4):522-5.

Wei FC, Chang YL, Chen HC, et al. Three successful digital replantations in a patient after 84, 86, and 94 hours of cold ischemia time. Plast Reconstr Surg. 1988 Aug. 82(2):346-50.

242
Q

A 41-year-old man is referred to the office because of a mass on his left breast that has been growing rapidly for 1 month. Examination of the specimen obtained on core needle biopsy confirms invasive mammary carcinoma. Genetic testing results are positive for BRCA2. In addition to an increased risk of male breast cancer, which of the following other types of cancer is most likely to be associated with this patient?

A) Colon
B) Lung
C) Pancreatic
D) Renal
E) Thyroid

A

The correct response is Option C.

Men with harmful BRCA1 mutations also have an increased risk of breast cancer and, possibly, pancreatic cancer, testicular cancer, and early onset prostate cancer. However, male breast cancer, pancreatic cancer, and prostate cancer appear to be more strongly associated with BRCA2 gene mutations. Colon, lung, renal, and thyroid cancers are not associated with the BRCA1 and BRCA2 gene mutations.

243
Q

A 42-year-old woman with a history of a cesarean delivery from a low-transverse abdominal incision is scheduled to undergo a unilateral deep inferior epigastric perforator (DIEP) flap breast reconstruction. BMI is 28 kg/m² and the distance from nipple to sternal notch is 24 cm per side. This patient’s history of cesarean delivery is most likely to have which of the following effects?

A) Decreased abdominal seroma
B) Decreased flap venous congestion
C) Increased flap arterial thrombosis
D) Increased flap fat necrosis
E) No overall effect

A

The correct response is Option B.

Pfannenstiel incisions are the preferred access for cesarean deliveries. They are not a contraindication for abdominal-based flaps for breast reconstruction because the deep inferior epigastric circulation is not disturbed. However, the superficial epigastric circulation may be divided. The net result appears to be a more robust venous circulation with a protective effect against fat necrosis in the flap. This venous division causes a delay-type phenomenon—during healing increased branches are formed from the superficial epigastric circulation, and there is some evidence of new connections to the venae comitantes of the deep epigastric venous circulation.

There is evidence, however, for an increased rate of abdominal healing problems, including seroma (15% versus 6%), wound healing problems, and fat necrosis in the abdomen. There is no evidence for an effect on the arterial circulation of the flap.

244
Q

A 9-year-old boy is brought to the office for evaluation of a nodule on the neck that appeared 1 week ago. The nodule measures 2 cm and is slightly to the right of midline. A photograph is shown. The lesion is slightly tender. Which of the following is the most appropriate management of this lesion?

A) Excision of lesion with any tract and a section of hyoid bone
B) Excision of lesion with any tract and a section of thyroid cartilage
C) Excision of lesion with any tract only
D) Fine-needle aspiration with cytology
E) Intralesional sclerotherapy under image guidance

A

The correct response is Option A.

Thyroglossal duct cysts are neck lesions that usually present during childhood, often after a localized inflammation or infection. They are usually neck lesions near the midline. Some may move with swallowing if they are close to the base of the tongue, but not all exhibit this finding. The congenital thyroglossal duct is the remnant of the descent of the developing thyroid gland, and it runs from the foramen cecum of the base of the tongue, down to the thyroid. Imaging is typically by ultrasound, but other modalities, including radioisotope scans to rule out ectopic thyroid tissue or confirm the location of the normal thyroid gland, have been suggested to avoid excising ectopic thyroid gland.

The Sistrunk procedure shows less recurrence and is a complete excision, including any associated tract, and about a 1-cm section of hyoid bone. Although proposed in 1928, this is still the most commonly recommended approach for thyroglossal duct cysts.

Fine-needle aspiration is appropriate for thyroid nodules, which are unlikely to present in an acute manner in this population.

Sclerotherapy is used for vascular anomalies and likely would have presented earlier on in childhood.

References

Sistrunk WL. Technique of removal of cysts and sinuses of the thyroglossal duct. Surg Gynecol Obstet. 1928;46:109–112.

Galluzzi F, Pignataro L, Gaini RM, et al. Risk of recurrence in children operated for thyroglossal duct cysts: A systematic review. J Pediatr Surg. 2013 Jan;48(1):222-7.

Li WY, Reinisch JF. Cysts, pits, and tumors. Plast Reconstr Surg. 2009 Jul;124(1 Suppl):106e-116e.

245
Q

A healthy 11-year-old boy is brought to the emergency department because of worsening redness and pain around the site of an injury to the middle finger of the left hand. The patient sustained the original injury 3 weeks ago while wrestling with his brother. The patient had swelling and pain of the finger, but the pain resolved quickly, so no medical care was sought. The patient’s mother reports that she noticed a small bump on the dorsum of the finger since then. However, the patient developed redness and pain 2 days ago at the same site, both of which have worsened. An image and x-ray studies are shown. Which of the following is the most likely cause of this patient’s symptoms?

A) Biting of nails
B) Exposure to Pasteurella species
C) Flexor tendon avulsion
D) Immunocompromised state
E) Trapped germinal matrix

A

This patient has a Seymour fracture—a juxta-epiphyseal open fracture—of the distal phalanx as evidenced by the eponychial disruption and fracture pattern on x-ray study. These are open fractures traditionally frequently with associated nail bed transection. Unfortunately, secondary to delay in treatment, the patient developed osteomyelitis.

These patients often present with a mallet-appearing deformity from flexion of the distal fracture segment, the nail may be disrupted (with the distal nail bed) and lay superficial to the eponychial fold, while the transected nail bed proximally (germinal matrix) becomes entrapped within the fracture, making closed reduction prone to failure. Acute treatment in the emergency department or operating room consists of nail plate removal, reduction of the trapped nail bed, irrigation and debridement of the fracture site, reduction of the fracture, nail bed repair, nail plate replacement, and immobilization. Instability or inadequate reduction warrants operative intervention and may require Kirschner wire fixation. Inadequate reduction and/or delayed treatment are critical influences of infection rates. In a study by Reyes and Ho in the Journal of Pediatric Orthopaedics, investigators reviewed acute appropriate reduction, acute partial treatment, and delayed treatment. No infections occurred in the acute appropriately managed group, whereas 15% of the patients developed infections in the partially treated group, and 45% of the delayed treatment group developed infections. Biting of nails has been associated with nontraumatic paronychial infections. There is no history of immunocompromised state, and healthy children can acquire infections with these injuries. This patient sustained the injury wrestling his brother as opposed to from an animal bite. Pasteurella is not the most frequent bacteria associated with infections in patients who have sustained Seymour fractures. Flexor tendon avulsions, also known as jersey fingers, are traditionally closed and would demonstrate lack of flexion of the distal interphalangeal joint.

246
Q

A 63-year-old man comes to the office because of pain of the right wrist and posttraumatic arthritis after a long-standing scapholunate tear that was untreated. Salvage reconstruction with proximal row carpectomy is planned. Which of the following articular surfaces should be intact in order to perform the procedure?

A) Capitolunate
B) Lunotriquetral
C) Radioscaphoid
D) Scaphotrapezial
E) Trapeziotrapezoid

A

The correct response is Option A.

The capitolunate articulation should be intact in order to perform proximal row carpectomy.

The patient described has a chronic scapholunate tear, which if left untreated, can lead to the consequences of scapholunate advanced collapse (SLAC) wrist. With ongoing progression, degenerative wrist arthritis and pain result.

Arthritis occurs in a predictable sequence, initially at the radioscaphoid joint, followed by the scaphocapitate joint and the capitolunate joint. The radiolunate joint is typically spared until advanced stages.

Proximal row carpectomy is a salvage wrist procedure that can be used in some cases of SLAC wrist. The proximal carpal bones of the wrist (scaphoid, lunate, and triquetral) are removed, and the capitate head is allowed to rest in the lunate fossa of the radius. In order for this procedure to be successful, the capitolunate joint should be free of arthritis. The patient should have preservation of cartilage on the capitate head and the lunate fossa of the radius, as this forms the new articulation of the wrist.

247
Q

A 45-year-old woman is evaluated because of asymmetry of the chest during physical activity. Medical history includes bilateral augmentation mammaplasty with silicone implants approximately 10 years ago. She recently began an aggressive exercise regimen that includes long-distance running and weight training. The patient’s major aesthetic concern is that the implants distort the appearance of her breasts during weight training exercises. Which of the following is the best treatment for this patient?

A) Remove and replace the implants with more cohesive silicone gel implants
B) Remove and replace the implants with saline implants
C) Remove the implants and insert them in the subglandular plane
D) Remove the implants and insert them in the subpectoral plane
E) Remove the implants, perform bilateral capsulectomy, and insert new silicone implants

A

The correct response is Option C.

Muscle contraction deformity and implant displacement are corrected with the removal of the implant and placement into a newly created subglandular plane from the current subpectoral plane. The deformity described is typical in patients involved in heavy weight training in which the pectoralis muscle is repeatedly activated.

Moving the implants from the subpectoral plane into the subglandular plane will compress the subpectoral pocket, thus preventing movement into the previous pocket. In addition, the pectoralis muscle may be resuspended to prevent fluid accumulation.

Removal and replacement of the implants with more highly cohesive implants may improve any rippling that may be seen, but it will not improve the animation deformity. Removal and replacement with saline implants will not make any significant improvements in the muscle-induced deformity. While capsulectomy may improve capsular contraction, it does not address the deformity cause by repeated pectoralis activation.

248
Q

A 48-year-old woman comes to the office because she is very unhappy with the appearance of her breasts following a bilateral mastopexy performed 1 year ago. Height is 5 ft 7 in (170 cm). BMI is 26 kg/m2. Which of the following findings on physical examination would be most difficult to correct?

A) Asymmetrical breast size
B) Dog ear of the inferior vertical scar
C) Nipple to inframammary crease distance of 16 cm
D) Nipple to sternal notch distance of 16 cm
E) Widened circumareolar scar

A

The correct response is Option D.

A sternal notch to nipple distance of 16 cm represents a high-riding nipple. Revisional surgery for correction of a high-riding nipple is complex, and it is difficult to achieve a favorable result because of the surgeon’s and patient’s desire to avoid a scar extending superior to the nipple areola. Further, the paucity of excess skin between the nipple and clavicle limits the reconstructive options.

Suggested strategies include direct reposition of the nipple-areola complex, expansion of the skin between the nipple and clavicle, and repositioning of the breast parenchyma and inframammary crease.

Breast size asymmetry can be improved with either liposuction or revision mastopexy/reduction. The operation is usually performed using the previous incisions.

A dog ear of the inferior vertical scar is easily revised with a small transverse scar within the inframammary crease. The majority of these early postoperative deformities will resolve without surgery.

Recurrence of ptosis or an elongation of the nipple to inframammary crease distance occurs with all mastopexy operations. When performing secondary mastopexy, this can be improved with shortening the vertical scar with wedge resection at the inframammary crease. Knowledge of the location of the previous nipple areola pedicle is helpful in minimizing vascular complications.

Widened circumareolar scars can be revised with excellent results. Utilizing a permanent suture around the areola helps control size of the areola and tension on the suture line.

249
Q

A 30-year-old woman comes to the office because of a 3-week history of unilateral swelling of the left breast. She underwent subglandular placement of textured silicone breast implants 4 years ago. She has had no trauma, fevers, or chills. A 1-week course of an oral antibiotic prescribed by her family physician has failed to resolve the swelling. On physical examination, the left breast is 300 to 400 mL larger than the right breast. No other abnormalities are noted. Ultrasonography report shows seroma and results are negative for hematoma or mass. Which of the following is the most likely diagnosis in this patient?

A) Anaplastic large cell lymphoma
B) Double capsule phenomenon
C) Giant fibroadenoma of the breast
D) Hematoma due to capsule tear
E) Periprosthetic abscess

A

The correct response is Option B.

The combination of late-onset swelling without signs of periprosthetic infection (fever, cellulitis), no history of trauma, and a negative ultrasonography suggests late-onset seroma, as can occur with a double capsule phenomenon. Late seromas occur as a complication in about 1% of reported breast implant series. This issue seems to be more common in the setting of textured implants, particularly those implants manufactured with an aggressive texturing process. At surgery, a capsule layer is seen lining the pocket, which often contains a substantial volume of serosangineous seroma fluid and a textured implant coated in a tight second capsule at the center of the pocket. Double capsule has been reported in both the subglandular and submuscular positions. A giant fibroadenoma of the breast would have a dominant mass, distortion of the breast shape, and would be visible on ultrasonography. Abscess would be likely to occur with fever, chills, and cellulitis of the breast. Hematoma of this size would be likely to have a history of trauma, breast pain, and external bruising. Although anaplastic large cell lymphoma is a possibility in the differential of late-onset seromas, it is a rare disorder. Seroma fluid, obtained either by ultrasound-guided aspiration or at the time of open surgery, should be sent for cytologic examination and immunohistochemistry to rule out this rare possibility.

250
Q

A 57-year-old woman with a history of modified left radical mastectomy followed by adjuvant chemotherapy and radiation therapy is evaluated six months after her last radiation treatment for breast reconstruction. Physical examination shows slight skin redundancy of the chest wall and grade 2 ptosis of the contralateral breast. She wears a size 36D brassiere and does not want to undergo symmetry procedures on the contralateral breast. Which of the following reconstructive options is most likely to achieve breast symmetry in this patient?

A) Deep inferior epigastric perforator flap
B) Latissimus dorsi flap
C) Thoracodorsal artery perforator flap
D) Tissue expander with acellular dermal matrix and staged implant placement
E) Total submuscular tissue expander and staged implant placement

A

The correct response is Option A.

Breast reconstruction requires reconstruction of both the soft-tissue envelope and volume. In the setting of radiated chest wall tissues, it would be difficult to recreate a breast envelope of sufficient size to match the contralateral breast with tissue expansion, even with acellular dermal matrix or total submuscular placement of a tissue expander. The history of radiation would likely cause an element of capsular contracture and decrease symmetry to the contralateral breast with grade 2 ptosis. Both the latissimus dorsi flap and the thoracodorsal artery perforator flap would recruit non-radiated skin to create an adequate envelope, but would not provide adequate volume alone. The deep inferior epigastric perforator (DIEP) flap would provide both envelope and volume.

251
Q
  1. A 20-year-old woman comes to the office for consultation regarding augmentation mammaplasty. Height is 5 ft 4 in (163 cm) and weight is 120 lb (54 kg). Physical examination shows mammary hypoplasia. She currently wears a size 34B brassiere and would like to wear a size C brassiere. Which of the following is the most appropriate option for breast enhancement?

A ) Autologous fat transfer

B ) Breast Enhancement and Shaping System (BRAVA)

C ) Saline prostheses

D ) Smooth gel prostheses

E ) Textured gel prostheses

A

The correct response is Option C.

Augmentation mammaplasty is one of the most common plastic surgery operations. During the moratorium on silicone gel prostheses between 1992 and 2006, the saline breast prosthesis became the prosthesis of choice. When a saline prosthesis ruptures, it decreases in size as the saline leaks out and is absorbed by the body. The deflated side is usually noticeable to the patient and can be compared to the nondeflated side for further distinction. The saline may leak out slowly, taking a week or longer to be noticeable.

When the Food and Drug Administration lifted the moratorium on silicone gel prostheses, it stipulated that women must be 22 years of age to use the gel prosthesis. Therefore, for the patient described, the only option is saline.

Saline prostheses are firm to the touch, and on very thin patients the normal rippling can be palpated through the skin, especially noticeable along the lower, outer pole where there is no pectoral muscle coverage.

Autologous fat transfer is reported in the literature but would be difficult to do on this very thin patient. Harvesting enough fat to achieve the goal of a size C brassiere would be difficult.

The BRAVA system can increase breast size but only minimally, so it is unlikely that this would give the patient enough volume for her goal of a size C brassiere.

The deflation rate of saline prostheses is debated in the literature, related to prosthesis type (textured versus smooth), fill volumes, and physician technique. It is agreed that the expected lifespan of the saline prosthesis is 10 years.

252
Q

A 47-year-old woman, gravida 2, para 2, who has grade III breast ptosis is evaluated for mastopexy. Attenuation of which of the following structures is the most likely cause of the ptosis?

A) Breast acini
B) Cooper ligaments
C) Lactiferous ducts
D) Scarpa’s fascia
E) Subdermal plexus

A

The correct response is Option B.

Breast ptosis is a complex interaction of events, informed by breast size, gravity, aging, lactation, and parity. It occurs through a combination of atrophy of the breast tissue, loss of elasticity of the skin envelope, and attenuation of Cooper ligaments.

While the breast is surrounded by fascia, the continuation of Scarpa’s fascia forms the posterior capsule of the breast.

The lactiferous ducts and breast acini do not contribute significantly to ptosis.

Subdermal plexus provides vascularity rather than support to the breast.

253
Q

A 21-year-old man sustains traumatic amputation of the right thumb at the level of the metacarpal base. Pollicization should include osteosynthesis of which of the following?

A) Index metacarpal base to trapezium
B) Index metacarpal to thumb metacarpal
C) Index middle phalanx to thumb metacarpal
D) Index proximal phalanx to thumb metacarpal
E) Index proximal phalanx to trapezium

A

The correct response is Option D.

Transfer of the index finger to the thumb position on the hand (pollicization) typically transfers the proximal phalanx to the thumb metacarpal, as long as the base of the thumb metacarpal is preserved. Transfer of the middle phalanx or metacarpal of the index would create a neo-thumb that is too short or too large, respectively. Obliterating an intact carpometacarpal joint by transferring the index metacarpal to the trapezium would eliminate palmar and ulnar abduction of the thumb and compromise global hand function.

References

Neligan P. Plastic Surgery, Volume 6, 3rd edition. Philadelphia: Elsevier: 2012;292.

Bravo CJ, Horton T, Moran SL, Shin AY. Traumatized index finger pollicization for thumb reconstruction. J Hand Surg Am. 2008;33(2):257-62.

Taghinia AH, Upton J. Index finger pollicization. J Hand Surg Am. 2011;36(2):333-9.

254
Q

A 45-year-old woman comes to the office for consultation regarding severe breast asymmetry after undergoing lumpectomy and radiation therapy for ductal carcinoma of the right breast 5 years ago. Physical examination shows the radiated right breast is tight and retracted, and the left breast is ptotic. Left-sided mastopexy for symmetry and autologous fat grafting to the radiated breast are recommended. The patient asks if the procedure will be covered by insurance. Which of the following is the most appropriate response?

A) Fat grafting the right breast will be covered by insurance but the mastopexy will be considered cosmetic and will not be covered
B) Insurance companies rarely cover the cost of immediate breast reconstruction
C) Insurance may not cover the procedure since insurers are not mandated to pay for reconstruction of lumpectomy defects
D) Procedures for both sides will be covered by insurance
E) The left-sided mastopexy will likely be covered but the fat grafting will not be covered

A

The correct response is Option C.

The Women’s Health and Cancer Rights Act, signed into law in 1998, requires insurance plans to cover the cost of breast reconstruction after mastectomy. The law includes all stages of reconstruction as well as contralateral procedures to provide symmetry. An often misunderstood aspect of the Women’s Health and Cancer Rights Act is that it does not apply to individuals undergoing breast conservation therapy (lumpectomy with radiation). As rates of breast conservation therapy have continued to increase (60% of women with stage I cancers), so have significant lumpectomy defects associated with the untoward effects of radiation. In the clinical scenario presented, the patient should be informed that the corrective operation (fat grafting and mastopexy) might not be covered by her insurance company.

255
Q

A 58-year-old man comes to the office with recurrent rectal cancer four years after undergoing low anterior resection with adjuvant radiotherapy. Open abdominoperineal resection is planned. Which of the following closure methods will most reliably reduce the incidence of perineal wound complications?

A) Gracilis flap closure
B) Negative pressure wound therapy
C) Primary closure
D) Rectus abdominis flap closure
E) Split-thickness skin grafting

A

The correct response is Option D.

Several retrospective studies and one randomized trial have shown that when compared to primary closure, the rectus abdominis myocutaneous flap reduces wound healing complications after abdominoperineal resection (APR). Gluteal and gracilis flaps have been used for reconstruction after APR; however, the data supporting their use is not as robust. Split-thickness skin grafting and negative pressure wound therapy are inappropriate for reconstruction of the APR due to the size of the wounds and the risk of evisceration.

256
Q

A 5-year-old boy who underwent closed reduction and pinning of a supracondylar humerus fracture is evaluated in the postanesthesia care unit because of absence of palpable radial pulse in the left wrist. Physical examination shows the left hand is warm with color similar to the right hand. No pulses are palpable at the level of the wrist. Without surgical exploration, which of the following is the most likely outcome for the left forearm?

A) Fibrosis of the deep flexor compartment
B) Ischemic necrosis of the radial hand
C) Median nerve palsy
D) Physeal arrest in the forearm bones
E) No functional impairment

A

The correct response is Option E.

Vascular impairment related to pediatric supracondylar fractures occurs in roughly 5 to 10% of patients. Given the nontrivial morbidity of exploration of the brachial artery after reduction of the supracondylar fracture, controversy exists over what criteria should prompt surgical intervention. Recent large, retrospective studies have demonstrated that absence of pulses in the wrist after reduction is not, by itself, associated with poor outcomes. Most patients who lack pulses immediately after reduction, but who have an otherwise perfused hand, go on to recover a palpable radial pulse within 24 to 48 hours after the reduction. In this patient with a perfused hand, observation can be expected to result in a good outcome.

Physeal arrest can occur secondary to ischemia, but this would not be the expected outcome based on the perfusion of the hand. Fibrosis of the deep flexor compartment (Volkmann ischemic contracture) is associated with compartment syndromes but would be rare in this setting. Median nerve impairment associated with the vascular insufficiency at the time of injury is advocated as an indication for surgical exploration. Development of a new median palsy as a late result of the vascular injury is unlikely.

Rare case reports of late development of ischemia in the forearm and hand occur, and families should be counseled to watch for color or temperature changes in the hand following discharge after the recommended observation period of 24 to 48 hours.

257
Q

Which of the following is the most likely chronic effect of post-mastectomy radiation therapy?

A) Desquamation
B) Dyspigmentation
C) Edema
D) Erythema
E) Ulceration

A

The correct response is Option B.

Radiation therapy induces tissue injury that can be categorized as acute or chronic. The spectrum of acute injury includes erythema, edema, desquamation, hyperpigmentation, and ulceration, ranging from mild to severe. Acute radiation dermatitis occurs in upward of 85% of treated patients. Chronic injury involves skin atrophy, dryness, telangiectasia, dyspigmentation, and dyschromia. In the breast, it leads to chronic fibrosis of the skin and subcutaneous tissues. This fibrosis and surrounding injury can lead to pain and restricted movement of the arm. The chronic changes from radiation can take months to years to fully manifest.

258
Q

A 51-year-old woman is scheduled to undergo a lower lip reduction. Blockade of the mental nerve is planned for anesthesia. The most appropriate site for injection of the mental foramen blockade is the mucosa below which of the following teeth?

A) Canine
B) Central incisor
C) Lateral incisor
D) Second bicuspid
E) Second molar

A

The correct response is Option D.

The mental nerve foramen is located near the second bicuspid or first molar along the border of the mandible. This nerve will give sensation to the lower lip. The other answers are too distal or mesial for the mental nerve foramen.

References

Chapter 30: Face and Scalp. In: Standring S. Gray’s Anatomy. Elsevier 2018:475-506.

Scalp and Face. In: Moses KP, Banks JC, Nava PB, Petersen DK. Atlas of Clinical Gross Anatomy. Elsevier Saunders 2018(4): 26-43.

259
Q

A 40-year-old man comes to the office because of an 8-month history of intermittent ischemic change to the right ring finger. The patient reports intermittent coolness, pallor, pain, and cold sensitivity. Angiogram demonstrates a tortuous ulnar artery at the wrist and faint radial digital artery runoff into the right ring finger. Digital brachial index (DBI) of the ring finger is 0.9. Which of the following is the most appropriate first step in management?

A) Botulinum toxin type A injection
B) Excision and vein grafting of the ulnar artery
C) Ligation of the thrombosed ulnar artery segment
D) Thrombectomy and heparin drip
E) Trial of acetylsalicylic acid and nifedipine

A

The correct response is Option E.

This patient presents with hypothenar hammer syndrome. The gold standard for establishing the diagnosis is angiography. Aortic arch and upper extremity arteriography is the study of choice. In hypothenar hammer syndrome, the pathognomonic angiographic features can include tortuosity of the ulnar artery with a corkscrew appearance, aneurysm formation, occlusion of the ulnar artery segment overlying the hook of the hamate, occluded digital arteries in the ulnar artery distribution, and demonstration of intraluminal emboli at sites of digital obstruction.

Treatment depends largely on the severity of the ischemia. The therapeutic strategy is controversial because there are limited studies on this problem. For most patients with milder or transient/intermittent symptoms, nonsurgical treatment will be sufficient, particularly in the setting of vasospasm with adequate collateral circulation. Conservative nonoperative care may include smoking cessation, avoidance of further trauma (may require change of occupation), padded protective gloves, cold avoidance, calcium channel blockers (nifedipine, diltiazem), antiplatelet agents or anticoagulation, local care of fingers with necrosis, and pentoxifylline to reduce blood viscosity.

More severe symptoms (persistent ischemia, soft tissue loss/gangrene, ulnar nerve symptoms) or symptoms refractory to nonoperative management require consideration of surgical intervention. Surgical options in this setting include arterial ligation (assuming an intact radial/palmar arch), resection of thrombosed arterial segment or aneurysm with end-to-end anastomosis, or resection and vascular reconstruction with vein or artery graft. Some argue that best outcomes are seen in those treated with surgical resection and reconstruction. The benefits of surgical treatment include removal of the source of embolism, removal of the painful mass, relief of ulnar nerve compression, and creation of a local periarterial sympathectomy. As this patient has mild and intermittent symptoms without evidence of soft-tissue loss or gangrene or any evidence of ulnar nerve irritation, a trial medical management is indicated. Botulinum toxin type A is indicated for vasospasm secondary to Raynaud syndrome or disease and would not be part of the medical management algorithm.

References

Ablett CT, Hackett LA. Hypothenar Hammer Syndrome: Case Reports and Brief Review. Clin Med Res. 2008 May; 6(1):3-8.

Yuen JC, Wright E, Johnson LA, Culp WC. Hypothenar hammer syndrome: an update with algorithms for diagnosis and treatment. Ann Plast Surg. 2011 Oct;67(4):429-38.

Endress RD, Johnson CH, Bishop AT, Shin AY. Hypothenar hammer syndrome: long-term results of vascular reconstruction. J Hand Surg Am. 2015 Apr;40(4):660-5.e2.

Zimmerman NB, Zimmerman SI, McClinton MA, et al. Long-term recovery following surgical treatment for ulnar artery occlusion. J Hand Surg Am. 1994 Jan;19(1):17-21.

260
Q

A 37-year-old woman is brought to the operating room after sustaining a crush injury to the left upper arm during a rollover motor vehicle collision that included prolonged extraction from the vehicle. In the operating room, the patient underwent bypass grafting to reconstruct the brachial artery after fixation of the humerus. Postoperatively, the patient reports increasing pain of the left forearm with increasing pain control requirement. Doppler shows intact radial and ulnar pulses. Which of the following clinical studies is most likely to help determine the treatment plan at this time?

A) Assessment of capillary refill of the finger tips
B) Doppler examination of the digital arteries
C) Duplex scan to check patency of graft
D) Measurement of the compartment pressures of the forearm
E) Pulse oximetry of the digits

A

The correct response is Option D.

In this scenario, the physician should rule out compartment syndrome for several reasons: the crush injury, the reperfusion state, and pain unrelieved by pain medications prior to providing more pain relief. Pain that is out of proportion to the expected level or out of proportion to examination findings should alert the clinician to the possibility of compartment syndrome.

The only study option provided that would give the clinician the ability to rule out compartment syndrome is the direct measurement of compartment pressures, which is recommended by several authors. Loss of peripheral artery pulses or perfusion to the distal skin would be very late presentations of the ischemic process, at a point where intervention, such as fasciotomy may not be effective. Missing this diagnosis in a patient such as this one, may lead to loss of muscular function (ischemic muscle contracture) even if the limb as a whole is salvaged.

The presence of palpable pulses is reassuring evidence for the patency of the bypass graft but does not tell the clinician about the perfusion in the capillary beds of the muscle. Similarly, perfusion of the skin and the digits does not guarantee flow in the muscle that was reperfused.

References

Kistler J, Ilyas A, Thoder JJ. Forearm Compartment Syndrome Evaluation and Management. Hand Clin. 2018 Feb;34(1):53-60.

Prasarn ML, Ouellette EA. Acute compartment syndrome of the upper extremity. J Am Acad Orthop Surg. 2011 Jan;19(1):49-58.

261
Q

A 45-year-old woman comes for evaluation 1 year after undergoing vertical mastopexy without placement of prostheses because she thinks her breasts have started to sag. An increase in which of the following breast dimensions has most likely occurred since the patient’s last visit?

A) Breast base diameter
B) Nipple to inframammary crease
C) Nipple-areola diameter
D) Suprasternal notch to inframammary crease
E) Suprasternal notch to nipple

A

The correct response is Option B.

The nipple-to-inframammary crease dimension is most likely to increase over time. This leads to pseudoptosis (bottoming out) and the appearance of a sagging breast. Pseudoptosis occurs when the breast gland migrates lower than the inframammary crease while the nipple stays in normal position. It is essential that patients be informed that their breasts will eventually sag following mastopexy. Procedures to prevent this from occurring include the use of permanent mesh encircling the breast mound. Mastopexy and reduction mammaplasty share similar operative strategies as well as complications. All techniques suffer bottoming out to different degrees.

Breast base diameter will change very little over time as long as the breast volume remains constant; eg, weight gain can increase breast volume.

An increase in the nipple-areola diameter is unlikely with vertical mastopexy; however, increased areola diameter is associated with periareolar mastopexy. To minimize this complication, a permanent purse-string suture is recommended. Suprasternal notch-to-inframammary crease distance changes very little in comparison with the nipple-to-inframammary crease distance.

The suprasternal notch-to-nipple distance changes very little postoperatively. When a prosthesis is used during mastopexy, this distance will increase; however, the nipple-to-inframammary crease will usually increase to a greater extent.

262
Q

A 63-year-old man with a BMI of 35 kg/m2 presents with an incisional hernia. The patient underwent a midline exploratory laparotomy for trauma one year ago. Primary fascial closure was achieved with a running polypropylene suture that was performed at the time of the initial operation. CT scan shows intact rectus muscles, and the hernia defect is measured to be 10 cm at the widest, which is in the supraumbilical region. Which of the following is the most effective treatment to prevent hernia recurrence following repair?

A) Component separation with bridging mesh repair
B) Component separation with overlay mesh repair
C) Component separation with primary fascial closure
D) Component separation with retrorectus mesh repair
E) Primary fascial closure

A

The correct response is Option D.

Hernia repair is associated with a high rate of recurrence, approaching 20% in many studies. Recurrence rates are lowest when primary fascial closure of the abdominal wall is reinforced with mesh placement as an underlay.

Primary fascial closure alone or with component separation results in a higher recurrence rate than primary fascial closure with mesh reinforcement. In this example, it is unlikely that primary fascial closure would be possible, given a 10-cm hernia defect. With regard to mesh placement, there are multiple planes at which the mesh can be placed. Using a bridging repair, the mesh is used to bridge across a fascial defect and is associated with the highest rates of recurrence. In a retrorectus repair, the mesh is placed deep to the rectus (Rives-Stoppa technique) or below the transversus abdominis (transversus abdominis release technique). This is performed underneath a primary fascial closure. Conversely, in an overlay repair, the mesh is secured superficial to the abdominal wall repair. Retrorectus placement of a mesh is associated with a significantly lower recurrence rate than placement of the mesh in another position.

263
Q

A 74-year-old woman comes to the office because of a 2-month history of increasing soreness over the left lateral chest wall. History includes bilateral radical mastectomy and adjuvant radiation to the left chest 30 years ago because of left-sided breast cancer. On examination, a small, nonhealing 1 × 2-cm wound is visible in the subaxillary region within an area of dense fibrotic skin. A photograph is shown. Which of the following is the most appropriate next step?

A) Biopsy of the nonhealing wound
B) Coverage with a left latissimus muscle flap and skin grafting
C) LHyperbaric oxygen treatment
D) MRI of the chest wall
E) Oral administration of antibiotics for 7 days

A

The correct response is Option A.

This patient’s presentation is consistent with late radiation tissue injury (LRTI), which occurs in 5 to 15% of long-term cancer survivors who have received radiation and can occur months to years (even decades) after treatment. It can vary significantly with age, dose, and site of treatment. Characteristics include progressive tissue deterioration secondary to decreased vascularity followed by replacement of normal soft-tissue architecture by dense fibrotic tissue until there is insufficient oxygen delivery to sustain normal function. When LTRI of the chest wall progresses to skin breakdown and ulceration, as has happened in this patient, a biopsy is always required to rule out recurrence of the primary tumor or a radiation-induced squamous cell carcinoma or soft-tissue sarcoma.

Hyperbaric oxygen therapy has been proposed as a treatment modality that can improve tissue quality and prevent tissue breakdown in irradiated areas. While most data on this relate to treatment of osteoradionecrosis of the mandible, there have been some reports of its use on the chest wall after irradiation due to breast cancer. However, the question specifically asks for the most appropriate next step, which would be biopsy rather than hyperbaric oxygen therapy.

The principles of management once malignancy has been ruled out include debridement of necrotic tissues (including ribs) and reconstruction with well-vascularized flaps. In this case, a left latissimus muscle flap and skin graft was used for reconstruction after aggressive debridement. No alloplastic material was placed or thoracic cage reconstruction performed. This is common in these types of patients as excessive fibrosis caused by the radiation to the chest wall prevents loss of respiratory efficiency through paradoxical motion.

CT scan or MRI may be useful in evaluating the size, extent, and nature of the problem, but does not take priority over tissue biopsy.

The redness that is visualized on the patient’s skin in the case is related to post-radiation changes and telangiectasias, which are common in LTRI, and is not infection. Therefore, antibiotics would be unnecessary.

264
Q

E-cigarettes are similar to traditional cigarettes in that the negative effect of nicotine on wound healing is primarily due to which of the following?

A) Endothelial cell injury
B) Increased inflammatory response
C) Increased prostacyclin production
D) Peripheral microvascular vasoconstriction
E) Platelet aggregation

A

The correct response is Option D.

Cigarette smoke contains thousands of different chemical substances including chemical toxins and carcinogens in addition to nicotine. While e-cigarettes are believed to have fewer health risks because tar and many other harmful chemicals have been removed, they do deliver vaporized nicotine. Nicotine-induced vasoconstriction in the periphery leads to relative skin ischemia and poor wound healing.

Platelet aggregation is incorrect. Nicotine does not have a direct effect on platelets. In some studies, platelet aggregation appears to be reduced with long-term administration of nicotine. Increased prostacyclin production is incorrect.

Prostacyclin is a local vasodilator with effects that minimize platelet aggregation.

Nicotine has been shown to inhibit synthesis of prostacyclin. Increased inflammatory response is incorrect. Nicotine causes a diminished inflammatory response by a weakened chemotaxis, reduced migration, impaired bacterial killing by inflammatory cells and a subnormal release of proteolytic enzymes and inhibitors. Endothelial cell injury is incorrect. While nicotine can alter the structural and functional characteristics of vascular smooth muscle and endothelial cells, it does not cause cell injury. Several studies show increased number of endothelial cells in venous blood after nicotine administration in cigarette smokers, but not in non-cigarette smokers, suggesting that other harmful chemicals found in tobacco may contribute to cell injury.

265
Q

A newborn has a pretragal cystic mass with a sinus tract visible in the external auditory meatus. Which of the following is the most likely diagnosis?

A) Dermoid cyst
B) First branchial cleft cyst
C) Second branchial cleft cyst
D) Third branchial cleft cyst
E) Vascular cystic lesion

A

The correct response is Option B.

The first branchial cleft develops into the external auditory canal. The second, third, and fourth branchial clefts merge to form the sinus of His, which will normally become involuted. When a branchial cleft is not properly involuted, a branchial cleft cyst forms. Occasionally, both the branchial pouch and branchial cleft fail to become involuted, and a complete fistula forms between the pharynx and skin.

First branchial cleft cysts are divided into type I and type II. Type I cysts are located near the external auditory canal. Most commonly, they are inferior and posterior to the tragus (base of the ear), but they may also be in the parotid gland. Type II cysts appear at the angle of the mandible and may involve the submandibular gland.

The second branchial cleft accounts for 95% of branchial anomalies, and they are most frequently identified along the anterior border of the upper third of the sternocleidomastoid muscle and adjacent to the muscle. However, these cysts may present anywhere along the course of a second branchial fistula, which proceeds from the skin of the lateral neck, between the internal and external carotid arteries, and into the palatine tonsil. Therefore, a second branchial cleft cyst is part of the differential diagnosis of a parapharyngeal mass.

Third branchial cleft cysts are rare. A third branchial fistula extends from the same skin location as a second branchial fistula (recall that the clefts merge during development); however, a third branchial fistula courses posterior to the carotid arteries and pierces the thyrohyoid membrane to enter the larynx, terminating on the lateral aspect of the pyriform sinus. Third branchial cleft cysts occur anywhere along that course (eg, inside the larynx), but they are characteristically located deep to the sternocleidomastoid muscle.

Congenital dermoid cysts of the face typically occur at the lateral orbit overlying the ZF suture, and vascular cyst lesions can be located anywhere on the face but do not present with sinus tracts into the EAC.

References

Fastenberg J, Nassar M. First Branchial Cleft Cyst. N Engl J Med. October 2016; 375:e33.

D’Souza AR, Uppal HS, De R, Zeitoun H. Updating concepts of first branchial cleft defects: a literature review. Int J Pediatr Otorhinolaryngol. 2002 Feb 1;62(2):103-9.

266
Q

Which of the following cranial nerves develops within the first branchial arch?

A) Facial (VII)
B) Glossopharyngeal (IX)
C) Hypoglossal (XII)
D) Trigeminal (V)
E) Vagus (X)

A

The correct response is Option D.

The trigeminal nerve (cranial nerve V) develops from the first branchial arch and gives rise to the malleus and incus. The ligaments associated with the first branchial arch are the anterior ligament of the malleus and the sphenomandibular ligament. The muscles of the first branchial arch include the muscles of mastication (masseter, temporalis, medial pterygoid, lateral pterygoid), tensor veli palatini, tensor tympani, mylohyoid, and the anterior belly of the digastric. The first pouch is associated with the external auditory canal and middle ear space.

The facial nerve (cranial nerve VII) develops from the second branchial arch and gives rise to the stapes, styloid, and the upper body of the hyoid. It includes the stylohyoid ligament. The associated muscles are facial expression muscles, stapedius, stylohyoid, and the posterior belly of the digastric. The second pouch is associated with the tonsillar fossa.

The glossopharyngeal nerve (cranial nerve IX) develops from the the third branchial arch and gives rise to the lower body of the hyoid. Its muscle is the stylopharyngeus, and the pouch is associated with the inferior parathyroid gland and the thymus.

The fourth branchial arch is associated with the larynx. The muscles associated with this arch are the laryngeal, pharyngeal, and soft palate. The pouch is associated with the superior parathyroid gland and the thyroid gland.

References

Irace A, Adil E. Embryology of congenital neck masses. Operat Tech Otolaryngol Head Neck Surg. 2017;28(3):138-142.

LaRiviere CA, Waldhausen JH. Congenital cervical cysts, sinuses, and fistulae in pediatric surgery. Surg Clin North Am. 2012;92(3):583-597.

267
Q

Which of the following innervates the nipple-areola complex?

A ) Intercostal

B ) Lateral pectoral

C ) Long thoracic

D ) Supraclavicular

E ) Thoracodorsal

A

The correct response is Option A.

The classic teaching ascribes nipple innervation to the fourth intercostal nerve. More recent anatomical studies have confirmed that the nipple is innervated by a rich subdermal plexus of nerves that provide both tactile and pressure sensation. This plexus receives innervation from the lateral and anterior cutaneous branches of the second to fifth intercostal nerves. This plexus explains why the nipple can retain sensation despite extensive surgical procedures.

The lateral pectoral innervates the pectoralis major muscle. The long thoracic innervates the serratus anterior muscle. The supraclavicular innervates the skin of the upper breast. The thoracodorsal innervates the latissimus dorsi muscle.

268
Q

During which of the following phases of wound healing is collagen deposition the greatest?

A) Collagen deposition occurs equally during all phases of wound healing
B) Collagen deposition does not occur during wound healing
C) Inflammatory
D) Proliferative
E) Remodeling

A

The correct response is Option D.

The proliferative phase of wound healing occurs in two phases. Fibrin and fibronectin are formed during the initial proliferative period. Around day 3 of wound healing, fibroblasts appear and begin production of collagen. The proliferative phase ends between 2 to 4 weeks of wound healing when collagen accumulation reaches a maximum and collagen remodeling begins, marking the beginning of the remodeling phase of wound healing.

Inflammatory phase is dominated by white blood cells. There is an influx of polymorphonuclear leukocytes (PMNs), macrophages, and lymphocytes. PMNs are not essential to wound healing, but macrophages are essential. Sterile incisions normally heal without PMNs.

Type III collagen is converted to type I in the remodeling phase. In this phase there are increased collagen cross-linking and increased tensile strength.

Collagen deposition does not occur during wound healing is incorrect. It occurs during all phases of wound healing.

Collagen deposition occurs equally during all phases of wounding healing is incorrect because collagen deposition is heaviest during the proliferative phase. Fibroblasts differentiate from resting mesenchymal cells in connective tissue. They are chemoattracted to the site, divide and produce components of ECM and their primary function is to synthesize collagen during that phase.

269
Q

A 26-year-old healthy woman comes to the office for consultation because she has constant pain 1 year after undergoing augmentation mammaplasty by another surgeon. She reports that he “botched” her surgery and that she is considering taking legal action against him. On physical examination, the breasts are quite firm and mildly tender. The relatively immobile subglandular implants are high on the chest wall. There are no overlying skin changes. Which of the following is the most appropriate response by the surgeon in this scenario?

A) Decline to establish care
B) Follow-up visit in one year
C) Perform a diagnostic intercostal nerve block
D) Prescribe a course of oral corticosteroids
E) Recommend surgical intervention

A

The correct response is Option E.

This patient has developed Baker grade IV capsular contracture as evidenced by hard, painful breast implants that are malpositioned. This is a known complication of augmentation mammaplasty, and the patient should be informed that it is treatable with another surgery. It would be reassuring to the patient to hear that it is a known post-operative complication that happens not uncommonly, and that it is unlikely the other surgeon directly did anything to cause this. Thoughtful analysis and contextualization are helpful in high-tension consultations such as these.

Providing the patient with a malpractice attorney’s contact information may be what she thinks she wants, but diffusing the situation is best for all involved. Dismissing the patient’s concerns outright without diagnosing her and suggesting a course of treatment would not be helpful. The new surgeon may even take it a step further by offering to speak with her previous surgeon to discuss the patient’s concerns and the findings seen during consultation. The patient may refuse to allow this, and she may have lost faith in her other surgeon, but at least offering to speak with the other surgeon is prudent and may restore the relationship between this patient and her surgeon. A pain management specialist and physical therapy may help somewhat with symptoms, but her problem ultimately requires and should respond well to a surgical solution.

270
Q

A 23-year-old man who works in an office undergoes evaluation of the left nondominant wrist after sustaining an injury from a fall 2 days ago. CT scan shows a nondisplaced distal scaphoid tubercle fracture. Examination reveals no other abnormalities. Which of the following is the most appropriate next step in management?

A) Autologous bone grafting and internal fixation plus immobilization for 3 to 6 months
B) Cast immobilization for 6 to 8 weeks
C) Compression screw fixation plus immobilization for 6 to 12 weeks
D) Physical therapy for 6 weeks
E) Observation until the wrist is nontender

A

The correct response is Option B.

Nondisplaced scaphoid fractures may not be apparent on plain radiographs and are better visualized on CT scan. Although the treatment options for nondisplaced scaphoid wrist fractures may include immobilization alone or surgical fixation, nondisplaced distal pole and tubercle fractures are felt to be more stable and can be treated with immobilization alone for 6 to 12 weeks. In addition, these fractures often have small fragments that are not as amenable to compression screw fixation.

Observation and physical therapy allow mobilization of the wrist and are not recommended because of the potential for delayed healing or nonunion.

Compression screw fixation is not generally necessary and is sometimes not possible for tubercle fractures, but it is a common treatment for scaphoid wrist fractures.

Autologous bone grafting and internal fixation is generally reserved for scaphoid nonunions.

271
Q

A 40-year-old nulliparous woman comes to the office because she is dissatisfied with the “saggy” appearance of her breasts following a 120-lb (54-kg) weight loss. Physical examination shows bilateral Grade 3 ptosis. Which of the following additional findings on examination of the breasts is most likely in this patient?

A ) Flatness of the upper pole

B ) High inframammary fold

C ) Lack of axillary fat roll

D ) Lack of excess skin

E ) Laterally displaced areolas

A

The correct response is Option A.

The types of breast deformities seen following massive weight loss are relatively new. To adequately manage these patients and assess outcomes, it is important to understand the defect. Classification systems exist for breast ptosis for other causes; however, these are based mainly on nipple position. Breast deformities after massive weight loss vary significantly. Patients typically present with severe breast ptosis (Grade III), medialization of the nipple-areola complex, lateralization of the breast mound, and extension to a lateral axillary fat roll, which often extends well into the back. The inframammary fold is often in a lower position because of deflation of the entire skin and connective tissue envelope. Beyond the typical breast changes of glandular tissue loss and ptosis, there tends to be more asymmetrical volume loss in the massive weight loss breast, and there is more of a deflated and flat appearance of the breast (particularly a flat upper pole). Skin laxity is very apparent, and the degree of excess skin can be significant.

272
Q

A 65-year-old man presents with an infection of the sternum following aortic valve repair. After sternal debridement, there is a 10-cm-wide, deep wound from the clavicle to the upper abdomen. Which of the following is the most appropriate flap to reconstruct the wound?

A) Latissimus dorsi
B) Omentum
C) Pectoralis major
D) Pectoralis minor
E) Serratus

A

The correct response is Option B.

The most appropriate flap to reconstruct the wound is omentum. Because of the large extent of the wound, the only flap listed that can adequately fill the defect and eliminate the dead space is the omentum. Pectoralis major flaps would not adequately fill the defect, particularly the inferior aspect of the wound. Pectoralis minor flaps are not used for sternal reconstruction and would not provide adequate tissue. The latissimus dorsi flap would not be able to fill the large sternal wound. Serratus flaps can be used for posterior chest wounds, but would not be able to reconstruct the large anterior chest wound

273
Q

A 35-year-old man comes to the office for evaluation of a previous amputation of the dominant thumb. Medical history includes factor V Leiden mutation. The patient states that he does not want any microsurgical flap reconstruction. X-ray study shows amputation at the metacarpal base level. Which of the following reconstruction methods is most appropriate for the best aesthetic and functional outcome in this patient?

A) Metacarpal distraction
B) Osteoplastic reconstruction with iliac bone and radial forearm flap coverage
C) Pollicization of the index finger
D) Silicone prosthesis
E) Web space deepening with Z-plasty

A

The correct response is Option C.

The thumb is considered to account for at least 40% of hand function. Essential characteristics to provide optimal function of the thumb include mobility, opposition, sensation, stability, strength, and normal shape. Metacarpal distraction can provide a strong, stable, and sensate thumb but is not very aesthetically pleasing as it will be larger than a normal thumb and lack a nail. Osteoplastic reconstruction and flap coverage provides a stable post but no mobility and poor sensation with the same visual concerns. Web space deepening with Z-plasty can provide a very functional stable thumb with more distal amputations but would not provide sufficient length in this patient. Silicone prostheses provide excellent cosmetic digits but are not functional. The most appropriate reconstruction in this patient without a toe transfer would be an index pollicization.

References

Bravo CJ, Horton T, Moran SL, Shin AY. Traumatized index finger pollicization for thumb reconstruction. J Hand Surg Am. 2008 Feb;33(2):257-62.

Graham DJ, Venkatramani H, Sabapathy SR. Current reconstruction options for traumatic thumb loss. J Hand Surg Am. 2016 Dec;41(12):1159-1169.

274
Q

A 22-year-old man who is a college student sustains a volar oblique fingertip amputation while chopping vegetables. Examination shows involvement of the hyponychium, but the nail is undamaged. The wound measures 1 × 1.5 cm, and no exposed bone is noted. Which of the following is the most appropriate treatment to encourage healing by secondary intention?

A) Apply negative pressure wound therapy
B) Apply povidone iodine to the wound daily and cover with dry gauze
C) Cover wound with semiocclusive dressing and change weekly
D) Leave wound open to air
E) Soak wound in hydrogen peroxide daily and cover with moist gauze

A

The correct response is Option C.

Fingertip or thumb tip amputations that result in small wounds (less than 1.5 cm2) and minimal exposed bone are best managed with healing by secondary intention. The only exception to this might be a laborer anxious to get back to work with a healed wound sooner than 3 to 4 weeks. Mennen reported a series of 200 such injuries treated with a semi-occlusive dressing, and average healing time was 20 days.

A semiocclusive dressing is semi-permeable and transparent, allowing air to pass through the dressing, but providing a barrier to moisture. Commonly available semipermeable dressings are marketed under brand names like Tegaderm (3M) and OPSITE (Smith & Nephew). These dressings maintain a moist wound environment, which speeds healing. If dressings are changed every 5 to 7 days, manipulation of the wound is minimized and, therefore, healing is less disrupted.

Leaving a wound open to air would allow tissues to dry out, which would delay healing. Likewise, the use of povidone-iodine and/or hydrogen peroxide would slow down healing due to drying of the wound. Although these topical agents are effective at eliminating bacteria from dirty or infected wounds, prolonged use will interfere with normal wound healing. Finally, a wound of this small size would not warrant negative pressure wound therapy. Even the small, intrinsically-powered negative pressure wound therapy devices would not offer any advantages over a semiocclusive dressing and would increase cost substantially.

References

Germann G, Rudolf KD, Levin SL, Hrabowski M. Fingertip and thumb tip wounds: changing algorithms for sensation, aesthetics, and function. J Hand Surg Am. 2017 Apr;42(4):274-284.

Germann G, Sauerbier M, Rudolf KD, Hrabowski M. Management of thumb tip injuries. J Hand Surg Am. 2015 Mar;40(3):614-622.

Mennen U, Wiese A. Fingertip injuries management with semi-occlusive dressing. J Hand Surg Br. 1993 Aug;18(4):416-422.

Panattoni JB, De Ona IR, Ahmed MM. Reconstruction of fingertip injuries: surgical tips and avoiding complications. J Hand Surg Am. 2015 May;40(5):1016-1024.

275
Q

A 37-year-old man who works as a carpenter is brought to the emergency department after falling 10 ft from a scaffold. He says he has pain in the left wrist and numbness of the thumb, index, and long fingers of the left hand. Trauma screening shows no other injury, and x-ray studies of the left wrist are obtained (shown). Which of the following is the most appropriate treatment?

A ) Application of finger traps and placement in traction

B ) Closed reduction and carpal tunnel release

C ) Open reduction and internal fixation

D ) Splinting of the wrist in 30 degrees of extension

E ) Total wrist fusion

A

The correct response is Option C.

Perilunate dislocations are relatively uncommon injuries that typically follow high-energy impact to the wrist. The pattern of injury traverses both the greater and lesser carpal arcs, disrupting the lunotriquetral ligament and either the scapholunate ligament or the body of the scaphoid (trans-scaphoid, perilunate dislocation). Although controversy surrounding the need for operative intervention existed in the past, poor long-term outcomes from closed reduction alone have showed the importance of early restoration of carpal alignment with repair of the injured ligaments. In the setting of trans-scaphoid, perilunate dislocation, this would include rigid internal fixation of the scaphoid.

Application of finger traps and placement in traction may assist in reduction of the dislocated lunate but will not, by itself, reduce the lunate and remove the need for internal fixation and repair of the injured ligaments.

Proximal row carpectomy and wrist function play a role in long-term salvage of the painful wrist following perilunate dislocations. Only in rare circumstances, such as nonreconstructible fractures of the proximal row bones, should acute proximal row carpectomy be considered.

Splinting of the wrist in extension often provides relief in the setting of idiopathic carpal tunnel syndrome; however, in the setting of lunate dislocation, the presence of the lunate in the carpal tunnel necessitates reduction of the lunate for relief of symptoms

276
Q

The inferior oblique muscle of the orbit is innervated by which of the following cranial nerves?

A) Oculomotor (III)
B) Trochlear (IV)
C) Trigeminal (V)
D) Abducens (VI)
E) Facial (VII)

A

The correct response is Option A.

The inferior oblique muscle receives its nerve supply from the oculomotor nerve, or cranial nerve III. The other voluntary muscles within the orbit that receive their innervation from the oculomotor nerve are the levator palpebrae superioris, superior rectus, medial rectus, and inferior medial rectus muscles. The superior oblique muscle is innervated by the trochlear nerve (cranial nerve IV). The lateral rectus muscle is innervated by the abducens nerve (cranial nerve VI).

The inferior oblique is the only one of these muscles that does not arise from the apex of the orbit. It originates from the medial floor of the orbit just posterior to the infraorbital rim. It runs laterally, posteriorly, and upward, crossing inferior to the inferior rectus, and inserting on the posterior half of the globe beneath the lateral rectus insertion. Its action is to elevate the globe, producing upward gaze of the pupil. Risk of iatrogenic injury to the inferior oblique is greatest with transconjunctival surgical approach to the orbit. Therefore, with this procedure, care must be exercised to place the periosteal incision along the anterior aspect of the infraorbital rim.

References

Hollinshead WH. The Head and Neck. 3rd ed. Philadelphia, PA: Harper & Row Publishers; 1982:124-132. Anatomy for Surgeons; vol 1.

Turvey TA, Golden BA. Orbital anatomy for the surgeon. Oral Maxillofac Surg Clin North Am. 2012;24(4):525-36.

277
Q

A 69-year-old man is brought to the emergency department because of acute onset of excruciating pain of the left forearm and a pale, pulseless, cool left hand. Medical history includes atrial fibrillation and steroid-dependent chronic obstructive pulmonary disease (COPD). Physical examination and hand-held Doppler interrogation are consistent with acute arterial blockage in the left ulnar artery. In addition to aspirin, which of the following therapeutic interventions should be administered promptly while assessing the patient’s ability to withstand surgery?

A) Apixaban
B) Clopidogrel
C) Heparin
D) Recombinant tPA
E) Verapamil

A

The correct response is Option C.

Iannuzzi and colleagues have summarized the treatment for acute arterial thrombosis of the hand. Prevention of further damage should be the first line of treatment while completing work-up of the patient. They recommend that heparin and aspirin should be administered to prevent propagation of further arterial occlusion. While the idea of reducing vasospasm in the surrounding vessels is appealing, Iannuzzi’s review of the literature is inconclusive of any benefit for tissue salvage outcomes. The article is also useful for comparison of the various imaging modalities for definitive diagnosis and approach to treatment.

In their meta-analysis for the Cochrane library, Berridge et al. surveyed the literature and came to the conclusion that distal limb salvage was similar at 30 days, 6 months, and 1 year with either surgical extraction of clot or thrombolysis by direct delivery of the agent to the artery in question. Bleeding and distal embolization were more common after use of thrombolytic agents at 30 days.

Robertson et al, also in meta-analysis for the Cochrane library, found some differences favoring tissue plasminogen activator (tPA) in initial vessel patency, but there were no differences in limb salvage outcomes with intra-arterial delivery of tPA or urokinase. In the streptokinase vs tPA studies, there were increased bleeding complications noted with streptokinase.

References

Robertson I, Kessel DO, Berridge DC. Fibrinolytic agents for peripheral arterial occlusion. Cochrane Database Syst Rev. 2010 Mar 17;(3):CD001099.

Iannuzzi NP, Higgins JP. Acute Arterial Thrombosis of the Hand. J Hand Surg Am. 2015 Oct;40(10):2099-106.

Berridge DC, Kessel DO, Robertson I. Surgery versus thrombolysis for initial management of acute limb ischaemia. Cochrane Database Syst Rev. 2013 Jun 6;(6):CD002784.

278
Q

A 65-year-old man develops a hemorrhagic stroke requiring decompressive craniotomy. The bone is found to be unusable and a customized polyetheretherketone prosthesis is planned. Which of the following is the most common complication of using this material?

A) Cerebrospinal fluid leak
B) Contour deformity
C) Dehiscence
D) Hematoma
E) Infection

A

The correct response is Option E.

Reports on using polyetheretherketone (PEEK) as an alloplast for cranial reconstruction vary in terms of outcomes and complications. The larger studies conclude that it is a reliable material compared with other alloplastic alternatives and has the advantage of being custom made for a variety of craniofacial defects. However, infection remains the most common complication, and choosing this material should be weighed against the risk for microorganism seeding through, wound dehiscence, hematogenous spread, or indolent colonization of the wound bed.

279
Q

A 24-year-old man is evaluated because of a 1-cm metacarpal defect after sustaining a gunshot wound to the hand. In addition to operative fixation of the fracture, which of the following materials placed into the defect is most likely to promote osteogenesis?

A) Calcium hydroxyapatite
B) Cancellous autograft
C) Cortical allograft
D) Demineralized bone matrix
E) Methylmethacrylate

A

The correct response is Option B.

The material that will most likely provide osteogenesis is cancellous autograft.

Bony defects can be constructed by a variety of methods, and there has been an increase in the number of biomaterials that can be used. Autograft bone is obtained from the same individual, while allograft bone is obtained from another human source (i.e., cadaveric or donor). Demineralized bone matrix does not contain calcium, but retains growth factors and proteins as the nonmineralized components of bone. Calcium hydroxyapatite is a bone substitute that mimics bone in mineral structure, and gradually becomes replaced with native bone. Methylmethacrylate is used in orthopedic bone cement, and is not biodegraded or replaced, but can provide rapid structural support.

Osteoconduction refers to the replacement of the graft material through a process known as creeping substitution, where native cells from the surrounding bone break down the material and replace it with new bone. This is the primary mechanism of healing of cortical bone grafts. It is also seen in biocompatible materials that are replaced with bone, such as calcium hydroxyapatite.

Osteoinduction refers to the stimulation of bone-forming cells from surrounding host tissues, resulting in activation of progenitor cells and differentiation into osteoblasts, leading to the creation of new bone. This process occurs due to growth factors that are present in the graft material, and can be seen in cancellous bone grafts, as well as demineralized bone matrix, which contains growth factors. Osteoinduction also occurs with cortical grafts, although to a lesser extent.

Osteogenesis refers to new bone formation, which is provided from surviving cells within the graft material. In order for osteogenesis to occur, viable cells must be transferred with the graft. This is seen in autograft materials, but not in allograft materials, which are processed and may be decellularized. Cells contained within autografts can survive and produce new bone. Vascularized bone transfer may give rise to more cell viability than traditional autografts, because it maintains perfusion to the grafted bone, rather than relying on nutrients from the bed

280
Q

A 65-year-old woman presents to the office with an ulcer on the right chest wall. She underwent right-sided mastectomy and adjuvant external beam radiation therapy for advanced breast cancer 5 years ago. Physical examination shows a 2-cm ulcer with surrounding radiation-damaged skin and no signs of acute infection. Which of the following is the most appropriate next step in management?

A) Biopsy of the wound
B) Excision of all radiation-damaged tissue and coverage with vascularized tissue
C) Excision of the ulcer and coverage with vascularized tissue
D) Hyperbaric oxygen therapy
E) Negative pressure therapy

A

The correct response is Option A.

Radiation causes production of reactive oxygen species, which causes injury to tissues and progenitor cells. Cytokine release results in chronic inflammation and ongoing tissue damage. Radiation therapy can cause soft-tissue ulcerations, osteoradionecrosis, and radiation-induced sarcomas. If a patient presents with a late ulcer after radiotherapy, malignancy needs to be ruled out. A biopsy of the ulcer edge should be performed.

Once malignancy has been ruled out, excision of all radiation-damaged tissue, rather than just the ulcer, will result in more durable reconstructive outcomes. Osteoradionecrosis of the chest wall presents as full-thickness chest wall ulcers and the involved ribs should be resected. The underlying pleura and lung may be adherent and, thus, limited lung resection may need to be performed. Reconstruction is performed with well-vascularized tissue, either local pedicled flaps or free flaps.

Negative pressure therapy utilizes subatmospheric pressure for local wound care. It provides local wound care by controlling exudate and, thus, keeping the wound clean. It is thought to promote wound healing by inducing cellular proliferation and increasing capillary blood flow. Malignancy in the wound is a contraindication to negative pressure therapy. Therefore, if suspected, malignancy should be ruled out prior to initiation of negative pressure therapy.

Hyperbaric oxygen is the administration of 100% oxygen in a pressurized chamber. This results in high tissue concentrations of oxygen, which promote neovascularization and wound healing. Hyperbaric oxygen has been shown to improve healing in soft-tissue radionecrosis and osteoradionecrosis. It can be used as an adjunct, especially when radical excision and reconstruction of radiation damaged tissue is not possible.

281
Q

A 50-year-old woman comes to the office for consultation about improving the appearance of her “saggy” breasts. She has lost 100 lb (45 kg) during the past 18 months by diet. Photographs are shown. Physical examination shows breast deflation and marked ptosis. A Wise pattern mastopexy with augmentation mammaplasty is planned. Which of the following arteries is most likely to provide circulation to the breast gland and nipple during submuscular augmentation in this patient?

A)Intercostal
B)Pectoral
C)Superior epigastric
D)Thoracoacromial
E)Thoracodorsal

A

The correct response is Option D.

The thoracoacromial artery and vein travel just deep to the pectoralis major muscle, supplying circulation to the overlying breast tissue and skin. Subglandular augmentation mammaplasty disrupts the connection between the thoracoacromial vessels and the overlying breast. This leads to a higher risk of wound-healing complications when placing the prosthesis in the subglandular plane. The submuscular plane of dissection maintains the connection between the thoracoacromial vessel and overlying breast and skin, allowing better potential healing.

Intercostal arteries are multiple and are not completely disconnected with either subglandular or subpectoral augmentation mammaplasty.

The superior epigastric artery provides circulation to the rectus abdominis muscle and abdomen. This artery would be injured with the mastopexy procedure.

The thoracodorsal artery supplies the latissimus dorsi muscle and not the chest.

282
Q

A 39-year-old woman comes to the office with a 6-month history of progressive firmness and superior fullness of the left breast. History includes bilateral augmentation mammaplasty with textured saline implants placed in a submuscular dual-plane pocket 15 years ago. On physical examination, the left breast appears larger and firmer with more upper pole fullness in comparison with the right breast. Which of the following is the most appropriate next step?

A) Capsulectomy and pocket change
B) Mammography
C) 3-Month trial of montelukast (Singulair)
D) MRI
E) Ultrasonography

A

The correct response is Option E.

Breast implant patients who present with late-onset enlargement of one of their breasts require evaluation for breast implant-associated anaplastic large cell lymphoma (BIA-ALCL). This disease usually presents with spontaneous onset of peri-prosthetic fluid. A late-onset seroma is usually accepted as occurring 1 year after surgery; however, they have presented as early as 4 months. It is often difficult to determine if late-onset firmness of the implant is secondary to fluid, capsular contracture, or both.

The initial workup should begin with an ultrasound to evaluate for peri-prosthetic fluid or capsular mass. If fluid is present, it should be sent for cytology, flow cytometry with immunohistochemistry looking for expression of T-cell CD30 cell surface protein.

BIA-ALCL is overwhelmingly associated with textured implants. It is important to remember that BIA-ALCL is extremely rare and that most patients presenting with a late seroma will not have lymphoma, but will have peri-prosthetic fluid from the textured surface pulling away from the capsule and forming a double capsule. The treatment for localized BIA-ALCL is bilateral total capsulectomy and explantation. Treatment for the more likely double capsule or capsule contracture is capsulectomy and pocket change; however, surgery is not indicated until the diagnosis is made.

Montelukast is a leukotriene antagonist that can inhibit the inflammatory cascade thought to be involved with capsular contracture. It seems to be more useful in patients with capsular contracture less than grade III. There is no consensus on its use or effectiveness.

The sensitivity and specificity of ultrasound for detecting a seroma has been equal or better than MRI or 3D mammography. After diagnosis of ALCL, MRI and PET scanning may be indicated. If the implants were silicone gel, MRI would be indicated to evaluate for implant rupture; however, ultrasound would still be recommended for seroma evaluation and aspiration.

283
Q

Which of the following cranial nerves develops with the first branchial arch?

A) Facial (VII)
B) Glossopharyngeal (IX)
C) Hypoglossal (XII)
D) Trigeminal (V)
E) Vagus (X)

A

The correct response is Option D.

The first branchial arch is associated with the trigeminal nerve (V), second branchial arch with the facial nerve (VII), third branchial arch with the glossopharyngeal (IX), fourth branchial arch with the superior laryngeal (X), and sixth branchial arch with the recurrent laryngeal (X).

284
Q

A 64-year-old woman who is postmenopausal asks why she has not been prescribed hormone replacement therapy with estrogen and progestin like her mother was. Supplementation with these hormones is associated with an increased risk for which of the following?

A) Coronary artery disease
B) Diabetes
C) Endometrial cancer
D) Invasive breast cancer
E) Osteoporosis

A

The correct response is Option D.

Hormone replacement therapy has fallen out of favor because of a risk profile that is believed to exceed the potential benefits. Combined estrogen and progestin supplementation is thought to be associated with an increased risk for invasive breast cancer but may decrease the risk for diabetes and osteoporosis. It is thought to not impact the risk for coronary artery disease or endometrial cancer.

285
Q

A 30-year-old woman comes to the office because of pain in the left breast. Two weeks ago, she underwent core needle biopsy of a breast mass that was diagnosed as benign. Family history does not include breast cancer. She does not smoke cigarettes. On examination, the left breast is erythematous and tender to palpation, and the skin of the breast is retracted laterally. There is a palpable, rope-like mass that courses longitudinally along the breast. Which of the following is the most appropriate diagnosis and treatment of this lesion?

A) Breast abscess, perform incision and drainage of the mass
B) Fibrocystic changes, perform biopsy
C) Mastodynia, treat with 10-day course of broad-spectrum antibiotics
D) Nipple papilloma, perform diagnostic mammography
E) Superficial thrombophlebitis, manage with analgesics

A

The correct response is Option E.

Mondor disease, or superficial thrombophlebitis of the breast, involves the superficial veins of the breast and anterior chest wall. It may occur following surgery, core biopsy, irradiation, or trauma. Clinical manifestations include pain, redness and swelling, and the presence of a thickened tender cord. This condition usually resolves in 4 to 6 weeks with symptomatic treatment using pain relief.

Nipple papillomas may be identified as a mass on breast imaging or may be found incidentally. They frequently present with bloody nipple discharge. While not concerning in and of themselves, these lesions may harbor areas of atypia or ductal carcinoma in situ, and are treated with core needle biopsy.

While a breast abscess is possible after a diagnostic procedure such as a biopsy, it would present as localized swelling, tenderness, and induration. The skin retraction and rope-like mass would not be present.

Fibrocystic changes in the breast present as a solitary mass and may cause patients to seek medical attention because of associated pain. They may fluctuate in size and tenderness during a patient’s menstrual cycle. Because no breast mass can be definitively declared benign on physical examination alone, imaging and/or biopsy may be considered.

Breast pain in the absence of a finding on physical examination may have a number of causes including menstrual changes, breast hypertrophy, diet, hormone replacement therapy, ductal ectasia, mastitis, malignancy, and hidradenitis. While the patient does have breast pain, the other physical findings rule out mastodynia alone.

286
Q

A 24-year-old woman with bilateral micromastia comes for consultation regarding augmentation mammaplasty. The patient says she would like her breasts to be “as big as possible.” On examination, which of the following is the most important factor in determining the maximum acceptable prosthesis size for this patient?

A) Breast base width
B) Diameter of the areola
C) Grade of nipple-areola ptosis
D) Maximum manufactured prosthesis volume
E) Pectoralis muscle height-to-prosthesis height ratio

A

The correct response is Option A.

The most important factor in determining the maximum acceptable prosthesis size in this patient is breast base width. Grade of nipple-areola ptosis, areola diameter, maximum manufactured prosthesis volume, and pectoralis height may all impact overall appearance of the breast but do not have an impact on breast prosthesis size choice.

287
Q

A 13-year-old girl is evaluated for breast asymmetry. Examination shows total absence of the left mammary gland tissue, with normal areola and nipple. Pectoral muscles are normal. No hand, facial, or other body abnormalities are noted. Which of the following is the most likely diagnosis?

A) Amastia
B) Amazia
C) Athelia
D) Ectodermal dysplasia
E) Poland sequence

A

The correct response is Option B.

There are a number of uncommon aplastic deformities of the breast. These include: total absence of the breast and nipple (amastia), absence of the nipple (athelia), and absence of the mammary gland (amazia), as described in this case. These anomalies may occur in isolation, or may be associated with various syndromes, such as Poland syndrome, where the absence of the breast is associated with absence of the pectoralis major muscle, rib cage and ipsilateral upper limb deformities. Ectodermal dysplasias can affect the breast, but two or more abnormalities of ectodermal structures – hair, teeth, nails, sweat glands, craniofacial structures – would be required to consider the diagnosis.

288
Q

A 30-year-old woman comes to the office for augmentation mammaplasty and mastopexy after a 50-lb (23-kg) weight loss. She wears a size 38B brassiere. Physical examination shows grade II ptosis and a sternal notch to nipple distance of 26 cm bilaterally. Simultaneous augmentation mammaplasty with short-T mastopexy using smooth saline-filled breast implants that will be implanted in a dual-plane configuration through an inframammary incision is planned. Which of the following factors puts this patient at highest risk for reoperation?

A) Inframammary implant insertion route
B) Presence of breast ptosis
C) Use of drains
D) Use of saline implants
E) Use of smooth-walled implants

A

The correct response is Option B.

It has long been realized that combination augmentation mammaplasty operations are more difficult and have a higher revision rate than either operation alone. A recent review of 177 primary augmentation mammaplasty cases found that, of the factors listed, preexisting breast ptosis and simultaneous mastopexy were both linked to a higher rate of reoperation when possible contributing factors were statistically analyzed. Furthermore, increasing grades of breast ptosis were linked with increasingly higher reoperation rates.

Although incision site for augmentation mammaplasty has been markedly linked to the rates of capsular contracture, inframammary incisions have been shown in at least two studies to date to have the lowest rate of capsule formation, with periareolar and transaxillary incisions showing 5 to 10 times higher rates of capsule-related complications.

289
Q

An otherwise healthy 46-year-old man comes to the emergency department because of pain in the right wrist 8 hours after falling onto his outstretched hand with the wrist extended. Plain anterior-posterior x-ray study shows widening of the joint between the scaphoid and lunate bones. Which of the following additional findings is most likely on lateral x-ray study?

A ) Dorsiflexion of the lunate bone

B ) Dorsiflexion of the lunate and scaphoid bones

C ) Dorsiflexion of the scaphoid bone

D ) Volar flexion of the lunate bone

E ) Volar flexion of the lunate and scaphoid bones

A

The correct response is Option A.

The mechanism of injury and the anterior-posterior x-ray study in the scenario described suggest dissociation between the scaphoid and lunate bones. This injury can result in a dorsal intercalated segmental instability (DISI), wherein the scaphoid bone loses the support of the scapholunate ligament and most commonly tips into volar or palmar flexion. In this scenario, the lunate then tips into dorsiflexion by the same mechanism; therefore, dorsiflexion of the lunate bone is correct. This also eliminates dorsiflexion of the scaphoid bone. A dissociation of the lunate and triquetral bones causes the opposite forces, and the lunate tips into volar flexion, eliminating volar flexion of the lunate bone. When the scaphoid and lunate bones dissociate, they can be expected to tip in opposite directions through the same forces, eliminating volar flexion of the lunate and scaphoid bones and dorsiflexion of the lunate and scaphoid bones. This would be a volar intercalated segmental instability (VISI).

290
Q

A 35-year-old woman comes to the office for consultation regarding prophylactic mastectomy and breast reconstruction. The patient’s mother and sister were diagnosed with bilateral breast cancer in their premenopausal years. Genetic testing for BRCA mutations is negative. Which of the following best estimates this patient’s lifetime risk of breast cancer?

A) 5%
B) 13%
C) 20%
D) 45%
E) 80%

A

The correct response is Option D.

The cumulative lifetime risk for a 35-year-old woman whose mother and sister had breast cancer is estimated to be approximately 15%. The risk may increase to as high as 45% if those cancers were premenopausal and bilateral. BRCA hereditary cancer is characterized by autosomal dominant genetics with multiple family members in each generation being affected. For patients with BRCA1 mutation, the risk of breast cancer has been estimated to be between 50 and 80% by age 65 years. The risk of developing ovarian cancer has been estimated to be 10% by age 60 years.

291
Q

A 23-year-old male rugby player is evaluated because of the inability to flex the ring finger at the distal interphalangeal (DIP) joint 2 days after injuring the finger during a match. The patient reports tenderness over the pulp and distal flexion crease. A clinical photograph and x-ray studies are shown. Which of the following is the most appropriate management?

A ) Closed reduction and percutaneous pin fixation
B ) DIP arthrodesis
C ) Extension block splinting
D ) Open reduction and internal fixation
E ) Volar plate arthroplasty

A

The correct response is Option D.

The clinical scenario, photograph, and x-ray studies demonstrate classic symptoms of a ?jersey finger,? or avulsion of the insertion of the flexor digitorum profundus (FDP) tendon. In this instance, the rupture involves a large bone fragment to which the FDP tendon remains attached (Type III). The large size of the bone fragment lends itself to repair via open reduction and internal fixation of the fracture. Use of mini-screws or transosseous wiring will provide suitable internal fixation. Type II (retraction of the tendon to the proximal interphalangeal [PIP] joint) and Type I (retraction to the palm) injuries can be addressed through transosseous suture reattachment or suture anchor.

Inadequate closed reduction would make percutaneous pin fixation a poor choice of treatment.

Arthrodesis offers a salvage procedure for failed or unrepaired FDP avulsion injuries. In this young patient with an acute condition, arthrodesis would be too aggressive an intervention.

Extension block splinting can be useful in volar plate injuries, which are often identified by a small, palmar avulsion fragment seen on the lateral x-ray study. This fracture would not be successfully reduced by attempts at flexion or splinting.

Volar plate arthroplasty can restore a functional articular surface following intra-articular fractures of the PIP joint but are not useful in the DIP.

292
Q

A 31-year-old woman with micromastia comes to the office to discuss bilateral augmentation mammaplasty. Which of the following surgical plans will most likely minimize the risk for breast implant–associated anaplastic large cell lymphoma?

A) Highly cohesive silicone gel
B) Nipple shields
C) Saline implants
D) Smooth wall implants
E) Subpectoral pocket

A

The correct response is Option D.

It is extremely rare, if ever seen, to have a diagnosis of breast implant–associated anaplastic large cell lymphoma (BIA-ALCL) in a smooth implant–only case. In those BIA-ALCL cases with smooth wall implants, the patient’s past surgical history revealed the use of textured implants (where adequate documentation was available).

Aggressive texturing (macrotexturing) is associated with a higher risk for BIA-ALCL. One theory is related to increased surface area and therefore higher numbers of bacteria.

The most common presentation for BIA-ALCL is a patient presenting with a late seroma (usually greater than one year). Workup requires aspiration of the seroma followed by cytologic evaluation of the fluid and flow cytometry looking for CD30 T-cell surface protein, which is an ALCL tumor marker.

Highly cohesive silicone gel, saline implants, and the type of implant pocket do not significantly impact the incidence of BIA-ALCL.

Nipple shields and antibacterial irrigation can lower the incidence of bacterial contamination and MAY play a part in reducing biofilm, capsule contracture, and possibly BIA-ALCL. This impact on BIA-ALCL is unknown and being investigated. There are multiple documented cases of BIA-ALCL in patients with textured implants who underwent antibacterial irrigation to reduce bacterial contamination. Nonetheless, avoiding textured implants is the most likely strategy for minimizing BIA-ALCL.

293
Q

A 22-year-old man comes to the emergency department after falling onto his outstretched left hand. An x-ray study and a clinical photograph are shown. Attempted reduction is unsuccessful. Reduction is most likely blocked by which of the following anatomical structures?

A) Central slip
B) Dorsal capsule and collateral ligaments
C) Flexor tendons and intrinsic muscles
D) Lateral bands
E) Sagittal bands

A

The correct response is Option C.

Dorsal metacarpophalangeal joint (MCP) dislocations are relatively uncommon. They occur with forced hyperextension injuries. The digit involved most commonly is the index finger, followed by the little finger.

With complete dislocation, the volar plate ruptures in the membranous proximal portion and becomes interposed in the joint. If this were the only structure blocking reduction, traction on the joint would be sufficient to draw the proximal edge of the volar plate over the metacarpophalangeal head. This is not possible in an irreducible MCP dislocation because additional taut medial and lateral structures are drawn around the narrow metacarpophalangeal neck. At the index finger, the structures include the lumbrical on the radial side and the flexor tendons on the ulnar side. At the little finger, the structures are the common tendon of the abductor digiti minimi and flexor digiti minimi on the ulnar side and the lumbrical and flexor tendons on the radial side. Central slip remains dorsal at the proximal interphalangeal joint and is not associated with the metacarpophalangeal joint. The other structures at the MCP joint would not result in a noose around the metacarpal neck and block reduction.S

294
Q

A 36-year-old man presents to the clinic 1 year after repair of an isolated brachial artery laceration. Prior to arterial repair, the hand and forearm were dysvascular. Fasciotomies were not performed at the time of repair. The patient is unable to extend his fingers actively or passively with the wrist held in neutral position, but he is able to actively make a full fist. Sensation is intact. Which of the following interventions is most appropriate to improve finger extension and preserve grip strength in this patient?

A) Flexor pronator slide
B) Free functional gracilis transfer
C) Joint release and tenolysis
D) Splinting

A

The correct response is Option A.

Volkmann ischemic contracture is a devastating condition with serious motor and sensory functional implications for the upper extremity, most typically the forearm. It is the result of an acute compartment syndrome, following severe soft-tissue trauma and accompanying vascular insult. The patient in the scenario demonstrates a moderate contracture that is best treated with a flexor pronator slide.

When treatment of acute compartment syndrome is delayed or neglected, the muscles of the forearm undergo necrosis and contracture due to secondary fibrosis, causing the typical flexed deformity. This results in impairment of hand and finger function.

Surgical treatment is based on severity of contracture and function of the residual motor units. Mild contractures allow for full passive extension of the fingers with the wrist in volar flexion and can be treated with tendon lengthening and skin release, or selective flexor pronator slide, depending on the source of constrainment. Patients with moderate contractures demonstrate an inability to passively extend the fingers with the wrist in flexion but retain flexor muscle function. These contractures require consideration for a flexor pronator slide alone or in conjunction with tendon lengthening. Complete loss of muscle function necessitates consideration of free functional muscle transfer. Superficialis to profundus transfers are a consideration in the setting of significant contracture and functional limitation. It is typically used to facilitate improved hygiene and confers limited function. This would not be as good of an option for the patient in this question as it would compromise his strength and function. Neurolysis should be considered in conjunction with any reconstructive procedures. Splinting is an important adjunct to any reconstructive procedure and potentially can be employed as an initial treatment prior to surgical intervention to prevent worsening contracture.

References

Farber A, Tan TW, Hamburg NM, et al. Early fasciotomy in patients with extremity vascular injury is associated with decreased risk of adverse limb outcomes: a review of the National Trauma Data Bank. Injury. 2012;43(9):1486-1491.

Pettitt DA, McArthur P. Clinical review: Volkmann’s ischaemic contracture. Eur J Trauma Emerg Surg. 2012;38(2):129-137.

Stevanovic M, Sharpe F. Management of established Volkmann’s contracture of the forearm in children. Hand Clin. 2006;22(1):99-111.

Tsuge K. Treatment of established Volkmann’s contracture. J Bone Joint Surg Am. 1975;57(7):925-929.

Braun RM, Vise GT, Roper B.J. Preliminary experience with superficialis-to-profundus tendon transfer in the hemiplegic upper extremity. Bone Joint Surg Am. 1974 Apr;56(3):466-72.

Stevanovic, Milan V.; Sharpe, Frances. “Compartment Syndrome and Volkmann Ischemic Contracture” Green’s Operative Hand Surgery. Pages 1763-1787.Published January 1, 2017.

295
Q

An 80-year-old woman undergoes excision and direct closure of a nasal sidewall carcinoma. The wound is closed with cuticular nylon suture and dressed with a temporary sterile gauze bandage in an uncomplicated office procedure with immediate discharge home. How long after surgery should this patient be advised to refrain from wetting the suture line with tap water rinses?

A) 48 Hours after surgery
B) 48 Hours after suture removal
C) Until suture removal
D) Until wound edge epithelization is complete
E) No restriction

A

The correct response is Option E.

Several studies have compared wet, moist, and dry wound healing following skin surgery without demonstrating an increase in infection rate when washing the wound with tap water at any point postoperatively as opposed to keeping the site dry for various lengths of time. This includes a rigorous randomized control trial in which patients with defects following skin lesion removal were divided into groups with tap water wound washing within 12 hours of surgery versus those asked to keep wound dry for 48 hours, where the incidence of surgical site infection in the wash group was not inferior to the dry group. On the other hand, unrestricted wound washing improves patient comfort, and multiple studies have demonstrated that wet or moist wounds promote reepithelialization and result in reduced scar formation with less inflammatory reaction compared to dry wounds.

296
Q

A 78-year-old woman has a 6.5-cm defect of the right cheek (shown) just below the lower eyelid after wide local excision for a lentigo maligna melanoma. All final pathologic margins are negative. Which of the following is the most appropriate management?

A ) Cervicofacial flap closure
B ) Full-thickness skin grafting
C ) Healing by secondary intention
D ) Internal mammary artery perforator flap closure
E ) Primary closure

A

The correct response is Option A.

Optimal color and texture matches for cheek reconstruction are obtained from using local cheek tissues (see photograph below). The cervicofacial flap is classically an inferomedially based flap that allows for transfer of large amounts of cutaneous and subcutaneous soft tissues from the loose preauricular and neck regions to the medial cheek. The incision begins at the superior margin of the defect and extends along the outer canthus toward the zygoma and down the preauricular crease. The incision ends in the retroauricular hairline or curves anteriorly in the region of the neck, preferably within a cervical rhytid. The flap is advanced and rotated into the defect, and primary closure of the donor site can usually be achieved via wide subcutaneous undermining. Anchoring of the flap to the zygoma is recommended to reduce tension on the lower eyelid that could lead to ectropion.

Full-thickness skin grafts tend to appear shiny and patch-like with a poor contour match. Split-thickness or very thin full-thickness grafts are also associated with graft contracture. Split- or full-thickness skin grafts are sometimes used as a ? emporary. method of reconstruction during a period of tumor surveillance to rule out early recurrence prior to definitive reconstruction.

Closure by secondary intention would require a prolonged period of healing and result in a poor final appearance with significant scar contracture, resulting in lower lid ectropion and possibly an upper lip deformity.

The internal mammary artery perforator (IMAP) flap is based on perforating blood vessels from the internal mammary artery and vein. The IMAP flap is an island variant of the deltopectoral flap and can be transferred as a free flap with good color match to the cheek compared with more distant tissues, such as the thigh or forearm. Use of the IMAP flap may result in breast distortion in a female and would require a microvascular anastomosis while not necessarily resulting in a superior cosmetic outcome to the cervicofacial flap.

Primary closure is often the reconstructive method of choice for small defects with excellent cosmetic results when the scar can be oriented along lines of minimal tension or natural cheek borders. However, for such a large defect, primary closure cannot be achieved without significant wound tension and excessive distortion of surrounding structures.

297
Q

Compared with standard wound dressings, postoperative negative pressure wound therapy is most likely to produce which of the following outcomes?

A) Better delayed primary fascial closure rates for salvage laparotomy
B) Better split-thickness skin graft incorporation
C) Increased inflammatory response
D) Increased postoperative dressing changes
E) Increased risk of infection

A

The correct response is Option B.

Multiple studies have shown the benefits of negative pressure wound therapy (NPWT) when used in conjunction with skin grafts, both as a bolster over a skin graft as well as wound bed preparation.

NPWT has been shown to decrease the risk of infection in complex and traumatic wounds in some studies, while others have shown no difference in infection rates in complex wounds when the patient has multiple comorbidities or when used to cover uncomplicated incisions for elective orthopedic operations. However, there is no evidence to suggest NPWT increases infection risk compared with standard wound dressings.

NPWT reduces both inflammatory response and edema formation.

When used for damage control laparotomy and abdominal compartment syndrome, studies have failed to show any benefit of NPWT over standard dressings. Furthermore, at least one study has suggested an increased rate of enteric fistula formation is associated with NPWT.

NPWT has been shown to decrease both the number of postoperative dressing changes and the number of additional operative interventions in complicated diabetic wounds.

298
Q

A 32-year-old man presents following a motor vehicle collision in which he sustained a dorsal perilunate dislocation of the nondominant left hand. He was treated with open reduction and internal fixation including ligament repair with suture anchors and Kirschner-wires two days after the injury. Which of the following is the expected long-term outcome for this patient?

A) Chronic pain and grip strength less than 50% of the normal side
B) Evidence of moderate post-traumatic arthritis and 80% of grip strength compared to the opposite side
C) Normal x-ray appearance with greater than 80% of motion compared with the opposite side
D) Normal x-ray appearance with poor wrist motion and poor grip strength
E) Severe post-traumatic arthritis requiring total wrist fusion

A

The correct response is Option B.

Peri-lunate dislocations (PLD) and peri-lunate fracture dislocations (PLFD) are considered complex, high-energy injuries with potentially difficult recovery for many patients. In terms of outcomes research, long-term data are considered to be follow-up greater than 10 years. The long-term data are retrospective but consistent across many studies.

The treatment of choice for PLD and PLFD is open reduction and internal fixation. Studies looking at closed reduction and casting or percutaneous pin fixation have shown inferior outcomes, and open treatment is recommend by most authors.

All studies agree that there will be the presence of moderate or even severe post-traumatic arthritic changes on x-ray in most patients (50 to 100%), which can be various degrees of SLAC, SNAC, or avascular necrosis. However, the data also show that the presence of radiographic arthritis does not necessarily correlate with functional outcomes. On average, patients will achieve 65 to 70% of wrist flexion-extension arc and 80% grip strength compared with the unaffected side.

Although some patients may develop severe complications of a PLD or PLFD such as advanced SLAC or SNAC or ulnar translation of the carpus, it is a rare finding. Most studies show outcomes in the good and fair range according to the Mayo wrist score and other outcome measures. Patients with the most severe arthritis usually do correspond to the worst symptoms and may require salvage procedures such as proximal row carpectomy or limited wrist fusion.

Persistent, chronic pain is a rare finding with long-term outcomes of perilunate injuries. Pain with heavy activity only is the most commonly reported outcome, although many patients are largely pain free.

Patients can have near normal looking x-rays after PLD or PLFD; however, this is rare. One would not expect >80% of wrist motion after an injury of this magnitude even with normal x-rays. Also, patients with minimal arthritic changes tend to show better functional outcomes

299
Q

A 75-year-old woman with a history of right mastectomy and irradiation therapy presents with a sarcoma that requires radical resection and partial sternectomy. A photograph is shown. A pedicled flap is planned to repair the defect. Which of the following arteries supplies the most appropriate flap in this situation?

A) Deep inferior epigastric
B) Internal mammary
C) Lateral thoracic
D) Thoracoacromial
E) Thoracodorsal

A

The correct response is Option E.

The most appropriate pedicled flap for this particular defect is a latissimus flap, shown in the photograph, supplied by the thoracodorsal artery. The latissimus flap is a Mathes/Nahai type V flap that can be transferred on its dominant pedicle (thoracodorsal artery) or on multiple segmental paraspinal perforators. The internal mammary artery terminates as the superior epigastric artery, which would provide blood supply for a superiorly based vertical rectus flap. The rectus flap is a Mathes/Nahai type III flap, with two dominant pedicles. However, this pedicle is not available because of the radical resection and prior irradiation. The lateral thoracic artery is one of two dominant pedicles supplying the serratus anterior muscle (Mathes/Nahai type III). This flap is an option, but it would not provide enough bulk necessary for the defect in this situation. The deep inferior epigastric artery (DIEA) supplies the DIEA perforator flap, which would be an option as a free tissue transfer but not as a pedicle flap for this situation.

300
Q

A 23-year-old man comes for evaluation after falling from a ladder onto the left wrist. A scaphoid fracture is suspected. Initial anterior-posterior, lateral, oblique, and scaphoid-view x-ray studies show no definitive fracture. Which of the following additional imaging studies is most sensitive and specific for detecting the suspected fracture?

A) Arthrography
B) Bone scan
C) CT scan
D) MRI
E) Ultrasonography

A

The correct response is Option D.

The imaging study that is most sensitive and specific for detecting an acute scaphoid fracture is MRI. Many authors have written about the best secondary imaging study for scaphoid fractures not evident on standard x-ray studies. MRI is the best test considering both sensitivity and specificity, followed by CT scan. The majority of the published data shows bone scan to be the most sensitive but less specific than MRI or CT scan. Ultrasonography is used for evaluation of long bone fractures but is not yet indicated for evaluation of carpal bone fractures.

301
Q

A 23-year-old woman comes to the office because she is interested in augmentation mammaplasty. Physical examination shows a right-sided sunken anterior chest wall, hypoplasia of the right breast with a superiorly placed nipple-areola complex, normal pectoralis muscle, and normal sternal position. This patient most likely has which of the following congenital deformities?

A) Amastia
B) Anterior thoracic hypoplasia
C) Pectus carinatum
D) Pectus excavatum
E) Poland syndrome

A

The correct response is Option B.

Amastia refers to an uncommon developmental condition in which the breast and nipple are absent. Some women are immediately given the diagnosis of Poland syndrome or pectus deformity when they exhibit abnormalities of the anterior chest wall. Poland syndrome involves an abnormal pectoralis muscle while pectus deformities do not. Pectus deformities involve alteration in the appearance or location of the sternum and its costal attachments. Another less commonly realized diagnosis is that of anterior thoracic hypoplasia in which patients share the same characteristics of unilateral sunken anterior chest wall, hypoplasia of the breast, superiorly placed nipple-areola complex, normal pectoralis muscle, and normal sternal position.

302
Q

A 45-year-old African American woman who underwent cardiac surgery 6 months ago seeks cosmesis of the surgical scar. Physical examination shows a raised, thick keloid scar that is pruritic. Which of the following nonsurgical treatments is likely to produce the most rapid improvement in the scar with the fewest adverse effects?

A) Intralesional 5-fluorouracil
B) Intralesional triamcinolone
C) Radiation therapy
D) Silicone sheeting
E) Topical triamcinolone

A

The correct response is Option A.

Injection of 5-fluorouracil demonstrates similar efficacy to intralesional corticosteroid therapy (triamcinolone) and has the advantage of a lower risk of hypopigmentation.

Silicone sheeting is effective for hypertrophic scars but requires serial application and is slower in visible effect. Topical corticosteroids are not as effective as intralesional treatment. Radiation carries the risk of adjacent tissue toxicity and also includes the risk of skin pigmentation changes.

303
Q

A 20-year-old man desires correction of a depressed, retracted, post-tracheostomy scar. Which of the following is the best recommendation for improving the scar?

A) Perform autologous fat grafting and laser resurfacing
B) Reconstruction tracheal ring and detach adhesions
C) Scar excision and interposition of acellular dermal matrix
D) Scar excision and reapproximation of strap muscles
E) Scar revision

A

The correct response is Option D.

After decannulation, the tracheostomy site heals by secondary intention. Often the patient is left with a soft, small asymptomatic scar. On occasion, the scar is painful and the skin has adhesions to tissue deep to the strap muscles. This may lead to pulling and retraction with swallowing as well as a scar that is not aesthetically pleasing to the patient. The depressed retracted tracheostomy scar requires reapproximation of platysma and approximation of the sternothyroid and sternohyoid for correction. Fat grafting is unlikely to address retraction or fully correct the depression. Laser resurfacing and fat grafting will have minimal improvement of retraction. Several studies support use of cadaver materials or fascia to support the coverage of the strap muscles when tissue is missing or heavily damaged. The tracheal ring does not need to be reconstructed for routine tracheostomy scar revision. Care must be taken when working around the trachea. Communication with anesthesia about oxygen content and fire risk is important for surgical safety.

304
Q

A 37-year-old woman comes to the clinic to be evaluated for augmentation mammaplasty to improve her breast shape. She is gravida 3, para 3, and breast-fed all of her children. On examination, she has decreased superior pole volume, and the distance from nipple to sternal notch is 28 cm. The nipple-areola complex is below the inframammary fold by 4 cm and is at the lower contour of the breast. Which of the following Regnault classifications of ptosis best describes these findings?

A) Grade I
B) Grade II
C) Grade III
D) Pseudoptosis

A

The correct response is Option C.

The Regnault classification of breast ptosis is based on the relationship of the nipple to the inframammary fold (IMF) and to the lower contour of the gland.

Pseudoptosis is the not true ptosis. In this situation, the nipple is above the level of the IMF but the breast parenchyma has descended below the IMF.

Grade I is minor ptosis with the nipple at the level of the IMF and above the lower contour of the gland.

Grade II is moderate ptosis with the nipple below the level of the IMF and above the lower contour of the gland.

Grade III is major ptosis with the nipple below the level of the IMF and at the lower contour of the gland.

305
Q

A 55-year-old man presents for a large abdominal midline hernia repair. A component separation is planned with a posterior approach and a retrorectus mesh placement. Which of the following layers can be divided to provide further release and preserve the innervation to the rectus muscle?

A) Anterior rectus sheath
B) External oblique
C) Internal oblique
D) Transversalis fascia
E) Transversus abdominis

A

The correct response is Option E.

In the posterior component separation approach for ventral hernia repair, transversus abdominis release (TAR) can provide further mobility and preserve the innervation to the rectus muscle. The posterior approach reinforces hernia repair with a sublay mesh placed between the rectus muscle and posterior sheath. The Rives-Stoppa approach is associated with a 3 to 6% recurrence rate. To avoid disruption of the segmental nerves to the rectus, classical dissection was limited medial to the linea semilunaris. This, however, limited the space and reserved this technique for small- to medium-sized hernias. To extend this dissection laterally for use in larger defects, either the internal oblique or the transversus abdominis muscle can be divided. Division of the internal oblique divides the nerves to the rectus muscle. Division of the transversus abdominis can preserve these nerves. With this technique, the anterior rectus sheath is preserved as well as the external oblique and transversalis fascia.

306
Q

A 4-year-old boy is seen following a pitbull bite to his face. The periorbital findings are shown in the photograph. A stent is placed and the ends of the stent are advanced across the canaliculus. The ends of the stent will enter the nasal cavity in which of the following locations?

A) Above the superior turbinate
B) Below the superior turbinate
C) Above the middle turbinate
D) Below the middle turbinate
E) Above the inferior turbinate
F) Below the inferior turbinate

A

The correct response is Option F.

Reconstruction of the lacrimal apparatus is a critical step in addressing this patient’s periorbital wounds. Failure to properly manage this aspect of the injury will lead to epiphora and the inevitable need for a secondary procedure to manage tear drainage. Delayed reconstruction of this injury would likely require a conjunctivodacryocystorhinostomy, a surgically created conduit between the eyelid and the nose. The best initial management of this lacrimal apparatus disruption is as described—placement of a stent through the canaliculi, into the lacrimal duct (contained within the maxillary bone), and into the nose. The exit point of the nasolacrimal duct is via the valve of Hasner, below the inferior turbinate. It is here that the ends of the stent can be identified and retrieved, although this is often quite challenging.

The frontal, maxillary, and anterior ethmoid sinus cells drain into the middle meatus, just below the middle turbinate. The sphenoid sinus and posterior ethmoid sinuses drain into the sphenoethmoid recess, between the nasal septum and the superior turbinate.

References

Netter, F. H. Atlas of Human Anatomy. Lacrimal apparatus. Ciba-Geigy. 1989.

Drake, R., Vogl, W., Mitchell. A.W.M., Tibbitts, R. and Richardson, P. Gray’s Atlas of Anatomy. 3rd. Edition. Churchill Livingstone. 2021.

307
Q

A 25-year-old man comes to the office 48 hours after “jamming” the right index finger while playing volleyball. The patient reports pain and swelling around the proximal interphalangeal (PIP) joint that prevents him from flexing the digit. Examination does not show malrotation or angulation of the fingertip, but range of motion is limited at the PIP joint because of pain. X-ray studies are shown. Which of the following is the most appropriate next step in management?

A) Arthrodesis of the PIP joint
B) Dorsal block splinting
C) Dynamic traction reduction
D) Hemi-hamate arthroplasty
E) Splint immobilization for 4 weeks

A

The correct response is Option C.

Dislocations of the proximal interphalangeal joint may be treated with splinting, typically dorsal block splinting, and early motion. In the setting of a volar dislocation, where injury to the central slip is suspected, splinting for 3 to 6 weeks in extension may be necessary. Splinting alone is inadequate treatment for fracture dislocations where the middle phalanx remains subluxed dorsally compared with the proximal phalanx condyles. This particular injury requires reduction prior to splinting or other methods of maintaining the joint congruity. Fracture/dislocations of the proximal interphalangeal joint that can be reduced and maintained with joint flexion can be successfully treated with either dorsal block splinting or percutaneous pinning as a dorsal block. Injury patterns amenable to this treatment typically involve <40% of the volar articular surface. The lateral radiograph demonstrates a comminuted fracture of 50 to 60% of the volar articular surface of the middle phalanx with the presence of a centrally depressed fragment; which is too large for a hemihamate graft. The AP radiograph shows comminution extending obliquely as well as in the sagittal plane, suggesting a pilon-type injury. This pattern of injury would not be reduced adequately with dorsal blocking alone. Complex fractures/dislocations of the base of the middle phalanx prove challenging to treat. Keys to success involve reduction of the dorsally dislocated middle phalanx base, restoration of articular congruity, and provision of early motion.

One useful approach to these injuries involves application of a dynamic traction device. Commercial devices are available; however, a construct composed of pins and rubber bands, described by Suzuki, allows creation of a dynamic traction device with materials available at any hospital. Outcomes from this treatment can achieve anywhere between 70 to 90% of the motion of the uninjured joint. When unstable, simple fracture/dislocations of the proximal interphalangeal joint can be treated with open reduction and internal fixation. Multiple techniques have been employed including dorsal, palmar, and midlateral approaches. Screws, cerclage wires, and even miniplates have been used. The lateral radiograph demonstrates this with the depressed articular fragment in the volar half of the joint surface. When severe comminution destroys enough of the volar half of the middle phalanx base so as to make restoration of stability unlikely, an osteochondral graft from the dorsum of the hamate between the little and ring finger metacarpal bases can be harvested and used to replace the lost articular surface. Use of this procedure requires the presence of a dorsal articular segment with some stability, to which the graft may be affixed. The AP radiograph demonstrates fracture lines extending through the dorsoradial articular surface of the middle phalanx, making hemi-hamate grafting difficult if not impossible. Arthrodesis is reserved as a salvage repair after attempts at primary repair have failed

308
Q

A 52-year-old woman comes to the office to discuss revision of breast reconstruction following mastectomy for breast cancer. She is undergoing adjuvant treatment with an agent that interferes with her body’s natural mechanisms that promote native breast growth, but she cannot remember its name. She is most likely being treated with which of the following agents?

A) Alkylating agent (cyclophosphamide)
B) Anthracycline (doxorubicin)
C) Aromatase inhibitor (anastrozole)
D) Platinum agent (cisplatin)
E) Taxane (paclitaxel)

A

The correct response is Option C.

Aromatase inhibitors such as anastrozole impair conversion of androgens to estrogens. Estrogens promote normal breast tissue growth as well as growth of many breast cancers. The other options (taxanes, anthracyclines, alkylating agents, and platinum agents) are all chemotherapeutic agents that do not particularly target hormones involved in normal breast growth mechanisms.

309
Q

A 23-year-old African-American man presents with a raised thickened scar on his anterior chest that he complains is pruritic and unattractive. It was removed by another provider 4 years earlier and has slowly recurred over the past year. On examination, the lesion extends beyond the initial borders of the scar and is firm and hyper-pigmented. On review of his prior pathology report, which of the following histologic characteristics is most likely?

A) Greater ratio of type III to type I collagen
B) Multitude of myofibroblasts and smooth muscle actin
C) Parallel collagen bundles
D) Thick, wavy, and randomly oriented collagen fibers

A

The correct response is Option D.

In patients with abnormal or excessive scar tissue formation, treatment and prognosis will be driven by the correct diagnosis of a keloid versus a hypertrophic scar. This patient presents with a recurrent keloid of the chest. His clinical history supports this diagnosis by recurrence after resection, growth extending beyond the original border of the lesion, late recurrence after several years, and continued growth over several years without regression or improvement. Hypertrophic scars are less likely to recur, contained within the original boundaries of the lesion, often regress somewhat within a year, and recur earlier in the postoperative period if they are to recur. Both hypertrophic scars and keloid scars can be pruritic.

Pathologic analysis of keloids reveals more type I collagen than type III collagen, similar to normal skin. Hypertrophic scars will exhibit increased type III collagen and pro-fibrotic collagen cross-linking. Keloid growth is thought to be impacted by cell-signaling between keratinocytes and fibroblasts, but hypertrophic scar production requires an abundance of myofibroblasts expressing smooth muscle actin. While hypertrophic scars have parallel collagen fibrils and bundles, keloids are characterized histologically by thick, randomly oriented collagen fibrils that are not organized into bundles.

References

Arno AI, Gauglitz GG, Barret JP, et al. Up-to-date approach to manage keloids and hypertrophic scars: a useful guide. Burns. 2014 Nov;40(7):1255-1266.

Jeong HS, Lee BH, Sung HM, et al. Effect of botulinum toxin type A on differentiation of fibroblasts derived from scar tissue. Plast Reconstr Surg. 2015 Aug;136(2):171e-178e.

310
Q

A 28-year-old woman comes for follow-up evaluation 2 weeks after undergoing bilateral augmentation mammaplasty with subpectoral placement of 325-mL, round, smooth saline prostheses. She is now concerned that both prostheses appear “too high.” Physical examination shows fullness in the upper quadrants of both breasts. Which of the following interventions is most appropriate?

A)Administration of oral zafirlukast
B)Application of a circumferential breast band
C)Injection of corticosteroid into the inframammary crease
D)Open capsulotomy
E)Percutaneous release of the inframammary crease

A

The correct response is Option B.

The most appropriate recommendation is breast band application. Breast shape following augmentation mammaplasty undergoes dynamic changes. The skin envelope and pectoralis muscle stretch under the expansion pressure of the prosthesis. The skin of the lower pole will stretch, allowing the prostheses to migrate inferiorly. Breast massage and a circumferential elastic breast band applied around the superior breast encourage this migration.

Zafirlukast is a leukotriene-antagonist that is used for the treatment of asthma. Preliminary studies suggest improvement in capsule contractures. This drug is associated with potential life-threatening liver complications as well as neuropsychiatric events. Because administration in the scenario described would constitute an off-label use of the drug, extensive discussion with the patient would be required prior to use.

In the past, steroid was injected into the saline compartment of a double-lumen prosthesis in an attempt to decrease the incidence of capsule contraction. This delivery system was uncontrolled and many prostheses migrated beyond the normal limits of the inframammary crease. Postoperative steroid injection has been used with some success for the prevention of recurrent capsule contracture following capsulectomy.

If residual inferior pectoralis muscle fibers are left intact along the rib or capsule contracture develops, open capsulotomy may be required; however, conservative treatment is indicated at this early postoperative period.

Percutaneous release would expose the patient to unnecessary complications of prosthesis injury, bleeding, and inframammary crease malposition.

311
Q

A 40-year-old man sustained traumatic amputation of all fingers of the dominant hand 3 months ago. Tripod pinch reconstruction is planned with a double second toe transfer. Which of the following arteries is most likely to be the dominant blood supply to the second toe transfer in this patient?

A) First dorsal metatarsal artery
B) First plantar metatarsal artery
C) Lateral plantar artery
D) Medial plantar artery
E) Third plantar metatarsal artery

A

The correct response is Option A.

The first dorsal metatarsal artery (FDMA) is the dominant blood supply (to the great toe and second toe) in approximately 70% of cases. The first plantar metatarsal artery (FPMA) is the dominant blood supply in 20% of cases. The FDMA and the FPMA have a similar vessel caliber in the remaining 10% of cases.

The dominant vascular pattern can be evaluated by careful retrograde dissection that begins at the dorsal aspect of the first web space. The junction of the lateral digital artery of the great toe and the medial digital artery of the second toe can be identified just above the intermetatarsal ligament. Proximal dissection continues dorsally and plantarly to evaluate the FDMA and FPMA.

If the FDMA is the larger caliber vessel or of similar caliber to the FPMA, then the toe transfer can be based on the FDMA. Proximal dissection of the FDMA to obtain length is relatively straightforward. In the setting of a plantar dominance, dissection of the FPMA is carried out proximally, which can be more challenging. Plantar proximal dissection is typically limited to the mid metatarsal level to avoid additional morbidity. If additional length is required on the FPMA pedicle, a vein graft can be used. It is important to note that in bilateral second toe transfers, the dominant vascular pattern can be asymmetric in 20% of patients.

References

Coskunfirat OK, Wei FC, Lin CH, Chen HC, Lin YT. Simultaneous double second toe transfer for reconstruction of adjacent fingers. Plast Reconstr Surg. 2005 Apr;115(4):1064-1069.

Henry SL, Wei FC. Thumb reconstruction with toe transfer. J Hand Microsurg. 2010 Dec;2(2):72-78.

Spanio S, Wei FC, Coskunfirat OK, Lin CH, Lin YT. Symmetry of vascular pedicle anatomy in the first web space of the foot related to toe harvest: clinical observations in 85 simultaneous bilateral second-toe transfer patients. Plast Reconstr Surg. 2005 Apr 15;115(5):1325-1327.

Wei FC, Henry SL. Toe-hand-transplantation. In: Wolfe SW, Hotchkiss RN, Pederson WC, Kozin SH, eds. Green’s Operative Hand Surgery. Philadelphia, PA: Elsevier Churchill Livingstone; 2011:1807-1837.

312
Q

A 27-year-old man is brought to emergency department because of a thumb avulsion injury measuring 3 × 3 cm. A photograph is shown. Which of the following is the best option for sensate, soft-tissue coverage?

A) Cross-finger flap
B) First dorsal metacarpal artery flap
C) Moberg flap
D) Pedicled groin flap
E) Skin grafting

A

The correct response is Option B.

There are several sensate options for thumb pulp deformities. These include neurovascular island flaps, Moberg flaps, free toe pulp flaps, and the first dorsal metacarpal artery flap (FDMA). Cross finger flaps, skin grafts, and pedicle groin flaps do not have innate innervation. The defect in the question involves the entire pulp of the thumb overlying the distal phalanx and is too large for a Moberg advancement flap.

The FDMA flap is supplied by its eponymous artery, which travels in the fascia overlying the index metacarpal and supplies the skin overlying the dorsum of the proximal phalanx. The vessel is accompanied superficially by a branch of the radial nerve that provides neural activation to the skin overlying the proximal phalanx of the index finger. The flap can be transposed to provide sensate coverage of the tip of the thumb, and can provide sufficient size to resurface relatively large defects.

Cortical reorientation is the fact that the brain recognizes a stimulus from the flap area as a stimulus from the thumb, and not from the index finger. This process takes some time, but is usually complete after 2 years. Average static two-point discrimination in these flaps utilized for thumb resurfacing is 10 to 11 mm.

Use of the FDMA flap for resurfacing of the thumb pulp has been compared to heterodigital island flaps in several studies. Both remain options to be considered, but the ease of elevation, limited dissection, and acceptable donor site morbidity make the FDMA flap a common primary option for thumb tip resurfacing.

References

Tränkle M, Sauerbier M, Heitmann C, Germann G. Restoration of thumb sensibility with the innervated first dorsal metacarpal artery island flap. J Hand Surg Am. 2003; 28(5):758–766.

Muyldermans T, Hierner R. First dorsal metacarpal artery flap for thumb reconstruction: a retrospective clinical study. Strat Traum Limb Recon. 2009; 4:27.

Delikonstantinou IP, Gravvanis AI, Dimitriou V, Zogogiannis I, Douma A, Tsoutsos DA. Foucher first dorsal metacarpal artery flap versus Littler heterodigital neurovascular flap in resurfacing thumb pulp loss defects. Ann Plast Surg. 2011 Aug;67(2):119-22.

313
Q

An otherwise healthy 67-year-old woman with advanced breast cancer is scheduled to undergo mastectomy and immediate reconstruction with a free transverse rectus abdominis musculocutaneous (TRAM) flap. BMI is 35 kg/m2. Which of the following is most appropriate for deep venous thrombosis prophylaxis?

A) Aspirin and intermittent pneumatic compression stockings
B) Elastic compression stockings only
C) Intermittent pneumatic compression stockings only
D) Low-molecular-weight heparin and intermittent pneumatic compression stockings
E) Positioning and early ambulation

A

The correct response is Option D.

This patient is at highest risk for deep venous thrombosis and she will require combination therapy of compression stockings and chemical prophylaxis.

Perhaps the most well-regarded set of guidelines in this matter comes from the American College of Chest Physicians. These guidelines provide treatment recommendations based on a patient’s risk classification. Caprini model offers a very user-friendly method of calculating patient risk factors and categorizing them into low, moderate, high, and highest risks.

The patient in question has risk factors (age, malignancy, and major surgery) that would put her in the highest risk category. In the highest risk category, prophylaxis will require combination therapy or warfarin.

314
Q

A 25-year-old woman comes to the office because she is dissatisfied after undergoing breast augmentation mammaplasty for correction of tuberous breast deformities. Physical examination shows two parallel creases running transversely across the lower pole of each breast with inferior displacement of the implant. Which of the following best describes the position of the original inframammary fold in this patient?

A) Above the superior and inferior transverse creases
B) At the inferior transverse crease
C) At the superior transverse crease
D) Below the superior and inferior transverse creases

A

The correct response is Option C.

A double-bubble breast deformity following breast augmentation mammaplasty is represented by the development of two parallel, curvilinear transverse lines in the lower pole of the breast. The native inframammary fold is disrupted and represented by the superior transverse line. The lower transverse line represents the lower limit of implant pocket dissection or the final position of implant descent.

Predisposing anatomic factors for the development of a double-bubble deformity include tuberous breasts, constricted inframammary folds, or a short inframammary fold-to-nipple distance. Other factors that can increase the risk for the development of a double-bubble deformity include glandular ptosis, postpartum involution of the breasts, excessive implant size, and overdissection of the implant pocket. Correction of the double-bubble deformity may require conversion of the implant to a subglandular position, capsulorrhaphies, use of form-stable implants, or dermal grafts.

315
Q

Incomplete involution of the mammary ridge during embryonic development is most likely to result in which of the following?

A) Amastia
B) Gynecomastia
C) Inverted nipple
D) Poland syndrome
E) Polymastia

A

The correct response is Option E.

The breast develops as the result of bilateral thickening of ectoderm along the milk line, or mammary ridge, from the axillary to the inguinal region. Mammary buds begin to develop as growths within the epidermis and invade the deeper mesenchyme. Much of the ridge disappears as the embryo develops as the result of apoptosis, except for the primary buds in the pectoral regions. Failure of regression of the mammary ridge can result in accessory breasts (polymastia) or accessory nipples (polythelia). Accessory breast tissue occurs in 1 to 2% of live births and commonly occurs in the axillae.

Amastia is the complete absence of the mammary gland. This occurs due to either the failure of the mammary ridge to develop or the complete involution of the mammary ridge.

Gynecomastia is defined as benign enlargement of the male breast. While pathologic cases can exist, it is most typically due to a normal response of the breast tissue to circulating levels of estrogen.

Inverted nipples are due to failure of the mesenchyme to proliferate above the level of the skin.

Poland syndrome can have the following components: hypoplasia of the breast and nipple, absence of the sternocostal portion of the pectoralis major muscle, absence of the pectoralis minor muscle, abnormalities of the chest wall, and anomalies of the upper extremity. Many etiologies have been hypothesized, with the most widely accepted being an interruption of the embryonic blood supply to the upper limb.

316
Q

A 36-year-old man undergoes rehabilitation following scapholunate ligament repair. Initial range of motion in therapy is planned to allow wrist movement while minimizing the movement between the scaphoid and lunate bones. Which of the following wrist movements is most likely to achieve this goal?

A) Neutral wrist extension to neutral wrist flexion
B) Radial deviation in extension to radial deviation in flexion
C) Radial deviation in extension to ulnar deviation in flexion
D) Ulnar deviation in extension to radial deviation in flexion
E) Ulnar deviation in extension to ulnar deviation in flexion

A

The correct response is Option C.

The most appropriate motion is from radial deviation in extension to ulnar deviation in flexion.

The dart-thrower’s motion, moving from radial deviation in extension to ulnar deviation in flexion, minimizes the movement between the scaphoid and lunate.

Studies have shown that during movement in this axis, from radial deviation and extension to ulnar deviation and flexion, the bones of the proximal carpal row (scaphoid, lunate, triquetrum), remain practically stationary, and motion occurs primarily through the midcarpal joint. As a result, this is felt to be the primary mechanical axis of movement in the wrist. Rehabilitation with movement in the dart-thrower’s axis will limit movement between the bones of the proximal carpal row and allow wrist range of motion while minimizing stress on a scapholunate repair.

Simulation of radioscapholunate fusion results in preservation of the dart-thrower’s motion, confirming this concept.

The remaining motions listed result in greater intercarpal movement of the proximal row.

317
Q

A 24-year-old nulliparous woman comes to the office for augmentation mammaplasty. She currently wears a size 34B brassiere and wants her brassiere size to be increased to a D cup. She is a good candidate for subglandular placement of implants. Which of the following risks is decreased by the use of the textured silicone shell compared with the smooth silicone shell?

A) Capsular contracture
B) Hematoma
C) Prosthesis malposition
D) Rippling
E) Symmastia

A

The correct response is Option A.

Texturing of the implant surface has been shown to decrease the rate of capsular contracture when compared with smooth implants when the implants are placed in the subglandular position. The benefit of textured implants may not be present when the implants are placed in a submuscular pocket.

There is no difference in hematoma rates for textured versus smooth implants. Both symmastia and implant malposition are related to pocket dissection and not related to the type of implant placed. In the case of symmastia, the pockets have encroached upon the sternum and are close to each other or are touching. Implant malposition can be related to factors such as inadequate dissection of the pocket, or over-dissection of the pocket. Finally, some studies have demonstrated an increase in rippling with textured implant when compared with smooth implants. However, rippling may be more related to cohesiveness of the gel and fill volumes of the shell, because early reports of experience with the form-stable implant (Natrelle 410) seem to show decreased rates of rippling.

318
Q

A 45-year-old woman undergoes mastectomy. Which of the following anatomical landmarks denotes a limit of the breast and, therefore, a limit to the extent of the mastectomy?

A) Anterior border of latissimus dorsi muscle
B) Inferior origin of pectoralis major muscle
C) Lateral pectoralis minor muscle
D) Superficial fascia of the serratus anterior muscle
E) Supraclavicular lymph node basin

A

The correct response is Option A.

The borders of the breast and, therefore, the limits of the mastectomy are the sternum, clavicle, inframammary fold, and anterior border of the latissimus dorsi muscle. The other options are not designated as borders of the breast tissue.

319
Q

A 53-year-old woman is evaluated for left-sided nipple reconstruction after mastectomy. She has scars on the left breast from a previous breast biopsy, as well as from the mastectomy itself. Nipple reconstruction must be designed around the scars. In single-pedicle nipple reconstruction, which of the following provides the blood supply to the pedicle?

A) Internal mammary artery perforators
B) Posterior intercostal arteries
C) Subdermal plexus
D) Superior intercostal artery
E) Thoracoacromial artery perforators

A

The correct response is Option C.

Single-pedicle nipple reconstructions, which include such techniques as the skate flap, star flap, C-V flap, and opposing tab flaps as well as other variations, create nipples from remaining mastectomy skin through adjacent tissue transfer. The flap derives its blood supply from the subdermal plexus.

The creation of the flap must keep this blood supply in mind. The flap design must avoid previous scars at the flap base and must integrate the subcutaneous fat at the base of the pedicle.

The internal mammary artery supplies the breast itself and the nipple-areola complex, and the thoracoacromial artery supplies the pectoralis muscle and the breast. The posterior intercostal arteries supply the intercostal spaces. The superior intercostal artery arises from the costocervical trunk, off of the subclavian artery, and supplies the intercostal spaces.

320
Q

Advances in Breast Reconstruction):23S-29S.

A 35-year-old woman presents with unilateral swelling that has developed over the past 3 months. She underwent bilateral nipple-sparing mastectomy with immediate implant reconstruction with textured, round silicone gel implants 8 years ago. Ultrasound confirms periprosthetic seroma without any masses. Which of the following is the most appropriate next step in the management of this patient?

A) Core needle biopsy
B) Fine-needle aspiration
C) Implant removal and capsulectomy
D) MRI
E) Positron emission tomography (PET) scan

A

The correct response is Option B.

The clinical scenario is concerning for breast implant–associated anaplastic large-cell lymphoma (BIA-ALCL). Aspiration of the fluid seen on ultrasonound and pathologic evaluation is necessary to confirm the diagnosis. Following the National Comprehensive Cancer Network guidelines, initial workup of an enlarged breast should include ultrasound evaluation specifically for a fluid collection, a breast mass, or enlarged regional lymph nodes (axillary, supraclavicular, and internal mammary).

MRI is appropriate for cases where ultrasound is indeterminate or requires further confirmation. This patient does not have an identifiable mass amenable to core biopsy. Positron emission tomography (PET) scan is beneficial in confirmed cases to identify associated masses, chest wall involvement, regional lymphadenopathy, and/or metastasis. Implant removal and capsulectomy is appropriate once the diagnosis is confirmed.

321
Q

A 45-year-old woman comes to the office for consultation regarding augmentation mammaplasty. She wears a size 32B brassiere; height is 5 ft 3 in (160 cm), and weight is 130 lb (59 kg). Subglandular placement of saline prostheses is planned. Which of the following is the primary advantage of using saline rather than silicone prostheses in this patient?

A)Easier detection of rupture
BLess capsular formation
C)Less wrinkling
D)Lighter prosthesis
E)Lower risk of leakage

A

The correct response is Option A.

Although both silicone and saline prostheses rupture at a similar rate, a saline rupture is more easily detectable because the saline is resorbed in the body. The deflated breast will be smaller in volume. Subtle changes, such as decreased upper pole fullness or increased softness, may be the only clues to silicone rupture on physical examination. Ultrasonography or MRI may be needed to confirm the diagnosis.

Saline prostheses are firmer than silicone; they are more likely to be palpable than silicone prostheses as well. Neither prosthesis has been associated with systemic immune syndromes, and both prostheses produce capsular contracture, wrinkling, and leakage.

322
Q

A 45-year-old woman presents with right breast cancer and is planning a nipple-sparing mastectomy and tissue expander placement. She is specifically interested in a carbon dioxide–based expander. Which of the following is a disadvantage of this device compared with a saline tissue expander?

A) Extrusion
B) Inability to deflate
C) Increase in wound dehiscence
D) Increase in wound infection
E) Possible device dislocation

A

The correct response is Option B.

The carbon dioxide-based tissue expander (AeroForm) is a fixed-volume device and has an inability to deflate the expander.

In a prospective, multicenter, randomized controlled trial comparing carbon dioxide–based expanders and saline tissue expanders, there were no statistically significant differences in rates of wound infection, extrusion, device dislocation, or wound dehiscence. Advantages of the carbon dioxide–based expander include a more rapid expansion process and a shorter time to implant exchange. The device is self-contained and patient-controlled, so there are no needles required and possibly fewer physician office visits.

323
Q

A 65-year-old man has a 4-cm defect in the mid portion of the metacarpal of the long finger after sustaining a gunshot wound to the left hand. Which of the following is the most appropriate treatment of this defect?

A ) Coverage with a free vascularized bone flap
B ) Distraction osteogenesis
C ) Injection of calcium phosphate cement
D ) Interpose an autologous bone graft
E ) Placement of demineralized bone matrix

A

The correct response is Option D.

In a patient with a noncritical (less than 6- to 8-cm) bone defect of the hand, reconstruction with an autologous bone graft provides the best combination of maximal healing and minimal morbidity. To provide the best chance of successful healing, the graft should allow rigid stabilization. Corticocancellous grafts from the iliac crest are the typical source.

Free vascularized bone flaps, such as the fibula, are essential tools when dealing with critical bone defects (greater than 6 to 8 cm). They do, however, add significant morbidity to the procedure and may not be feasible in individuals with severe peripheral vascular disease. In this patient, the noncritical defect would argue against the need for vascularized bone.

Distraction osteogenesis works well for bone defects ranging from 1.5 to 13.5 cm. A prerequisite, however, is adequate bone stock to allow pin placement for the distractor. In this case, a 4-cm defect would leave insufficient bone at the metacarpal base and head to allow distractor placement.

Calcium phosphate cement (Norian, Synthes) offers an osteoconductive substrate in bone defects that are already stabilized. In addition, its use is contraindicated in infected or potentially infected wounds.

Demineralized bone matrix is osteoinductive, but, like calcium phosphate cement, will not offer any structural stability.

324
Q

A 53-year-old woman comes to the office for evaluation of breast asymmetry. Reduction of the left breast and augmentation of the right breast with implant and autologous fat transfer are planned. She is concerned about fat injection and cancer risk. Which of the following is the most appropriate response regarding mammographic changes after fat transfer?

A) Calcifications warranting biopsy are more likely on the fat transfer side
B) Calcifications warranting biopsy are more likely on the reduction side
C) Masses requiring biopsy are more likely on the reduction side
D) Scarring will be decreased on the reduction side
E) There are no differences between mammographic findings in fat transfer and reduction

A

The correct response is Option C.

Fat transfer to the breast remains a controversial procedure. There are some concerns about the oncologic safety of fat transfer, and for this reason some authors do not recommend fat transfer in patients with a history of cancer. Another concern about fat transfer is the potential difficulty in screening for malignancy. Rubin, et al. compared mammographic changes after fat transfer with changes after reduction mammaplasty. In this blinded study, radiologists reviewed pre- and postoperative mammograms of patients who had undergone augmentation and fat transfer and reduction mammaplasty. In the reduction cohort, masses requiring biopsy and scarring were more common; other abnormalities, including oil cysts, benign calcifications, and calcifications requiring biopsy showed no differences between the groups.

325
Q

An otherwise healthy 28-year-old woman comes to the physician requesting removal of bilateral axillary masses. She states that the masses fluctuate in size and tenderness with her menstrual cycle. She reports that the masses have not had discharge or drainage. Physical examination shows smooth, spongy masses in both axillae. A photograph is shown. Which of the following is the most appropriate next step in management of this patient?

A) Excise the bilateral axillary masses and skin
B) Obtain bilateral mammograms of the axillary masses
C) Order an MRI of the chest
D) Perform a core biopsy of both axillary masses
E) Perform liposuction

A

The correct response is Option A.

This patient presents with ectopic breast tissue. In utero, the milk line (galactic band) forms at 5 weeks of gestation. This bilateral structure courses from the axillae to the groin, and normal breasts form in the prepectoral region after there has been regression of the rest of the galactic band. When there is failure of this regression, breast tissue remains in locations outside of the normal breast. The most common location for ectopic breast tissue is in the axillae, although it can be found anywhere along the milk line from the axillae to the groin. Ectopic breast tissue outside of the milk line has been described and is termed aberrant breast tissue.

The tissue found in these ectopic locations is breast tissue with the same characteristics and propensity for disease as normally located breast tissue, and breast cancer has been described in these tissues. In the absence of pathologic findings such as a mass, pain, and skin changes that are associated with breast cancer, there is no strong oncologic indication for excision. If there are findings concerning for a neoplasm, then work-up should be initiated and might include further imaging, core biopsy, and surgery. However, most cases present without pathologic findings and are excised to achieve a more reasonable appearance for the patient, the ability to don clothing more comfortably, and for the obvious social advantages.

In this case, the patient is young, has no complaints, and has no physical findings to suggest a neoplasm. Excision should be offered.

Obtaining bilateral mammograms is incorrect because there is no indication for imaging in this patient based on her age, history, and physical examination. In addition, mammograms of axillary breast tissue are technically unfeasible.

Performing a core biopsy is incorrect as there is no concern for malignancy in this case. In the case of a mass noted within the ectopic axillary breast tissue, then an oncologic workup should be initiated which might include a core biopsy.

An MRI of the chest is incorrect because there is no indication for imaging in this patient based on her age, history, and physical examination.

Reassuring the patient with no further action is not the most appropriate management, as it will not address the patient’s concerns and desires. In the patient who does not request excision or is not an appropriate surgical candidate, then reassurance and surveillance are appropriate.

As this is a young female with axillary breast tissue, liposuction will not improve the excess breast tissue or skin.

326
Q

A 17-year-old girl with a history of bilateral cleft lip/palate presents for evaluation. She underwent lip and palate repair as a child, and alveolar bone grafting in mixed dentition. She has hypernasal speech. Physical examination shows severe mid face hypoplasia with 12 mm of negative overjet. On cephalometric analysis, SNA angle is 73 degrees (N 80–82), and SNB angle is 79 degrees (N 79–81). She is concerned about her appearance and her speech. Which of the following is the best initial option to address the patient’s concerns?

A) Bilateral sagittal split osteotomy with mandibular setback
B) Combined Le Fort I advancement and mandibular advancement
C) Maxillary distraction
D) Pharyngoplasty
E) Speech therapy

A

The correct response is Option C.

This patient has severe maxillary retrusion and mid face hypoplasia in the setting of significant velopharyngeal deficiency. These two issues are at odds with one another. Advancing her mid face with a standard Le Fort I will worsen her velopharyngeal dysfunction (VPD), while addressing her VPD with a pharyngeal flap will make advancing her maxilla challenging. Only anterior segmental maxillary distraction will maintain her current velopharyngeal anatomy but allow for improved mid face projection. A pharyngoplasty can be performed relatively easily at a later date or as a secondary procedure. Performing a mandibular setback is not appropriate because she has normal mandibular projection.

327
Q

A 58-year-old man is to undergo excision of a painful ulnar artery aneurysm of the palm, which has been causing ulnar nerve compression. Preoperative examination shows a digital/brachial index (DBI) of 0.5 in the small finger. After excision of the diseased segment, which of the following is the most appropriate next step in management?

A) Arterial reconstruction
B) Botulinum toxin type A injection
C) Extended periarterial sympathectomy
D) Periarterial injection of 2% lidocaine
E) Postoperative anticoagulation

A

The correct response is Option A.

Ulnar artery aneurysms may cause symptoms because of local mass effect, distal embolization, and/or episodic vasospasm. Ligation of the ulnar artery to exclude the aneurysm from hand circulation can effectively eliminate risk for embolism, but may rob the digits of necessary blood flow if there is not enough collateral circulation from the deep arch or other sources. Measuring the digital-brachial index (DBI) is an effective way to assess whether or not there is sufficient blood flow to the digits. A normal DBI is between 0.75 and 0.97. Values equal to or less than 0.7 indicate inadequate perfusion. Below a DBI of 0.5, tissue loss is inevitable. Following ulnar artery ligation, if the DBI is below 0.7, then reconstruction of the ulnar artery is recommended rather than simple aneurysm excision or ligation. This is typically accomplished with a reversed vein graft or an arterial graft (e.g., from the lateral femoral circumflex system).

Anticoagulation alone, or anti-vasospastic drugs, such as botulinum toxin type A or lidocaine, are not sufficient in this clinical situation, where blood flow is limited because of blockage of flow. While sympathectomy could improve circulation in cases of vasospasm, this patient had no history of this, and sympathectomy alone would not be a substitute for arterial reconstruction.

References

McClinton MA. Reconstruction for ulnar artery aneurysm at the wrist. J Hand Surg Am. 2011;36(2):328-32.

Berger AC, Kleinert JM. Noninvasive vascular studies: a comparison with arteriography and surgical findings in the upper extremity. J Hand Surg Am. 1992;17(2):206-10.

Zimmerman NB. Occlusive vascular disorders of the upper extremity. Hand Clin. 1993 Feb;9(1):139-50.

Hui-Chou HG, McClinton MA. Current options for treatment of hypothenar hammer syndrome. Hand Clin. 2015 Feb;31(1):53-62

328
Q

A 40-year-old man with a history of seizure disorder presents with jaw pain and malocclusion 12 hours after he was punched in the face during a brawl. X-ray studies show a displaced left angle fracture and right parasymphyseal fracture. He smokes one pack of cigarettes daily. Open reduction and internal fixation of the fractures is planned. Which of the following factors in this patient’s clinical presentation places him at greatest risk for postoperative wound infection?

A) History of seizures
B) History of tobacco use
C) Patient age
D) Patient gender
E) Surgery delayed until 5 days after injury

A

The correct response is Option B.

Although early treatment of mandible fractures would help relieve pain, delay of treatment in multiple series has not been directly correlated with an increased wound infection rate. In a recent review, tobacco use has been associated with a sixfold increase in wound infection compared with nonsmokers. Although the incidence of complications after mandible fractures is lower in children, in the adult population, patient age and gender do correlate with an increased complication rate. Factors that have correlated with a higher complication rate include the number of mandible fractures present and incision location, intraoral and combined intraoral, and combined intraoral and extraoral incisions were associated with higher infection rates than only extraoral incisions.

329
Q

A 43-year-old woman undergoes the second stage of tissue expander–based breast reconstruction. Exchange of the tissue expander for a smooth round silicone implant is planned along with a superior capsulotomy and fat grafting to the upper pole for contour improvement. Which of the following is an increased risk associated with fat grafting to the breast in this patient?

A) Anaplastic large cell lymphoma
B) Benign lesions
C) Hypopigmentation
D) Infection
E) Recurrent breast cancer

A

The correct response is Option B.

Autologous fat grafting is a widely accepted technique in breast reconstruction. A large systematic review recently confirmed the oncologic safety of this technique but did report a significant incidence of benign lesions including cysts and calcifications. Fat grafting is not associated with an increased risk of recurrent breast cancer, infection or hypopigmentation.

330
Q

A 62-year-old woman is evaluated because of a new 2 × 2-cm open area near her left axillary fold. Medical history is significant for left breast cancer previously treated with bilateral mastectomies, left axillary node dissection, and adjuvant chemoradiation 10 years ago. A photograph is shown. She has been compliant with postoperative oncologic surveillance and has had no recent trauma. Which of the following underlying conditions is most likely responsible for her current presentation?

A) Empyema with spontaneous drainage
B) Lymphedema drainage tract
C) Osteoradionecrosis of the underlying rib(s)
D) Recurrent breast cancer
E) Skin ulceration from intertriginous shearing forces

A

The correct response is Option C.

The effects of ionizing radiation are permanent and may present either acutely or in delayed fashion, even years after the original radiation insult. The mechanism of injury from this radiation is through free radical production which, in turn, directly damages the DNA. In the acute period, the effects of radiation may manifest themselves as erythema and edema of the skin, vasodilation with endothelial edema, and lymphatic obliteration. This eventually leads to capillary thrombosis and subsequent inadequate tissue oxygenation. Over time, nonhealing ulcers can spontaneously develop, sometimes years later.

Although recurrent cancer is always a concern in patients with a personal history of cancer, proper, regular, and thorough surveillance can often detect recurrences early, especially in compliant patients. Most recurrences occur within the first 5 years.

Abscesses usually present initially with pain, erythema, and localized fluctuance, and often with associated fever and/or malaise. Spontaneous necessitation to the skin would also result in purulent drainage.

Lymphedema can be a chronic condition after mastectomy and axillary node dissection, and is usually manifested as generalized edema of the ipsilateral upper extremity. Sinus tract formation is rare.

Intertriginous shearing would most often present as superficial epidermal loss with possible superinfection with yeast due to moisture.

331
Q

A 30-year-old woman comes to the office because she is unable to flex the distal interphalangeal (DIP) joint of the right long finger. An x-ray study is shown. She reports that she injured the finger 2 days ago when attempting to restrain her dog. Physical examination shows no active flexion of the DIP joint; however, the DIP joint can be passively flexed from 0 to 80 degrees. During surgical exploration, the distal end of the flexor digitorum profundus tendon is most likely to be found at the level of which of the following structures?

A ) A4 pulley

B ) Camper chiasm

C ) Central slip

D ) Sagittal band

E ) Terminal tendon

A

The correct response is Option A.

The injury to the patient described is commonly referred to as a €œjersey finger. € Patients have the sudden failure of fingertip-level grasp, like a football player attempting to tackle an opponent by the jersey. Injury may involve a pure soft-tissue rupture of the flexor digitorum profundus (FDP) tendon, or a portion of the volar proximal aspect of the distal phalanx may be avulsed along with the tendon. Injuries are classified based on the type of fracture and how proximally the FDP tendon has retracted. In the patient described, a large fragment of the distal phalanx base remained attached to the FDP tendon. The tendon and fragment are held in this position by the A4 pulley.

Camper chiasm is where the flexor digitorum superficialis (FDS) tendon splits to pass dorsal to the FDP tendon en route to its insertion at the base of the middle phalanx. Camper chiasm does not exist distal to the proximal interphalangeal (PIP) joint. The sagittal band is a stabilizer of the extrinsic extensor tendons over the dorsum of the metacarpophalangeal (MCP) joint. It also does not contact the FDP tendon. The terminal tendon and central slip are components of the finger extensor mechanism. They do not contact the FDP tendon.

The postoperative x-ray study shown demonstrates reduction of the bone fragments to the shaft of the distal phalanx. A Bunnell suture was used to capture the tendon and the multiple bone fragments. This suture was then passed through the distal phalanx and tied dorsally over a button.

332
Q

A 45-year-old woman comes to the office 1 week after sustaining an injury to her right hand in a golfing accident. Physical examination shows tenderness at the ulnar base of the palm and numbness of the little finger. Which of the following injuries is best exposed using a carpal tunnel x-ray view of the wrist?

A ) Hook of the hamate fracture
B ) Lunotriquetral separation
C ) Scaphoid fracture
D ) Scapholunate dissociation
E ) Trapezium body fracture

A

The correct response is Option A.

Specialized views of the wrist can provide better information regarding bony relationships and fractures, in addition to standard anteroposterior, lateral, and oblique films. There are many different specialized views that the plastic surgeon should be familiar with. Among these are the scaphoid, stress, and carpal tunnel views. The carpal tunnel view is a hyperextended wrist view displaying the carpal bone to carpal tunnel relationships. This view allows visualization of the hook of the hamate and the pisotriquetral joint, as well as the palmar surfaces of the trapezium, pisiform, and triquetrum.

333
Q

A 22-year-old woman presents for consideration of aesthetic breast surgery to address asymmetry. Physical examination shows a unilateral hypoplastic breast with a constricted, elevated base and a herniated nipple-areola complex. Which of the following is the most likely diagnosis?

A) Amastia
B) Micromastia
C) Poland syndrome
D) Tuberous breast
E) Virginal mammary hypertrophy

A

The correct response is Option D.

A tuberous breast is classically defined as hypoplastic with a constricted and elevated base, insufficient inferior skin, and a herniated nipple-areola complex.

Amastia would manifest without a nipple. Poland syndrome is classically described as missing the pectoralis muscle with variable breast and nipple effects. A constricted base and herniated areola are not usually associated with Poland syndrome. Hypertrophy would likely present with a broader base and increased volume. Micromastia would not manifest with a herniated areola.

334
Q

A 15-year-old girl with a history of a bilateral cleft lip and palate is evaluated because she is concerned about her nasal-sounding speech. On examination, she has a wide and poorly projected nasal tip, a bilateral cleft lip scar with a whistle deformity, mid face hypoplasia, persistent alveolar clefts with a protrusive premaxilla, a large fistula at the incisive foramen, and a straight line scar on the palate. Which of the following is the most appropriate first step in addressing her multiple cleft-related problems?

A) Alveolar bone graft
B) Fistula closure with premaxillary setback
C) Lip revision
D) Maxillary advancement
E) Rhinoplasty

A

The correct response is Option B.

Patients with cleft palate who are late-presenting and have undergone previous procedures that were performed out of the preferred sequence can be especially challenging cases. This patient, with a bilateral cleft lip and palate, has only had her lip and palate repaired. Yet she has significant speech, skeletal, and soft tissue sequelae of her cleft including unrepaired anterior hard palate fistulae and alveolar fistulae, inadequate central lip fullness (whistle deformity), a wide and poorly projected nose, mid face hypoplasia, and velopharyngeal insufficiency.

Sequencing procedures to address these issues is crucial to having favorable surgical outcomes. Addressing speech is the patient’s main concern and should be performed first. This includes closing of the oronasal fistula along with a premaxillary setback followed by a pharyngoplasty, if necessary. Alveolar bone grafts should be performed to stabilize the maxillary dental arch and minimize tooth injury. This should be followed by a lip revision to establish the lip-tooth relations prior to undergoing Le Fort I advancement. Finally, a rhinoplasty can be performed once the maxilla has been advanced.

335
Q

A 54-year-old woman is evaluated for nipple-areola complex reconstruction after mastectomy and silicone implant-based reconstruction. During discussion of the risks and benefits of a C-V flap, the patient asks about the long-term results of different techniques. Which of the following is the most likely long-term complication of a single-pedicle nipple-areola reconstruction?

A) Atrophic scarring
B) Delayed nipple necrosis
C) Hypertrophic scarring
D) Implant exposure
E) Loss of projection

A

) Loss of projection

The correct response is Option E.

Single-pedicle nipple reconstructions, which include techniques such as the skate flap, star flap, C-V flap, and opposing tab flaps as well as other variations, create nipples from remaining mastectomy skin through adjacent tissue transfer. The flaps derive their random-pattern blood supply from the subdermal plexus.

The creation of the flap must keep this blood supply in mind. The surgical technique must avoid previous scars at the base of the flap design and must integrate the subcutaneous fat at the base of the pedicle.

While hypertrophic and atrophic scarring can occur, they are not the most common long-term effects, and are more a function of patient characteristics than flap characteristics.

Implant exposure can occur with scar breakdown, but this is an early rather than a late complication.

Delayed nipple necrosis is technically not correct because the nipple is no longer present, and is not correct of the nipple reconstruction because necrosis of the flaps, if it occurs, usually occurs early.

336
Q

A 60-year-old woman receives low-molecular-weight heparin (LMWH) 40 U subcutaneously 1 hour before undergoing breast reconstruction using a unilateral transverse rectus abdominis musculocutaneous (TRAM) flap. Weight is 185 lb (84 kg); BMI is 32 kg/m2. Which of the following is the most likely effect of the LMWH on perioperative risks in this patient?

A ) Decreased risk of flap failure

B ) Decreased risk of postoperative hematoma

C ) Decreased risk of pulmonary thromboembolism

D ) Increased risk of blood transfusion

E ) Increased risk of infection

A

The correct response is Option C.

There are no uniform standards or guidelines for the routine use of chemoprophylaxis of venous thromboembolism in plastic surgery. At a minimum, lower extremity mechanical compression devices should be used on all patients undergoing general anesthesia. The patient described has multiple risk factors for developing deep venous thrombosis (DVT) and pulmonary embolism, such as an age over 50, prolonged surgery time, malignancy, and being overweight.

In such high-risk patients, data support the use of chemoprophylaxis with either standard heparin or low-molecular-weight heparin given subcutaneously. As the risk of developing DVT begins with anesthesia induction, it is generally recommended that heparin therapy be started before surgery. Out of concern for bleeding, some surgeons start heparin therapy in the early postoperative period. While more data are needed to clarify the optimal start time of therapy, it is clear that starting therapy before surgical incision is generally safe when dosed appropriately.

In large retrospective studies, chemoprophylaxis patients were not more likely to require blood transfusion, though they do demonstrate a slightly greater decrease in postoperative hemoglobin as compared with control patients. There is clearly a decreased risk of postoperative DVT and pulmonary embolism, both clinically apparent and asymptomatic. The data regarding postoperative hematoma are less clear, with some studies showing no increase in €œtake-backs € to the operating room for hematoma with chemoprophylaxis. One study did show an increase in hematomas in oncologic breast surgery with LMWH versus standard, unfractionated heparin. There is no association between flap survival or infection and subcutaneous heparin use.

Until reliable prospective, randomized data of sufficient study size are available to demonstrate optimal treatment, the studies seem to support the routine use of mechanical and perioperative subcutaneous heparin prophylaxis in high-risk patients.

337
Q

A 25-year-old man sustained traumatic amputation of the nondominant index finger 3 hours ago and requests replantation. Which of the following factors has the greatest influence on survival of the injured digit after replantation?

A) Mechanism of injury
B) Number of vessels repaired
C) Patient’s smoking status
D) Time from injury to replantation
E) Use of anticoagulation

A

The correct response is Option A.

The mechanism of injury has the greatest influence on survival of replanted digits. Injuries from sharp devices that leave a clean cut with little or no crush component are the most amenable to replantation. The more the tissue is crushed or avulsed, resulting in greater vessel injury, the less likely the digit will survive. No studies have shown that the use of anticoagulants changes survival rates. Smoking decreases blood flow in digits, but has not been widely studied in replantation. Fingers have no muscle, which is the tissue most susceptible to ischemia, so digits can tolerate long delays as long as they are treated correctly. At least two veins per artery have been shown to help prevent venous congestion.

References

Prucz RB, Friedrich JB. Upper extremity replantation: current concepts. Plast Reconstr Surg. 2014 Feb;133(2):333-42.

Sears ED and Chung KC. Replantation of Finger Avulsion Injuries: A Systematic Review of Survival and Functional Outcomes. J Hand Surg Am. 2011;36A:686– 694.

338
Q

A 48-year-old woman comes to the office because she is dissatisfied with the “sagging” appearance of her breasts. Physical examination shows the location of the nipples 1 cm above the inframammary fold bilaterally. The majority of breast tissue is below the fold. Which of the following is the most likely diagnosis?

A ) Grade 1 ptosis

B ) Grade 2 ptosis

C ) Grade 3 ptosis

D ) Pseudoptosis

A

The correct response is Option D.

Regnault defined the degree of ptosis by evaluating the relationship of the nipple to the inframammary fold.

In pseudoptosis, the nipple is above or at the level of the inframammary fold, with the majority of the breast tissue below. This gives the impression of ptosis.

In Grade 1, or mild ptosis, the nipple is within 1 cm of the level of the inframammary fold and above the lower contour of the breast and skin envelopes. In Grade 2, or moderate ptosis, the nipple is 1 to 3 cm below the inframammary fold but above the lower contour of the breast and skin envelopes. In Grade 3, or severe ptosis, the nipple is more than 3 cm below the inframammary fold and below the lower contour of the breast and skin envelopes.

339
Q

A patient undergoes extraction of a fully impacted mandibular third molar. During corticotomy of the mandible, protection of the adjacent soft tissue is necessary to avoid injury to which of the following nerves?

A) Facial
B) Hypoglossal
C) Inferior alveolar
D) Infraorbital
E) Lingual

A

The correct response is Option E.

Protection of the lingual border of the mandible during extraction of mandible wisdom teeth is critically necessary because of the close proximity of the lingual nerve to the lingual border of the mandible. This nerve can be inadvertently injured if not routinely protected during third molar extractions.

References

Martos-Fernández M, de-Pablo-Garcia-Cuenca A, Bescós-Atín MS. Lingual nerve injury after third molar removal: unilateral atrophy of fungiform papillae. J Clin Exp Dent. 2014;6(2):e193-e196.

Richards AT. Surgical exposures for the nerves of the neck. In: Tubbs RS, Rizk E, Shoja MM, Loukas M, Barbaro N, Spinner RJ, eds. Pain, Treatment, Injury, Disease, and Future Directions. London, United Kingdom: Elsevier; 2015:chap 14. Nerve and Nerve Injuries; vol 2.

340
Q

A 48-year-old woman is evaluated for bilateral microsurgical breast reconstruction. Compared with the deep inferior epigastric perforator (DIEP) flap, the superficial inferior epigastric artery (SIEA) flap places the patient at a greater risk for which of the following complications?

A) Abdominal bulge
B) Donor site dehiscence
C) Fat necrosis
D) Flap failure
E) Umbilical necrosis

A

The correct response is Option D.

Several comparative studies have reported a higher incidence of anastomotic thrombosis and failure with the superficial inferior epigastric artery (SIEA) flap than with flaps based on the deep inferior epigastric artery (DIEA). These failure rates range from 7.35 to 14%. Most of these failures were arterial in nature. Since SIEA flaps do not require an incision into the anterior rectus sheath or rectus muscle, bulges do not occur. Reported fat necrosis rates are similar between SIEA and DIEA flaps. There is no evidence for a difference in donor site dehiscence or umbilical necrosis rates.

341
Q

A healthy 45-year-old woman presents for consultation for mastopexy. Examination of the breasts shows grade II ptosis, large areolae, and dense breast tissue. No prior surgical scars are noted. An inverted T mastopexy is planned. On the day of surgery, the angle of the vertical limbs has to be marked wider than anticipated because of the large areolae. The large angle of divergence of the vertical limbs most likely increases the risk for which of the following?

A) Lower pole deformity
B) Nipple-areola malposition
C) Nipple-areola slough
D) Parenchymal fat necrosis
E) Pedicle overresection

A

The correct response is Option A.

For a patient undergoing a full-scar, inverted T skin resection as part of mastopexy, large areolae may require that the vertical limbs diverge more widely than would otherwise be necessary to tighten excess skin. This can create lower pole deformities such as flattening or boxiness.

The position of the nipple-areola complex is usually set at the Pitanguy point—the level determined by transposition of the inframammary crease onto the breast—and is not affected by large areolae in a patient who is a candidate for inverted T mastopexy. Necrosis of tissues such as the nipple-areola complex or breast parenchyma relate to surgical technique and preservation of blood supply to these areas. Pedicle overresection can lead to nipple-areola necrosis and is a result of poor surgical technique in developing the pedicle; pedicle design and resection are independent from the design of skin resection, as seen in this patient.

342
Q

A newborn male infant who is born at 36 weeks’ gestation via cesarean delivery has a large defect of the anterior abdominal wall. Examination shows matted bowel loops coming through the defect lateral to the umbilical cord. No other abnormalities are noted. Which of the following associated findings is/are most likely?

A) Abnormal karyotype
B) Constriction rings with limb and digital amputations
C) Elevated maternal serum alpha fetoprotein (MSAFP)
D) Hypoglycemia, macrosomia, and macroglossia
E) Translucent membrane covering bowel

A

The correct response is Option C.

Omphalocele (OC) and gastroschisis (GS) represent the two most common congenital abdominal wall defects, with a prevalence of approximately 3 to 4 per 10,000 live births/fetal deaths/stillbirths/pregnancy terminations each. Precise pathoetiologies are unclear, but developmental pathways and characteristics at the time of birth are notably distinct. OC is characteristically a midline partial-thickness abdominal wall defect covered by a membrane of amnion and peritoneum occurring within the umbilical ring and containing abdominal contents. GS is characteristically a full-thickness, paraumbilical abdominal wall defect associated with eviscerated bowel.

Both OC and GS are associated with elevated maternal serum alpha fetoprotein (MSAFP). For comparison, MSAFP values average twice that recorded in pregnancies with open spina bifida, and similar to values recorded with anencephaly. An elevated MSAFP is an indication for thorough ultrasound examination of the fetus for anatomical abnormalities.

Multiple chromosomal abnormalities have been associated with at least 60% OC cases, including trisomy -18, -13, -21, Turner syndrome, and triploidy. By contrast, GS is associated with abnormal karyotype in about 1% of cases, usually in the setting of other congenital abnormalities.

The definite treatment of both OC and GS is surgical once optimal resuscitation is achieved. Primary closure is associated with better survival rates if it can be achieved without compromise of intestinal blood flow or other hemodynamic or respiratory embarrassment. Large defects are frequently managed with temporary abdominal silos which are gradually reduced over the course of days to weeks in a form of visceral tissue expansion followed by delayed abdominal wall closure. The long-term outcome in isolated cases of OC and GS are generally good, although they can be associated with gut motility impairment, gastroesophageal reflux, ventral hernias, and late obstructive episodes.

Constriction rings with limb and digital amputations are found in amniotic band sequence but are not characteristic of OC or GS. GS is not characteristically associated with hypoglycemia, macrosomia, or macroglossia.

343
Q

Which of these situations is best suited for the use of a topical skin adhesive (polymerizing cyanoacrylate) for closure?

A) Burst laceration along the eyebrow
B) Over an intradermal repair of a vertical forehead laceration
C) Over a suture repaired dog bite of the ear
D) Straight line laceration on the cheek with 4 mm of separation
E) Well apposed lip laceration crossing the vermilion cutaneous border

A

The correct response is Option B.

Cyanoacrylate skin adhesives are sold as monomers that polymerize by an exothermic reaction on contact with air and fluids. They can be used in conjunction with other skin closure mechanisms such as sutures or as a primary skin closure device. There is evidence across multiple surgical specialties and situations that skin glues can save time in the operating room. When used correctly, the cosmesis is similar or better than external suturing. There are in vitro studies suggesting inhibition of Gram-positive cocci and clinical anecdotal evidence of decreased infection.

There is evidence that when wounds are closed with skin glue as the only closure device that dehiscence rates are increased.

If the adhesive leaks below the skin when applied, it can hold the edges open and delay or prevent healing with increased scarring or poorer cosmesis. Adhesives should be applied to well apposed skin edges only.

The U.S. Food & Drug Administration–approved package insert for a major brand of skin adhesive (eg, Johnson & Johnson, Ethicon Dermabond) specifically indicates use on mucosa and over dirty wounds such as dog bites to be contraindicated.

A burst laceration along the eyebrow and a straight line laceration on the cheek with 4 mm of separation would be contraindicated because of the lack of excellent epithelial continuity. Adhesive over a suture-repaired dog bite of the ear and a well-opposed lip laceration crossing the vermilion cutaneous border are specifically contraindicated on the package insert. Studies across multiple surgical specialties are supportive of cyanoacrylate skin adhesives over an intradermal repair of a vertical forehead laceration.

344
Q

A 19-year-old man comes to the office because of persistent left wrist pain after an ATV accident 4 months ago. Medical history includes cast treatment of a wrist fracture. X-ray study shows a proximal pole scaphoid fracture without arthritic changes or collapse. Which of the following is the most appropriate imaging for assessing the vascularity of the bone fragment in this patient?

A) Angiography
B) CT scan
C) MRI
D) Triple phase bone scan
E) Ultrasonography

A

The correct response is Option C.

The most appropriate imaging modality is MRI.

This patient has presented with a delayed proximal pole scaphoid fracture/nonunion, which was previously untreated. The primary blood supply to the scaphoid enters distally and travels proximally. As a result, perfusion to the proximal portion of the scaphoid occurs in a retrograde fashion. Fractures of the proximal pole of the scaphoid are located at the furthest distance from the blood supply, and these fragments are at risk for nonunion and avascular necrosis.

MRI (particularly with gadolinium enhancement) would be the best imaging study for evaluating the blood supply to the scaphoid fragment and looking for the presence of avascular necrosis. MRI can also provide anatomical information regarding the fracture. Direct intraoperative visualization of bleeding of the fragment has also been advocated in assessing vascularity.

Angiography can show blood flow patterns, but would not provide anatomic information.

CT scan is useful for detailed anatomic analysis of fractures and assessment of healing, but would be less helpful than MRI in determining avascular necrosis.

Bone scan shows the presence of inflammatory activity and can be used in identifying the presence of occult fractures (high sensitivity, albeit with low specificity), but has low resolution and would not be helpful in determining avascular necrosis.

Ultrasonography has been used in the diagnosis of acute fractures, but would not determine vascularity.

345
Q

A 60-year-old woman with breast cancer undergoes a transverse rectus abdominis musculocutaneous flap breast reconstruction after mastectomy. She has no allergies. Weight is 200 lb (91 kg). Estimated blood loss is 200 mL. Duration of the operation is 3 hours and 50 minutes. Administration of cefazolin before skin incision is planned as prophylaxis against surgical site infection. Which of the following is the most appropriate dosage and timing of this injection?Dose/Timing/Redosing

A) 1g/5 minutes prior/after 150 mL of blood loss

B) 1g/15 minutes prior/no

C)1g/40 minutes prior/no

D)2g/15 minutes prior/no

E)2g/40 minutes prior/no

A

The correct response is Option E.

There has been a renewed interest in perioperative antibiotics in recent years toward more appropriate use to decrease surgical site infection (SSI) while decreasing the incidence of resistant bacteria.

Current recommendations are to administer a single perioperative dose of antibiotics against common skin flora (gram positive), usually using a first-generation cephalosporin. However, the following recommendations may be underappreciated:

Cefazolin intravenous:1 g if <80kg; 2g if >80kg

Alternatives:clindamycin 600 to 900mg intravenously;
vancomycin 1 to 1.5g intravenously

In this case, the patient weighs 200lb (91kg), so 2g of cefazolin is the recommended dosage.

Additionally, an important factor is the timing of the administration of the perioperative antibiotics in order to achieve proper skin levels before incision. In one study by Classen et al., published in the New England Journal of Medicine in 1992, the optimal time was between 2 hours before the operation and skin incision, as greater than 2 hours before and any time after skin incision led to marked increases in the relative risk of SSIs. A follow-up by Weber et al. in 2008 narrowed the most appropriate window to between 30 and 59 minutes before skin incision.

Finally, there is the issue of redosing. Current recommendations are to redose if there is excessive blood loss (>1500 mL) or if there are long procedures where one exceeds the half-life of the antibiotic used.

In the clinical scenario described, the most appropriate choice is 2 g of cefazolin, because the patient’s weight is above 80 kg, administered between 30 to 59 minutes before skin incision. Redosing on the basis of blood loss is unnecessary, although one could consider redosing at approximately 4 hours on the basis of half-life.

346
Q

A 48-year-old man presents with pain 4 days after he underwent elective surgery of the right hand. The procedure included injection of 1% lidocaine with 1:100,000 epinephrine into the palm. Physical examination shows cold, pale digits, with prolonged capillary refill. Which of the following is the most appropriate management?

A) Inpatient admission and hourly wound checks for signs of necrosis
B) Local phentolamine infiltration
C) Topical nitroglycerin with warm water immersion
D) Topical terbutaline infiltration
E) No further management is necessary

A

The correct response is Option B.

Case reports have been documented of ischemia and subsequent tissue necrosis following elective hand surgery using lidocaine with epinephrine. The vasoconstrictive effect of epinephrine is a result of its stimulation of alpha-adrenergic receptors. Phentolamine, an alpha-adrenergic antagonist, has been used effectively to reverse the vasoconstrictive effect of epinephrine. When used in the hand, phentolamine rescue is carried out by injecting 1 to 2 mg of phentolamine in 1 to 5 mL of saline into the area where epinephrine was used. The reversal of vasoconstriction should result within 1 hour. Digital ischemia following accidental EpiPen injection into the hand has also been reported. The use of topical terbutaline infiltration has been attempted in such cases. In one case series, terbutaline infiltration was found to be effective in reversing vasoconstriction in some, but not all cases. The conclusion reached in the study was that terbutaline should be considered as an alternative when phentolamine is not available. The use of topical nitroglycerin and warm water immersion has not been proven to be an effective method to reverse the alpha-adrenergic effect of epinephrine. If prolonged ischemia is a concern following the use of lidocaine with epinephrine, further management, such as phentolamine rescue, should be considered given that complications such as distal digital amputation have occurred.

References

Zhang JX, Gray J, Lalonde DH, et al. Digital Necrosis After Lidocaine and Epinephrine Injection in the Flexor Tendon Sheath Without Phentolamine Rescue. J Hand Surg Am. 2017 Feb;42(2):e119-e123.

Velissariou I, Cottrell S, Berry K, et al. Management of adrenaline (epinephrine) induced digital ischaemia in children after accidental injection from an EpiPen. Emerg Med J. 2004 May;21(3):387-8.

347
Q

A 43-year-old woman presents with moderately large breasts with mild ptosis, and the surgical plan is a bilateral periareolar mastopexy to minimize the length of surgical scars. Which of the following postoperative complications is most commonly associated with this technique?

A) Areolar spreading
B) Constriction of the lower pole of the breast
C) Loss of nipple-areolar sensation
D) Pseudoherniation of the nipple-areolar complex
E) Synmastia

A

The correct response is Option A.

Mastopexy is a procedure designed to improve the appearance of the ptotic breast. The goal is to improve breast shape while minimizing visible scars. The periareolar mastopexy is best suited for correcting very minimal degrees of mammary ptosis. When the procedure is used to attempt to correct moderate to severe ptosis, complications can occur. These include flattening of the central breast mound; widening of the areolar diameter; and irregularity, widening, and even hypertrophy of the circumareolar surgical scar. Additionally, recurrent ptosis, or “bottoming out,” of the breast can occur.

Areolar spreading is the most common complication of this technique. Loss of nipple-areolar sensation is associated with breast tissue resection. Synmastia is associated with large implants, and pseudoherniation of the nipple areolar complex and constriction of the lower pole of the breast are associated with the tuberous breast deformity.

348
Q

A 25-year-old woman is considering augmentation mammaplasty with silicone prostheses. The patient asks about the associated risks of developing connective tissue disease. Which of the following risk assessments is most accurate in this patient?

A)Increased risk of extracapsular leak only
B)Increased risk of intra- and extracapsular leak
C)Increased risk only if the silicone migrates to the lymph node
D)Increased risk only in the pre-1990 prostheses
E)No increased risk

A

The correct response is Option E.

Concern regarding an association between silicone breast prostheses and connective tissue disease was raised in the 1980s and early 1990s, eventually leading to the US Food and Drug Administration (FDA) moratorium of the use of silicone breast prostheses in augmentation mammaplasty. Since then, multiple cohort studies and case control studies in Europe and North America have failed to determine a causative association between silicone breast prostheses and any traditional or atypical connective tissue diseases.

349
Q

A 75-year-old woman is evaluated after undergoing bilateral autologous breast reconstruction with deep inferior epigastric artery perforator (DIEP) flaps 2 days ago. The patient had been recovering well until this morning, when she developed acute respiratory insufficiency with hypoxia, tachycardia, and tachypnea. Pulmonary embolism is suspected, and pulmonary CT angiography has been ordered. Baseline creatinine level is 1.1 mg/dL. Which of the following steps should be taken to prevent contrast-induced nephropathy in this patient?

A) Hydration with hydroxyethyl starch (colloid)
B) Hydration with 0.9% saline
C) No preventive measures are needed
D) Pretreatment with the antioxidant N-acetylcysteine
E) Pretreatment with 0.9% saline and diphenhydramine

A

The correct response is Option B.

Although the risk of contrast-induced nephropathy (CIN) is overall relatively low, geriatric patients are at higher risk for this complication secondary to their high prevalence of risk factors for renal dysfunction. In this patient population, even normal laboratory creatinine concentrations are not always indicative of normal glomerular filtration rate (GFR), and every effort should be made to prevent the development of CIN. Important prophylactic strategies to avoid this adverse effect include optimization of fluids and avoidance of dehydration, use of non-ionic contrast media, and avoiding repeated exposure to contrast at close intervals.

This geriatric patient should be well hydrated prior to undergoing exposure to this high-contrast study. Minimizing fluid and performing no intervention will increase the patient’s risk of developing CIN even with a normal creatinine concentration. Antioxidants such as N-acetylcysteine, vasodilators such as dopamine, and colloid solutions such as Hespan have not proven to be beneficial in preventing renal dysfunction. Pretreatment with 0.9% saline and diphenhydramine is used in patients who have allergies to the contrast dye to minimize the allergic reaction and plays no role in preventing CIN.

350
Q

A 42-year-old nurse is scheduled to undergo elective non–implant-based surgery of the left breast. Medical history includes no abnormalities, and she has no allergies. She smokes 1 pack of cigarettes daily. To decrease the incidence of surgical site infection, which of the following is the most effective perioperative strategy?

A) Administer cefazolin intravenously within 5 minutes of skin incision
B) Administer an insulin drip to keep blood glucose concentration less than 100 mg/dL
C) Have the patient practice complete smoking cessation for 7 days prior to surgery
D) Prescribe nasal mupirocin and chlorhexidine baths for 5 days prior to surgery
E) Use povidone-iodine skin preparation rather than chlorhexidine/isopropyl alcohol

A

The correct response is Option D.

A 2010 randomized controlled trial of over 6700 patients published in the New England Journal of Medicine documented a nearly 60% decrease in Staphylococcus aureus infections in patients if mupirocin was applied twice a day to the nares as well as a full-body wash with chlorhexidine for 5 days prior to surgery. The mean hospital stay was already reduced almost 2 days. A meta-analysis also demonstrated the same findings, with a nearly 45% decrease in surgical site infections (SSIs).

Data would suggest that isopropyl alcohol–containing skin preparations for surgery decrease SSI rates more effectively than povidone-iodine alone.

Perioperative antibiotics should be administered with enough advance time to achieve proper and adequate rates of skin penetration. With cefazolin, this is 30 to 59 minutes before skin incision, with an odds ratio of 1.0 (vs. 2.0 if given within 30 minutes, and 1.7 if given after 60 minutes). Five minutes prior to skin incision is not sufficient for the SSI-reducing effect to be achieved.

Blood glucose control is critical to decreasing SSI rates, with optimal rates usually being quoted as less than 180 mg/dL. However, this patient is not diabetic; she is otherwise healthy, so an insulin drip would not be appropriate. In addition, hypoglycemia can also have detrimental physiologic effects and also should be avoided.

Smoking cessation decreases SSI rates if the patient does not smoke for 4 weeks before or after surgery. A 2012 systematic review and meta-analysis of nearly 500,000 patients demonstrated this. However, 7 days of smoking cessation is insufficient time in advance of surgery to obtain these statistically significant benefits.

351
Q

A 12-year-old girl with Beckwith-Wiedemann syndrome develops profound breast enlargement at the onset of puberty. Physical examination shows two distinct masses in the right breast and severe hyperplasia consuming the left breast. A photograph is shown. Which of the following is the most appropriate surgical management?

A) Hormone suppression therapy
B) Right lumpectomies; left mastectomy with skin reduction with application of nipple as skin graft
C) Right simple mastectomy; left mastectomy with skin reduction with application of nipple as skin graft
D) Right simple mastectomy; left simple mastectomy with sentinel lymph node biopsy
E) Right total mastectomy; left modified radical mastectomy

A

The correct response is Option B.

Epithelial hyperplasia is a benign pathological process. Therefore, modified radical mastectomy or sentinel lymph node biopsy would not be indicated in the absence of cancer. Mastectomy on the right is not indicated because the masses are discrete and separate from the normal breast parenchyma. As such, a viable breast mound can be preserved in this 12-year-old girl. Because of the expansive nature of her hyperplasia on the left, skin reduction is required to ensure smooth contour of her skin flaps. The nipple may be spared because of the benign disease. Reconstruction can be performed in a delayed fashion after development is complete. An adjustable expander/prosthesis can also provide a reasonable breast mound until definitive reconstruction is performed.

Hormone suppression therapy may temporarily slow the growth of the hyperplasia but could adversely affect this child’s normal development. It would not comprise definitive management.

352
Q

Which of the following is the primary role of adipose-derived stem cells (ADSC) in wound healing?

A) Assist in chemotaxis of platelets and granulocytes
B) Differentiate directly into fibroblasts and keratinocytes
C) Induce development of hair and sweat follicles
D) Provide a scaffold for deposition of granulation tissues
E) Register and organize pro-collagen fibrils

A

The correct response is Option B.

Adipose-derived stems cells (ADSC) have had extensive study in vitro and in vivo because there are ready sources of them from adult patients, which bypasses many ethical and regulatory issues of embryonic stem cells.

ADSC have both direct structural and paracrine roles in wound healing. They can directly differentiate into keratinocytes, endothelial cells, and dermal fibroblasts. ADSCs, through paracrine phenomena, are modulators of the inflammatory environment of the wound healing milieu but are not involved in the immediate chemotaxis during the inflammatory period nor do they function as a scaffold during the proliferative phase. Lysyl oxidase is the extracellular enzyme responsible for final alignment of collagen fibrils.

Presence of skin adnexa such as hair follicles and sweat glands are hallmarks of scarless, fetal healing. Hair follicle formation typically only occurs during embryonic development and involves interaction of ectodermal and mesenchymal cells influenced by signaling pathways including Wnt/b-catenin and BMPl but not ADSCs.

353
Q

A 45-year-old man comes to the office for consultation regarding breast cancer after undergoing gene testing and learning that he is a carrier of the BRCA2 gene mutation. He has a strong family history of breast, prostate, and ovarian cancers. Which of the following best represents his lifetime risk for developing breast cancer?

A ) 6%

B ) 15%

C ) 30%

D ) 60%

E ) 85%

A

The correct response is Option A.

Since the identification of the first breast cancer gene in 1990, the field of molecular breast cancer testing has grown enormously. It is estimated that approximately 5% to 10% of patients diagnosed with breast cancer last year carry highly penetrant cancer susceptibility genes. It is important to identify these patients early, as the presence of genetic mutations or other high-risk factors may alter management strategies for patients, both prior and subsequent to the diagnosis of breast cancer.

Male breast cancer accounts for less than 1% of all breast cancer cases in the United States and is more common in men with a family history of breast cancer. The relationship between male breast cancer and a deleterious BRCA2 mutation has been well established. In the largest study to date, the lifetime risk of developing breast cancer in a BRCA2 male carrier is approximately 7%. The correlation with BRCA1 and male breast cancer is less clear, but it seems that the presence of BRCA1 mutation confers a lower lifetime risk of developing breast cancer than a BRCA2 mutation. The relative risk of developing breast cancer is highest for men in their 30s and 40s, and it decreases with increasing age.

The lifetime risk for the development of breast cancer in female carriers of BRCA1 and BRCA2 mutations is significantly higher than for males. Female BRCA1 carriers have an 85% risk of developing breast cancer and a 62% risk of developing ovarian cancer. Female BRCA2 carriers have an 85% risk of developing breast cancer and a 25% risk of developing ovarian cancer.

354
Q

A 15-year-old boy sustained a traumatic amputation of the left index finger at the proximal interphalangeal joint level from a sharp injury. Replantation of the digit is performed, with vein grafting of the radial digital artery and vein. The distal tip of the digit appears congested, so medicinal leeching is instituted. Which of the following antibiotics is the most appropriate prophylaxis for this patient?

A) Amoxicillin and clavulanic acid
B) Ampicillin
C) Cephalexin
D) Ciprofloxacin
E) Vancomycin

A

The correct response is Option D.

The antibiotic choice that constitutes the best prophylaxis for this patient undergoing leech therapy is ciprofloxacin. Hirudo medicinalis is the most common leech species used in medicine, and its gut flora includes Morganella, Rikenella, and Aeromonas isolates. These bacteria are all sensitive to ciprofloxacin. Doxycycline or ceftriaxone are alternative treatments for Aeromonas prophylaxis.

Animal toxicology data available with the first quinolone compounds revealed an association with inflammation and subsequent destruction of weight-bearing joints in canine puppies. This observation limited further development or large-scale evaluation of this class of antibiotic agents in children at that time. However, there continued to be increased use of fluoroquinolones for pediatric patients over the past 30 years with data on the lack of toxicity when used in children. In 2004, ciprofloxacin became the first fluoroquinolone agent approved for use in children 1 through 17 years of age.

Cephalexin (Keflex) is a first-generation cephalosporin that is used to treat respiratory tract, middle ear, skin, bone, and urinary tract infections. Most Aeromonas strains are resistant to penicillin, ampicillin, carbenicillin, and ticarcillin. And most Aeromonas and Morganella strains have complete or intermediate resistance to amoxicillin and clavulanic acid (Augmentin). Vancomycin is a macrolide antibiotic, and has limited effectiveness for Aeromonas strains with high levels of antibiotic resistance.

References

Jackson MA and Schutze GE. The Use of Systemic and Topical Fluoroquinolones. Pediatrics. 2016 Nov; 138(5):e1-e13.

Stevens DL, Bisno AL, Chambers HF, et al. Practice guidelines for the diagnosis and management of skin and soft tissue infections: 2014 update by the Infectious Diseases Society of America. Clin Infect Dis. 2014 Jul 15;59(2):e10-52.

Whitaker IS, Elmiyeh B, Wright DJ. Hirudo medicinalis: the need for prophylactic antibiotics. Plast Reconstr Surg. 2003 Sep 15;112(4):1185-1186.

Whitaker IS, Maltz M, Siddall ME, et al. Characterization of the digestive tract microbiota of Hirudo orientalis (medicinal leech) and antibiotic resistance profile. Plast Reconstr Surg. 2014 Mar;133(3):408e-418e.

355
Q

The mammary glands develop from which of the following embryologic structures?

A) Bilateral mesenchymal condensations
B) Ingrowths from the ectoderm
C) Ingrowths from the mesoderm
D) Proliferating masses of endoderm
E) Proliferating masses of mesenchyme

A

The correct response is Option B.

The breasts, or mammary glands, are modified sweat glands. They are ingrowths from the ectoderm that form the lactiferous ducts and alveoli. They begin as linear mammary ridges with 15 to 20 buds. During the seventh week in utero, these buds undergo apoptosis, leaving a single pair of solid buds—the primary mammary buds—at the fourth or fifth intercostal space.

Proliferating masses of mesenchyme are at the center of each limb bud. The mesoderm gives rise to organs, musculature, vasculature, and connective tissues. The endoderm becomes the epithelial lining of the alimentary tract. Bilateral mesenchymatous condensations develop into the sternum.

356
Q

A 29-year-old man comes to the office because of pain and the inability to flex the little finger of his dominant right hand 6 weeks after injuring it during a recreational football game. He reports that he “jammed” the finger and then pulled it back into place. He did not seek medical care at the time of the injury. He is concerned about maintaining an active lifestyle and preserving the maximum range of motion. Physical examination shows the inability to flex at the proximal interphalangeal joint. Lateral x-ray study of the finger is shown. Which of the following is the most appropriate management?

A) Arthrodesis
B) Dynamic external fixation
C) Hemi-hamate arthroplasty
D) Open reduction and internal fixation
E) Silicone implant reconstruction

A

The correct response is Option C.

Proximal interphalangeal (PIP) joint fracture dislocations are common finger injuries that result in pain and loss of motion. Normal PIP joint range is 0 to 110 degrees.

This type of injury normally results from an axial load applied to the finger in a hyperextended position. If more than 30 to 50% of the volar base of the middle phalanx breaks off, the joint will become unstable.

If identified early, the fracture can be managed with either open reduction and internal fixation or dynamic external fixation. At 6 weeks post injury, the fracture fragments would not be mobile, making dynamic external fixation unsuccessful. In addition, it is normally not possible to mobilize the fracture fragments in a manner that they can be reduced and fixated this far out from injury.

Silicone implant arthroplasty can provide pain relief and preserve limited PIP motion in low-demand patients such as those with rheumatoid or osteoarthritis. It would not be sufficiently durable to tolerate the lifestyle of this patient and does not provide as much range of motion as a hemi-hamate graft.

Arthrodesis will provide durable stability and pain relief even in a young patient. However, it sacrifices all motion at the PIP joint. As such, it should be reserved as a salvage option if motion-preserving options fail.

Since its description by Hastings in 1999, hemi-hamate arthroplasty has become a reconstruction of choice for unstable late dorsal fracture-dislocations, particularly those involving more than 50% of the articular surface. The dorsal central portion of the hamate is harvested as an osteocartilaginous graft. Due to the thicker articular cartilage of the distal hamate compared with the base of the middle phalanx, the bone surfaces may appear uneven even though the cartilage surface (and, therefore, the joint surface) is confluent.

357
Q

A 23-year-old woodworker sustains an injury to the dominant left thumb that involves the loss of less than 2 cm of the distal pulp with exposed bone from a planing machine. Which of the following reconstruction methods is most likely to provide normal sensation to the volar pulp of this patient’s thumb?

A) Cross-finger flap from the long finger
B) Islandized Moberg flap
C) Flag flap
D) Thenar flap
E) Venous flow-through flap

A

The correct response is Option B.

The venous flow-through flap was described for small defects of the dorsum of a digit or hand where end-to-end anastomoses of the included veins on the proximal and distal edges of the flap can provide venous outflow for the digit and supply the flap. A defect from the distal, volar surface of the thumb would not have any veins large enough to use. A cross-finger flap is a classical solution to cover the volar aspect of a digit. The other mentioned flaps are also excellent options for volar thumb coverage, except for the thenar flap. The thenar flap is used for distal pulp defects of the fingers in children. The only flap that maintains the normal sensation of the thumb pulp is the Moberg flap, which advances the volar aspect of the thumb on its neurovascular pedicles. The islandized (O’Brien) modification was to make a transverse incision at the base of the thumb and dissect the neurovascular bundles to allow the flap to travel further distally, and then placing a skin graft over the proximal defect.

References

Graham DJ, Venkatramani H, Sabapathy SR. Current Reconstructive options for traumatic thumb loss. J Hand Surg Am. 2016 Dec;41(12):1159–1169.

Germann G, Sauerbier M, Rudolf KD, Hrabowski M. Management of thumb tip injuries. J Hand Surg Am. 2015 Mar;40(3):614–22; quiz 623.

358
Q

Which of the following structures contributes to the formation of the mandibular body and ramus?

A) First branchial arch
B) First branchial cleft
C) Second branchial arch
D) Second branchial cleft

A

The correct response is Option A.

The first branchial arch contributes to the formation of the mandible, the tragus, and the anterior helix.

The first branchial cleft gives rise to the external auditory canal. The second branchial arch contributes to the formation of the majority of the external ear–the antitragus, remainder of the helix, antihelix, and crura all arise from the second branchial arch. The second branchial cleft is typically obliterated during development, but may persist in the form of a second branchial cleft cyst.

References

Adams A, Mankad K, Offiah C, Childs L. Branchial cleft anomalies: a pictorial review of embryological development and spectrum of imaging findings. Insights Imaging. 2016 Feb;7(1):69-76.

Parada C, Chang Y. Mandible and tongue development. Curr Top Dev Biol. 2015;115:31-58.

359
Q

A 23-year-old man comes for evaluation because of pain and swelling of the left wrist 6 hours after he fell onto his outstretched left hand. On physical examination, he has tenderness to palpation in the anatomical snuffbox. An occult fracture is suspected. In addition to standard x-ray study views of the wrist, which of the following x-ray views is the most appropriate to confirm the diagnosis?

A) Anteroposterior view with neutral alignment and the beam angled at neutral
B) Clenched fist view with the wrist at neutral and the beam angled at 30 degrees distal to proximal
C) Lateral view with the wrist flexed 30 degrees and the beam angled 45 degrees distal to proximal
D) Oblique view with radial deviation and the beam angled at neutral
E) Posteroanterior view with ulnar deviation and the beam angled 20 degrees distal to proximal

A

The correct response is Option E.

The scaphoid oblique view is a posteroanterior (PA) view with the wrist in ulnar deviation and the beam angled 20 degrees distal to proximal. This view often will show scaphoid fractures not seen on standard PA, oblique, or lateral views. The other views would not extend the scaphoid and the scaphoid would not be seen as clearly.

360
Q

A newborn is noted to have a lesion of the midline of the lower back consisting of a protruding membrane which covers meninges, cerebrospinal fluid (CSF), and neural structures. Which of the following is the primary goal of surgical repair?

A) Hydrocephalus mitigation
B) Increase in lower extremity strength
C) Infection prevention
D) Prevention of tethered cord syndrome
E) Restoration of bowel or bladder function

A

The correct response is Option C.

Meningomyelocele is the most common neural tube defect. It involves dorsal herniation of the meninges and spinal cord through the vertebrae and may produce motor and sensory nerve deficits. It is often diagnosed prenatally by elevated maternal serum alpha fetoprotein and ultrasonography. Treatment of larger defects often involves both neurosurgery and plastic surgery teams. After repair of the neural placode, the goals of soft tissue reconstruction are to cover and protect the neural element, prevent infection, and avoid any cerebrospinal fluid leak. Ideally this is performed within the first 24 to 48 hours of life. Larger defects are often best reconstructed with muscle flaps, fasciocutaneous flaps, or a combination of both. Many different flaps have been described, but considerations for adequate vascularity (such as inclusion of perforator blood vessels within geometrically designed flaps) and closure without tension are paramount.

While hydrocephalus is a common finding in patients with meningomyelocele, it is treated with cerebrospinal fluid shunting if required.

Meningomyelocele repair does not regain or improve neural abilities that are not present at birth, such as bowel and bladder function, and lower extremity motor and sensory function.

Symptoms related to tethering of the spinal cord may develop as the patient grows in as many as 20 to 50% of children who undergo meningomyelocele repair shortly after birth and many may require surgery to release the scar tissue attached to the cord. However, this condition is not prevented by meningomyelocele repair.

361
Q

A 24-year-old woman is undergoing endoscopic transaxillary augmentation mammaplasty. Which of the following is most appropriate to preserve sensation in the medial aspect of the upper extremity?

A ) Avoiding dissection into the axillary fat

B ) Blunt dissection near the clavicle

C ) Identification of the sensory nerves within the axilla

D ) Positioning of the prosthesis subpectorally

E ) Preservation of the lateral pectoral nerve

A

The correct response is Option A.

During transaxillary augmentation mammaplasty, prevention of sensory changes to the medial aspect of the upper extremity requires a subdermal dissection and avoids dissection into the axillary fat. Branches of the intercostobrachial and medial brachial cutaneous nerves provide sensory innervation to the medial upper extremity. Both nerves course superficially through the axillary fat posterior to the lateral border of the pectoralis major muscle. Dissection within the axillary fat risks injury to these nerves with subsequent anesthesia or paresthesia of the inner arm.

Identification of the nerves within the axilla requires dissection into axillary fat and risks injury to the sensory nerves. Sensory innervation to the medial aspect of the upper extremity is not affected by the positioning of the prosthesis (subpectoral versus subglandular) or dissection near the clavicle. The lateral pectoral nerve provides motor innervation to the lower third of the pectoralis major muscle.

362
Q

A 20-year-old man is evaluated after falling on his outstretched hand. Physical examination shows tenderness of the snuffbox. X-ray studies of the wrist show no fracture. Which of the following is the most appropriate initial step in management?

A) Application of a sugar-tong splint
B) Application of a thumb spica splint
C) Application of a volar wrist splint
D) Application of an ulnar gutter splint
E) No treatment is necessary

A

The correct response is Option B.

The patient described may have a scaphoid fracture that is not apparent on initial x-ray studies. Prudent management involves placement of a thumb spica splint until definitive diagnosis can be made. Repeating x-ray studies in 2 weeks or obtaining further radiologic studies, such as CT scans, can make the definitive diagnosis. Casting would be suboptimal in an acute injury such as this because soft-tissue swelling can cause constriction. Surgical exploration is not warranted at this time. A wrist-control, sugar-tong, or ulnar gutter splint would not adequately immobilize the scaphoid, and therefore, would not be adequate management.

Acute scaphoid fractures can often be missed on initial x-ray studies, with reported sensitivities ranging from 84 to 98%. When clinical suspicion of a scaphoid fracture is high and plain films are negative, the traditional recommendation is for these patients to be immobilized in a thumb spica splint or cast with repeat x-ray studies after about 2 weeks.

Even on the repeated x-ray study after 10 to 14 days propagated by many clinicians in cases of occult fracture, a scaphoid fracture is often missed since the additional sensitivity is low, although in a case of sclerosis, an x-ray study could confirm the suspected diagnosis. Further studies that may confirm the diagnosis include CT scan, MRI, and bone scan.

363
Q

A 45-year-old woman comes to the office for consultation about immediate bilateral breast reconstruction of a right-sided tumor measuring 2.5 cm. Biopsy reveals a HER-2/neu negative invasive ductal carcinoma without lymphovascular invasion. The patient requests nipple-sparing mastectomy. Physical examination shows a palpable mass is located in the right upper outer quadrant approximately 1.5 cm from the nipple-areola complex and is freely mobile. There is no lymphadenopathy on exam. Based on current literature, which of the following best describes this patient’s candidacy for the requested procedure?

A) Good candidate based on current presentation
B) Not a candidate because of lymph node status
C) Not a candidate because of tumor location
D) Not a candidate because of tumor pathology
E) Not a candidate because of tumor size

A

The correct response is Option C.

Nipple-sparing mastectomy or total skin-sparing mastectomy is becoming an increasingly popular choice for women because of the excellent cosmetic outcomes and the ability to save the nipple-areola complex that may provide psychological benefits with increased patient satisfaction as well. Nipple-sparing mastectomy appears to be oncologically safe with low risks of cancer recurrence in the literature thus far. However, there has been little long-term follow-up, so this approach is still somewhat controversial because the oncologic safety and locoregional recurrence have not been examined definitively. Although certain centers are pushing the envelope regarding the use of this technique in a wide range of patients, the current literature supports the following exclusion criteria:

A. Tumor size greater than 5 cm B. Tumor location less than 2 cm from the nipple C. Evidence of axillary disease D. Tumor involvement on retroareolar biopsy E. Lymphovascular invasion, human epidermal growth factor receptor-2 positivity, and/or HER-2/neu positivity on biopsy

The current patient’s tumor was found to have a tumor-to-nipple distance of 1.5 cm which is a relative contraindication to nipple-sparing mastectomy in this case.

364
Q

A 50-year-old woman with type 2 diabetes mellitus is scheduled to undergo ligament reconstruction tendon interposition (LRTI) surgery for trapeziometacarpal joint arthritis. The procedure is expected to last 90 minutes. Which of the following is the most appropriate antibiotic prophylaxis for this patient?

A) Oral antibiotics for 3 days following surgery
B) Single dose intravenous antibiotic within 1 hour of surgery
C) Single dose intravenous antibiotic within 1 hour of surgery and oral antibiotics for 24 hours following surgery
D) Single dose intravenous antibiotic within 1 hour of surgery and oral antibiotics for 3 days following surgery
E) No antibiotic prophylaxis is indicated

A

The correct response is Option E.

Multidrug resistant bacterial infections continue to rise and antimicrobial overuse is the leading cause for antibiotic resistance. There is growing evidence that prophylactic antibiotic use is not necessary for clean plastic surgery cases, aside from breast surgery cases. Despite consensus guidelines, the use of prophylactic antibiotics for elective Hand Surgery cases continues to increase. Level I evidence exists that demonstrates prophylactic antibiotics are not necessary for clean Hand Surgery cases lasting less than 2 hours. Although there has been concern regarding diabetes and surgical infection risk, this has not been demonstrated in larger studies with multivariate analyses.

365
Q

A 53-year-old woman with a BMI of 27 kg/m2, gigantomastia, and grade III ptosis is considering unilateral mastectomy and autologous reconstruction with abdominal tissue. Which of the following is the most appropriate advice regarding the contralateral breast?

A) A contralateral prophylactic mastectomy should be performed to maximize symmetry
B) Contralateral reduction should be accompanied by placement of an implant on that side to address upper pole symmetry with the autologous reconstruction
C) Simultaneous contralateral reduction may be performed with an acceptable risk profile
D) Symmetry is unlikely to be achieved, so a contralateral matching procedure would not be recommended
E) Symmetry with autologous tissue is likely to be achieved without the contralateral matching procedure

A

The correct response is Option C.

Studies have shown that contralateral symmetry procedures performed synchronously with unilateral autologous tissue reconstruction after mastectomy (including reconstructions with a free perforator flap) are acceptable. Situations requiring reduction rather than augmentation or mastopexy seem to be the most suited to this timing. A patient with a lower BMI and gigantomastia is unlikely to achieve symmetry without a contralateral procedure. Autologous tissue reconstruction likely would not need a contralateral implant for upper pole symmetry. An attempt at improvement in symmetry using contralateral surgery would be acceptable should the patient so choose, but recommending a prophylactic mastectomy on the contralateral side solely for symmetry and not for risk reduction may be overly aggressive when other methods such as reduction mammaplasty exist with a likelihood of acceptable postoperative symmetry.

366
Q

A 38-year-old man comes to the office because of central wrist pain 7 months after falling on his outstretched right hand. He did not seek treatment at the time of his original injury. An anteroposterior x-ray study is shown. In a lateral x-ray study view, the scapholunate angle is most likely to be which of the following?

A) Less than 20 degrees
B) 20 to 40 degrees
C) 41 to 60 degrees
D) Greater than 60 degrees

A

The correct response is Option D.

The patient described has scapholunate separation that results not only in widening of the gap between the scaphoid and the lunate, as depicted in the x-ray study shown (Terry-Thomas sign), but also in flexion of the scaphoid. This is seen on the anteroposterior x-ray study as a ?ring sign? as a result of the distal pole of the scaphoid moving relatively closer to the proximal scaphoid cortex and being viewed end-on.

Carpal bone malalignment is also determined by angles on lateral x-ray studies. As the scaphoid flexes progressively, the lunate (still tethered to the triquetrum) goes into an extension dorsal intercalated segment instability deformity.

In a patient with rotary subluxation at the scaphoid, the scapholunate angle is expected to be increased on lateral x-ray study. Normal values range from 30 to 60 degrees. Angles greater than 80 degrees are considered a definite indication of scapholunate dissociation.

367
Q

In patients with basilar joint arthritis of the thumb, treatment with trapeziectomy alone compared with trapeziectomy with ligament reconstruction and tendon interposition is most likely to result in which of the following?

A) Decreased complication rate
B) Increased risk for recurrent pain
C) Longer recovery time
D) Worse functional outcomes

A

The correct response is Option A.

Simple trapeziectomy was first described in 1947 and though it is only utilized by a small minority of hand surgeons in the United States as solitary treatment for basilar thumb arthritis, its efficacy has not been demonstrated to be inferior to the more commonly employed trapeziectomy with ligament reconstruction and tendon interposition. Meta-analysis has demonstrated that simple trapeziectomy is equally beneficial in terms of pain relief and function as trapeziectomy with ligament reconstruction and tendon interposition, but results in fewer complications.

A variety of reconstructive techniques have been employed in management of basilar thumb arthritis. Most commonly employed in the United States is the trapeziectomy with ligament reconstruction and tendon interposition utilizing the flexor carpi radialis tendon. Other described techniques include CMC joint implant arthroplasty, partial trapeziectomy with interposition arthroplasty, and thumb metacarpal extension osteotomy for early-stage arthritis. None of these techniques have demonstrated superiority over simple trapeziectomy, and synthetic implants have been associated with significant complications.

368
Q

A 54-year-old woman is evaluated for an injury to the proximal interphalangeal (PIP) joint of the long finger of the left hand that she sustained during a motor vehicle collision. X-ray study is shown. Which of the following methods is the most appropriate surgical management of this patient’s fracture?

A) Dynamic external fixation
B) Hemihamate arthroplasty
C) Lag screw fixation
D) PIP joint arthrodesis
E) Volar plate arthroplasty

A

This patient presents with an intra-articular, or pilon, fracture of the middle phalanx base. Typically, pilon fractures are axial load injuries to the finger resulting in impaction, comminution, central depression, and splaying of the middle phalanx fracture fragments both volarly and dorsally.

Fractures involving the proximal interphalangeal (PIP) joint are generally divided into three categories: volar rim fractures, dorsal rim fractures, and pilon fractures (volar and dorsal rim) with or without joint dislocation. Joint dislocation occurs secondary to ligamentous injury and correlates to the amount of articular surface involved in the fracture. Management of these injuries depends on the fracture type, severity, degree of articular surface involvement, extent of dislocation, and associated soft-tissue injuries. Pilon fractures due to the comminution and crush component of the middle phalanx base can be very difficult to treat. Post-traumatic osteoarthritis causing stiffness, impaired function, and pain is a known complication.

Multiple different types of dynamic external fixation devices have been described in the literature. These are mostly fabricated from K-wires with or without a rubber band traction system. The devices work by the principles of a lever and ligamentotaxis to maintain fracture reduction and joint congruency and to allow early motion for joint surface remodeling. The devices are inexpensive and relatively easy to place with experience, and can be placed using local anesthesia. Dynamic distraction external fixation has been indicated for use in pilon fractures and unstable PIP joint fracture dislocations. The constructs are usually removed 4 to 6 weeks after fracture consolidation and clinical healing is present. Most studies report 60 to 88 degrees of PIP motion with good patient satisfaction and return of function. The presence of arthritic changes on x-ray studies is common after these injuries. Pin tract infections up to 40% have been reported in retrospective studies; however, these seem to be treated with oral antibiotics alone and do not require pin removal. They do not progress to osteomyelitis.

PIP joint arthrodesis is an option for treatment in very severe pilon fractures or crush injuries with severe soft-tissue involvement; however, this sacrifices motion and should be reserved for cases where there are no other viable treatment options.

Hemi-hamate arthroplasty and volar plate arthroplasty are operations described to treat PIP joint fracture dislocations with isolated volar rim involvement. These procedures require an intact dorsal cortex of the middle phalanx base to achieve stable fixation. Both procedures are indicated for comminuted fractures involving 30 to 50% of the articular surface of the middle phalanx base.

Open reduction and internal fixation of pilon fracture with locking plates has been described, but is technically challenging and requires significant soft-tissue dissection compared with percutaneous dynamic external fixation. Lag screws are inadequate fixation in general for pilon fractures. Isolated volar rim fractures with a larger fracture fragment can be treated successfully with lag screws.

369
Q

A 30-year-old man sustains significant mid face injuries following a motor vehicle collision, and has a large laceration in the vicinity of the medial canthal region. Canalicular injury is confirmed intra-operatively. Which of the following is the most appropriate method for repairing this patient’s canalicular injury?

A) Delayed dacryocystorhinostomy
B) Direct microsurgical suture repair
C) Healing by secondary intention
D) Immediate dacryocystorhinostomy
E) Placement of silicone canalicular stents

A

The correct response is Option E.

When canalicular injury is suspected, the lacrimal system should be investigated for patency. Typically, this involves performing a Jones I and II test to determine if fluorescein navigates from the lower lid fornix into the nose. If canalicular interruption is suspected and identified, the proximal and distal stumps of the canaliculus are joined by placing a silicone stent and leaving this in place for 3 to 6 months to allow for healing.

Direct microsurgical suturing is not preferred because of the high likelihood of cicatricial obstruction.

Dacryocystorhinostomy is generally reserved as a “salvage” procedure for patients who have lacrimal obstruction after being treated with a stent. Healing by secondary intention is incorrect since it would likely result in canalicular obstruction.

370
Q

A 26-year-old woman with a strong family history of breast cancer undergoes genetic testing. She is found to have a deleterious mutation of the BRCA1 gene. Which of the following best describes her lifetime risk for the development of breast cancer when compared with women without this mutation?

A) Two times greater
B) Six times greater
C) Ten times greater
D) Fifteen times greater
E) Twenty times greater

A

The correct response is Option B.

A woman’s lifetime risk of developing breast and/or ovarian cancer is greatly increased if she inherits an altered BRCA1 or BRCA2 gene. Women with an inherited alteration in one of these genes have an increased risk of developing these cancers at a young age (before menopause) and often have multiple close family members with the disease. These women may also have an increased chance of developing colon cancer.

Men with an altered BRCA1 or BRCA2 gene also have an increased risk of breast cancer (primarily if the alteration is in BRCA2) and possibly prostate cancer. Alterations in the BRCA2 gene have also been associated with an increased risk of lymphoma, melanoma, and cancers of the pancreas, gallbladder, bile duct, and stomach in some men and women.

According to estimates of lifetime risk, approximately 13.2% (132 of 1000 individuals) of women in the general population will develop breast cancer, compared with estimates of 36 to 85% (360 to 850 of 1000) of women with an altered BRCA1 or BRCA2 gene. In other words, women with an altered BRCA1 or BRCA2 gene are 3 to 7 times more likely to develop breast cancer than women without alterations in those genes. Lifetime risk estimates of ovarian cancer for women in the general population indicate that 1.7% (17 of 1000) will get ovarian cancer, compared with 16 to 60% (160 to 600 of 1000) of women with altered BRCA1 or BRCA2 genes. No data are available from long-term studies of the general population comparing the cancer risk in women who have a BRCA1 or BRCA2 alteration with women who do not have an alteration in these genes. Therefore, these figures are estimated ranges that may change as more research data are added.

371
Q

A 45-year-old woman who is obese is considering unilateral mastectomy and reconstruction of the left breast because of invasive ductal carcinoma. Which of the following patient characteristics is associated with the lowest risk for complications from a nipple-sparing mastectomy?

A) BMI of 41 kg/m2
B) Grade III ptosis of the breast
C) Nipple retraction
D) Tumor distance from nipple of 5 cm
E) Tumor size of 6 cm

A

The correct response is Option D.

Nipple-sparing mastectomy is increasing in popularity. To decrease the risk for surgical complications as well as oncologic complications, smaller tumors located further from the nipple in patients without morbid obesity or severe ptosis are considered better candidates for treatment with nipple-sparing mastectomy. Clinical involvement of the nipple, including retraction, would suggest that nipple-sparing mastectomy should not be performed.

372
Q

A 48-year-old woman undergoes radical resection of the left breast, left hemisternectomy, four-rib resection, and visceral pleurectomy because of invasive inflammatory breast cancer extending through the thoracic wall into the mediastinum and chest. Examination of specimens obtained from intraoperative frozen section biopsies shows no residual disease. Postoperative adjuvant chemotherapy and radiation therapy to the area are planned. An intraoperative photograph of the 17 × 17-cm defect is shown. Which of the following is the most appropriate technique for soft-tissue reconstruction?

A) Left latissimus muscle flap with skin graft
B) Left rectus muscle turnover flap and skin graft
C) Omental flap and skin graft
D) Reverse abdominoplasty flap
E) Right transverse rectus abdominis musculocutaneous (TRAM) flap

A

The correct response is Option E.

The most appropriate option for this patient would be a right TRAM flap. The defect encompasses a composite defect of the left side of the sternum, including the left internal mammary artery, four contiguous ribs, and the entire left breast. The question specifically asks for soft-tissue thoracic reconstruction. The analysis of the defect yields a very large soft-tissue requirement. Given this, the most logical choice of flaps to provide this amount of soft tissue on a reliable pedicle would be a right (contralateral) TRAM flap. Furthermore, this flap could also be shaped to provide a breast mound versus all the other choices.

A left latissimus muscle flap with skin graft is not the optimal choice as it would not provide sufficient soft-tissue coverage with its associated skin paddle without undue donor site morbidity in this particular patient with a large defect that spans parasternal and lateral chest wall. As a muscle-only flap with skin graft, however, it may be considered as a backup option in case of primary flap failure.

An omental flap with skin graft is an option for reconstruction of this defect; however, it would not be the most appropriate option in this case as it would necessitate an otherwise unnecessary intra-abdominal procedure and would not reshape a breast mound. It has been shown, however, to be optimal in contaminated areas due to its associated lymphatic vessels and nodes associated with its pedicle, so in different circumstances, this may be a preferred choice.

A left rectus turnover flap with skin graft would not be a good option for two reasons: the left internal mammary artery has been harvested, thereby compromising the superior epigastric vessel on which this flap would be based; and it would not cover the entirety of the defect.

A reverse abdominoplasty flap would not be able to reach the full superior extent of the defect.

373
Q

A 43-year-old woman who is BRCA-positive is scheduled to undergo bilateral mastectomy. Tissue expander–based reconstruction is planned. Which of the following is the optimal duration of antibiotic prophylaxis for this patient?

A) No preoperative antibiotic
B) One preoperative antibiotic dose and another dose during skin closure
C) One preoperative antibiotic dose, followed by 24 hours of treatment while in the hospital
D) One preoperative antibiotic dose, followed by 24 hours of treatment while in the hospital and then discharge on oral antibiotics until drains are removed
E) One preoperative antibiotic dose, followed by 24 hours of treatment while in the hospital and then maintenance on oral antibiotics until tissue expanders are exchanged

A

The correct response is Option C.

The overall complication rate in breast reconstructive surgery is as high as 60%. Infection rates can exceed 20%, much higher than in clean elective surgery. The CDC guidelines suggest only 24 hours of peri-operative antibiotics beginning thirty minutes prior to skin incision. However, not all plastic surgeons agree with this. A 2013 meta-analysis found when comparing combined patient cohorts receiving no antibiotics, antibiotics for less than 24 hours, and antibiotics for greater than 24 hours, the average infection rates were 14.4, 5.8, and 5.8%, respectively. This demonstrated that the administration of antibiotics made a difference, however duration beyond 24 hours did not.

A study was published in 2013 evaluating the difference in surgical site infection between two different prophylactic antibiotic durations (24 hours and until drain removal). In this prospective, randomized, controlled non-inferiority trial, 24 hours of antibiotics is equivalent to extended oral antibiotics for surgical-site infection in tissue expander immediate breast reconstruction patients.

374
Q

A 28-year-old woman is scheduled to undergo vertical mastopexy. She has no history of previous breast surgery. A superior pedicle technique is planned. Which of the following is the dominant blood supply for this pedicle?

A) Deep branches of the internal mammary artery from the fourth interspace
B) Deep branches of the internal mammary artery from the fifth interspace
C) Superficial branches of the internal mammary artery from the second interspace
D) Superficial branches of the internal mammary artery from the fourth interspace
E) Superficial branches of the lateral thoracic artery

A

The correct response is Option C.

The breast receives its arterial blood supply from multiple sources, and this fact is used to design multiple pedicles for the nipple-areola complex that can work reliably for both mastopexy and reduction mammaplasty procedures.

The superior pedicle receives its arterial blood supply primarily from the internal mammary branch from the second interspace. It is usually about 1 to 2 cm below the surface of the skin just medial to the breast meridian as it approaches the areola and may be localized with a handheld Doppler device during preoperative planning.

The inferior pedicle and central pedicle designs are primarily supplied by branches of the internal mammary system from the fourth interspace. Additionally, there is some accessory input from the intercostal branches at the level of the inframammary fold with the inferior pedicle design. These secondary vessels are typically interrupted in a central pedicle operation.

The medial pedicle design receives its arterial input mainly from the third superficial branch of the internal mammary artery. This vessel may be damaged by previous augmentation mammaplasty.

The lateral pedicle design receives its arterial supply from superficial branches of the lateral thoracic artery.

375
Q

A 52-year-old woman is evaluated for breast reconstruction after modified radical mastectomy with adjuvant chemotherapy and radiation therapy 18 months ago. Her last radiation treatment was 8 months ago. BMI is 29 kg/m2. Examination today shows hyperpigmentation of the right chest wall with no redundancy of the mastectomy skin flaps. Her contralateral breast is a D cup with grade III ptosis. Which of the following methods will create the best symmetry for this patient?

A) Deep inferior epigastric perforator flap
B) Gel breast implant and acellular dermal matrix
C) Latissimus dorsi musculocutaneous flap
D) Tissue expander and acellular dermal matrix placement with planned staged exchange for permanent gel implant
E) Tissue expander placement alone with planned staged exchange for permanent gel implant

A

The correct response is Option A.

The deep inferior epigastric perforator flap would give the patient autologous tissue reconstruction with ample tissue for skin resurfacing and soft tissue for volume. In this radiated patient with a tight skin envelope, a tissue expander/implant, with or without acellular dermal matrix, would be difficult to create an appropriately ample skin envelope and the patient would be at higher risk for wound-healing problems and capsular contracture. The latissimus dorsi musculocutaneous flap, although an autologous tissue reconstruction, would have insufficient volume to adequately match this patient’s contralateral side. It would have to be combined with an implant.

376
Q

A 2-day-old female newborn is evaluated in the neonatal intensive care unit. Physical examination shows epicanthal folds, a flat nasal bridge, and a wide U-shaped cleft palate. She has micrognathia with 12 mm of overjet, substernal and costal retractions and desaturations while supine, and is not able to feed orally. Ophthalmologic evaluation shows bilateral cataracts. Which of the following metabolic abnormalities is most likely responsible for this patient’s condition?

A) Abnormal cellular response to fibroblast growth factor
B) Abnormal formation of collagen II
C) Abnormal regulation of craniofacial bone and suture embryogenesis
D) Dysregulation of embryogenesis caused by a multi-gene deletion on chromosome 22
E) Inadequate production of surfactant by the lungs

A

The correct response is Option B.

The neonatal patient described in the vignette has clear signs of respiratory distress in the setting of micrognathia and glossoptosis, also known as Pierre Robin sequence (PRS). PRS can be isolated or associated with a broad metabolic disturbance, the most common of which is Stickler syndrome. Stickler syndrome is a disruption of the formation of collagen, which can lead to multiple congenital anomalies including a flat nasal bridge, hearing loss, cleft palate, limb anomalies, micrognathia, and ophthalmologic issues including cataracts.

Mutations in the fibroblast growth factor receptor can lead to syndromic forms of craniosynostosis such as Apert, Crouzon, or Pfeiffer syndromes. Disruption of suture embryogenesis is caused by a mutation in the TWIST1 gene and gives rise to Saethre-Chotzen syndrome. Deletion of the small arm of chromosome 22 leads to 22q11.2 deletion syndrome, previously known as DiGeorge syndrome or Velocardiofacial syndrome. Lack of lung surfactant is most often caused by prematurity and is not associated with PRS.

References

Bütow KW, Morkel JA, Naidoo S, Zwahlen RA. Pierre Robin sequence: subdivision, data, theories, and treatment - part 2: syndromic and nonsyndromic Pierre Robin sequence. Ann Maxillofac Surg. 2016 Jan-Jun;6(1):35-37.

Gomez-Ospina N, Bernstein JA. Clinical, cytogenetic, and molecular outcomes in a series of 66 patients with Pierre Robin sequence and literature review: 22q11.2 deletion is less common than other chromosomal anomalies. Am J Med Genet A. 2016 Apr;170A(4):870-880.

Mingo KM, Sidman JD, Sampson DE, Lander TA, Tibesar RJ, Scott AR. Use of external distractors and the role of imaging prior to mandibular distraction in infants with isolated Pierre Robin sequence and Stickler syndrome. JAMA Facial Plast Surg. 2016 Mar-Apr;18(2):95-100.

377
Q

A 5-year-old girl has craniosynostosis, a low-set hairline, ptosis, and 4th/5th-digit syndactyly of both hands. Genetic testing is most likely to show a mutation in which of the following genes?

A) EFNB1
B) FGFR2
C) FGFR3
D) MPDZ1
E) TWIST1

A

The correct response is Option E.

The clinical picture is consistent with Saethre-Chotzen syndrome. It is an autosomal dominant condition defined by a genetic mutation or deletion affecting the TWIST1 gene or chromosome 7p21. FGFR2 mutations are predominantly associated with Apert, Crouzon, and Pfeiffer syndromes. FGFR3 mutations are associated with Muenke syndrome and Crouzon syndrome with acanthosis nigricans. EFNB1 is associated with craniofrontonasal syndrome. MPDZ1 is associated with hydrocephalus.

References

Cho E, Yang TH, Shin ES, Byeon JH, Kim GH, Eun BL. Saethre-Chotzen syndrome with an atypical phenotype: identification of TWIST microdeletion by array CGH. Childs Nerv Syst. 2013 Nov;29(11):2101-4.

Chun K, Teebi AS, Jung JH, et al. Genetic analysis of patients with the Saethre-Chotzen phenotype. Am J Med Genet. 2002 Jun 15;110(2):136-43.

de Heer IM, de Klein A, van den Ouweland AM, et al. Clinical and genetic analysis of patients with Saethre-Chotzen syndrome. Plast Reconstr Surg. 2005 Jun;115(7):1894-902; discussion 1903-5.

Wilkie AOM, Johnson D, Wall SA. Clinical genetics of craniosynostosis. Curr Opin Pediatr. 2017 Dec;29(6):622-628.

378
Q

A 2-year-old boy presents with swelling over the bridge of the nose that has been present since birth. The swelling has been slowly increasing in size and he has hypertelorism. The swelling is soft, compressible, and it transilluminates. There are visible and palpable pulsations, and the mass enlarges when the patient cries. Which of the following is the most likely diagnosis?

A) Encephalocele
B) Glioma
C) Hemangioma
D) Nasal dermoid cyst
E) Nasopharyngeal angiofibroma

A

The correct response is Option A.

Encephaloceles are neural tube defects that result in sac-like protrusions of the meninges (meningocele) or brain and meninges (meningoencephalocele) in various locations along the cranium, such as between the forehead and nose (including naso-orbital, frontonasal, and nasoethmoidal locations) or on the back of the skull. They tend to be soft, compressible masses that transilluminate that may be sessile or pedunculated. Biopsy may result in a cerebrospinal fluid leak.

Glioma is a mass of ectopic neural tissue that does not transilluminate.

Hemangiomas are benign vascular lesions that are present at birth and characterized by a rapid growth phase around the age of 1 to 6 months, followed by gradual involution over 1 to 12 years. They have no intracranial connection and no cerebral pulsations. Nasopharyngeal angiofibromas, also known as juvenile nasopharyngeal angiofibromas, are benign but locally invasive vascular tumors that occur almost exclusively in adolescent males. They present with unilateral or bilateral nasal obstruction, frequent epistaxis or blood-tinged nasal discharge. Nasal dermoid cyst is a benign cystic lesion that does not pulsate and does not transilluminate.

References

Holm C, Thus M, Hans A, et al. Extracranial correction of frontoethmoidal meningoencephaloceles: feasibility and outcome in 52 consecutive cases. Plast Reconstr Surg. 2008 Jun;121(6):386e-395e

Tirumandas M, Sharma A, Gbenimacho I et al. Nasal encephaloceles: a review of etiology, pathophysiology, clinical presentations, diagnosis, treatment, and complications. Childs Nerv Syst. 2013 May;29(5):739-44.

379
Q

A 23-year-old woman with severe progressive hemifacial atrophy that has been stable for 3 years now desires a long-term stable reconstruction. Which of the following is the most appropriate recommendation for reconstruction of this patient’s facial asymmetry?

A) Alloplastic bony augmentation
B) Contralateral suction lipectomy
C) Delay reconstruction until it has been stable for 10 years
D) Free tissue transfer
E) Hyaluronic acid injections

A

The correct response is Option D.

Progressive hemifacial atrophy is also known as Parry-Romberg syndrome. The progression is ultimately self-limiting. Reconstruction 2 years or more after burn out is commonly accepted. For very mild asymmetry, hyaluronic acid fillers can improve symmetry, but require recurrent treatments. For mild to moderate asymmetry, microfat grafting can restore symmetry. Multiple sessions may be required to achieve long-term correction. For severe asymmetry, free muscle flap with parascapular flap or anterolateral thigh flap can provide enough soft tissue bulk for long-term correction.

Alloplastic bony augmentation would correct any potential bony deficiencies, but would not address any soft tissue deficiencies.

References

Chen JT, Schmid DB, Israel JS, Siebert JW. A 26-year experience with microsurgical reconstruction of hemifacial atrophy and linear scleroderma. Plast Reconstr Surg. 2018 Nov;142(5):1275-1283.

El-Kehdy J, Abbas O, Rubeiz N. A review of Parry-Romberg syndrome. J Am Acad Dermatol. 2012 Oct;67(4):769-784.

Slack GC, Tabit CJ, Allam KA, Kawamoto HK, Bradley JP. Parry-Romberg reconstruction: optimal timing for hard and soft tissue procedures. J Craniofac Surg. 2012 Nov;23(7 Suppl 1):1969-1973.

380
Q

A 15-year-old girl has a 12-month history of pain and fullness in the right supraorbital rim. She has café-au-lait spots, a history of precocious puberty, and a recent pathologic rib fracture. Which of the following is the most likely pathology of the lesion?

A) Dermoid cyst
B) Fibrous dysplasia
C) Neurofibroma
D) Osteoblastoma
E) Rhabdomyosarcoma

A

The correct response is Option B.

This patient has a classic presentation of McCune-Albright syndrome. Patients with McCune-Albright present with a triad of polyostotic fibrous dysplasia, precocious puberty, and skin pigmentation (eg, café au lait spots). Additionally, they may have hyperfunctioning endocrinopathies such as growth hormone excess. If these patients present with intramuscular myxomas, it is known as Mazabraud syndrome. Malignant degeneration of fibrous dysplasia has been reported in up to 4% of patients with McCune-Albright syndrome. Management depends on the clinical presentation and functional impact of the lesions, and is primarily surgical.

Patients with neurofibroma may have café au lait spots, but not precocious puberty or pathologic fractures. Dermoid cysts generally do not present with pain and are often noted at a much younger age than the patient described.

References

Havlik R. Miscellaneous craniofacial conditions. In: Thorne CH, Chung KC, Gosain AK, et al, eds. Grabb and Smith’s Plastic Surgery. 7th ed. Philadelphia, PA: Lippincott Williams and Wilkins; 2004:295-310.

Riminucci M. Craniofacial Fibrous Dysplasia. In: Lin K, Ogle RC, Jane JA, eds. Craniofacial Surgery Science and Surgical Technique. Philadelphia, PA: WB Saunders; 2004:366.

381
Q

A 14-year-old girl with Crouzon syndrome presents with a severe Angle Class III malocclusion, mid face retrusion, and severe sleep apnea. She is scheduled to undergo Le Fort III advancement using distraction osteogenesis. The risk for complications with this procedure is closest to which of the following?

A) 5%
B) 20%
C) 40%
D) 60%
E) 80%

A

The correct response is Option B.

There are several important advantages of distraction osteogenesis for Le Fort III advancement versus conventional single-stage advancement with bone grafting and these include: less regression, greater advancement distance, and no need for bone grafting. Le Fort III distraction is not without its issues. Major and minor complications have been reported in nearly 20% of patients undergoing this procedure; these complications include bone loss, pin migration, loss of fixation, meningitis, seizures, and cerebrospinal fluid leaks. Several recent reports show that these complications occur in approximately 20% of cases.

References

Goldstein JA, Paliga JT, Taylor JA, Bartlett SP. Complications in 54 frontofacial distraction procedures in patients with syndromic craniosynostosis. J Craniofac Surg. 2015 Jan;26(1):124-128.

Knackstedt R, Bassiri Gharb B, Papay F, Rampazzo A. Comparison of complication rate between le fort III and monobloc advancement with or without distraction osteogenesis. J Craniofac Surg. 2018 Jan;29(1):144-148.

Raposo-Amaral CE, Denadai R, Pereira-Filho JC, Vieira PH, Ghizoni E, Raposo-Amaral CA. Serious complications after le fort III distraction osteogenesis in syndromic craniosynostosis: evolution of preventive and therapeutic strategies. J Craniofac Surg. 2018 Sep;29(6):1397-1401.

382
Q

Which of the following genetic mutations is most likely to be found in a patient with orofacial clefting and popliteal pterygium?

A) Gain-of-function mutation in FGFR2
B) Gain-of-function mutation in NF1
C) Gain-of-function mutation in PIK3CA
D) Loss-of-function mutation in IRF6
E) Loss-of-function mutation in TCOF1

A

The correct response is Option D.

IRF6 mutations that result in loss-of-function have been reported in both syndromic Van der Woude syndrome as well as nonsyndromic orofacial clefting; Van der Woude syndrome can include popliteal pterygium.

PIK3CA gain-of-function mutations have been reported with venous malformations and lymphatic malformations. Mutations in TCOF1 are associated with Treacher Collins syndrome; these patients may have cleft palate, but they do not have lip pits. Gain-of-function mutations in FGFR2 have been implicated in syndromic craniosynostosis (Apert syndrome, Crouzon syndrome, and others). Mutations in NF1 result in increased RAS/MAPK signaling and neurofibromatosis type 1.

References

Azoury SC, Reddy S, Shukla V, Deng CX. Fibroblast growth factor receptor 2 (FGFR2) mutation related to syndromic craniosynostosis. Int J Biol Sci. 2017 Nov 2;13(12):1479-1488.

Green AK, Goss JA. Vascular Anomalies: From a clinicohistologic to a genetic framework. Plast Reconstr Surg. 2018 May;141(5):709e-717e.

Hao S, Jin L, Wang H et al. Mutational analysis of TCOF1, GSC, and HOSA2 in patients with treacher collins syndrome. J Craniofac Surg. 2016 Sep;27(6):e583-e586.

Leslie EJ, Koboldt DC, Kang CJ et al. IRF6 mutation screening in non-syndromic orofacial clefting: analysis of 1521 families. Clin Genet. 2016 Jul;90(1):28-34.

Wu-Chou YH, Hung TC, Lin YT et al. Genetic diagnosis of neurofibromatosis type 1: targeted next-generation sequencing with multiple ligation-dependent probe amplification analysis. J Biomed Sci. 2018 Oct 5;25(1):72.

383
Q

A 2-month-old infant is referred for evaluation because he has an abnormal head shape. Physical examination shows low-set ears; short, webbed fingers; and duplicate great toes. A CT scan shows sagittal and lambdoid synostosis. A mutation in which of the following genes is most likely responsible for these findings?

A) FGFR1
B) FGFR2
C) FGFR3
D) RAB23
E) TWIST1

A

The correct response is Option D.

This child has Carpenter syndrome. This syndrome is caused by a mutation in the RAB23 gene, which is located on chromosome 6. Carpenter syndrome is inherited in an autosomal recessive manner, but it can also be caused by de novo mutation in RAB23. In addition to synostosis, symbrachydactyly and preaxial polydactyly are found in patients with Carpenter syndrome.

Mutations in the other genes listed are all associated with syndromic craniosynostoses. Fibroblast growth factor receptor (FGFR) mutations have been associated with several differing syndromes: FGFR1 mutations cause Pfeiffer syndrome, FGFR2 mutations cause Apert and Crouzon syndromes, and FGFR3 mutations cause Muenke syndrome. A mutation of the TWIST1 gene causes Saethre-Chotzen syndrome.

References

Alessandri JL, Dagoneau N, Laville JM, Baruteau J, Hébert JC, Cormier-Daire V. RAB23 mutation in a large family from Comoros Islands with Carpenter syndrome. Am J Med Genet A. 2010 Apr;152A(4):982-986.

Buchanan EP, Xue AS, Hollier LH Jr. Craniofacial syndromes. Plast Reconstr Surg. 2014 Jul;134(1):128e-153e.

Jenkins D, Seelow D, Jehee FS, et al. RAB23 mutations in Carpenter syndrome imply an unexpected role for hedgehog signaling in cranial-suture development and obesity. Am J Hum Genet. 2007 Jun;80(6):1162-1170.

384
Q

A 6-month-old female infant presents with a wide, tall forehead, low-set ears, and supraorbital rim retrusion. CT scan demonstrates bicoronal synostosis. Genetic testing is positive for TWIST mutation. Which of the following additional findings is/are characteristic of this patient’s disorder?

A) Cervical spine anomalies
B) Complete cartilaginous tracheal rings
C) Eyelid ptosis
D) Gastroschisis
E) Thumb duplication

A

The correct response is Option C.

The patient described has Saethre-Chotzen syndrome as confirmed by bilateral coronal synostosis, low-set ears, and mutations of the TWIST gene. In addition to these findings, patients with Saethre-Chotzen syndrome often have eyelid ptosis, which is a distinguishing feature from other forms of syndromic craniosynostosis. It is important to recognize ptosis in infants and young children in order to maintain adequate visual pathway development. Thumb duplication is not found in patients with Saethre-Chotzen syndrome. Tracheal anomalies are associated with Pfeiffer syndrome. Cervical spine anomalies can be found in both Apert and Pfeiffer syndromes. Gastroschisis is not associated with syndromic craniosynostosis.

References

Jadico SK, Huebner A, McDonald-McGinn DM, et al. Ocular phenotype correlations in patients with TWIST versus FGFR3 genetic mutations. J AAPOS. 2006 Oct;10(5):435-44.

Tahiri Y, Bastidas N, McDonald-McGinn DM, et al. New Pattern of Sutural Synostosis Associated With TWIST Gene Mutation and Saethre-Chotzen Syndrome: Peace Sign Synostosis. Bartlett SP. J Craniofac Surg. 2015 Jul;26(5):1564-7.

Taylor JA, Bartlett SP. What’s New in Syndromic Craniosynostosis Surgery? Plast Reconstr Surg. 2017 Jul;140(1):82e-93e.

385
Q

A 4-year-old boy is referred to the clinic for evaluation. Medical history includes a cardiac defect requiring surgery, submucous cleft palate, hypernasality, and developmental delay. His parents report that he undergoes cardiology follow-up annually. Further testing is most likely to detect an abnormality that will require consultation with which of the following specialties?

A) Endocrinology
B) Gastroenterology
C) Immunology
D) Nephrology
E) Psychiatry

A

The correct response is Option C.

The patient described has 22q11.2 deletion syndrome (formerly known as velocardiofacial syndrome or DiGeorge syndrome). This syndrome is the most common chromosomal deletion error in fetuses, with a prevalence of 1 in 3000 to 6000 live births. 22q11.2 Deletion syndrome is a common cause of hypernasality. Children with congenital heart defects and hypernasality should be worked up for 22q11.2 deletion syndrome. Either a FISH probe or microarray can detect the chromosomal deletion that occurs in the LCR22A–LCR22D region of the chromosome.

Children with 22q11.2 deletion syndrome can present with a myriad of clinical manifestations. The most common clinical manifestation is a congenital cardiac defect, particularly of the outflow tracts (e.g., tetralogy of Fallot). Congenital cardiac disease remains the primary cause of mortality in this patient population.

Hypernasality is another common finding within this patient population, occurring in approximately 65% of patients with 22q11.2 deletion syndrome. Classic workup for this involves imaging of the velopharyngeal mechanism (either nasopharyngoscopy or video fluoroscopy) and imaging of the posterior pharyngeal pharynx with MRI and evaluation of aberrant/medialization of the internal carotid arteries.

Immunologic abnormalities are the most common of the group. A referral to immunology should be initiated in all patients with 22q11.2 deletion syndrome since up to 75% of this patient population can have thymic hypoplasia and diminished T cell production. Children with 22q11.2 deletion syndrome can have chronic infections and poor responses to vaccinations.

Gastrointestinal conditions such as poor feeding, gastroesophageal reflux, and vomiting or constipation occur in approximately 30% of patients with 22q11.2 deletion syndrome. More concerning GI conditions such as malrotation or tracheoesophageal fistula have been found in patients with this syndrome.

Hypocalcemia secondary to hypoparathyroidism can present as tetany, seizures, or feeding issues. Hypocalcemia presents in approximately 50 to 65% of patients with 22q11.2 deletion syndrome. Thyroid function can also be abnormal with hypothyroidism possible.

Nephrology consultation should be considered if abdominal ultrasonography shows renal agenesis, duplication of the collecting system, or cystic kidney disease. About 33% of patients with 22q11.2 deletion syndrome have some abnormality related to the genitourinary system.

Patients with 22q11.2 deletion syndrome are at increased risk for psychiatric disorders such as attention-deficit/hyperactivity disorder (ADHD), autism spectrum disorder, and anxiety (particularly when they progress into adulthood). The rate of schizophrenia is increased in this patient population compared with unaffected individuals.

References

Bassett AS, McDonald-McGinn DM, Devriendt K, et al. Practical guidelines for managing patients with 22q11. 2 deletion syndrome. J Pediatr. 2011 Aug;159(2):332-9.e1.

McDonald-McGinn DM, Sullivan KE. Chromosome 22q11. 2 deletion syndrome (DiGeorge syndrome/velocardiofacial syndrome). Medicine (Baltimore). 2011 Jan;90(1):1-18.

McDonald-McGinn DM, Sullivan KE, Marino B, et al. 22q11. 2 deletion syndrome. Nat Rev Dis Primers. 2015 Nov 19;1:15071.

Hoeffding LK, Trabjerg BB, Olsen L, et al. Risk of psychiatric disorders among individuals with 22q11.2 deletion or duplication: a Danish nationwide, register-based study. JAMA Psychiatry. 2017 Mar 1;74(3):282-290.

386
Q

Patients with which of the following conditions have the highest incidence of accessory auricle?

A) Apert syndrome
B) Goldenhar syndrome
C) Treacher Collins syndrome
D) VACTERL association
E) Van der Woude syndrome

A

The correct response is Option B.

In a recent review of the literature about congenital accessory auricle, patients with Goldenhar syndrome were found to have an accessory auricle reported 100% of the time.

Of the options listed, VACTERL (Vertebral, Anal, Cardiac, Tracheal, Esophageal, Renal, and Limb association) has the lowest association with an accessory auricle at 16%. Patients with VACTERL typically are found to have vertebral defects, anal atresia, cardiac anomalies, tracheoesophageal fistula, renal anomalies, and limb differences.

Patients with Treacher Collins syndrome are found to have an accessory auricle present 30 to 40% of the time. These patients often have mandibular hypoplasia, zygomatic hypoplasia, external ear anomalies, conductive hearing loss, eyelid colobomas, cleft palate, and dental anomalies. It is associated with several different genes, including TCOF1, POLR1C, and POLR1D.

Patients with Apert syndrome and Van Der Woude syndromes do not typically have accessory auricles.

References

Online Mendelian Inheritance in Man. VATER/VACTERL ASSOCIATION. https://omim.org/entry/192350. Accessed 12/17/2017.

Online Mendelian Inheritance in Man. Treacher-Collins Syndrome. https://omim.org/entry/154500. Accessed 12/17/2017.

Amirhassankhami S, Lloyd MS. Accessory Auricles: Systematic Review of Definition Associated Conditions, and Recommendations for Clinical Practice. J Craniofac Surg. 2017 Dec 12.

387
Q

A 6-year-old girl is referred for velopharyngeal dysfunction and on physical examination is found to have a submucous cleft and bifid uvula. Which of the following additional clinical findings is most likely to support the diagnosis of 22q11.2 deletion syndrome?

A) Hypocalcemia
B) Hypothyroidism
C) Microtia
D) Overjet
E) Retinal detachment

A

The correct response is Option A.

22q11.2 Deletion syndrome (also referred to as DiGeorge syndrome, velocardiofacial syndrome, and CATCH-22) is a congenital disorder caused by the deletion of a segment of chromosome 22. Symptoms of this syndrome often include anomalous carotid arteries and conotruncal cardiac abnormalities such as truncus arteriosus and tetralogy of Fallot. The prevalence of atrial septal defect (ASD) has been reported to be 12% in patients with velocardiofacial syndrome. Other symptoms and findings include absent or hypoplastic thymus, developmental delay, cleft palate, and hypocalcemia related to hypoparathyroidism. Patients with velocardiofacial syndrome also have abnormal facial features including a broad nasal root, low-set ears, retrognathia, elongated face or maxillary excess, and epicanthal folds. Overjet and retinal detachment are not associated with velocardiofacial syndrome. Hypothyroidism is not as common as hypocalcemia in this patient population.

References

Yoo D, Kim HJ, Cho KH, et al. Delayed diagnosis of 22q11 deletion syndrome due to late onset hypocalcemia in an 11 year old girl with imperforate anus. 2017. Ann Pediatr Endocrinol Metab; 22:133-138.

Cancrini C, Puliafito P, Digilio MC, et al. Clinical features and follow-up in patients with 22q11.2 deletion syndrome. J Pediatr. 2014 Jun;164(6):1475-80.e2.

388
Q

An 18-month-old child is brought to the office after undergoing fronto-orbital advancement for metopic craniosynostosis. Which of the following is the earliest age the surgeon can order the x-ray studies and expect to be able to see frontal sinus development?

A) 1 year
B) 2 years
C) 4 years
D) 6 years
E) 10 years

A

The correct response is Option D.

Frontal sinus development is associated with specific age-related periods of growth of the skull. The frontal sinus is absent at birth and during the initial phase of growth of the skull. The sinus is visible only in x-ray studies at the end of the first period of skull growth. This is the time when the endocranial table of the skull ceases to grow and conforms to the general shape of the brain. This is not seen on x-ray studies until 6 years of age or 72 months.

References

Moore K, Ross A. Frontal sinus development and juvenile age estimation. The Anatomical Record. 2017;300(9):1609-1617.

Verma P, Verma KG, Khosa R, et al. Combined use of frontal sinus and nasal septum patterns as an aid in forensics: A digital radiographic study. North Am J Med Sci. 2015;7:47-52.

389
Q

A female infant is born with severe Treacher Collins syndrome and bilateral Pruzansky III mandible (absence of condyle). Tracheostomy is performed for respiratory distress. Which of the following surgeries is most likely to allow decannulation?

A) Alloplastic condylar reconstruction
B) Bilateral sagittal split osteotomy
C) Costochondral rib grafts
D) Mandibular distraction
E) Tongue-lip adhesion

A

The correct response is Option C.

Patients with Treacher Collins syndrome may have a varied presentation. The mandible hypoplasia may be mild or severe. In this case the patient has no temporomandibular fossa or condyles. Tongue-lip adhesion and mandibular distraction are used in severe Pierre Robin sequence. Because of the absence of condyles and temporal mandibular joints, distraction, and sagittal split osteotomy are not the best options. A costochondral graft will provide better airway support and can also be distracted in the future.

References

Flores RL, Tholpady SS, Sati S, et al. The surgical correction of Pierre Robin sequence: mandibular distraction osteogenesis versus tongue-lip adhesion. Plast Reconstr Surg. 2014;133(6):1433–9.

Tahiri Y, Viezel-Mathieu A, Aldekhayel S, et al. The effectiveness of mandibular distraction in improving airway obstruction in the pediatric population. Plast Reconstr Surg. 2014;133(3):352e–359e.

390
Q

Which of the following statements is correct about Tessier clefts No. 3, No. 4, and No. 5?

A) Tessier No. 3 involves the alveolar ridge, while Tessier No. 5 does not
B) Tessier No. 3 is medial to the infraorbital nerve, while Tessier No. 4 is lateral
C) Tessier No. 3 only affects the oral region, while Tessier No. 4 only affects the orbital region
D) Tessier No. 4 involves the piriform aperture, while Tessier No. 5 does not
E) Tessier No. 4 is medial to the infraorbital nerve, while Tessier No. 5 is lateral

A

The correct response is Option E.

Tessier No. 3 and No. 4 are medial to the infraorbital nerve, but Tessier No. 5 is lateral.

Tessier No. 3 involves clefts of the nose, orbit, and lip (naso-oral-ocular cleft), whereas Tessier No. 4 involves the lip and orbit (oral-ocular cleft), and the nose is uninvolved.

Tessier No. 5 involves oral, cheek (maxillary sinus), and orbital cleft and is the rarest.

References

Racz C, Dakpe S, Kadlub N, et al. Phenotypic spectrum of Tessier facial cleft number 5. J Cranio-Maxillofac Surg. 2018, 46: 22-27.

Spolyar JL, Hnatiuk M, Shaheen KW, et al. Tessier No. 3 and No. 4 clefts: Sequential treatment in infancy by pre-surgical orthopedic skeletal contraction, comprehensive reconstruction, and novel surgical lengthening of the ala base-canthal distance. J Craniomaxillofac Surg. 2015 Sep;43(7):1261-8.

S. M. Balaji. Two-stage Corrections of Rare Facial Tessier’s Cleft - 3,4,5,6,7. Ann Maxillofac Surg. 2017 Jul-Dec; 7(2): 287–290.

391
Q

Which of the following is the most common cause of scaphocephaly without craniosynostosis?

A) Fibromatosis colli
B) Hereditary dolichocephaly
C) Ocular torticollis
D) Prematurity
E) Ventriculoperitoneal shunt

A

The correct response is Option D.

Positional plagiocephaly is very common since the American Academy of Pediatrics began the ‘back to sleep’ campaign to decrease sudden infant death syndrome in 1992. Occipital flattening is the most common type seen. Positional plagiocephaly risk factors include prematurity, intrauterine position, congenital muscular torticollis, plural birth, motor delays, and hypotonia. Ventriculoperitoneal (VP) shunts may induce a secondary craniosynostosis if over-shunting occurs. Scaphocephaly head shape without craniosynostosis is noted in the premature babies in the NICU due to positioning and tone. This corrects with growth, increased tone, and repositioning. Presence of a VP shunt does not lead to the head shape of scaphocephaly without craniosynostosis but may be seen in children after intraventricular hemorrhage in the NICU. Scaphocephaly without craniosynostosis does occur in older infants and children but is not known to be a hereditary condition. Both ocular torticollis and congenital muscular torticollis can cause a head tilt and are associated with deformational plagiocephaly. Fibromatosis colli is seen with severe muscular torticollis and is associated with plagiocephaly.

References

Joganic JL, Lynch JM, Littlefield TR, Verrelli BC. Risk factors associated with deformational plagiocephaly. Pediatrics. 2009 Dec;124(6):e1126-33.

Beckett JS, Pfaff MJ, Diluna M, Steinbacher DM. Dolichocephaly Without Craniosynostosis. J Craniofac Surg. 2013 Sep;24(5):1713-5.

Rogers GF. Deformational plagiocephaly, brachycephaly, and scaphocephaly. Part I: terminology, diagnosis, and etiopathogenesis. J Craniofac Surg. 2011 Jan;22(1):9-16.